You are on page 1of 495

C.

T. C. BapaneHKoe. B. 77 AeMudoeuH, B. A
M. Koean, r Jl JJyHit, E
noptuneea,
C. B. <Ppo/io8> P. fl. UlocmaK, A. P.

H. Ctweea,

SAflAMM H VnPA)KHEHHfl

no

MATEMATM H ECKOMV
AHAJ1H3V

I7od
B.

H.

AE

rocydapcmeeHHoe
a

G. Baranenkov* B. Drmidovich V. Efimenko, S. Kogan,


G. Lunts>> E. Porshncva, E. bychfia, S. frolov, /?. bhostak,
A. Yanpolsky

PROBLEMS
IN

MATHEMATICAL
ANALYSIS
Under
B.

the editorship
of

DEMIDOVICH

Translated from the Russian


by
G.

YANKOVSKV

MIR PUBLISHERS
Moscow

TO THE READER
MIR
opinion

of

Publishers would be
the translation and

glad
the

to

have your

design

of

this

book.

Please send your suggestions to 2, Pervy Rtzhtky


Pereulok, Moscow, U. S. S. R.

Second Printing

Printed

in

the

Union

of

Soviet Socialist

Republic*

CONTENTS
9

Preface

Chapter
Sec.

I.

INTRODUCTION TO ANALYSIS

1.

Functions

Sec. 2

Graphs

Sec. 3

Limits

of

11

Elementary Functions

16

22

Infinitely Small and Large Quantities


Sec. 5. Continuity of Functions

33

Sec. 4

Chapter II
Sec
Sec

1.

36

DIFFERENTIATION OF FUNCTIONS
Calculating Derivatives Directly
Tabular Differentiation

42
46

Sec. 3 The Derivatwes of Functions Not Represented Explicitly


Sec. 4. Geometrical and Mechanical Applications of the Derivative
Sec 5 Derivatives of Higier Orders

Sec

Sec

Sec. 8

56

60

66

and Higher Orders


Mean Value Theorems
Taylor's Formula
Differentials of First

9 The L'Hospital-Bernoulli Rule


Forms

Sec

for

Evaluating

71

75

77

Indeterminate

78

THE EXTREMA OF A FUNCTION AND THE GEOMETRIC


APPLICATIONS OF A DERIVATIVE

Chapter III

Sec.

1.

Sec. 2

Sec

Sec

4.

Sec. 5.

Chapter IV
Sec.

Sec

Sec

Sec. 4

Sec. 5.

The Extrema of a Function


The Direction of Concavity

of

One Argument

83

Points of Inflection

Asymptotes
Graphing Functions by Characteristic Points
Differential of an Arc Curvature

91

93

96
.

101

INDEFINITE INTEGRALS
Direct Integration
Integration by Substitution
Integration by Parts

Standard Integrals Containing a Quadratic Trinomial


Integration of Rational Functions

107

113
116

....

118
121

Contents

Sec. 6.

Integrating Certain Irrational Functions

125

Sec

Integrating Trigoncrretric Functions


Integration of Hyperbolic Functions

128

7.

Sec. 8

Sec

9.

133

Using Ingonometric and Hyperbolic Substitutions

integrals of the

Form

(x,

^a^ + bx + c) dx,

for

Where R

Finding
is

Ra-

tional Function

Sec

10

Sec

11

Sec.

12.

133

Integration of

Vanou* Transcendental Functions

135

Using Reduction Formulas


Miscellaneous Examples on Integration

135
136

DEFINITE INTEGRALS

Chapter V
Sec.

1.

Sec

The Definite Integral

as the Limit of a

Sum

138

Evaluating Ccfirite Integrals by Means of Indefinite Integrals 140

Sec. 3

Improper Integrals

Sec

Charge

of

143

Variable in a Definite Integral

146

Integration by Parts

Sec. 5.

149

Mean-Value Theorem
Sec. 7. The Areas of Plane Figures
Sec 8. The Arc Length of a Curve
Sec 9 Volumes of Solids
Sec 10 The Area of a Surface of Revolution

Sec

Sec

11

torrents

Sec

12.

Applying Definite Integrals

Centres

of

Gravity

150

153
158
161

166

Guldin's Theorems

to the Solution of Physical

168

Prob-

lems

Chapter VI.
Sec.

1.

173

FUNCTIONS OF SEVERAL VARIABLES


Basic Notions

180

Sec. 2. Continuity

184

Sec

Partial Derivatives

185

Sec

Total Differential of a Function

187

Sec

Differentiation of Composite Functions


190
Given Direction and the Gradient of a Function 193

Sec. 6. Derivative in a
Sec. 7

Sec
Sec
Sec

HigKei -Order Derivatives and Differentials

197

Integration of Total Differentials


9 Differentiation of Implicit Functions
10 Change of Variables

202
205

.211
217

15

The Tangent Plane and the Normal to a Surface


for a Function of Several Variables
.
The Extremum of a Function of Several Variables
....
*
Firdirg the Greatest and tallest Values of Functions
Smcular Points of Plane Curves

16

Envelope

232

Sec.

11.

Sec

12

Sec.

13

Sec

14

Sec

Sec

Taylor's Formula

Sec. 17. Arc Length o! a Space

Curve

220
222
227

230
234

Contents
Sec.

18.

Sec.

19

The Vector Function of a Scalar Argument


The Natural Trihedron of a Space Curve
Curvature and Torsion of a Space Curve

Sec. 20.

Chapter VII.
Sec.

The Double

Integral in Rectangular Coordinates

Sec.
Sec.

7.

Triple Integrals

Sec.

8.

Improper Integrals

Sec.
Sec.

Sec.

238

242

MULTIPLE AND LINE INTEGRALS

Change of Variables in a Double Integral


3. Computing Areas
4. Computing Volumes
5. Computing the Areas of Surfaces
6 Applications of the Double Integral in Mechanics

Sec.

235

246
252

256
258

259
230
262

Dependent

on

Parameter.

Improper
269

Multifle Integrals
9 Line Integrals

273

Sec.

Sec.

10.

Surface Integrals

284

Sec.

11.

Sec.

12.

The Ostrogradsky-Gauss Formula


Fundamentals of Field Theory

288

286

Chapter VIII. SERIES


Sec.

Number

1.

293

Series

Sec. 2. Functional Series

304

Sec. 3. Taylor's Series


Sec. 4. Fourier's Series

318

Chapter
Sec.

311

IX DIFFERENTIAL EQUATIONS
1.

Verifying Solutions. Forming Differential Equations


Curves. Initial Conditions

of

Fami-

lies of

Sec. 2

Sec.

3.

327

Orthogonal Trajectories
Sec. 4

322

324
First-Order Differential Equations
First-Order Diflerential Equations with Variables Separable.
First-Order

Sec. 5. First-Order

Homogeneous
Linear

Differential

Differential

330

Equations
Equations.

Bernoulli's

332
Equation
335
Sec. 6 Exact Differential Equations. Integrating Factor
Sec 7 First-Order Differential Equations not Solved for the Derivative 337
339
Sec. 8. The Lagrange and Clairaut Equations
Sec. 9. Miscellaneous Exercises on First-Order Differential Equations 340
345
Sec. 10. Higher-Order Differential Equations
349
Sec. 11. Linear Differential Equations
Sec.

12.

Linear Differential Equations of Second Order with Constant

Coefficients

351

Contents

8
Sec. 13. Linear

Differential

Equations of Order

Higher

than

Two
356

with Constant Coefficients


Sec

14.

Euler's Equations

Sec

15.

Systems

of

Differential

357

Integration of Differential Equations by

Sec. 16.

359

Equations

Means

of

Power

Se-

361

ries

Sec

17.

Chapter X.

Problems on Fourier's Method

363

APPROXIMATE CALCULATIONS
367

Sec. 2.

Operations on Approximate Numbers


Interpolation of Functions

Sec.

Computing the^Rcal Roots

376

Sec.

3.

Sec. 4

Numerical, Integration of

Nun

Equations
Functions

Integration of Ordinary DilUrtntial Equations


Sec. 6. Approximating Ftuncr's Coefficients
Sec. 5.

372

of

er:ca1

382
.

384

3>3

ANSWERS

396

APPENDIX

475

I.

II.

Greek Alphabet
Some Constants

Inverse Quantities, Powers, Roots, Logarithms


Trigonometric Functions
V. Exponential, Hyperbolic and Trigonometric Functions
VI. Some Curves

475

475

III.

476

IV

478
479
480

PREFACE
This collection of problems and exercises in mathematical analcovers the maximum requirements of general courses in
ysis
higher mathematics for higher technical schools. It contains over
3,000 problems sequentially arranged in Chapters I to X covering
branches of higher mathematics (with the exception of analytical geometry) given in college courses. Particular attention is
given to the most important sections of the course that require
established skills (the finding of limits, differentiation techniques,
the graphing of functions, integration techniques, the applications
all

of definite integrals, series, the solution of differential equations).


Since some institutes have extended courses of mathematics,

the authors have included problems on field theory,

method,

and

the

Fourier

approximate calculaiions. Experience shows that


problems given in this book not only fully satisfies

the number of
the requireiren s of the student, as far as practical mas!ering of
the various sections of the course goes, but also enables the instructor to supply a varied choice of problems in each section
to select problems for tests and examinations.
Each chap.er begins with a brief theoretical introduction that
covers the basic definitions and formulas of that section of the
course. Here the most important typical problems are worked out
in full. We believe that this will greatly simplify the work of
the student. Answers are given to all computational problems;
one asterisk indicates that hints to the solution are given in
the answers,
two asterisks, that the solution is given. The
are
frequently illustrated by drawings.
problems
This collection of problems is the result of many years of
teaching higher mathematics in the technical schools of the Soviet
Union. It includes, in addition to original problems and examples, a large number of commonly used problems.

and

Chapter I

INTRODUCTION TO ANALYSIS

Sec.

1.

Functions

1. Real nurrbers. Rational and irrational numbers are collectively known


numbers The absolute value of a real number a is understood to be
the nonnegative number \a\ defined by the conditions' \a\=a if a^O, and
= a if a < 0. The following inequality holds for all real numbers a
|aj
as real

ana

b:

2. Definition of a function. If to every value*) of a variable x, which


belongs to son.e collection (set) E, there corresponds one and only one finite
value of the quantity /, then y is said to be a function (single-valued) of x
or a dependent tariable defined on the set E. x is the a r gument or independent variable The fact that y is a Junction of x is expressed in brief form
by the notation y~l(x) or y = F (A), and the 1'ke
If to every value of x belonging to some set E there
corresponds one or
several values of the variable /y, then y is called a multiple- valued function
of x defined on E. From now on we shall use the word "function" only in
the meaning of a single-valued function, if not otherwise stated
3
The domain of definition of a function. The collection of values of x for
which the given function is defined is called the domain of definition (or the
domain) of this function. In the simplest cases, the domain of a function is
either a closed interval [a.b\, which is the set of real numbers x that satisfy
the inequalities
or an open intenal (a.b), which :s the set of real
numbers that satisfy the inequalities a
a
more comx
b. Also possible is
plex structure of the domain of definition of a function (see, for instance, Prob-

a^^^b,

< <

lem 21)

Example

1.

Determine the domain

of definition of the function


1

Solution.

The function

is

defined

x
that

is,

if

|x|>

1.

oo<x<

vals:

Thus, the
1

and

if

-l>0,

domain

<x<

-\-

of the function is a set

of

two

inter-

oo

4. Inverse functions. If the equation t/ /(x) may be solved uniquely for


the variable x, that is, if there is a function x
g(y) such that y^*

*)

stated.

Hencetorth

all

values will be

considered

as

real,

if

not

otherwise

Introduction to Analysis

12

\Ch.

then the function x = g(y), or, in standard notation, y=g(x), is the inverse
of y = f(x). Obviously, g[f(x)]s&x, that is, the function f (x) is the inverse
of g(x) (and vice ve^sa).
In He fereia! case, the equation y
f(x) defines a multiple-valued inverse function x
f~ (y) such that y ==[[(-* (y)\ for all y that are values of
the function f (x)
Lxanple 2. Determine the inverse of the function

y=l-2-*.
Solution. Solving equation

(1) for x,

(1)

we have

2-*=l

and

log(l-y)

log 2

'

of the function (2) is


the domain of
oo </<!.
Definition
5. Corrposite and irrplicit functicns. A function y of x defined by a seof equalities y = /(), where u = 9 (x), etc., is called a comoosite function,

Obviously,
ries

or a function of a function.
A function defined by an equation not solved for the dependent variable
the equation x*+i/*=l defines
is called an implicit (unction. For example,
y as an implicit function of x.
6. The graph of a function. A set of points (x, y) in an ;o/-plane, whose
coordinates are connected by the equation y
f(x), is called the graph of
the given function.

1**.

Prove that

if

a and b are real numbers then

\\a\-\b\\<\a-b\<\a\ + \b\.
Prove the following equalities:

2.

a)

|ab|Ha|.|b|;
2

b)

|a|

d)

Solve the inequalities:

3.

1|<3;

a)

|x

b)

|x+l|>2;

Find /(-1),/(0),/(1),

4.

4- llx

c)

d) |jt-

/(2), /(3), /(4),

if

/(*)

= *'- 6*' -f

6.

5.

Find

6.

/ (x)

7.

The function

and /(2)
*)

=a

c)

/(O),

-arc cos (log x). Find

Log x

/(-4). /(-^/(T)
f(x)

is

linear.

/!,
Find

7^.

if/

/(I), /(10).

this function, if/(

3.

is

the logarithm of the

number x

to the base 10.

1)

=2

Sec

Functions

1]

13

Find the rational integral function f(x) of degree two,


= and /(3) = 5.
9. Given that f(4) =
2, /(5) = 6. Approximate the value
if we consider the function / (x) on the interval
3)
/(4,
linear (linear interpolation of a function).
10. Write the function
8.

0,

if

of

if

as a single formula using the absolute-value sign.


Determine the domains oi definition of the following functions:
11.

a)

y=

x+\;

y=

16.
17.

a),= ?E2L
= *VV-2.
14**.
=1/2 + * **.

= lo

13.

b)

21.

19.

(/

Determine the domain

22. f(jc)

= 2A:

SA;'

5x

+ 6A:

(-*)l

function

Find

10.

and

^(^)

defined in a symmetric region


/
and
orfd
if
x)
f(x).
f(
f(x)
/(
x)
Determine which of the following functions are even and which

23.
is

of definition of the function

= |/sin 2x.

y
4

t/=

called euen

f (x)

if

are odd:

e)

24.

Prove

/<;*</

that

may

any function
be represented

defined
in the interval
the form of the sum of an

f(x)
in

even function and an odd function.

In t roduction to Analysis

14

[Ch.

25. Prove that the product of two even functions or of two odd
functions is an even function, and that the product of an even
function by an odd function is an odd function.
26. A function f (x) is called periodic if there exists a positive
numter T (the period of the function) such that f(x+ T)^f(x)
for all valves of x within the dcmain of definition of f(x).
Determine uhich of the following functions are periodic, and
for the periodic functions find their least period T:

= 10 sin 3 *,
=
a sin \K + b cos tar;
(*)

= sin *;
= sin (J/*).
/ (x)
1

a) / (x)

d) / (x)

b)

e)

c)

and the area S


Express the length of the segment y =
as a function of
Construct
(Fig
1).
the graphs of these functions.

27.

x=AM

AMN

of the figure

28. The linear density (that is,


mass per unit length) of a rod AB = l
(Fig. 2) on the
segments AC
l^
CD = 1 2 and DB = l\ (/ + l t + / 3 =- 1)
to </,, q z and q^ respecis equal
t

AfsM"

MN

B
Fig.

Fig. 2

AM =

x of this
of a variable segment
tively. Express the mass
rod as a function of x. Construct the graph of this function.
29.

Find cp|\M*)| and

30.

Find

31.

Find

H/Um,
/U+l),

Let f(n) be the


that

32.

Show

33.

Show

that

i|?

jtp(x)J,

if

v(x)

= x?

and q(x)

= 2*.

if

f(x

if

sum

l)

of

= x\

n terms

of

an arithmetic progression.

if

f(x)

and the numbers jc lf xt x t form an arithmetic progression, then


the numbers J (x ) f (x t ) and / (xj likewise form such a pro,

gression.

Sec.

__
Functions

1]

15

Prove that if f(x) is an exponential function, that is,


and the numbers x v *,, x t form an arithmetic
progression, then the numbers /(*,), f (* 2 ) and /(jcj form a geo34.

/ (x)

= a x (a >0),

metric progression.
35. Let

Show

that

Let

36.

Show

<p

(*)

= !(* + a-*)

that
f

(*+</)

<p

and
ty

37.

(x

+ y)

and

<p

(AT)

t|)

(y)

= 1 (a*

+ * (*)

^>

a-*).

(y)

<p

Find /(-I), /(O), /(I)


arc sin
arc tan

if

^ ^
< # c + oo.

x for
x for

0,

38. Determine the roots (zeros) of the


rrgion of positivity
of the region of negativity of the function
y if:
a)

b)
c)

39.

In
40.

r/=l

-f-x;

y=2+x
0=1

d)

y = x*

ff

2
;

and

3x;

_ ioo

2jc

Find the inverse of the function y

if:

what regions will these inverse functions be defined?


Find the inverse of the function
/

x,
x*,

if

if

Write the given functions as a series of equalities each


of which contains a simple elementary function (poweri
exponential, trigonometric, and the like):
41.

member

a)

b)

= (2*-5r;
*
y=2
i/

c)

d)

COS

= arc sin (3-*

).

Introduction to Analysts

16

[Ch.

Write as a single equation the composite functions repre-

42.

sented as a series of equalities:


a)

b)

y=

= sin#;
# = arctan, u = Yv,
w

u*>

w,
,

if

= log#;

t/<0,

ifu>0;

*_!.
43. Write, explicitly, functions of
a)

b) 10*
c)

arc cos y

y defined by the equations:

= n;

+10' =10;

+ \y\ = 2y.

Find the domains of definition of the given implicit functions.


Sec, 2. Graphs of Elementary Functions

Graphs of functions # /(*) are mainly constructed by marking a suffiand


/ (*,-) (/ = 0, 1, 2, ...)
ciently dense net of points Ai /(*,-, //), where */,
by connecting the points with a line that takes account of intermediate points.
Calculations are best done by a slide rule.

Fig. 3

Graphs of the basic elementary functions (see Ap pendix VI) are


learned through their construction. Proceeding from the graph of
y

we

= f(x),

readily

(T)

get the graphs of the following functions by means of simple geometric


constructions:
M*) js th * mirror image of the graph T about the *-axis;
1) 0i
2) 0i=/(
*) is the mirror image of the graph F about the #-axis;

Sec. 2]

3) #i
4) 1/4

Graphs

= /(*)

= & + /(*)

the

is
is

F graph displaced along

the F

graph

The

to the right

desired line

is

17

th? x-axis by an
Uve (/-axis by

along

displaced

(Fg. 3).
Example. Construct the graph

Solution.

Elementary functions

of

amount a;
an amount

of the function

y = sinx displaced along the *-axis

a sine curve

by an amount -j

(Fig. 4)

Fig. 4

Construct

the

(straight lines):
44. y
kx, if
45.
46.

i/

=
= x+

6,

fc

if

= 0,
6 = 0,

1,

the

of

graphs

2,

1/2,
2,

1,

following

functions

-1, -2.
1,

2.

0=1. 5* +2.

Construct the graphs of rational integral

two

linear

functions of degree

(parabolas).
3

47. y=--ax , if a=l, 2,


48. //
*'-{- c, if c=0,
49. ,/=(*-*)', if *.

1/2,

2,

1,

1,

2, 0.

1.

= 1, 2, -1.
= y, 4 (x-l)\
= 0, 1, 2, -1.
= 2, c = 3; 2) a =
51*. y = ax* + bx + c,
1) a=l, b
6 = 6, c = 0.
= 2 f x x*. Find the points ol intersection of this
52.
50. y

if

</

if:

t/

rabola with the

2,

pa-

Ac-axis.

Construct the graphs of

the

following rational integral func-

tions of degree above two:


x* (cubic parabola).
53*. y
2 +(*-!)'.
54. </
x'
55. t/
3x-\-2.
x\
56. y
2x'-x*.
57. y

=
=
=
=
=

Construct the graphs of


tions (hyperbolas):
68*.

0=4.

the

following

linear

fractional func-

_ _
Introduction to Analysis

18

-.

59.

y=
*-

61*.

62 *-

Construct the graphs of the fractional rational functions:


63.

=*+

</

-.

65*.

y=.

66.

67*.

=r
(WtYc/i of Agnesl).
,^rj
2x
=x
(Newton's serpentine).

i/

68.

*/

69.

y=*-h^.

=^

70. y

(trident of Newton).

Construct the graphs of the irrational functions:


71*. */=]/*:
72.

73*.

y=t/x._2
y=-*/x

74.

y=x\fx

(semicubical parabola).

y=j/25

75*.
76.

(Niele's parabola).

f/=

Vx

(hyperbola).

y- >=,.
J^l-A __
78*. y = + x
y ^^

77.

(ellipse).

(cissoid of Diodes).

==

x 1/25
x
79. r/
Construct the graphs of the trigonometric functions:
83*. */=-cotjc.
80*. y
sinx.
84*. y
sec x.
81*. y
cosx.
85*. y = cosec x.
82*. /=-tanx.
.

86.

87*.

{/-/4sinx

y^smnx,
/

if
if

y= sin(x
89*. y = 5sin(2x

88.

cp),

/4

= 1,

n=l,
*

if

3).

10,
2,

9 = 0,
rv

2.

1/2,
3,

1/2.

Jl

3ll

-J-,

-j-

ft

~T*

[Ch. 1

Sec. 2]

Graphs

of

Elementary functions

19

Construct the graphs of the exponential and logarithmic functions:

101.

= ax
a = 2, l (?(e = 2, 718 ...)*).
a =10, 2, 1,
y = \oga x,
-x
x
=
where
e
sinhx,
y
sinhx=l/2(e
= coshx, where coshx =
= tanhx, where

102*.

if

*.

if

103*.
104*.

).

f/

105*.

f/

106. 0=10*.
2
107*. y=-e~* (probability curve).

= 2""

108. ^

*3

109. //-logx
2
110. y-=log A:.
111. //=-log(logx).

114.

112.

117

the

graphs

118*. y--=arc sin*.


119*.

(/=--

=
= log"(cosx).
^ = 2-^ sin*.

115.
116.

/y==rV-log X

Construct

y=

113.

{/

t/

of the

inverse trigonometric functions?

= arcsin~.
x
=
arc cos--.
#
=
+ arc cot x.
^

122. #

= arccosx.
# = arc tan*.

123.

j/

120*.
124.
A:
121*, (/= arc cot x.
Construct the graphs of the functions:
125.

y=\x\.

127.

= ^(x + \x\).
= x\x\\
a) y

128.

a)

126. y

129.

*)

b)

t/= sinA:+ sin jt|;


b)
2
x when |jc|< 1.
3

^-<

y = log^^l x
f/= sin x
sinx|.
\-

_l. whcn

About the number

* see p. 22 for

more

details.

Introduction to Analysis

20

[C/i.

=
#=[*], b) y x[x], where [x] is the integral part
number x, that is, the greatest in.eger less than or equal

130. a)
of the

to

x.

Construct the graphs of the following


coordinate system (r, cp) (r^O):
131.

132*.

133*.

= l.
f = 7r
=
r =
<*>

/-

134*.

in the polar

functions

(spiral of Archimedes).

(logarithmic spiral).

(hyperbolic spiral).

= 2cosip (circle).
' =
-^- (straight line).
= sec*y (parabola).

135. r
136.
137.

/-

138*. r=10sin3(p (three-leafed rose)


139*. r
a(l fcoscp) (a>0) (cardioid).
2
I
143*. r
a cos2(p (a>0) (lemniscate).
Cjnstruct the graphs of the functions represented parametri-

=
=

cally:

t* (semicubical parabola).
141*. x
t\ y
142*. *=10 cos/, y=sin/ (ellipse).
3
1
143*. *=10cos /,
10 sin / (astroid).
144*. jc
a(cos/-f / sin/), t/
a(sm / /cos/)

y=

(involute of a

circle).

^'

145*. ^

146

'

^3,

=
rTT'

^0//wm ^

Descartes).

/==

2- (branch of
y=2
=
# = 2 sin
(segment of
*-/t\
y=t
x^a (2 cos/ cos2/), = a(2sin/

147. xasfc'-t^143. jc
2cos f f

149.
150.

J/

a hyperbola).
a straight line).

*/

sin 2/) (cardioid).

Cjnstruct 'the graphs of the following functions defined implicitly:

151*.x
152.

25 (circle).
xy--= 12 (hyperbola).
*/

= 2jc (parabola).
154. ^1 + ^! =
= jc'(10
155.
156*. x T + y T =;aT (astroid).
153*.

i/

j/*

157*. x
158. *'

Graphs

Sec. 2]

of

Elementary Functions

21

a"

=e
=

*
159*. |/V
y
(logarithmic spiral).
8
160*. x*
3x//
(folium of Descartes).
y
161. Derive the conversion formula Irom the Celsius scale (Q
to the Fahrenheit scale (F) if it is known that
corresponds

0C

to

32F

and 100C corresponds to 212F.


Construct the graph of the function obtained.
162. Inscribed in a triangle (base 6^=10, altitude h

rectangle (Fig. 5).


tion of the base x.

Fig. 5

Fig

Construct the graph of this function and


value.
163. Given a triangle ACB with BC
a, AC

ACB = x (Fig. 6).


angle
area
ABC as a function of
Express #
of this function and find its greatest value.

Give

164.

a) 2x'

b) x*
c)

find

=b

x.

a graphic solution of the equations:

5x + 2 =
x
1=0;

0;

d)
e)

= 0.1jc;
logJt

f)

I0'

x=l
cot

= x\
4

x^x

xy=10, x

b)

xr/-6,

c)

d)

e)

#=sinx,

5sin;c;

(0<jc<jt).

= 7\

2
2

+
j/

= cos#

(0

<x<

2jt).

its

greatest

and a variable

Plot the graph

165. Solve tjie systems of equations graphically:


a)

6) is a
as
a
of
the
functhe
area
y
rectangle
Express

22

Introdnction to Analysis

(C/t.

Sec. 3. Lfmits

xn

lim
n >
if

for

\xn

e>0

any
|

<e

there

when

Example

1.

the limit of a sequence

is

a,

such that

(e)

n> N.

Show

that

5L + 1.2.

Urn
n

Solution.

oo

N=N

number

is

number

The

1. The limit of a sequence.


x lt .... X0, .... or

*!

Form

-*

rt-r

(1)

the difference

2*

+1

Evaluating the absolute value

of this difference,

-2

<

we have:
e,

(2)

if

n>-\ = N
Thus, for

every

number

positive

(e).

there will

be a number

Af=

such

>

N we will have inequality (2) Consequently, the number 2 is


that for n
the limit of the sequence x n
(2n-\- l)/(n-fl), hence, formula (1) is true.
2. The limit of a function. We say that a function / (x) -*- A as x -+ a
(A and a are numbers), or
lim f(x)
x -a
if

for

every 8

>

\f(x)A <e

we have

<

for

=6

()

>

such that

f(jO

= 4,

a|<6.

|x

Similarly',

lim
*

\f( X

if

-> 00

)A\<*

The following conventional notation


lim

*-*a

which means that

f (x)

>

for

/(x)

<

for

|x|>

/V(e).

also used:

is

= oo,

< 6 (E),

where E

number

positive

3. One-sided
x -- a

= A,

limits.

0; similarly,

f(a

ii

0)=
*

If

>a
a

and x
-+ a,

-* a,

then

f(x) and /(a

lim
-

<a

and x
o

we

is

an arbitrary

we write conventionally
write * -^ a-f-0. The numbers

then

+ 0)=

lim

(x)

*-*a + o

*re called, respectively, the limit on the left of the function f (x} at the point a
and the //mi/ on the right of the function / (x) at the point a (if these

numbers

exist).

Limits

Sec. 3]

23

For the existence of the limit of a function


and sufficient to have the following equality:

the limits

as jc-^o,

it

is

necessary

O)-/

/(a
If

/ (x)

lim /, (x) and lim f 2 (x) exist, then the


x ->a
x -> a

following theorems.

old:
1)

x
2)

3)

lim
-+ a

[/,

(*)

+ /, (*)] = x lima

[f,

(x) f 2

(jc)J

lim [f ,
x- o

W/^

(JK)J

lim

x-*a

+ xlimaf

/, (x)

-+

(x);

lim f, (x). lim f t (x);


x -* a
x -* a

lim
*

The following two

-*

lim

/, (x)l

-#

-*

jc

^ (x)

(lim f , (x)
Jt

-^

^ 0).

limits are frequently used:

ILi=i

lim
AP

->-0

and
lim

Example

2.

1-J--L

lim

(l

Find the limits on the


/ (x)

a
a)

right

=* = 2

and

left

71828

of the function

= arc tan-

as x ->-0.

Solution.

We

have
lim
x

.+

arc tan
)= 2
x J
+o fa
\

and

f(-0)=

lim
-

x->.

Obviously, the function

166.

is

Prove that as n

/ (x)

faictanlW-42
/
A:

o \

in this case

has no limit as x--0.

*oo the limit of the sequence

equal to zero. For which values of n will

we have

the inequal-

= 0.01;

c) 8

ity

an arbitrary positive number)?


Calcula e numerically for a) e
0.1; b) e
167. Prove that the limit of the sequence

(e is

= 0.001

24

_ _

as

Introduction to Analysis

>oo

rt

is

which

For

unity.

the inequality

(e

n>N

values of

an arbitrary positive number)?


Find N for a) e = 0.1; b) e = 0.01;
168. Prove that
liin x*=:4.
X ->

How

we have

will

K-l|<e

is

= 0.001.

c)

should one choose, for a given positive number

number

positive

[CH. 1

e,

some

6 so that the inequality


2

-4|<8

|*

should follow from

Compute 6

for a)

= 0.1;

= 0.01;

b)

c)

Give the exact meaning of the


Hoi log* =
oo; b) lim 2*= +00; c)

169.
a)
*

->

+0

170.

c)

_ 2*3'
1 _

a)
*}
b)
V)

X + +

Find the limits

i,

'

1/2",

'

'

(-

)""

_2n_
'

...

2/i~l

'

'

'

1/21/21/2",
.

'

limits:

a+4 + l,+ ...


*\*

lirn
fl

'

'

Hm
n

173

-> 00

of the sequences:

0.23, 0.233, 0.2333,

Find the

172.

1/2

1/2;

d) 0.2,

171.

'

liai/(x)
X

00

= 0.001.

following

-oo

Hm

C+D

"*

)(>.

+ 3)

n
l)

2n+11
2

178*. lim
n

-* CD

J'

notations:

= oo.

Limits

Sec. 3]

Hm (Vn +

179.

n -+
-o/%

\f~n).

<

180. lim

When
x -+

the

seeking
is

it

oo,

useful

of a ratio of two integral polynomials in


n
to divide both terms of the ratio by x , where

limit

first

the highest decree of these polynomials.


A similar procedure is also possible in
ing irrational terms.

Example
lim

2.

*
lim

lim

181.
r

182.

cases for fractions contain-

1.

J2^-3)(3t-f^)(4A'-6)

Example

many

-. or

lim

=.

^rrr.
*

*86.

^^.

187.

~'

lim

lim
*

.,

3*
O ^2

184.

185.

lim

J"

__

+5*

8v

lim

1/

-11-

jc

lim
->*

-\- \

10-j-

A:

Y L

4-

188.

+7

X*

lim
*

lim

1.

^~~^=J.

<x>

00

183.

* as
n is

189.

lirn

-r-r~c

^5

190.

lim

Vx + Vx
If

P(A-)

and Q

(x)

are integral

polynomials and P

(u)

or

(a)

then the limit of the rational fraction


lim

is

obtained directly.
But if P(a)
Q(a)=0, then

out of the fraction

Example

P
Q

(x)

it

is

advisable to camel the binomial *

once or several times.

3.

lim

/'T

lim !*""!!)

xf ??

Hm

^^4.

Introduction to Analysis

26

101.

^{.

lim

*-.

192. lim
* _.

*-+>

The expressions containing irrational


ized by introducing a new variable.
Example

196. lim

\Ch.

Um ^

198.

terms are

fl

in

cases rational-

many

Find

4.

lim

Solution. Putting

we

!+* =

have

E=1

lim

199. lim
X -

Mm

*
-4^-.
*~

200. lim

lim

"

,'~
201.
,,. lim

*/',

3/
t/x

x
.

Another way of finding the limit of an irrational expression is to transfer the irrational term from the numerator to the denominator, or vice versa,
from the denominator to the numerator.

Example

5.

lim

lim

x -+a(X

a)(Vx

_^
+ V a)

lim

*-> a

203.

204.

-49Q-.

lim

li.n

*-*

j-^=

206.

207.

jc

-f

lim

lim
*-+<

205.

lim
*-+'

^L""

208.

lim
^-*o

2\f~i

-=f.
__

Limits

Sec. #]

_
209.

27

212.

lim

213

[/*(*

^i
214.

Hm(]/xfa

+ a)

xj.

-6* 4 6-*).

'

210. lim

211.

lim
X-

li.

Jf--fCO

215.

The formula
llm
X-

-i

r frequently used when solving the following examples.


cos a.
sin a and lim cos
granted that lim sin *

Example

a)

!!!

lim;

lim

227. a)

lim

X-*

-> CO
sill

,.

3x

228.

xsinl;

lim x sin

b)

b)li.n^.
X
217.

lim

00

x) tan

(1

-~-

lim
X -0

sin

5*

sin

2*

229.

'

Jt-M

218.

taken

is

6.
lira

216.

It

*=

lim cot 2x cot f-^


*

* -+0

x).
/

sin JTX

219.

lim
M

sin BJIJC

230.

lim
*-

220.

lim ( n
n-*cc

221.

lim

222.

lim

223.

lim

224.

lim

225.

lim

226.

lim

sin-).
n I

Jt

231.

lim

232.

lim

cosmx

arc sin

crs^
tan*

236.
'

sui

A:

lim
JC

- cosn
V*

tan

233.

ji

1-2

lim

'

sin six

for

"28

_ _
Introduction to Analysis

m.

24

.=T.

ta

!!?.

one should bear

mind

in

C=4";
lim (p(x)

if

= ^
/l

cp

A and

(x)

and

if

(x)

where

\|?

lim ty(x)^=

is

= B,

(x)

oo,

then the problem of finding

we

Hm a

(x)

Hm

(x)

= 2.718

Example

7.

...

is

Napier's number.

Find
lim

Solution. Here,
lim

(5111=2
X

and

lim

Jf-^O \

hence,
lim
x-*o

Example

Solution.

put

q>(x)=

+a(x),

as x -+ a and, lien^e,

-*

lim

solved in straightforward fashion;


lim \|) (x)
then
co,
lini(pU)=l and

the limit of (3)

(3)

that:

lim

where a

-=C

there are final limits

if

3)

'"""

taking limits of the form


lim l<p(Jt)]* U)
X * fl

2)

*""r

Jt

-* 1

When

then

nx

1)

[Ch. 1

8.

Find

We

have

lim
1

r-^(

end

Hm

*2

J^

[(p

(x)

- ij

ty (x)

^Sec

Limits

3]

Therefore,
lim

=0.

Example

Find

9.

lim

Solution.

We

x ~~
\
t

have

^11=

lim
X-+ 06 X

-4- 1

lim
(- CO

i
.

+T
,

Transforming, as indicated above, we have

In this case

easier to find the limit without resorting to the general

is

it

procedure:

it

Generally,

is

useful to

remember

that

lim

250.

244. lim(*
X

"" 2 *"

/^i

245>

].

Hmfl

... V

251. lirn(l

252**. a) lim (cos x)

Iim(l

J_
*
;

X ~*

-)/

b)

247

+ sinjc) *.

-.o

2 \*a

Jill ( 2?+T )
/

246.

3x4-2/

X-K>\

li

f I)*.

30

_ _
Introduction to Analysis

When solving the problems that follow,


limit lim/(x) exists and is positive, then
lim [In /(*)]
x-+a

Solution.

lim
X-*0

Formula

We

that

if

f (x)].

X-+Q

have

X-+Q

frequently used in the solution of problems.

253. lim [In

(2*+!)

*-

254.

li

255.

"
limfjlnl/J-i^).
lX/

260*.

,_*<> \

llmn(^/a
^

V)

256. lim *[ln(jt+l)

Inx].

261. lim-

<-*--- 00

257.

lim.

258*.

Hm=.

259*.

ital!

262.

-^o

(a

lim

b)

lim

>0).
(see

Find

the*

264. a)

sin

263. a)

*_^

Hm
*" +

b)Jirn*

p===.
267

a/lLutanh*;
*-*-*

b)

limtanh*,
*->+

where tanh^ =
266. a)

x*

Problems 103 and

fa

265.

following limits that occur on one side:

lira

lira

a ) lim

*-b) Hm
*-*+

^^~.

(a>0).

--

li

\)

ptX

pCLX

268. a) lim
b) |im

1+ ' T

the

ln

is

(*)

[Hm

know

Prove that

tO.

Example

= In

useful to

is

it

[Ch.

104).

Limits

Sec. 31

269. a)

31

270. a)

lim-^4i;
'

Hm-^-;
x~*

Construct the graphs of the following functions:


2
\im (cos "*).
271**. y

n->oo

272*.

y=lim
n-*c

= \im

273. y

(x^O).

xn

J/V-t-a

2
.

a->o

274.

t/

275.

= li;n|

t/

li

-*<

276. Transform
a

common

the

following

fraction:

Regard
277.

it

What

mixed

periodic

fraction

into

= 0.13555...

as the limit of the corresponding finite fraction.


will happen to the roots of the quadratic equation

if the coefficient a approaches zero while the coefficients b and c


are constant, and fc^=0?
278. Find the limit of the interior angle of a regular n-gon
>
oo.
as n
279. Find the limit of the perimeters of regular n-gons inscribed
oo.
in a circle of radius R and circumscribed about it as n
20. Find the limit of the sum of the lengths of the ordinates

of the

curve

= e~*cos nx,

x 0, 1, 2, ..., n, as n *oo.
281. Find the limit of the sum of the areas of the squares
constructed on the ordinates of the curve

drawn

as

at the points

on bases, where x=^l, 2, 3, ..., n, provided that n


282. Find the limit of the perimeter of a broken line

*oo.

M^.. .Mn

inscribed in a logarithmic spiral

Introduction to Analysis

[Ch.

oo), if the vertices of this broken line have, respectively,


(as n
the polar angles
<P,

== 0,

<PJ

=y

q>rt

=y

A segment AB = a

(Fig. 7) is divided into n equal parts,


each pnrt serving as the base of an isoscelos triangle with base
angles u ^45. Show that the limit of the perimeter of the broken line thus formed dilTers from the

283.

length of AB despite the fact that in


the limit the broken line "geometrically
merges with the segment AB".

Fig

Fig.

in half; the
point C, divides a segment AB---1
the
in
AC
a
half;
divides
point C, divides a
segment
point C 2
C
C 3 in half, and so
C
divides
the
in
half;
2
point
4
segment C,C
on. Determine the limiting position of the point C n when /i--oo.
285. The side a of a right triangle is divided into n equal
which is constructed an inscribed rectangle
parts, on each of
the limit of the area of the step-like figure
Determine
8).
(Fig.
*ou.
thus formed if n
286. Find the constants k and b from the equation

The

284.

0.

(1)

What

is the geometric meaning of (1)?


287*. A certain chemical process proceeds in such fashion
that the increase in quantity of a substance during each interval
of time r out of the infinite sequence of intervals (tr, (i -f l)t)
substance
(/~0, 1, 2, ...) is proportional to the quantity of the
available at the commencement of each interval and to the length
of the interval. Assuming that the quantity of substance at the
n}
after
determine the quantity of substance Q
nth
each
takes
if
the increase
part of
the elapse of time t
place

initial

time

is

the time interval

Find

Q^lhi

*=

Sec. 4]

33

Small and Large Quantities

Infinitely

Sec. 4. Infinitely Small and Large Quantities

1.

Infinitely small quantities (infinitesimals).

If

lim a (x) = 0,
x->a
i.e.,

<

|a(x)|<e when

if

as

infinitesimal
* oo.
as x

<

then the function a (x) is an


a
fi(e),
similar fashion we define the infinitesimal a (x)

In

a.

The sum and product of


x-+a.

limited

number

infinitesimals as x

of

+a are

also infinitesimals as
If

a(x) and p

*a and

(x) are infinitesimals as

lim SlJfUc,
x-+a P (x)

where C

is some number different from zero, then the functions a(x) and p(x)
are called infinitesimals of the same order; but if C
0, then we say that the
function a (x) is an infinitesimal of Higher order than p (x). The function
u (x) is called an infinitesimal of order n compared with the function p (x) if

lim

where

<

C|

<

Q(x)

-C

"

'

-f oo.

If

then the functions a

For example,

for

(x)

and p

are called equivalent functions as x

*a:

we have

>

sinx~x;
and so

(A*)

tanx~ x;

ln(l-fx)~ x

forth.

The sum

two

infinitesimals of different orders is equivalent to the


lower.
The limit of a ratio of two infinitesimals remains unchanged if the terms
of the ratio are replaced by equivalent quantities. By virtue of this theorem,
when taking the limit of a fraction
of

term whose order

is

lim

!>

aPW
,

as x
*a we can subtract from (or add to)
where a (x)
>.0 and p (x)
the numerator or denominator infinitesimals of higher orders chosen so that
the resultant quantities should be equivalent to the original quantities.
>

Example

1.

j/?T2? = lim a/7

'

,.

lim

i-

,.

*-*o

2.
ber

Af

2x

Infinitely large quantities (infinites). If for an arbitrarily large numa


x
exists a 6(N) such that when
6(N) we have the

<

there

inequality

lfMI>tf.
then the function f(x)

2-1900

is

called an infinite as x

>a.

<

_ _
Introduction to Analysis

34

> co

The definition of an infinite f (x) as x


of infinitesimals, we introduce the concept

is

[Ch.

analogous. As in the case

of infinites of different orders.

288. Prove that the function

is

an infinitesimal as x

*oo. For what

values

of

is

the ine-

quality

l/WI<e
fulfilled

if

e is

an arbitrary number?

Calculate for: a) e
0.1; b) e-0.01;
the function
289. Prove that

is

an infinitesimal for x

>1.

c)

e-0.001.

For what values

of

is

the ine-

quality

!/(*)!<
fulfilled

if

e is

an arbitrary positive number? Calculate numeri-

0.01; c) e
a) e-0.1; b) e
290. Prove that the function

cally for:

= 0.001.

x
is

an infinite

for

*2. In

what neighbourhoods

2|<8

of |x

is

the inequality

lf(x)\>N
fulfilled

if

is

an arbitrary positive number?


Find 5 if
a) #=10;

J2^

c)

b)

#=100;

#=1000.

291. Determine the order of smallness


the surface of a sphere, b) the volume
of a sphere if the radius of the sphere r
is an
infinitesimal of order one. What
will the orders be of the radius of the
sphere and the volume of the sphere with
respect to its surface?
292. Let the central angle a of a circular sector ABO (Fig. 9) with radius R
tend to zero. Determine the orders of
the infinitesimals relative to the infinitesimal a: a) of the
chord AB\ b) of the line CD; c) of the area of A/4BD.
o

of: a)

Infinitely Small

Sec. 4]

*0 determine the orders

293. For

and Large Quantities


of

35

smallness relative to

of the functions:
*\
*)

^*

cos

d)

+x

tan

e)

*'*

sin

A:

A:.

b)
c)

$/*'-

294. Prove that the length of an infinitesimal arc of a circle


constant radius is equivalent to the length of its chord.
295. Can we say that an infinitesimally small segment and
an infinitesimally small semicircle constructed on this segment
as a diameter are equivalent?
of

Using the theorem


si "

296. lim

!"

arc sin

^o

infinitesimals, find

lim^

298.

=
_^-*

297. lim

two

of the ratio of

5*

3*' s

299. lim

ln(l--*)

300. Prove that

when x

*0

the quantities

VT+T1 +
Applying formula
a)

1/L06;

b)

+xl
\x\ is

(1)

approximate the following:

(1),

1/0^7;

Y\

when

and

are equivalent. Using this result, demonstrate that


small we have the approximate equality

c)

/lO;

d)

/T20

and compare the values obtained with tabular data.


we have the following approxi301. Prove that when x
mate equalities accurate to terms of order x 2
:

b)
n
+x) &\

c)

(1

d)

log(l+x)

where

+ nx

(n

a positive integer);

is

= Afx,
Af = log e = 0.43429...

Using these formulas, approximate:


*>

02

Compare

2>

0^7

3>

I<55

4)

^16;

5)

1.04'; 6) 0.93*; 7) log 1.1.

the values obtained with tabular data.

_ _
Introduction to Analysis

36

302.

Show

X+OQ the
~
P (x) = a.x n + a,x n +
that for

[Ch.

rational integral function

-f

an

is an infinitely large quantity equivalent to the term of highest


n
degree a x
303. Let x*oo. Taking x to bean infinite of the first order,
determine the order of growth of the functions:
.

a)

b)

*>-

100* -1,000;

c)

7+2-

Sec. 5. Continuity of Functions

1. Definition of continuity.

function

continuous

is

/ (x)

when x =

(or "at the point g"), if: 1) this function is defined at the point g, that is,
there exists a number / (g); 2) there exists a finite limit lim f (x); 3) this lim-

x-4
is

it

equal to the value of the function at the point

llmf

(*)

g,

i.e.,

= /().

(1)

*-*fc

Putting

where Ag

^0, condition
lim

(1)

may be

A/(g) =

lim

rewritten as

l/(g+ Ag)-f

(g)]

= 0.

(2)

or the function / (x) is continuous at the point g if (and only if) at this point
to an infinitesimal increment in the argument there corresponds an infinitesimal increment in the function.
If a function is continuous at every point of some region (interval, etc.),
then it is said to be continuous in this region.
Example 1. Prove that the function

y
fs

continuous
Solution.

Ay = sin

<*

for every

We

= sin x

value of the argument

x.

have

+ A*)-sin x = 2 sin

cos

x+

sin

cos f

Since
si

lim

T=

and

2
it

follows that for any x

we have
lim A(/

Hence, the function sin*

is

continuous when

oo<x< +

x+

Sec. 5]

Continuity of Functions

37

2. Points of discontinuity of a function. We say that a function


/(x)has
discontinuity 'at
(or at the point XQ) within the domain of definition
of the function or on the boundary of this domain if there is a break in
the
continuity of the function at this point.

x=*

Example
when x=l.

2.

The

This

function

function

f(x)=

is

10 a)

(Fig.

not defined at the point x

is
1,

discontinuous

and no matter

1-2

how we

choose the number /(I), the redefined


tinuous for *=1.
If the function f (x) has finite limits:

Hm
and not

all

three

/(*)

= f(*

numbers f(x

-0)
),

a discontinuity of the first kind.

then *

is

Urn

/(*)

+ Q)

)
/(*
f (x
In particular,

/ (x)

will not be con-

= /(
are equal, the nxQ

is

called

if

called a removable discontinuity.


of a function f(x) at a point

For continuity
ficient that

and

function

JC

Q,

it

is

necessary and suf-

= 0.

The function

3.

Example
at

[Ch. I

In t reduction to Analysis

38

/(jc)=j-y

has a discontinuity of the

kind

first

Indeed, here,
/

+ 0)=

5!!L

lim

==+ i

and

/(_0)=

lim

jc-*-o

Example 4. The iunction y = E(x), where E(x) denotes the integral


number x [i.e., E (x) is an integer that satisfies the equality x = E(x)

of the

where
1,

0<<7<1],
i2,

and

...,

discontinuous

(Fig.
106) at every integral
all the discontinuities are of the first kind.

is

point: x

part

+ q.
= 0,

+ =

Indeed, if n is an integer, then


/i.
At all
(/i
0)-=/il and
(/i
0)
other points this function is, obviously, continuous.
Discontinuities of a function that are not of the first kind are called
discontinuities of the second kind.
Infinite discontinuities also belong to discontinuities of the second kind.
These ane points * such that at least one of the one-sided limits, /(*<>
0) or
/(*o
0) is equal to oo (see Example 2).

Example

The function # = cos

5.

lim cos 5L
X
X-+-0

the

lOc) at

(Fig.

discontinuity of the second kind, since


here:

both one-sided

and

lim

cosi

Jt->

point

x=0

has a

limits are nonexistent

3. Properties of continuous functions. When testing functions for continuity, bear in mind the following theorems:
of a limited number of functions continuous in
1) the sum and product
some region is a function that is continuous in this region;
2) the quotient of two functions continuous in some region is a continuous
function for all values of the argument of this region that do not make the
divisor zero;
3) if a function

f (x) is continuous in an interval (a, b), and a set of its


values is contained in the interval (A, B), and a function cp (x) is continuous
in (A B), then the composite function cp[/(*)J is continuous in (a, b).
A function f (x) continuous in an interval [a, b] has the following propert

ties:
is

1) f (x)

boanded on

|/(*)|<M when
2) / (x) has a
3) / (x) takes

a<*<6;

[a,

minimum and

6J,

there

i.e.,

maximum

some number

is

value on

[a,

such that

b]\

between the two given values;


matter what the
if
and
fl (a<a<p<6), then no
ithat
is,
number C between A and B, there will be at least one value JC Y (<Y<P)
such that f(y)=*C.
In particular, if f(a)/(p)<0, then the equation
on

f(a) = A

has

at least

304.
of the

one

Show

all

intermediate

=
/(P)

values

real root in the interval

that the function y

argument

x.

(a,

=x

p).

is

continuous for any value

_ _
Continuity of Functions

Sec. 5]

39

305. Prove that the rational integral tunction

is

continuous for any value of x.


306. Prove that the rational fractional function

is continuous for all values of x except those that make the denominator zero.
307*. Prove that the function y = Yx is continuous for x&zQ.
308. Prove that if the function f (x) is continuous and non-

negative in the interval

is

then the function

(a, 6),

likewise continuous in this interval.


309*. Prove that the function y
cos x is continuous for any x.
310. For what values of x are the functions a) tan* and

b) cotjc

continuous?

Show

that the function #


|#| is continuous.
graph of this function.
312. Prove that the absolute value of a continuous
is a continuous function.
313. A function is defined by the formulas

311*.

How

Plot the

function

should one choose the value of the function A=f(2) so


2?
the thus redefined function f(x) is continuous for #
Plot the graph of the function y
f(x).
314. The right side of the equation

that

f(x)
is

meaningless for x =

0.

= lx sin

How

so that f(x) is continuous for


315. The function
f(*)
is

in
for

should one choose the value /(O)


jc

= 0?

= arctan--^

meaningless for x=--2. Is it possible to define the value of /(2)


such a way that the redefined function should be continuous
jc

= 2?

Introduction to Analysis

40

316. The
so that fix)

function f(x)

f(x)

= ^ y\__1
y
x

b)

/(*)

c)

/(*)

= ln(\+x)

d)

/(

f)

/(*)

/]

a)

is

not

continuous for x =

is

cos*

[Ch.

defined
0,

for

x=

0.

Define /(O)

if:

(n is a positive

integer);

A*
111(1

= * cot*.

Investigate the following functions for continuity:


317.

y=

318.

y=

3,9.

yssi

320.

^=.
b)

322.

*/

-.

324.

-.
=

y=

xs\n-.

</

325. y

= arc tan

326.

= (!+

329.

</

A:)

arc tan
,

= -/

323.

y=\n(cosx).

330.

y=J

**
\

2#+l

for

'"T
x>3.

P'ot the graph of this function.

331. Prove that the Dirichlet function %(x) which is zero for
and unity for rational x, is discontinuous for every
t

irrational x
value of x.

Investigate the following functions for continuity and construct


their graphs:

= \in\
y = lim (x arc tan nx).

332. y
333.

Continuity of Functions

Sec. 5]

334. a) y

= sgnx,

function sgn x

is

b)

=x

sgnx,

c)

i/

= sgn(sinjt),

where the

defined by the formulas:

sgn x

1,

if

*>0,

0,

if

x=

-1,

if

*<0.

0,

335. a) y
xE(x), b) y xE(x), where E (x) is the integral
part of the number x.
336. Give an example to show that the sum of two discontinuous functions may be a continuous function.
337*. Let a be a regular positive fraction tending to zero
(0<a<l). Can we put the limit of a into the equality

=
which

is

338.

(l

a)

l,

true for all values of a?


that the equation

Show

has a real root in the interval


339. Prove

that

any

(1,2).

polynomial

one real root.


340. Prove that the equation

least

has an infinite number of real roots.

Approximate
P (x) of odd

this root.

power has

at

Chapter II

DIFFERENTIATION OF FUNCTIONS

Sec. 1. Calculating Derivatives Directly

1. Increment of the argument and increment of the function. If x and x l


are values of the argument x, and y
f(x) and t/ 1
/(jc 1 ) are corresponding
values of the function y
f(x), then

^x~x
is

called the increment of the argument x in the interval

A0=0i

(x,

xj, and

or
/

= f (x,) -f (x) = f (x + A*) -

<n

Fig.
is

11

called the increment of the function y in the


and by AN). The ratio

where

&x=MA

MN

same

interval

(jc,

*,) (Fig.

11,

the slope of the secant


of the
graph of the function y=*f(x) (Fig. 11)
is called the mean
rate of change of the function y over the interval
(x, *-f Ax).
Example t. For the function

5s

and

Calculating Derivatives Directly

Sec. 1]

calculate

fromx=l

b)

from

a)

Ax=l. 1

x=3
Solution. We

to x=l.l;
to x
2.

have

1=0.1,

= (l.l 5-1.1 + 6)
Ax = 2 3 =
At/ = (2*
5-2-1-6)
(3*
2

Ai/

b)

to a change in the argument:

Ax and A#, corresponding

a)

1+6) =

5-

(I

2.

5-3 -f- 6)--=0.

In the case of the hyperbola y

secant passing through the points

3,

--

Solution.
AJ/

Ax~~

2. The

Ax=10

Here,

and

the

find

10,

-r^

the

of

'

=7

slope

Ay = ^
1U
1

and

4=
o

5*5U

Hence,

1
30'

The derivative

derivative.

spect to the argument x


that

0.29;

1,

Example

43

is

y'=j-

of a function

the limit of the ratio -r^

y-=f(x) with

when Ax approaches

re-

zero;

is.

y>=

lim
AJC -> o

The magnitude

>.

A*

of the derivative yields the slope of the


f(x) at the point x (Fig. 11):

graph of the function y

tan

y'

tangent

MT

to

the

q>.

usually called differentiation of the function. The


derivative y'=f' (x) is the rate of change of the function at the point x.
Example 3. Find the derivative of the function

Finding the derivative

/'

is

y
Solution.

From formula

= x*.

we have

(1)

Ay = (*+

xi

A*)*

2*Ax+ (Ax) 1

and

Hence,

'=

L^

lim

Ax
5*.

lim
AJC->O

One-sided derivatives. The expressions

/'_(*)= lim
AJ:-*--O

(*+**)-/(*)
Ax

and

/(x)=

lim

Ax

'

44

[Ch. 2

Differentiation of Functions

are called, respectively, the left-hand or right-hand derivative of the function


f(x) at the point x. For /' (x) to exist, it is necessary and sufficient that
/'.(*)

Example 4 Find

By the

Solution.

and

/'_ (0)

f^

4.

Infinite derivative.

(0)

the function

we have

lim

lim
A*--t-o

Ax

some point we have

at

If

(0) of

/'

definition
/'_ (0)

= /+(*).

/(*+**)-/(*)_.
Uoo,

lim

we say that the continuous function / (x) has an infinite derivative at x.


In this case, the tangent to the graph of the function y
f(x) is perpendicular to the x-axis.
Example 5. Find /' (0) of the function

then

Solution.

We

V=V*

have

/'0)=llm *~
Ax

]im

-=-=<-

341. Find the increment of the function


to a

change

a)

b)
c)

in

from x=
from x =
from A: =

argument:
1

to

to

to

xt
xl
A:,

= 2;
=
= + h.
1

342. Find A// of the function


a)

b)
c)

*=

0,

^=8,

^=

a,

2
y = x that corresponds

y=-i/xil:

AA:- 0.001;

^ ==

AA:

9;
/Z.

343. Why can we, for the function y = 2;c


3, determine the
increment Ay if all we know is the corresponding increment
Ax = 5, while for the function y x 2 this cannot be done?

344. Find

the

increment

by and

the ratio

tions:

a)

=
( Jg

f 01

"*^

2)

b)

y=l/"x

forx-0

c)

for

= \ogx

x -100,000

andAjc =

^
A
*

for the func-

0.4;

and AJC = 0.0001;


and A*90,000.

Sec.

Calculating Derivatives Directly

1]

which correspond
Ay and
ment fromx to x-(- Ax for the functions:
345. Find

a)

y-ax +

b)

y-x';

d) y

6;

e)

45

to

change in argu-

= /x;
= 2*\

346. Find the slope of the secant to the parabola

==

~x

the abscissas of the points of intersection are equal:

if

a)

x,-l, x a -2;

c)

x^l,' x 2 =l+fc.

To what
if

limit does the slope of the secant tend in the latter case

/i->0?

What

347.
in the

is

mean

the

l^x^4?

interval

rate of change of the

function y

= x*

2/
348. The law of motion of a point is s
3/
5, where
the distance s is given in centimetres and the time t is in seconds.
What is the average velocity of the point over the interval of
time from t~\ to ^
5?
2* in the interval
349. Find the mean rise of the curve y

mean

350. Find the


[x,

x+Ax].
351. What

rise of the

curve

j/

= /(x)

in the interval

f(x)
by the rise of the curve y
given point x?
352. Define: a) the mean rate of rotation; b) the instantaneous
rate of rotation.
353. A hot body placed in a medium of lower temperature
cools off. What is to be understood by: a) the mean rate of
cooling; b) the rate of cooling at a given instant?
354. What is to be understood by the rate of reaction of a substance in a chemical reaction?
355. Let
/(X) be the mass of a non- homogeneous rod over
the interval [0, x]. What is to be understood by: a) the mean
linear density of the rod on the interval [x, x+Ax]; b) the linear
density of the rod at a point x?
is

to be understood

at a

M=

356.

x=

2,

if:

ative y'

Find

the

a)

Ax-1;

ratio

when x^2?

b)

of

the

Ax = 0.1;

c)

function

*/

Ax -0.01. What

at the point
is

the deriv-

_ _
Differentiation of Functions

46

357**. Find the derivative of the function y


358. Find {/'=

a)

=x

t/

c)

lirn

[C/t.

= ianx.

of the functions:

359. Calculate f'(8), if


1
360. Find /'(0), /'(I), /'(2), if /(*)
*(*- 1) (x-2)V
361. At what points does the derivative of the function
#* coincide numerically with the value of the function itself,
/(#)
that is, /(*)
/'(*)?
362. The law of motion of a point is s
5/*, where the distance s is in metres and the time t is in seconds. Find the speed

at

= 3.

Find

363.

the

of

slope

the

tangent

drawn

at a point with abscissa x


2.
364. Find the slope of the tangent

the point

the

to

to the

curve y = Q.lx*

curve y=sinjt at

0).

(ji,

f (*)
\
/

= -i

are the slopes of the tangents to the curves y

=~

365. Find the value of the derivative of the function


at the point

366*.

x = X Q (x

What

of their intersection?

Find the

367**. Show that the following functions


derivatives at the indicated points:

y=^?_

b) y
c)

=l/xl

at

x
x

= |cosx|

at

*=

Sec. 2. Tabular

1. Basic
v

ty(x) are

at

5)

(*)'=,;

6)

3)

)'-'
r

4)

fc

= 0,

1,

(cu)'=cu

have

finite

2,

rules for finding a derivative. If c is a constant


functions that have derivatives, then

2)

(c)'

jt,

do not

angle be-

Differentiation

= 0;

1)

+ 0).

and y = x* at the point


tween these tangents.

a)

t;';

7)-

==

(v

* 0).

and

o>(jc) '

Sec. 2]

__

2. Table
I.

of derivatives of basic functions


n
n- 1

(x )'

III.

= nx

(sinx)'

IV. (cosx)'

V1T

VIII.

47

= cos*.
= sin*.

(arrdn*)':=

ZL.

(arccos*)'=

IX. (arc

Tabular Differentiation

<

1).

<|*|<1).

i*.

jo'^y-pj.

X.
XI.
XII.

XIII.

XIV.

XV.
XVI.
XVII.
XVIII. (cothx)'

XIX.

"

cosh 1 x

(arcsinhjt)'

."""

= -^J==r.
V\+x*
1

XXI.

(arc

tanh

x)'

(|jc|<l).

XXII.

3. Rule
that

is,

/==/

composite function. If y f(u) and u


where the functions y and u have derivatives, then

for differentiating a
[<p (A-)],

y
or in other notations

^/
dx~~ du dx'
This rule extends to a series of any finite

number

of differentiate functions.

48

_ _
Differentiation of Functions

Example

Find the derivative

1.

=a

Solution. Putting #

5
,

[Ch. 2

of the function

where w =

2jc

(*

+ 3),

by

formula

we

(1)

will

have

= (u*)'u (* -2x + 3); = 5u


2

y'

Example

Find the derivative

2.

(/

we

*;

2
(jt

of the function

y = sin
Solution. Putting

= 10 (x-1)

(2x-2)

4*.

= sinu;

4jc,

find
>-4

Find the derivatives


a

differentiating
368-408).

= 12sin

4xcos4jt.

of the

composite

following functions (the rule for


function is not used in problems

A. Algebraic Functions
s

QO

AV

,.
ouo. y

Q5Q
oOy.

<>

i/

A V8
~~ TEA

==:

373.

v^

-f-

&A:

Q7 i/.
Of

*j.
4
A
U.OAV

if

+6

y==+n n

Q7IS4T
o/O

QJA
v3

11
t^

= Xv*

Ov
~~~ ^A.

~f~

Av~'

/" +f*
y X

+ C.

(,-a(-H-W".
V^a

374.

~]~

Ov
AA

;r-Jt-pJT

'":

370. y--=ax*

372

2.

379. il-

380.

2jc

JC

=i

381.

B. Inverse Circular and Trigonometric Functions


382.

383.

(/

t/

= 5 sin ^ + 3 cos x.
= tanx cotx.

386.
387.

388

S85.j/-2/sin(-(''-2)cos(.

389.

y=arctan^-h
f/

arc cot x.

=
-

_('+"')' '"'-'.
^

Sec. 2]

Tabular Differentiation

49

C. Exponential and Logarithmic Functions

390.

= e*arc sin x.

396. y

y^K*-e*.

391. y

= (x-l)e*.

397.

y^ ~.

392.

= 5-

398

r/

393. (/==

399.

J.

394. / (x)
e* cos
2
395. #=:(A;
2

400.

jc.

*
.

\j

=7
y = \nx\ogx

*/

In

a log a

jc.

D. Hyperbolic and Inverse Hyperbolic Functions


401.

t/

= Jtsinhjt.

y=-Vcosh x

402. y
403.

404

//

t/
^

405.
406.

= tanhA:

407.

*.

= ^iiL
Inx

= arctanx
= arc
J
(/

arctanh

t/

(/

408. ^
//

==

x-

E. Composite Functions
In problems 409 to 466, use the rule for differentiating a composite function with one intermediate argument.
Find the derivatives of the following functions:

40Q**
"v
u

H
i
\

Solution.

2
~~~- ^r \*
-i_^v
*jAi
i~ \jAt

Denote

= w;
=
^ 30w M

3jt

5jc*

i^

- 30a 29 -(3

L. \

\0x)

then

t/

=w

=3
u'^

j<)
.

We

10*;

=30 (1 + 3jc

410. i/=|
411.
412.

410
,._
10> f/

>

"~56(2*

1)'

414. t/=J/T^J?".
415.

y=^/

416. w=(a''.

24(2^1)'

'

40(2x

I)''

have:

Differentiation of Functions

50

417.

t/

= (3

2 sin*)

Solution.

2 sin x) 4

4
Jt)

2 sin x)'

-(3

419. y
r/=J/coU

/coU.

423. y
j/

427.

428

426.

429.

430.
431.

= |/ 3sin*--2cos*
y=

/.

t/

y=/2ex
+ tart
y = sin 3* + cos
-|-

Solution

^
^

sin
Sm

f^

= cos 3^3*)' -sin 4 f 4V +


5

5 /

cos 2

x
Y/-

x
5 "^2

432. t/=sin(x

433. /(x)
cos(ct;
434. /(0=sin/si
,._

"

l+cos2*

= acot~
= ~
y = arc sin 2x.

436. /(x)
437.

438.

t/

Solution, y'

439. y

= arcsin^.

440. /(x)

2 cos x)

-- CGSJC
= 30-^-3
cos *

424. y

= cosec ^+sec
425.
f
f(x) =
6(1 _ 3cosx)
{/= 1/1 + arc sin x.
(arc sin x)
y = J/arc tan *

422.

2 sin x)*

= 2x + 5 cos' *.

421*. x

= 5 (3

418. j/=tanjc

[Ch. 2

= 5 (32 sin
y'

- 10 cos x (3

420. y

= arccosJ/7.

441. y

= arc tan.

442. y

Tabular Differentiation

Sec. 2]

443.

AAA
444.

= 5e~*.
447. y = arc cose*.
448 0=1
= X
= logsinjc.
5
449.
= x 10'*.
450. y= ln(l
*').
=
=
\n* *
451.
ts'm2
y
In(lnjc).
f(t)
5
4
arc
x
sin
sinx).
y== \n(e* +
= arctan (lnA:) + ln(arctan^).
y = /In x+l + In (1/7+1).
t/

>

t/

j/

445.

446.
452.

453.
454.

j/

t/

F. Miscellaneous Functions

455**. y=sin'5jccos*y.

15

10
3)'

458.

j/=

460.

az

461. y
462.

463.

f/

^-i-jc 2
x*

=
:

= |-

y=44

465.

t/

=x

(a

__J
"2
|/

470. z

468.

469.

471.

/(0=(2/-

(Jt-i-2)

1 '

51

52

Differentiation of Functions

[C/t.

--

= ln(]/l+e*-l)-ln(/l
# = ^ cos'x (3 cos * 5).

473. y

474.
...

475

= (tan

l)(tan

x-HOtan 2 *-fl)

476. y=-ian*5x.

= ^ sin (x

477. y

478. j/=sin
479.

493.
494.

A;+sin'x.

ianx + x.

tan *

486. y

= arc sin

487. y

488. y

= 4~- af c sin fx

491.

jc

*
.

cos *

^
V

= K^

490. t/=jt/a

y=/a sin + p cos x.


y = arc sinjc + arccosA;
2

= arc sin

489. y

^f +cotx.

^-T +a
8

y=arcsin(l

= -^ (arc sin*)

arc cos jt.

=jc-I
y = ln(arcsin5x).
y = arc sin (Inx).
=

495.

496.

</

497.

t/

498.

t/

499.
500.

501.

502.

/
i/

arc tan

= 36
=
=
=

F(
F

5tan-i
-

arc tan

1/2 arc cot

(36

=x.

I/
*

-)
a /

+ a arc sin

x*

484. y

492.

(O-

-o-

481. y

483.

).

= 3sinA:cos

*/

480. w

482.

485. #

+ 2*) Vbxx*.

arc

.
CL

sin-.

Tabular Differentiation

Sec. 2]

504.

y = ~e- (3sm3x
x

cos 3*).

507

= x"a-*".
508. y=
K
r
=
= I/cos *a
509.
506.
510. y = x
2 1/7+2 In (!+/*)
= ln(a + *+/2ax + *'). 514*.
511.
5I2 ^ =
!^'
515. y
y
^_
=
513. y
_,_

505. y

<

*.

j/

516.

517. y
518.

y=lnln(3

=5
^=

519. y
520.

522.

t/

x-sin(lnx

^-)
1

524. /(*)

525. y

= \l

526.

=2

i/

arc sln
sin

527.

528.

+ (!

ux

u^?^^-

;/

COS X

= -Uln

arc cos 3x)

'

'

sm 8

1
.

m
'

3 cos' bx

tan^+2-:
=
n

529. y

530.
531.

= arc

tan In x.

y^lnarc sinx +
f/

= arctanln

In

jc-|

arc sin In x.

z
x

Differentiation of Functions

54

633. t/=

[C/t.

1Vsin*

1 arc tan x.
= 1 In Ji- + 1 In
J^| +
1 In (1 + *)- In (*'-*+ 1) +
/(*) =

534. y
535.

536. f(*)

= s'mh*2x.
= e * cosh px.
y = ianh*2x.
y = Insinh2jc.

545.

= arcsinh~.

546.

537. y
538. y
539.

540.
541.

t/

542. y=r. arc cosh In*.


arc tanh (tan x).

=
# = arc cot h (sec*).
y = arc tanh y^

543.
544.

t/

= -2-(

A:a

-^+--

547. r/=

548. Find y',


a)

arc tan

if:

y=\x\\

b)

= *|*|.

Construct the graphs of the functions y and


549. Find #'
550. Find

(x)

if

551. Calculate

/'

(0)

f
/'

x<0,

for

if

/(*)
Solution,

y'.

if

= e~ x (3 sin 3x)
= e (3 sin 0) e
(0)

-* cos
3*.
e~* cos

(x)

cos

3jc;
1

552.
553.

f(x)-ln(l+x)
1

y-tan ^.

554. Find

b) / (x)

+ (0)

/'

= arc sin
Y

l+e

+ arcsin.

Find

2 ;

/'(I).

(-gj^.

and /1(0)

^^

Find

of the functions:

e)

f(x)

= x sin

-^

Tabular Differentiation

Sec. 2]

55

555. Find / (0) -f xf (0) of the function f(x)=e-*.


556. Find /(3)
(x
3)/' (3) of the function f(x)
557. Given the functions f(x)
tar\x and (p(*)

= Y\ + x.
= ln(l x)>

n0)

find
tind

q/(or

Given the functions /(x)=l

558.

2^

find
nna

ff

and

cp(jc)

sin^

(1)

559. Prove that the derivative of an even function is an odd


function, and the derivative of an odd function is an even function.

560. Prove that the

derivative of a periodic

a periodic function.

561.

Show

that

562.

Show

that the

563.

Show

that the function

the

function

xy' = d-x)y-

xy'

function

tion

xy' = y(y\i\x

is

also

y = xe~*

satisfies

the

equation

= xe~?

satisfies

the

e <l uati

= (\-x*)y.

function

y=

satisfies
lrlx

the equa-

1).

G. Logarithmic Derivative

A logarithmic derivative of a function


logarithm of this function; that is,

is

= f(x)

is

the

derivative

sometimes simplified by

first

of the

exponential function

where u = y(x) and v


ty(x).
Solution. Taking logarithms we get
In

v In u.

Differentiate both sides of this equation with respect to x:


(In y)'

= v' In u + v (In a)',

or
1

whence

~~

the

taking logs of the func-

tion.

Example. Find the derivative

of

fM

~y
Finding the derivative

_ _

56
or

[Ch. 2

Differentiation of Functions

564. Find y'

Solution.

In

y'

if

2
= ^In x + In (1

x)

21,
--ui(1)
= _-

_-2x i-4-32

whence

y'

=y

565. Find

y\

Solution.

In

/'

if

x In sinx;

2x

-/'

cosx

sin x

1-j-x

-^

y = (smx)

(1

+ x + 3 In sin x +2 In cos x;
2

In

-~pj

-2smx
cosx

+ 3cotx--2 tan xj

x
.

= In sin x + x cot x;

= (sin x) x (In sin x + x cot x).

,Ln the following problems find y' after


function y
f(x):

first

taking logs of the

Sec. 3. The Derivatives of Functions Not Represented Explicitly


1. The derivative of an
derivative

y'x

^ 0,

then

the

inverse

function.

I!

derivative of the inverse

*~7

function

function

y=f(x) has

x=/-

(t/)

is

The Derivatives

Sec. 3]

Functions Not Represented Explicitly

of

57

or

_
dy'

dy

Tx
Example

1.

Solution.

\s

Find the derivative x

We

if

=1+1=^1
x
x

have y x

hence,

*>

-7-.
x-\- 1

2. The derivatives of functions represented parametrically.


related to an argument x by means of a parameter t

If

a function

*-

then

*t
or, in other notation,
*JL

t^dx'
dt

Example

2.

Find

^,
dx

if

a cos

y = a sin
Solution.

We

find

t,
/

a sin/ and -r-

d\

dt

_
dx

3. The derivative
and y

is

of an

= acosf.

implicit

sin

Whence

>

function.

If

the

relationship between

given in implicit form,

F(x,y) = Q,

(I)

then to find the derivative

y'x
y' in the simplest cases it is sufficient: 1) to
calculate the derivative, with respect to x, of the left side of equation (1),
taking y as a function of x\ 2) to equate this derivative to zero, that is, to put

~F(A:,f/)

and

= 0,

3) to solve the resulting equation for

Example

3.

Find the derivative

yx

(2)

/'.

if

0.

Solution.
ito

zero,

we

Forming the derivative

get
3*'

of the left

(3)

side

+ 3y V -3a (y + xy') = 0,

of (3)

and equating

it

58

Differentiation of Functions

[Ch. 2

whence
2

581. Find the derivative

,_* ay
~~axy*'
xy

a)

f/

c)

if

= 0.
.dy

In the following problems, find the derivative

#'=^

the

functions y represented parametrically:


582.

589.

x = acos*f,

= b sin*
x = acos*
y=b sin

\ y

590.

583.

t.

t,

cos

1.

T^nr
591.

584.

sin

?=

V coslr

585.

= arc cos

arc sin

592.

__

586.

'

y~=e:

= a( In tan + cos
= a(sin + cosO.
-2-

587.

588.
/

595. Calculate

cos/).

when

///i

/t cn
a
sin

= 4= a(t
= a(l

Solution.

*~ ^

593.

sin/

-r-~

a(l

cosO

cos/

if

sin

/),

cos/).

sin

^)

>

The Derivatives

Sec. 3]

Functions Not Represented Explicitly

of

59

and
S1

fdy\

^
**

when

"T
I

596.

Find

if<

= tlnt,
in/
y=^
x = e cosf,
= ^ sm^./
\
x

'

~.

597.

dv
dx

Find

when

= 44
ji

-r

if

<

<

f/

698.

Prove that a function # represented parametrically by the

equations

satisfies

599.

the equation

When x = 2

the following equation


jc

Does

when

it

x = 2?

= Va*

x*.

(x*)'

Is

= :r

5//+10 = 0.

2x

602.

5 + ? =1
s

604.

x'-

605.

l/^ +

606.

l/S + /~* =

607.

/'=
y

--=a

possible to perform term-by-term

+ y*^0'?
it

required to find the deriva-

is

609.

a cos

6I0

'

tan//

611.

xy-

613.

^=

603.

-t-y

= (2x)

implicit functions y.

of the

601.

608.

it

examples that follow

In the
y'

true:

follow from this that

600. Let y
differentiation of

tive

is

= 2x.

K^ = /"a.

0.3 sin y

'/a*.

= *.

614.

616.

arctan

^-

= -^

60

Differentiation of Functions

+ y = care tan
2

617.

1/x

619.

Find

Solution.

#=1, we

at the point

y'

Differentiating,

obtain

2*/'

= l+3f/',

620. Find the


indicated points:
a)

b)

for
for

c)

we get
whence

for

if

==y* + 3xy*y'.

Putting

2y'
0'

derivatives y' of

y)

x+l
ye = e
# =
y

A! (1,1),

x*

618.

(Ch.

and

1.

functions y at the

specified

x=

*=
x=

and
and

y=l;

and

r/=l.

*/=!;

Sec. 4. Geometrical and Mechanical Applications of the Derivative

1. Equations of the tangent and the normal. From the geometric significance of a derivative it follows that the equation of the tangent to a curve
will be
t/ )
(*
y = f(x) or F(x,y)=Q at a point

where

value of the derivative y' at the point


(X Q y Q ). The straight
through the point of tangency perpendicularly to the tangent is
called the normal to the curve. For the
normal we have the equation

y'Q is the

line passing

Y\
2. The angle between
angle between the curves

curves.

The

d
and

at their

10
{Z

the

is

common

A and

point

point
Using a familiar formula of analytic geometry,

to

these

y Q ) (Fig. 12)
tangents
curves at the

(*

between

co

angle

the

we

get

3. Segments associated with the tangent and the normal in a rectangular


coordinate system. The tangent and the normal determine the following four

Sec

Geometrical and Mechanical Applications of the Deriiative

4]

segments (Fig.

13):
t

= TM

is

NM

the so-called

is

S t = TK

is

S n = KN

is

segment

and tan

y = y'Q

Sn N

/f

Fig.

KM = \y

of the tangent,

the subtangent,
the segment of the normal,
the subnormal.

St

Since

61

13

it

follows that

j/o

4. Segments

associated with the tangent

tern of coordinates. If a
en in polar coordinates

tion

= /(q>),

then

formed by the tangent

OM

the

MT

u.

angle

and the

radius vector r
(Fig. 14),
defined by the following formula:

The tangent

and the normal

curve is givby the equa-

is

MN

MT

and the normal


together with the radithe point of tangency
and with the perpendicular to the
radius vector drawn through the pole
determine the following four segat the point
us vector of

ments

(see Fig.

\Af

Fig.

14

14):
t

= MT

n=

MN

S t = OT
S n = ON

is
is
is

is

the segment of the polar tangent,


the segment of the polar normal,
the polar subtangent,

the polar subnormal.

in a polar sys-

[Ch. 2

Differentiation of Functions

62

These segments are expressed by the following formulas:

to

621. What angles cp are formed with the x-axis by the tangents
2
x x at points with abscissas:
the curve y

x = 0;

a)

x=l/2;

b)

x=l?

c)

2x. Whence
Solution. We have y'^\
45; b) tan 9 0, q>=0;
l, <p
a) tan cp
135
1,
(Fig. 15).
c) tan q>
q>

=
=

622. At

the

sect

what angles do the sine


and y= s'm2x inter-

axis

of

abscissas

at

the

origin?
623. At what angle does the tanianx intersect the
gent curve y

Fig. 15

axis of abscissas at the origin?


tx
624. At what angle does the curve y
e*'
intersect the
line
x
2?
straight
625. Find the points at which the tangents to the curve
4
20 are parallel to the jc-axis.
12x*
y =z 3* -f. 4x*
626. At what point is the tangent to the parabola

+
=

3
0?
5x y
627. Find the equation of the parabola y~x*-}-bx-\-c that is
tangent to the straight line x
y at the point (1,1).
628. Determine the slope of the tangent to the curve x*+y*
Q at the point (1,2).
2
629. At what point of the curve y
2x* is the tangent perto
2
the
line
0?
straight
pendicular
630. Write the equation of the tangent and the normal to the
parallel to the straight line

xy7 =

=
+
4x3y =

parabola

y=

at the point
Solution.

We

= [y'] x= i = -T-

with abscissa x = 4.
have y

,/-

whence

7=;

the

slope

the

of

Since the point of tangency has coordinates

#2 =

= 4,

follows that the equation of the tangent is


1/4 (*
4) or x
Since the slope of the normal must be perpendicular,
*,

whence the equation

of the

normal:

tangent

y = 2,

4 (x

4) or

4x

+y

It

4# + 4 = 0.

= -4;
t/2 =

is

18

0.

Geometrical and Mechanical Applications of the Derivative

Sec. 4\

63

631. Write the equations of the tangent and the normal to the
4# 3 at the point (2,5).
2x*
curve y = x'
632. Find the equations of the tangent and the normal to the
curve

at the point (1,0).

633. Form the equations of the tangent and the normal to the
curves at the indicated points:
tan2x at the origin;
a) y
b)

=
y = arc

sin

^^

at

the

point

of

intersection

with

the

A:-axis;
c)

d)

y = arc cos 3x at the point of intersection with the y-axis;


at the point of intersection with the #-axis;
y = ln*
~ x*
at
the points of intersection with the straight
y=e
}

e)

line

y=

1.

634. Write the equations of the tangent


point (2,2) to the curve
t*

and the normal

at the

'

635. Write the equations of the tangent to the curve

at the origin

and

at the

point

= j-

636. Write the equations of the tangent and the normal to the
x*
6=0 at the point with ordinate y 3.
2x
y*
637. Write the equation of the tangent to the curve x*
y*

curve

+ +

2xy = Q at the point (1,1).


638. Write the equations of the tangents and the normals to
the curve y = (x
3) at the points of its intersection
2)(x
l)(jt
with the #-axis.
639. Write the equations of the tangent and the normal to the
4
curve y* = 4x
6xy at the point (1,2).
640*. Show that the segment of the tangent to the hyperbola
xy = a* (the segment lies between the coordinate axes) is divided
in two at the point of tangency.
641. Show that in the case of the astroid x 2 8 + y*t* = a*/ J the
segment of the tangent between the coordinate axes has a constant value equal to a.

___

[Ch. 2

Differentiation of Functions

64

642.

Show

that the normals to the involute of the circle

x = a(cost

y = a(sinf

sin/),

cost)

are tangents to the circle


643. Find the angle at which the parabolas y
(x
2
_.
x i ntersect.
4 _|_ 6*
y
2
x and y
644. At what angle do the parabolas y

Sect?
645.

Show

that the curves y


4x
2x 8 and y
are tangent to each other at the point (3,34). Will

same thing
646.

at

Show

2)

and

= x*

inter-

= x*

we have

-\-

10

the

(2,4)?
that the hyperbolas

intersect at a right angle.


647. Given a parabola y*
4x. At the point (1,2) evaluate the
lengths of the segments of the subtangent, subnormal, tangent,

and normal.
648. Find the length of the segment
2* at any point of it.
curve y
649.

Show

that in the

equilateral

of the

subtangent of the
2

a the
y
hyperbola x
to
the
radius
vector
equal

length of the normal at any point is


of this point.
650. Show that the length of the segment of the subnormal
2
2
2
a at any point is equal to the abscissa
in the hyperbola x
y

of this point.

651.
x*

Show

that the segments of the subtangents of the ellipse

jjr+frl

an d the circle

x -+y

= a*

at

points

with the same

abscissas are equal. What procedure of construction of the tangent to the ellipse follows from this?
652. Find the length of the segment ol the tangent, the normal, the subtangent, and the subnormal of the cycloid
(
\

x = a(t
a(ts'mt),
y = a(l

an arbitrary point t~t


653. Find the angle between the tangent and the radius vector
of the point of tangency in the case of the logarithmic spiral
at

Find the angle between the tangent and the radius vecthe point of tangency in the case of the lemniscate
1
a cos 2q>.

654.
tor
r*

of

Sec. 4]

Geometrical and Mechanical Applications of the Derivative

65

655. Find the lengths of the segments of the polar subtangent,


subnormal, tangent and normal, and also the angle between the
tangent and the radius vector of the point of tangency in the
case of the spiral of Archimedes

at a

point with polar angle

<p

= 2jt.

Find the lengths of the segments of the polar subtangent,


subnormal, tangent, and normal, and also the angle between the tanat an
gent and the radius vector in the hyperbolic spiral r=
656.

r
rQ
cp
arbitrary point cp
657. The law of motion of a point
;

Find the velocity of the point at /


in centimetres and / is in seconds).

on the *-axis

= 0, ^

and

is

(x

is

658. Moving along the #-axis are two points that have the
2
5t and #=l/2/
where t^O.
following laws of motion: x=\00
With what speed are these points receding from each other at
the time of encounter (x is in centimetres and / is in seconds)?

659. The end-points of a segment AB


the coordinate axes OX and OY (Fig. 16).

^5 m
A

is

are sliding along


at 2 m/sec.

moving

A
Fig.

17

is the rate of motion of B when A is at a distance OA = 3 m


from the origin?
660*. The law of motion of a material point thrown up at an
angle a to the horizon with initial velocity V Q (in the vertical
plane OXY in Fig. 17) is given by the formulas (air resistance is

What

3-1900

66

_ _
Differentiation of Functions

[Ch. 2

disregarded):

#=i; /cosa,
where

trajectory of motion
speed of motion and

661.

i>

/sin a

^-,

point

g is the acceleration of gravity. Find the


and the distance covered. Also determine the

is in

its

direction.

motion along a hyperbola

r/

so that its

x increases uniformly at a rate of 1 unit per second.


the rate of change of its ordinate when the point passes

abscissa

What

the time and

is

*/

is

through (5,2)?
662. At what point of the parabola

y*=\8x does the ordinate


increase at twice the rate of the abscissa?
663. One side of a rectangle, a
10 cm, is of constant length,
while the other side, b, increases at a constant rate of 4 cm,'sec.
At what rate are the diagonal of the rectangle and its area increas30 cm?
ing when 6
664. The radius of a sphere is increasing at a uniform rate
of 5 cm/sec. At what rate are the area of the surface of the
sphere and the volume of the sphere increasing when the radius

becomes 50 cm?
665.

=10

point

is

in

motion along the spiral

of

Archimedes

so that the angular velocity of rotation of its radius


constant and equal to 6 per second. Determine the rate
of elongation of the radius vector r when r
25 cm.
666. A nonhomogeneous rod AB is 12 cm long. The mass of a
part of it, AM, increases with the square of the distance of the
from the end A and is 10 gm when
2 cm.
moving point,
Find the mass of the entire rod AB and the linear density at
any point M. What is the linear density of the rod at A and S?

(a

vector

cm)

is

AM =

Sec. 5. Derivatives of Higher Orders

1. Definition of higher derivatives. A derivative of the second order, or


Ihe second derivative, of the function y=f(x) is the derivative of its derivative; that is,
</"

(</')'.

The second derivative may be denoted


</".

If

* = /(/)

is

or

^.

as
or

f"(x).

the law of rectilinear motion of a point, then

eration of this motion.

is

the accel-

Sec. 5]

Derivatives of Higher Orders

Generally, the ith

a derivative of order (n

derivative of a function y
f(x) is the derivative
1). For the nth derivative we use the notation

(v

Example

or

~^,

or

Find the second derivative

t.

= \n(\

(n)

67
of

(x).

function

of the

x).

/.JZL; /

Solution.

2. Leibniz rule. If the functions u q>(x) and v=ty(x) have derivatives


to the nth order inclusive, then to evaluate the nth derivative of a product of these functions we can use the Leibniz rule (or formula):
up

(uv)

<">

= u<"

3. Higher-order derivatives

= -r

then the derivatives y'x

functions

of
(

i^

==
f/^jc

represented

can

^2

successively

xt

we have

For a second derivative

Example

Solution.

2.

F^nd

We

the formula

w
/

if

have
-

If

_
/

(a cos*),

& cos
.

&

"~~ "~~~
.

asm*

LUl

and
.

(acosO

If

= q>(0,
= *(0,

by the formulas

xt

parametricaKy.

-asln<

osln

be

calculated

68

Differentiation of Functions

[Ch.

A. Higher-Order Derivatives of Explicit Functions


In the examples that follow, find the second derivative of
given function.
667. y
668. y

669.

= x* + 7x'
= e*

5x

+ 4.

671.

672.

= \n t/\+x

Show

y= (arc sin x)
= acosh
u

673.
674.

V^

675.

y=sm*x.

670. j
y

//

th<

OY

*/

y=

that the function

-\

satisfies the

differ

ential equation

676.
tial

Show

that the function y

2y'+y = e

equation y"
677.

Show

equation y" 4-3y'


678.
tion y"
679.
680.
681.
682.
683.

Show

=
-|-2y

the

satisfies

differen

y=-C

e"

for all constants

the

that

function

the

that

= -^-x e x

function y

+ C 2 e' 2x
C and C
l

= e 2x s'm5x

satisfies th
2

satisfies the equa


4y
s
2
5x + 7x
2.
Find y"' if y = x
5
Find /'"(3) if /(*) - (2^:
3)
Find y v of the function # = ln(l+x).
Find t/ VI of the function y==sin2x.
Show that the function y = e~ x cosx satisfies the differ
ential equation y lv + 4y = Q.
684. Find /(O), f (0), T(0) and /'"(O;
f

+29y = 0.
,

if

= e x sinx.

f(x)
685.

The equation of motion of a poin


along the jc-axis is

O.OOU 8

X-100-H5/

Find the velocity and the acceleration


the
f

point

for

times

=10.

= 0,

=\

an

is in motion around
with constant anguls
Fig- 18
velocity CD. Find the law of motion of i1
projection M, on the x-axis if at time / =
the point is at
Q (a, 0) (Fig.
18). Find the velocity and the ac
celeration of motion of M,.
at the in
What is the velocity and the acceleration of
tial time and when it passes through the origin?
What are the maximum values of the absolute velocity and th

686.

circle

point
2

+y = a

absolute acceleration of Ai,?

Sec. 5]

69

Derivatives of Higher Orders

687. Find

the nth derivative of the function


3 natural number.
688. Find the nth derivatives of the functions:

where n

y=

js

a)

and

y^T^x*

y^^**'

b)

689. Find the /zth derivative of the functions:

j/=sinx;

a)

e)

b)4, = cos2*;
= e~ *;
c) y

f)

y=^j\
= yJ;
J/

g)

|/=ln(l+x);

d)

h)

y=sin*jr,

y=

690. Using the Leibniz rule, find y


a)

y = x.f\

b)

x
y = jc .e-*

{n

d)y =

c)

691. Find

//

= (!

(n)

(0),

if:

2
A:

e)

= x*

cos x\

if

B. Higher-Order Derivatives of Functions Represented


Parametrically and of Implicit Functions
d^u

In the following problems find

692. a)

693. _,

= \nt,

'

b)

:asin/;
:

= 0cos'/,

= arc tan/,
\0-l
x

"'

iv

c)

* = arc sin/

= a(l -cos/);
x = a (sin/- /cos/),
= a(cos/-f-/ sin/).

\ y

696. Find

[Ch. 2

Differentiation of Functions

70

d*u

697. Find
698.
tions x

ax 2

Show

== sin

if

y=

{
\

that y (as a function

^
+ be~ iV
y = ae'
2

t,

* = ln(l-f-/

/=0,

for

satisfies the differential

= sec,
{
=
tan/.
\ y
= ^"'cos/,
{xy-e-<*tnt.
x

699.

703.
jt'"

701.

Solution.

By

/,

=
y'

r/

y = f(x),
x = f~*(y).

/=-

and

if

'

find

composite

^^

+ **_
*

function

we have

-2^;^.

if

if

required to determine
represented implicitly.

is

it

= f(x)

the

|y

l.

707.

# = *-}- arc tan

708.

Having the equation y = x + \ny

709. Find

|/.

at the point (1,1)

5xy +

710. Find y" at (0,1)

711. a)

2x

find

-j

= 0.

if

The function y

is

defined

implicitly

= 0.

at the point (1,1).


b)

and

if

Find

the derivatives x",

finally get:

In the following examples


derivative y" of the function y
705. y*

^+f=

^.

differentiating

-jwe
^

706.

the equaand 6

+ y*=*l.

Substituting the value of #',

x = e -1
=
^

function

the rule for

2* + 2i0'=0; whence

by

any constants

7 2 ' Find

x'

if

<
1

of the inverse function

704. Find

defined

x)

y'"
f

the

of

for

'

Knowing

),

equation

In the following examples find


(

Find

if

i/

==a 8 .

by the equation

Sec. 6]

Differentials of First

and Higher Orders

71

Sec. 6. Differentials of First and Higher Orders


1. First-order

differential. The differential (first-order) of a


function
the principal part of its increment, which part is linear relative
to the increment Ax
dx of the independent variable x. The differential of a

= f(x)

is

19

Fig.

function

is

equal to the

product of its

derivative

by the differential of the

independent variable
dy--=y'dx,

whence
u
y

MN

If
is an arc of the graph
tangent at M. (x, y) and

dy
dx

'

the function

of

= f(x)

(Fig.

MT

19),

is

the

PQ = Ax-=dx,

then the increment in the ordinate of the tangent

and the segment AN


by.
Example 1. Find the increment
2
=
3x
x.
y
Solution. First method:
A//

= 3 (x + Ax)

and

(x

the

differential

+ Ax)

3x 2

of

the

function

+x

or
At/

Hence,

= (6*

dy = (6x

Ax + 3 (Ax) 2

1)

1)

Ax = (6x

1)

dx.

Second method:
t/'

Example

2.

and Ax = 0.01.
Solution.

= 6x

Calculate

A/ =

At/

1;

df/

= j/'dx = (6x

and dy

of

the

1)

dx.

function

y = 3x

l)-Ax

(6x

<fy

(6jt

+ 3 (Ax) = 5- 0.0 + 3- (0.01 = 0.0503


Ax = 5- 0.01 = 0.0500.
2

and

1)

for

x=l

72

Differentiation of Functions

[C/t.

2. Principal properties of differentials.


= 0, where c = const.
1) dc
2) d*- Ax, where x is an independent variable.
3)

4)
5)

= cdu.
dv.
v) = du
d (uv)
udv + v du.
d(cu)

d(u

7)

differential to approximate calculations. If the increment


argument x is small in absolute value, then the differential dy of the
function y = f(x) and the increment At/ of the function are approximately

3. Applying the

A*

of the

equal:
that

A # =^ dy,

is,

whence
3. By how much (approximately) does the side of a square change
2
to 9.1 m 2 ?
area increases from 9
Solution. If x is the area of the square and y is its side, then

Example

if

its

It

is

given that #

The increment
mately as follows:

At/

=9
in

and A*

0.1.

the side of the square

ky^zdy--=y' Ax

4. Higher-order

differentials.

j=z

-0.1

may

be calculated approxi-

= 0. 016m.

second-order differential

is

the differential

of a first-order differential:

We

similarly define the differentials of the third and higher orders.


If y
f(x) and x is an independent variable, then

But

if

= /(),

where w

d*y

and so

forth.

= cp(x),

then

= y"' (du) + 3y" du


9

d*u

+ y' d'u

(Here the primes denote derivatives with respect to

M).

712. Find the increment Ay and the differentia! dy of the func2


# 5* -f x for x 2 and A# 0.001.

tion

Differentials of First

Sec. 6}

713.

and Higher Orders

Without calculating the derivative,

find

73

d(l-x')
for

x=\
714.

and Ax =

The area

square S with

of a

Find the increment and the

side x

differential of

is

this

x*.
given by S
function and ex-

plain the geometric significance of the latter.


715. Give a geometric interpretation of the increment and
differential of the following functions:
nx*\
a) the area of a circle, S
b) the volume of a cube, v=^x\
716. Show that when Ax
*0, the increment in the function

= 2X

corresponding to an increment Ax in x, is, for any x,


equivalent to the expression 2* In 2 A*.
717. For what value of x is the differential of the function
2
y = x not equivalent to the increment in this function as Ax >0?
718. Has the function y = \x\ a differential for x = 0?
719. Using the derivative, find the differential of the function
//

cos x for x

=y

and Ax

--=

720. Find the differential of the

for

for

function

Ax-

0.01.
9 and
x
721. Calculate the differential of the function

x-^-J

Ax^.

and

problems find the differentials of the given


functions for arbitrary values of the argument and its increment.
In the following

727. y

722.

y^'-m-

723.

<,=

724.

#= arc sin

= cot
s = arc

729. r

730.

725. //--=arctan~.

= x\nx

q>

x.

-f

cosec

(p.

lane*.

726. y
e~ \
731 Find d//

if

x*

+ 2xy

y*

= a*.

Solution. Taking advantage of the invariancy of the form of a differential,


x dy)
2y dy
(y dx

we obtain 2x dx + 2
Whence

74

Differentiation of Functions

[Ch. 2

In the following examples find the differentials of the functions


defined implicitly.
732.

733.

734.
X

y'y = 6x*.

735. Find dy at the point (1,2), if


736. Find the approximate value of sin
Solution. Putting

3) we have

(see

sin

arc30=-jr-

and Ax = arc

+ ^~ cos

31=^ sin 30

31.

1=^, from formula

(1)

30=0.500+0.017-J~^=0.515.

737. Replacing the increment of the function by the differencalculate approximately:

tial,

a) cos

b) tan
c)

61;
44;

d) In 0.9;
e) arc

tan 1.05.

e*\

738. What will be the approximate increase in the volume of


a sphere if its radius
15 cm increases by 2
739. Derive the approximate formula (for
\&x\ that are small
compared to x)

#=

mm?

A*

Using

it,

V5

approximate

Y\7, /70, /640.

740. Derive the approximate formula

and find approximate values


741.

for

for

*=1.03;
* = 0.2;

for

* = 0.1;

for
__

c)

d)

j!/TO,

j/70, jI/200.

Approximate the functions:

/(x)- !/"}=
y=e

~ x*

x =1.05.

for
/

742. Approximate tan 453 20".


743. Find the approximate value of arc sin 0.54.
744.

Approximate

Sec. 7]

_ _
Mean-Value Theorems

745. Using

Ohm's

law,

= --,

show

the current, due to a small change


found approximately by the formula

2/

the

in

resistance,

in

may be

A.

A/
746. Show that,
relative error of 1/

that a small change

75

of the radius, a
error of approximately
in calculating the area of a circle and the surface of a sphere.
747. Compute d*y, if y
co$5x.

determining the

in

length

results in a relative

Solut ion.

748. u
749.
750.

//

d y = y" (dx =
2

25 cos 5* (dx) 2

=
x\
= arccosx,

find d*u.

{/^sinxlnx,

find d*y.
find d*y.

= ^, find d'z.
z = **-*, find d'z.
z =
flnd d4
2=TJ,
M = 3sin(2jt-f 5), find
= e* cosa sin(;t sin u),

751.

752.

753.

*'

754.

755.

//

d"w.
n

find d y.

Sec. 7. Mean-Value Theorems

1. Rolle's
b,

then the

theorem. If
has a derivative

argument

has

at

the interval
is continuous on
f (x)
every interior point of this interval, and

function

at

/' (x)

least

one

value

where a

< 5<

b,

such that

2. Lagrange's theorem. If a function f (*) is continuous on the interval


and has a derivative at every interior point of this interval, then
where a

<5<

ft.

3. Cauchy's theorem.

If

the functions

a<x<b

a^x^b

and for
simultaneously, and F(b)^F(a)
interval

-f (a)
756.

Show

l<x<0

that

and

the
g.

and F

(x) are

derivatives

continuous on the
do not vanish

that

then

_
function

0<x<l

appropriate values of

f (x)

have

f(x)

= xx*

satisfies the

Rolle

on

the

intervals
the

theorem. Find

76

Differentiation of Functions

The function / (x) is continuous and different! able for all values
= /(0) = /(1)=0. Hence, the Rolle theorem is applicable on
1)
intervals
and 0<*<_1, To find
form the equation
_ we

Solution.

of x,

the

and /(

Kx<0

n*) = l-3* = 0. Whence


2

0<E <

and

[Ch. 2

757.

The function

fc^-J/l;

J/l

-i< g, <o

where

1.

takes on
f(x)
2)
\/(x
equal values
the end-points of the interval [0.4]. Does

/(4)
j/4 at
the Rolle theorem hold for this function on [0.4]?
758. Does the Rolle theorem hold for the function
/(0)

on the interval
759.

Show

[0,

JT]?

Let

that the equation


/'(*)

has three real roots.


760.

The equation

obviously has a root x

any other

real

= 0.

Show

that this equation cannot have

root.

761. Test whether the Lagrange theorem holds for the function

on

the interval
value of

[2,1] and

the

find

appropriate

intermediate

= xx*

all

Solution. The function f(x)


values of A:, and /' (x)= 1
3x 2

h3ve

/(l)-/(-2y = 0-6

2
2 and
3^
2
inequality
1

continuous and difTerentiable for


by the Lagrange formula, we
= [l-(-2)]/ (E), that
is,
Hence,
the only suitable value is
1, for which the
is

Whence,
/

/'(E)2
=

g=l;
< < holds
1

762.

Test the validity of the Lagrange theorem and find the


intermediate point
for the function f(x)
x 4/s on
the interval [
1,1].
2
763. Given a segment of the parabola y
x
lying between
two points A (1,1) and 3(3,9), find a point the tangent to which
is parallel to the chord
AB.
764. Using the Lagrange theorem, prove the formula

appropriate

sin (x 4- h)

where

sin

x = h cos

Taylor's Formula

Sec. 8]

77

+2

*
765. a) For the functions /(x)
whether the Cauchy theorem holds on
find E;

b)

do

same with

the

on the interval

~1

To,

and F(x)

the

respect to /(*)

= x'

interval

= sin*

test

and

[1,2]

and F(x)

= cosx

Sec. 8. Taylor's Formula


If

(n
is

function

f (x)

is

continuous and has continuous derivatives up to the

l)th order inclusive on the


a finite derivative f (n} (x) at

lor's

a<x<6

interval

each interior point

&<*<a),

(or
of the interval,

and there
then Tay-

formula

fw = f (<o + (*-*) r (o) +

-V(a)

-V

w+

nl

where

= +

a
0(jc
In particular,
f

W =/

(0)

a)

and

when

+xf

(0)

0<6<1,

holds true on the interval.

we have (Maclaurin's formula)

r (0) +

"-'>

(0)

/<>

(I),

where

=
?

0<9<1.

0jc,

766. Expand the polynomial /(A:)


tive integral powers of the binomial x

n*)=3jt

Solution.
for

4A-

= 6^

2;c

A:

+ 3;

/'

f'(2)

= 7;r(2) = 8;r(2)

(jc)

+ 3^ + 5

in posi-

2.
4;

/'"

(x)=6;

n^4. Whence
H;

=6.

Therefore,

or

2x z

8
Jt

767.

+ 3x + 5 =

(n)

Solution.

+7

(.v

2)

+ 4 (x

x
Expand the function f(x)=e

term containing (x-f-1)

where

1 1

(x)

= e*

in

2)

2)

(A-

powers

of

to

the

8
.

for all

n,

p)(

1)=JL. Hence,

+6(*+ 1); 0<G<1.

768. Expand the function /(x)


the term with (x
1)*.

lnjt in

powers

of

up

to

78

[Ch. 2

Differentiation of Functions

= sin x

in powers of x up to the term containing x


e* in powers of x up to the term contain770. Expand f(x)
n~ l
ing x
771. Show that sin(a
/i) differs from

769.

Expand
9

/ (x)

and to the term containing x*.

+ h cos a

sin a
2

by not more than l/2/i


772. Determine the origin
.

of the

VT+x&l+x y*
yi+i&l+x~x*,
2

a)

b)

and evaluate their errors.


773. Evaluate the error

Due

774.

to

line

a catenary

in

of the

shape

775*.

\x\<l,

\x\<\

is

formula

in the

weight, a heavy

y = a cosh.

thread

Show

own

its

approximate formulas:

Show

approximately

suspended thread

that

the

by the parabola

expressed

that for \x\<^a, to within

small \x\

for

lies

(^-J

we

have

the

approximate equality

Sec. 9.

The L'Hospital-Bernoulli Rule

Evaluating Indeterminate Forms

for

oo

1. Evaluating the indeterminate forms

and

and
u

Let

oo

the

single-valued

a </i; the derivative


be differentiate for 0<|#
of one of them does not vanish.
* a\ that
If f(x) and q>(*) are both infinitesimals or both infinites as x
functions

if

is,

/ (x)

the quotient

(p

(x)

^-4

at

x = a,

is

one

of the

indeterminate forms --

oo
i

then

00

*-+<*

provided that the limit

lim /(*)
(p (x)

^lim
x-+a

(*)

q>' (

x)

of the ratio of derivatives exists.

or

V Hospital-Bernoulli

Sec. 9]

The

rule

is

when a =

also applicable

fix)

If

the quotient

,,

Rule for Indeterminate Forms


00.

yields an

again

indeterminate

at the point

form,

= a,

of one of the two above-mentioned types and /' (x) and


the requirements that have been stated for f(x) and q? (x),
pass to the ratio of second derivatives, etc.

79

q>'

However,

may

exist,

it

should

borne

be

whereas the ratios

mind that the

in

limit of the ratio -^-~

derivatives do not tend to any limit

the

of

(x) satisfy

we can then

all

Example 809).
2. Other indeterminate forms. To evaluate an indeterminate form like
0oo, transform the appropriate product fi(x)*f t (x), wne re lim/, (jt) = and
(see

K+O.

/(*)
lim/ 2 (*)

= oo,

^^

into thequetient

form

(the

*->a

(T^T\
/i (X)

( the

orm -).
oo

M*)
In the case

of

the indeterminate form oo

appropriate difference /,(*)

f 2 (x)

oo,

one should transform the

into the product / t (x)

first

evaluate the indeterminate form 7*7^;


r

lim

if

(X)

and

/ 1

7^7^=1,
\x

x-+a i\

x )j

then

we

re-

duce the expression to the form

form

(the

).

/Tw
The indeterminate forms I, 0, 00 are evaluated by first faking loga
rithms and then finding the limit of the logarithm of the power [f l (x)]^ (x}
(which requires evaluating a form like 0oo).
In certain cases it is useful to combine the L'Hospital rule with tht
finding of limits by elementary techniques.

Example

1.

Compute
lim

JL1

*->o cot

(form ").
7
oo

Solution. Applying the L'Hospital rule

we have

JEfL^llm pL*r
x+ocotx jc-o(cot*)

lim

lim

We

get

the indeterminate form

-jp

jc-*o

however,

L'Hospital rule, since

Um
C-frO

We

sint

Hm

sin

~~*-H)

thus finally get


"

JC->0

COt X

we

do

not

need to

use the

[Ch. 2

Differentiation of Functions

80

Example

Compute

2.

limf-J- x

*-M)V sin

Reducing

xL^
J

(form

common denominator, we

to a

lim

oo

oo).

get

(4__J_Ulim*.!z^lf
x
x J
x-o x sin x

x-+o \ sin

v
(fonn

'

Before applying the L'Hospital rule, we replace the denominator of the


2
2
fraction by an equivalent infinitesimal (Ch. 1, Sec. 4) * sin A;~ x*.
obtain
ter

lat-

We

The L'Hospital

rule gives

lim

L)

Then,

in

A "~ S

= lim

m *=lim-

elementary fashion, we find

_ ___
2

x-<> \ sin

Example

3.

cos 2x

2 sin 2 *

,.

Compute
8

*2

lim (cos 2x)

00

(form

X-M)

Taking logarithms and applying the L'Hospital

lim In (cos
X-*0

lim
2*p = X-+Q

31ncos2*
X

rule,

= _ 6 lim

we

%X

x-+Q

get

= _6

J^

Hence, lim

(cos 2x)

*^e-*.

Jf->0

ind the indicated limits of functions in the following

ples.
x '-

77G.

lim
,_>,

*'

Solution. lim

"
;
X->1

777.

lim

xcosx
v

^r 2 "~~ 7/

OJi

~~

sinx
,

779.

lir

*7on

is

I-*

tj
11IU

'

ji

*->il_si n :r

780.

lim

tan*

sin*

exam-

sec

,.

781.

2tan*
1+COS4*
2

'

785.

lim-

*-cot^
2

lim^.
tan5x

782.

^Ji

786.

lim

*-"

787.

i .

1.

lim

[,

cos x) cot

(1

x->o

n x
lim -r-?=r

lim
1

783.

81

A-

x +

lim

Solution,

(1

^cos

x) cot

= Hm

cos A

'^- lim
X->0

Sill AT

Forms

L' Hospital-Bernoulli Rule for Indeterminate

Sec. 9]

Sill

linrZi!iJ^(

788.

lim(l

A*)

Ian

792.

,V-^l

X^X

lim arc sin x cot x.

789.

-,
V

lim x" sin

793.

'

hnilnxln

(x

1).

X-+Q
ri

790.

n>0.

lim(jc e?"*),
v^o

791. lim x sin

794.

lim
^

*i

f^
\

n"^
lr
%

Solution.

[-

A'

= lim

lim
j

^^MiiA-l

795.

lim

796.

lim

(A

1)

11

= lim

\nx

-1

*^

A
-7

T-

h~?
2
A'

L__l

limf-^^'
*

y
797

~ >l

^Vc

_^/

A)

AO J

^}
2cosx/

798.

lim

A;*.

Solution.

lim

We

= lim
j~

have

0,

**

r/;

whence lim//=l,

In

y=?x

that

In

is,

A".

ImiA^

lim In

l.

t/

= limjtln x

82

(Ch. 2

Differentiation of Functions

799. limx*.

804.

li

V-H
tan

800. limx4 * "*.

805. Hmftan^f)
4 /

X-+l\

801. linue s/n

806. lim (cot x) ln

*.

*->0

*.

X-H)
cos

802.

lim(l-*)

803.

lim(l+x

ta

807.

)*-

lta(I)
x /
x-*o \

".

808. lim (cot x)* in

*.

X-+0

809. Prove that the limits of

X
a)

cannot

sin*

be found

by the L'Hospital-Bernoulli

rule.

Find these

limits directly.

810*. Show that the area of a circular segment with minor


and
central angle a, which has a chord
(Fig. 20), is

AB=b

CD=A

approximately

with an arbitrarily small relative error when a

->0.

Chapter III

THE EXTREMA OF A FUNCTION AND THE GEOMETRIC


APPLICATIONS OF A DERIVATIVE

Sec.

1.

The Extrema

1. Increase and

of

a Function of One Argument

of tunctions. Tlu Junction y


f(x) is called
on some interval if, fo. any points x and x 2 which
belong to this interval, from the inequality A',<A- 2 we get the inequality / (*,)<
21a)
</(*i) (Fig
[/(*,)>/ (A,) (Fm. 21&)]. I! f(x) is continuous on the
interval [a, b] and /' (x)>0 [/' (A')<OJ for a< .<b, then /(A) increases (decreases) on the interval [a, b\.

decrease

increasing (decreasing)

ffxj

xz

i,

(a)
Fifi.

Fig. 22

21

In the simplest cases, the domain of definition of f (x) may be subdividincrease and decrease of the funca finite number of intervals of
tion (intervals of monotonicity). These intervals are bounded by ciitic-'
or f' (x) does not exist].
points x [where /'(jc)
Example 1. Test the following function for increase and decrease:

ed into

Solution.

We

find the derivative


t/'

Whence

y'

for

x=l. On

= 2x
a

= 2(*

number

scale

1).

we

get

two intervals

of

monot-

oo<x<l, then
onicity: (00, 1) and (1, -f oo). From (1) we have: 1) if
oo, 1); 2)
i/'<0, and, hence, the function f (x) decreases in the interval (
if l<A'<
oo, then j/'>0, and, hence, the function /(*) increases in the interval (1, +00) (Fig. 22).

Extrema and the Geometric Applications

84

Example

Determine the intervals

2.

of

and decrease

of increase

[Ch. 3

a Derivative

of the func-

tion

Solution.

,
i

o\\2

<Q

oo<*<
Example

Here,

*^~

for

Hence, the

the

of

discontinuity

and

function

(/'

decreases in the intervals

function y

and
Test the following function for increase or decrease:

3.

/i
y

Solution

is

*v s

*y a

'

Here,
(2)

find the points x l


Q, we
Solving the equation x* -x
0, x,= l
1, *2
at which the derivative y' vanishes. Since y' can
change sign only when
passing through points at which it vanishes or becomes discontinuous (in the
given case, y' has no discontinuities), the derivative in each of the intervals
1, 0), (0,1) and (1,
(00,
1), (
+00) retains its sign; for this reason, the
function under investigation is monotonic in each of these intervals. To
determine in which of the indicated intervals the function increases and in
which it decreases, one has to determine the sign of the derivative in each
of the intervals, To determine what the sign of y' is in the interval
00,
inter1), it is sufficient to determine the sign of y' at some point of the
val; for example, taking x=
2, we get from (2) f/'
12>0, hence, y'>Q in
the interval (00,
increases Similar1) and the function in this interval
we find that y'<Q in the interval (1, 0) (as a check, we can take
ly,
(

'

the

in

Y\

/'A\

interval

(0,1)

**

(here,

we can

interval

(1,

x=l/2) and y'>0

use

in

the

+00).

the function being tested inThus,


creases in the interval (
oo,
1), decreases
in the interval
1) and again increases
in the interval (1, -f oo).
2. Extremum of a function. If there

(1,

exists a two-sided neighbourhood of a point


X Q such that for every point X^X of this
Q

neighbourhood we have the


inequality
f(x)>f(x Q ) then the point x is called the
J

Fig

23

minimum

point

of

while the number

mum

the

/ (x

of the function y
neighbourhood of the point

function
is

f(x),

called the

mini-

f(x).

Similarly,

if

x lf the inequality
any point xj^x l of some
f(*)<f(x\) is fulfilled, then *, is called the maximum point of the function
and
is
the maximum of the function (Fig. 23). The minimum
f(x),
/(*j)
point or maximum point of a function is its extremal point (bending point),
and the minimum or maximum of a function is called the extremum of the
function. If xn is an extremal point of the function f
then /' (* ) = 0, or
(x),
for

The Extrema

Sec. 1]

The

of a

Function of One Argument

85

and absence of an
are given by the following rules:

extremum

sufficient conditions for the existence

continuous function

/ (x)

of a

1. If
there exists a neighbourhood (X Q
6, *
6) of a critical point *
such that /'(x)>0 for X Q
d<x<xQ and /'(jt)<0 for xQ <x<xQ d, then * is
the maximum point of the function / (*); and if /' (*)<0 for *
6<*<x
is the minimum point of the
and /' (x)>0 for x <x<xe
function
6\ then *

/(*)-

Finally,

function
2.
if

there
for

/ (x).
r
If
f (XQ)

f' ( XQ )

(*

if

unchanged

sign

=Q

= 0,

is

some

0<|jc

positive

number 6 such

then x

X Q |<6,

is

that /' (x) retains its


not an extremal point of the

then X Q is the maximum


and /"(*<,)<(),
)>0, then x is the minimum point; but if f
)^0, then the point X Q is not an extremal point.

and

f"

and /'"

(*

(*

point;
(*

= 0,

More generally:

let the first of the derivatives (not equal to zero at the


function f (x) be of the order k. Then, if k is even, the
is
an extremal point, namely, the maximum point, if f (k) (* )<0;
and it is the minimum point, if / (ft) (x )>0 But if k is odd, then A- O is not
n extremal point.
Example 4. Find the extrema of the function

point x
point X Q

the

of

<

i/

==2*

+3

Solution. Find the derivative


(3)

Equating the derivative

Whence,

x-

x-=
function

if

we
-/i,

find

then

is

we

get:

x l =-

a sufficiently small

Hence,

/'<0*).

and //max=-l.
Equating the denominator
r/

to zero,

the critical point

where h

\+h,

y'

*,

1.

From formula

(3)

we

have:

if

positive number, then /y'>0; but


I
is the maximum point of the

of

the expression of y' in (3) to zero,

we

get

\\e find the second critical point of the function A'2


there
0, where
for*/! we have
no derivative //' For *==
/i, we obviously have //<0;
is
the minimum point of the function y, and
//>0. Consequently, * 2
m jn
(Fig. 24). It is also
possible to test the behaviour of the function
1
at the point x
by means of the second derivative

whence
is

i/

/=--4^=

1
is the maximum
I
x,
and, hence, *,
point of the
function.
3. Greatest and least values. The least (greatest) value of a continuous
function f (x) on a given interval [a, b] is attained either at the critical
points of the function or at the end-points of the interval [a, b].

Here, r/"<0 for

to determine the sign of the derivative y', one can


*) If it is difficult
calculate arithmetically by taking for h a sufficiently small positive number.

86

Extrema and the Geometric Applications

Example

of a

Find the greatest and least values

5.

on the interval

Derivative

[Cfi.

of the function

P/2

Solution. Since

it

follows that the critical points of the function y are *,

and

Fig. 24
of the function at these points and the values
function at the end-points of the given interval

Comparing the values

we conclude
the

(Fig.

x=l (at
*=2 /i

point

at the point

that the function attains

25)

the

minimum

point),

and

its least

the greatest value

Determine the

812.
813.
814.

y=l

m=l,

at

A4 = ll

(at the right-hand end-point of the interval).

intervals

of decrease

and increase

1ions:
811.

value,

of the

4*

jf.

= (* 2)
y = (A:+4)
= *'(*- 3).

*>-

{/

817.

818.

= (x

{/

of the func-

The Extrema

Sec. 1]

819.

of

a Function

y^ \-V~x.

822.

y = 2e*

823.

= x -f sin x.
y = x\nx.
= arcsin(l-f-x).

820. y

821.

One Argument

of

'

825

^T"

We

find

the

zero,

we

for extrema:

= +

Solution.

-'*.

_. 2~<*.

24

t/

Test the following functions


x*
826. y
4*4-6.

derivative

of

the

given function,
value of the argument x=
2.
Since i/'<0 when x<
2 is
2, and y'>Q when *>
2, it follows that *=
the minimum point of the function, and #min
2. We get the same result
of the second derivative at the critical point y"~<
by utilizing the
~ sign

Equating

to

y'

the

get

critical

827. y
828. {/
829. (/

--

=
=

Solution,

We

find the derivative

= 6*

y'

4-

6x

12

=6

+*

2
(jc

2).

2
Equating the derivative y' to zero, we get the critical points x,=
and *,= !. To determine the nature of the extremum, we calculate the
second derivative ^"^ 6 (2* 4-1). Since /(
2
2)<0, it follows that x,=
is the maximum point of the function y, and #max
25. Similarly, we have
the function y and
t/*(l)>0; therefore, x 2 =l is the minimum point of

i=

2.

<-

12 )

2
'

840.

(*

I)

y-

2)'.

841.

836. V

= rr4=^.

837. t/=

w=J/(^

OQQ
ooy.

it
*/ ===

Determine the
indicated

843. y

844.

847.

citi ^. v
-+ sin
^k*.
I

least

intervals

,_

846. y

1)'.

O sin
cin ZA
Ov
z

842. #

(if

f/

/1ft
//
M
oto.

= x'e-*.
= -.
X
-

ar/* fan
^
idii ^t.
/t-~drc

interval

is

of the functions on the


not given, determine the

and greatest values


the

ln(l+*).

je

845>

^_.

838.

t/

88

greatest

Extrema and the Geometric Applications

and

[Ch. 8

of a Derivative

values of the function throughout the domain

least

of definition).
849.
850.
851.

= rih&.
-=
y
x(lOx).
y= sin + cos
A;

imum

interval

= 2x* + 3*

A;.

12*

a)

b)

on the interval
on the interval

that for positive values of

856. Determine

nomial

V = x* on the

854. y

= arc cos x.

Show

852. #
855.

853

!/

*we

1,3].

1,6];

[10,12],

have the inequality

coefficients p and q of the quadratic triso that this trinomial should have a minJt= 1. Explain the result in geometrical terms.

the

y*=x*+px + q

3 when
t/
857. Prove the inequality
e*

> +x
1

when x

4* 0.

Solution. Consider the function

In the usual

way we

find

lhat

this function

has a single

minimum

/(0)

Hence,

/(*)>/ (0)
and so e*
as

we

set

> +x
1

when x
when x

0,

^ 0,

out to prove.

Prove the inequalities:


x
sin x
858. x

<

^<
o

when *>0.

when

~<ln(l +x)<x

when

859.

cos*>l

860.

A:

JL

861. Separate a given positive number a into two summands


such that their product is the greatest possible.
862. Bend a piece of wire of length / into a rectangle so that
the area of the latter is greatest.
863. What right triangle of given perimeter 2p has the greatest area?
864. It is required to build a rectangular playground so that
should have a wire net on three sides and a long stone wall
it
on the fourth. What is the optimum (in the sense of area) shape
of the playground if / metres of wire netting are available?

Sec.

1]

The Extrema

of

a Function

of

One Argument

89

865. It is required to make an open rectangular box of greatest


capacity out of a square sheet of cardboard with side a by cutting
squares at each of the angles and bending up the ends of the
resulting cross-like figure.
866. An open tank with a square base must have a capacity
of v litres. What size will it be if the least amount of tin is used?
867. Which cylinder of a given volume has the least overall
surface?
868. In a given sphere inscribe a cylinder with the greatest volume.

869.

In a given sphere inscribe a cylinder having the greatest

lateral surface.

870. In a given sphere inscribe a cone with the greatest volume.


871. Inscribe in a given sphere a right circular cone with the
greatest lateral surface.
872. About a given cylinder circumscribe a right cone of least
volume (the planes and centres of their circular bases coincide).
873. Which of the cones circumscribed about a given sphere
has the least volume?
874. A sheet of tin of width a has to be bent into an open
cylindrical channel (Fig. 26). What should the central angle cp be
so that the channel will have maximum capacity?

D
N

M
Fig. 27

875. Out of a circular sheet cut a sector such that when made
into a funnel it will have the greatest possible capacity.
876. An open vessel consists of a cylinder with a hemisphere
at the bottom; the walls are of constant thickness. What will the
dimensions of the vessel be if a minimum of material is used for
a

given capacity?

OB of the door of a ver877. Determine the least height h


so that this door can pass a rigid rod
tower
of
length /, the end of which, M, slides along a horizontal straight
line AB. The width of the tower is
(Fig. 27).

tical

MN

ABCD

d<l

90

Extrema and the Geometric Applications

of

a Derivative

[Ch. 3

878. A point
(x
# ) lies in the first quadrant of a coordinate plane. Draw a straight line through this point so that the
triangle which it forms with the positive semi-axes is of least area.
879. Inscribe in a given ellipse a rectangle of largest area with
sides parallel to the axes of the ellipse.
880. Inscribe a rectangle of maximum area in a segment of
the parabola y*
2px cut off by the straight line x 2a.
,

881.

On

the curve y

find a point at

-\-

Xt

which the tangent

forms with the A>axis the greatest (in absolute value) angle.
882. A messenger leaving A on one side of a river has to get
to B on the other side. Knowing that the velocity along the bank
that on the water, determine the angle at which the
is k times
messenger has to cross the river so as to reach B in the shortest
possible time. The width of the river is h and the distance between A and B along the bank is d.
883. On a straight line AB=a connecting two sources of light A
that
(of intensity p) and B (of intensity </), find the point
receives least light (the intensity of illumination is inversely pro-

square of the distance from the light source).


suspended above the centre of a round table
At what distance should the lamp be above the table

portional to the
884. A lamp
of radius

r.

is

an object on the edge of the table will get the greatest


illumination? (The intensity of illumination is directly proportional to the cosine of the angle of incidence of the light rays and
is inversely
proportional to the square of the distance from the
so that

light source.)

885. It is required to cut a beam of rectangular cross-section


ont of a round log of diameter d. What should the width x and
the height y be of this cross-section
so that the beam will offer maximum
resistance a) to compression and b) to

bending?

I
i/J

Note. The resistance of a beam to compresof its crossis proportional to the area
to the product of the
section, to bending
width of the cross-section by the square of
sion

its

height.

886. A homogeneous rod AB, which


can rotate about a point A (Fig. 28),
is carrying a load
Q kilograms at a distance of a cm from A
and is held in equilibrium by a vertical force P applied to the
free end B of the rod. A linear centimetre of the rod weighs
q kilograms. Determine the length of the rod x so that the force P
should be least, and find P mln
Fig.

Sec. 2]

The Direction

887*. The centres


on a single straight
locity v strikes
strikes C of mass

fi,

of Concavity.

Points of Inflection

91

of three elastic spheres A, B\ C are situated


Sphere A of mass
moving with vewhich, having acquired a certain velocity,

line.

m. What mass should B have so that C will


have the greatest possible velocity?
888. N identical electric cells can be formed into a
battery
in different ways by combining n cells in series and then combining the
allel.

in pargroups (the number of groups is


]
The current supplied by this battery is given by the formula

resulting

where

NnS

~~

the electromotive force of one cell, r is its internal


and R is its external resistance.
For what value of n will the battery produce the greatest
<

is

resistance,

current?
889. Determine

the

diameter y of a circular opening in the


which the discharge of water per second Q
will be greatest, if Q = cy Vhtj, where h is the depth of the
lowest point of the opening (h and the empirical coefficient c are

body

of a

dam

for

constant).
890. If x lf # 2 ..., x n are the results of measurements of equal
precision of a quantity x, then its most probable value will be
that for which the sum of the squares of the errors
,

(*-*,)
0=2
1=1
value (the principle of least squares).

is

of least

of

Prove that the most probable value of x


the measurements.

is

the arithmetic

mean

Sec. 2. The Direction of Concavity. Points of Inflection

1. The concavity of the graph of a function. We say that the graph of a


differentiable function y
f(x) is concave down in the interval (a,b) [concave
the arc of the curve is below (or
up in the interval (a p 6,)] if for
for
the tangent drawn at any point of the interval (a, b)
lt above)
or of the interval (a,, &.)] (Fig. 29). A sufficient condition for the concavity
downwards (upwards) of a graph y f(x) is that the following inequality befulfilled in the appropriate interval:

a.<x<b

a<x<6
=

rw<o

irw>oj.

2. Points of inflection. A point [* f (jc )] at which the direction of concavity of the graph of some function changes is called a point of inflection
,

(Fig. 29).

Extrema and the Geometric Applications

92

of

[Ch. 3

a Derivative

point of inflection x of the graph of a function


derivative f (* )
or /" (x ). Points at which
Q or f (x) does not exist are called critical points of the second kind.
f'(x)
The critical point of the second kind x is the abscissa of the point of inflec* an
*
the intervals x
6
tion if I" (x) retains constant
signs in
.d

For the abscissa

f (x)

there

is

of the

no second

<x<

< <

+ 6,

where 6 is some positive number; provided these signs are

x?

Jc

And it is not a point of


inflection if the signs of f (x) are the
same in the above-indicated intervals.

opposite.

y~f(x)

Example 1. Determine the intervals of concavity and convexity and


also the points of inflection of the
Gaussian curve

Solution.

We

have

bx

a,

b,

and
Fig. 29

Equating the second

we

derivative y* to zero,

find the critical points of tHe

second kind

and

7=r

*o

T=-

These points divide the number scale


OO<A:< OO into three intervals:
1 (00, xj, II (*j, x 2 ), and III (x
+00). The signs of t/' will be, respec2
,

Fig. 31
-f(this is obvious if, for example, we take one point in each
lively, +,
of the intervals and substitute the corresponding values of x into y ) Therefore:
,

1)

the curve

curve

is

is

concave up when

concave down when

oo<
-=^

<

<x<

7= and
F2
==

F=

< x <-f oo;

2) the

V 2

The points

-=^

\V2

r=]

are

VeJ

points of inflection (Fig. 30).


It will be noted that due to the symmetry of the Gaussian curve about
the #-axis, it would be sufficient to investigate the sign of the concavity of
x
this curve on the semiaxis
+00 alone.

< <

__

Sec. 3]

Asymptotes

Example

Find the points

2.

of

93

inflection of the graph of the function

y=*/7+2.

Solution.

It

is

We

have:

obvious that y" does not vanish anywhere.


Equating to zero the denominator of the fraction on the

right of (1), we
find that y" does not exist for x
2. Since y"
for
2 and f/"<0 for
2, it follows that (
2,0) is the point of inflection (Fig. 31). The tangent at this point is parallel to the axis of ordinates, since the first derivative y'
is infinite at x
2.

x<

>

*>

Find the

intervals of concavity and the points


graphs of the following functions:
6x*
12x + 4.
891. y = x*
896. y
cosx.

of

inflection

of the

892.
893.

y = (x + l)\
y = -4r
o

897.

898.

X-\-

894.
895.

ff

899.

.
,

12

900.

\2x.

y=i/4x*

=
y=x
y=x

sin*.
2

In x.

= arc tanx
y = (l+x*)e*.
//

x.

Sec. 3. Asymptotes

1. Definition. If a point (#,/) is in continuous motion along a curve


f(x) in such a way that at least one of its coordinates approaches infinity
(and at the same time the distance of the point from some straight line tends
to zero), then this straight line is called an asymptote of the curve.
2. Vertical asymptotes. If there is a number a such that
y

Jim /(v)--=

then the straight line x


3

a
Inclined asymptotes.

is

If

00,

an asymptote (vertical asymptote).


there are limits
llm
X

->>

+ 00

and
lim [/(*)X-++ 00

Ml = *i.

then the straight line y


k l x+b l will be an asymptote
0, a right horizontal asymptote).
asymptote or, when ^

If

there are limits


llm

(a

right inclined

Extrema and the Geometric Applications

94

of a Derivative

[Ch. 3

and
Urn

k zx
y
b^ is an asymptote (a left inclined asymptote
horizontal asymptote). The graph of the function y
f(x)
(we assume the function is single-valued) cannot have more than one right
(inclined or horizontal) and more than one left (inclined or horizontal) asymptote.
Example 1. Find the asymptotes of the curve

then the straight


or,

when

fe

= 0,

line

a left

Solution. Equating the denominator

to zero,

we

get

two

vertical asyinp-

lotos-

x=
We

x=l.

and

seek the inclined asymptotes. For x

kl

lim

lim

x)

(//
*

+ oo

= lim

we obtain

~l,

lim

*-+o>
b =-

>

}^x z
x

y^2

=0,

\
\

-/
Fig. 32

hence, the straight line y

=x

the right asymptote. Similarly,

is

when*

oo,

we have
fc

fc

Hm

~=

1;

lim
AC->~

Thus, the
totes

is

Example

2.

y= -x

(Fig. 32). Testing a curve for asympconsideration the symmetry of the curve.
Find the asymptotes of the curve

left

simplified

asymptote
if

we take

Is

into

__

Sec. 3]

95

Asymptotes

Solution. Since

lim

t/

oo,

is a vertical
the straight line x
asymptote (lower). Let us now
curve only for the inclined right asymptote (since x>0).
We have:

k=
b

lim (y
*-*+ 00

lim

X++OD X
x)

lim
#->+<

test

the

1,

\nx

oo.

Hence, there is no inclined asymptote.


If a curve is represented by the parametric equations x
cp(0i */
^(0
then we first test to find out whether there are any values of the parameter /
for which one of the functions cp (t) or \|> (/) becomes infinite, while the other
remains finite. When (p(/ )=oo and ty(t ) = c, the curve has a horizontal
asymptote y c. When \j)(f ) = oo and (p(V ) = c, the curve has a vertical
asymptote x = c.
If <pU ) = *(*o)=<
and

lim

then the curve has an inclined asymptote y


kx+ b.
If the curve is
represented by a polar equation r
/(cp), then we can
find its asymptotes by the preceding rule after transforming the equation of
the curve to the parametric form by the formulas x
r cos cp
/((p) cos q>;
r sin <p = / (q>) sin (p.
y

Find the asymptotes of the following curves:


901.

11

903. y

^rr.

-,

908. u

=x

909. y

= e-

910. i/=

911.

905.

y^Y^^l.

912.

906.

y==-

913

907. </=

* ~~"

914. x

/;

j/

915. Find the asymptote of the hyperbolic

spiral r

of a Derivative

Extrema and the Geometric Applications

96

[Ch. 3]

Sec. 4. Graphing Functions by Characteristic Points


In constructing the graph of a function,

find its

first

domain

of definition

and then determine the behaviour of the function on the boundary of this
domain. It is also useful to note any peculiarities of the function (if there
are any), such as symmetry, periodicity, constancy of sign, monotonicity, etc.
Then find any points of discontinuity, bending points, points of inflection,
asymptotes, etc. These elements help to determine the general nature of the
graph of the function and to obtain a mathematically correct outline of it.
Example 1. Construct the graph of the function

The function exists everywhere except at the points x


1.
and therefore the graph is symmetric about the point

Solution, a)

The function
0(0,

This simplifies construction of the graph

0).

The

b)

lim

odd,

is

discontinuities

are

x=

hence, the straight lines

t/=oo;

and

jc

and

1;

lim

and

oo

J/=

V-M + O
are vertical asymptotes of the

#=1

X->--10
graph.
c)

We

seek inclined asymptotes, and find

,=

lim
X -> +

bl

oo

lim

-x

= 0,

oo,

#->-t-oo

thus, there is no right asymptote. From the symmetry of the curve it follows
that there is no left-hand asymptote either.
d) We find the critical points of the first and second kinds, that is,
points at which the first (or, respectively, the second) derivative of the given
function vanishes or does not exist.

We

have:

x=l,

The

derivatives y' and \f are nonexistent only at


that is, only at
points where the function y itself does not exist; and so the critical points
are only those at which y' and y" vanish.
From (1) and (2) it follows that

when x=
when x =

y'=Q
r/"

y' retains a

Thus,

(-V3,
intervals

l),
(

(1,

00,

V$\
and

x=

3.

in each of the intervals


constant_ sign

1),

3),

(l,
3,

V$)

and (V~3

1),

(1,

0),

+00),
(0,

1),

and
(1,

/
3)

00,

in

each

and

(3,

J/T),
of

+00).

the

To determine the signs of y' (or, respectively, y") in each of the indicated
intervals, it is sufficient to determine the sign of y' (or y") at some one point
of each of these intervals.

Sec

Graphing Functions by Characteristic Points

4]

97

It is convenient to tabulate the results of such an


investigation (Table I),
calculating also the ordinates of the characteristic points of the graph of the
function. It will be noted that due to the oddness of the function r/, it is
enough to calculate only for Jc^O; the left-hand half of the graph is constructed by the principle of odd symmetry.

Table I

e)

Usin^ the results

function (Fig

of

the investigation,

33).

-/

Fig. 33

4-1900

we

construct

the

graph

of

the

Extrema and the Geometric Applications

Example

2.

of

a Derivative

[Ch. 3]

Graph the function


In x

x
Solution, a) The domain of definition of the function is 0<x<-f-oo.
b) There are no discontinuities in the domain of definition, but as
0) of the domain of definition we have
approach the boundary point (#

=
limw = lim

JL? =

oo

X-*0

JC->

we

Hence, the straight line jc


(ordinate axis) is a vertical asymptote.
c) We seek the right asymptote (there is no left asymptote, since x cannot tend to
oo ):

k=

lim
X<-++

The

00

-^
X

lim #
x->+<

0;

= 0.

right asymptote is the axis of abscissas:


find the critical points; and have
d)

j/

= 0.

We

Inx

y
3

y'

and

y" exist at all

of the

points

y' = Q

domain

when ln*=l,

that

definition of the function and

of
is,

when x =

<?;

(/'=0

We

when

form a table,, including


io the characteristic points

Inx^y,

the
it

is

that

is,

when x~e*l*.

characteristic points (Table 11). In addition


useful to find the points of intersection of

34

1
the curve with the coordinate axes. Putting /
0, we find *
(the point
of intersection of the curve with the axis of abscissas); the curve does not
intersect the axis of ordinates
the
e) Utilizing the results of investigation, we construct the graph of
lunction (Fig. 34).

-h

CM

CO

CM

'

"
>

non

> C
a c.u

-~

Extrema and the Geometric Applications

100

its

of

[Ch. 3

a Derivative

Graph the following functions and determine for each function


domain of definition, discontinuities, extremal points, inter-

vals of increase and decrease, points of inflection of


the direction of concavity, and also the asymptotes.
916. y

=x

3x*.

"

918.

u = (x

919.

y-

921.
922.
923.

924.
925.

=
=
y=
y=
=
(/

(/

</

926. y==

928.
929.

930.

__

=,-

,6

3*'+!
932.
933.
934.
935.

936.

938.

y = 2x + 2-3'l/(xl-

963

'

#=

its

graph,

Sec.

an Arc. Curvature

Differential of

/>]

964. y

sin

-7-

y=

976.

4 /

978. ,,=

967.

979. ^

=
y = *-l-sin*.
y = arc sin (1
/F).

970.
971.
972.

and
973.

974.

953.

982
tan

983.

984

<>arcsin

f/

= lnA:-arc tan*.
In cos x.
y = cos^
= arc tan(ln
= arc sin In (*' 4-1).
,/

,/

985.

je).

y==x

-^-

987. y

= lnsin*.

A good

exercise

ples 826-848.
Construct the

is

to

>

Af

i/

= garcun*
= n sin x

,,

A;.

= x arc tan - when


when * = 0.
y=
=
2 arc cot*.
= + arc tan*.

975. y

arc cosh

965. */= sin*- sin 2*.


966. (/
COS*-COS2*.
968.

101

graph the functions indicated


following functions

the

graphs of

parainelrically.
l
2/.
988. x=--t*
//----/
2t,
sin/ (a>0).
989. x=--acob*/,
te~
990. jc
/e',
y
fl
/ 4-g991. x
i/=2/ + ea (cosh /
a (sinh/
992. x
/), i/

in

Fxam-

represented

y^a
=

=
=
=

I)

(a>0).

Sec. 5. Differential of an Arc. Curvature

1. Differential of an arc. The differential of an arc s of


represented by an equation in Cartesian coordinates x and y
the formula

dshere,

the equation of the curve

if

a)

J/~(d*)

//

= /(*),

b)* = /,Urt.

then ds

then

= 0,

ds^-

d)

^(*,

f/)

of the

',

form

ds

then

(dy)

= q>(0, y = +(0,

c)

is

ds-

V F* + F

then

'

2
/;

V F'

a
is

plane curve
expressed by

Extrema and the Geometric Applications

102

of

a Derivative

[Ch. 3

Denoting by a the angle formed by the tangent (in the direction


increasing arc of the curve s) with the positive ^-direction, we get

of

dx

cos

a = -3-

ds

dy
-r

sina

ds

In polar coordinates,

Denoting by p the angle between the radius vector of the point


curve and the tangent to the curve at this point, we have
a
008 P

sin

= dr

of the

'

2. Curvature of a curve. The curvature K of a curve at one of its


is
the limit of the ratio of the angle between the positive direcpoints
and N of the curve (angle of contintions of the tangents at the points

gence) to the length of

the

M
^MN^\s

arc

K=

iim

As
\\hore

point

is

the angle between

The radius
i.

circle

line (/C

the

AS

when

.V

(Fig.

35),

that

is,

= ^,

positive

rfs

directions

of

the

tangent jt the

arid the .v-axis.

curvature,

The

Au

= 0)

of

curvature

is

the reciprocal

of

radius

the

the absolute value of the

e.,

K=

where a

is

the

are lines of constant curvature.

of

circle)

and the straight

Sec

an Arc. Curvature

Differential of

5]

103

We have the following formulas for computing the curvature in rectangular coordinates (accurate to within the sign):
1) if the curve is given by an equation explicitly, y
f(x), then

2)

if

the curve

given by an equation implicitly, F(x,

is

0,

then

represented by equations in parametric form,

*=

Fxx.

Fx

Fxy
'

Flx

y)

F'yy

'y

Fv

F'

/j

3)
/

if

(/),

\j)

the curve

is

then
f

y
y

*,'"

where
dx

dy
-

'

~~dt*
In polar coordinates,

when the curve

is

given by the

equation

/(q)),

we have
r

+ 2r'

rr"

where
,

= dr

d"r

and

dcp

dtp

3. Circle of curvature. The circle of curvature (or osculating circle) of a


is the
curve at the point
limiting position of a circle drawn through
v
and two other points of the curve, P and Q, as P
and Q
M.
The radius of the circle of curvature is equal to the radius of curvature,
and the centre of the circle of curvature (the centre of curvature) lies on the
in the direction of concavity of
normal to the curve drawn at the point

>

the curve.

The coordinates X and Y


computed from the formulas

X=x-

the centre of

of

L,,

curvature

-jf-r

of

the

curve

are

{j

The evolute

of a

curve

is

the locus

of

the

centres

of

curvature

of

the

curve.
If

ture

in the

formulas

for

we regard X and Y

determining the coordinates


as the current coordinates

of

of the centre of

curvapoint of the evoof


the evolute \vith

then these formulas yield parametric equations


parameter x or y (or /, if the curve itself is represented by equations
parametric form)
z
Example 1. Find the equation of the evolute of the parabola // x

lute,

in

Extrema and the Geometric Applications

104

Solution.

X=

4* 8

Y--

+ 6*

involute

curve

of

curve

is

the parameter x,

Eliminating

the equation of the evolute in explicit form,

The

[Ch. 3

a Derivative

of

o'

we

+ ^lT")

which the given curve

for

find

an

is

evolute.

MC

The normal

CC

length of the arc

involute P 2

of the

of the evolute

is

is

the evolute

to

tangent

equal to the corresponding

P,;

the

increment

the
radius of curvature CC,
in
AfC;
M,C,
that is why the involute P 2 is also called the
evolvent of the curve P, obtained by unwinding
a taut thread wound onto P, (Fig. 36). To each
evolute there corresponds an infinitude of invowhich are related to different initial
lutes,
lengths of thread.
4. Vertices of a curve. The vertex of a curve
of the curve at which the curvature
is a point
has a maximum or a minimum. To determine
the vertices of a curve, we form the expression
of the curvature K and find its extremal points.
In place of the curvature K we can take the

radius

curvature

of

points

Since

//
J

= sinh
a

a cosh

,
2

and, hence,

*=
of

is

X
.

0).

cosh
a

We

follows

it

that

tf

rf/?

have -j- = sinh2


dx
a

Equating
M
6

Q,

the point

we

the

we

get sinh 2

Computing

(0,

get

we

whence

0,

-r-y-

,=

A:

cosh2

catenary.

The vertex

a
a
curvature (or of

of

the

catenary

the

sole

putting

into

find

and

the second derivative

a
point of the radius of

minimum

the

curvature) of

thus,

and J(/"

= acosh

to zero,

=Q

point *

value x

the

/?

>

(a

I")

ax

it

extremal

the derivative -j
critical

its

the computations are simpler in this case.


2. Find the vertex of the catenary

a cosh

Example

36

Solution.

if

and seek

7-7^

> 0.

Therefore,

the

maximum

f/

= acosh

is,

a).

Find the differential of the arc, and also the cosine and sine
of the angle formed, with the positive ^-direction, by the tangent
to each of the following curves:
993. *

2
*/

=a

(circle).

+ ^-=l (ellipse).
y* = 2px (parabola).

994. ~2

995

Sec. 5]

996. x 2 / 8

997.
998.
999.

-f

2/

t/

Differential of an Arc. Curvature

=a

2 /'

(astroid).

105

y = acosh
(catenary).
x = a(ts\nt)\ y = a(lcost) (cycloid).
x = acos*t, y = asm*t (astroid).

Find the differential of the arc, and also the cosine or sine
of the angle formed by the radius vector and the tangent to each
of the following curves:
1000. r^atp (spiral of Archimedes).
1001.
1002.

= (hyperbolic spiral).
r =
asec*-|- (parabola).
r

1003. r

= acos*-

(cardioid).

1004. r=za.v (logarithmic spiral).


2
a
1005. r
a cos2q) (lemniscate).
Compute the curvature of the given

points:
1006. y
1007. x*

= x* 4x* ISA'
+ xy + y* = 3 at
2

+
=
=

1008.

=1

at the

coordinate origin.

the point

the vertices

at

curves at the indicated

(1,

1).

and 5(0,

0)

(a,

b).

*' at the point (1,


1009. *
/*, f/
1).
2
2
2a eos2q> at the vertices cp
1010. r
2
1011. At what point of the parabola t/

and

= 8x

<p

is

= n.
the curvature

equal to 0.12S?
1

1012. Find the vertex of the curve y^-e*


Find the radii of curvature (at any point) of the given
x* (cubic parabola).
1013. y
.

lines:

1014.

5 + S =1

(ellipse).

= -!^.
* = acos
y = as\n*t (astroid).
= a(cosM sin 0; y = a(s\nt

1015. *
1016.
1017.
circle).

/;

A:

/?osO

curvature of

the

y* = 2px.

1021. Prove

y = acosh

of

1018. r
ae k v (logarithmic spiral).
1019. r- a(l -f-coscp) (cardioid).
1020. Find the least value of the radius of

parabola

involute

that

the

is equal to a

radius of

segment

curvature

coordinates of the centre of


Compute
at
the
indicated points:
curves
given
the

of

the

catenary

of the normal.

curvature of

the

Extrema and the Geometric Applications

106

1022. xy=l at the point (1, 1).


x* at the point (a, a).
1023. ay*
Write the equations of the circles
curves at the indicated points:

of

a Derivative

[Ch. 3

=
=

x*
1024. y
Gjc+10 at the point
1025. y
e* at the point (0, 1).
Find the evolutes of the curves:
1026. y*
2px (parabola).

1027.

=
J + g=l

of

(3,

curvature of the given


1).

(ellipse).

1028. Prove that the evolute of the cycloid

x~a(t
is

sin/),

y = a(l

cost)

a displaced cycloid.

1029. Prove that the evolute of the logarithmic spiral


r
is

also a

1030.

logarithmic spiral with the same pole.


that the curve (the involute of a

Show
x

is

= a (cos +
/

sin

the involute of the circle

/),

;c

#=-a(sin

= acob/;

//

circle)

/cos

= asm/.

/)

Chapter IV

INDEFINITE INTEGRALS

Sec.

Direct Integration

1.

1. Basic rules of integration.


1) If F' (A-)--- MA), then

where C
2)
3)

4)

an arbitrary constant.
f f (x) dx, where A

is

^Af(x)dx=-A
[ft

U'H:

f f

If

f2

v )l

dv

( fj

(*)/*

and

(x)dx-~F(\-) -f-C

is

f f2

/-cf

a constant quantity.

O)

dx.

(v),

then

In particular,

ax

2. Table

II.

of standard

dA'

f*

III

,*r

IV

==

-r-:

\
2
s
J X -f- a
dx
C
\
)

-=2

;5

a*

A'

dx
r

V.

integrals.

dx

\:

arctan

~-,T-

2a

a+v

|-C

v:

arc cot

+C

(a

-f-C

(a*Q).

yS-

0).

(a^O).

VI.

VII.

(>0); fc x d* =

+C
'

(a

^ 0).
'

__

108

VIII.

IX.

\
f

cosx +

sinxdA;=

[Ch. 4

Indefinite Integrals

C.

cosxdx=sinx + C.
COS 2

XI.

J
XII

--=
x
sin

cotx+C.

'

tan

smx

XIII.

XIV.

H-C = In

cosec x

cot x |-f-C.

cosx
f

sinhxdx=coshx-|-C.

XV.
dx

XVL
XVIL
Example
f (ax

1.

+ bx + c) dx=

Applying the basic rules


tion,

1,

2,

3 and the formulas of integra-

find the following integrals:

5dVd*.

1031.

1040.

1032-5(6^ + 8^3)^.
1

033.

(x

r
1

034.

J (a

1035.

-i

a) (x 4 b) dx.

+ bx

)* clx.

yZpxdx.

(V

1042
' U4 ^'

-- dxv

V x)

j/51

1043.

1036.
'~ n

1037.
J

(nx)

"

1045

^x
'

dx.

1046

'

1047

'

1038.

1039.

^+ljc-/x-dx.

^4+^

'

,,

'

Sec

Direct Integration

1]

tan *dx;

1048*. a)

109

1049. a)

Jtanh *d*.

b)

b)

1050.

Jcoth'jtdx.

$3Vdx.

differential. Rule 4 considerably


sign
expands the table of standard integrals: by virtue of this rule the table of
integrals holds true irrespective of whether the variable of integration is an
independent variable or a differentiate function.

3. Integration under

Example

of the

the

2.

where we put u

Exa m p.e

We

5*

We

took advantage of Rule 4 and tabular integral

xdx

d(x

1.

2
1

3.

implied u

Example

2.

4.

jc

and use was made

( x 2 e xl}

dx-^~

( e*' d

3
(jc )

of

Rule

and tabular

i-e^ + C by

integral V.

virtue of Rule 4 and

tabular integral VII.


In examples 2, 3, and
form before making use of

of

we reduced the given

/ (<P (*)) <p' (*)

This type

a tabular

dx\

transformation

is

Some common transformations


ples 2 and
a)

3,

I (u)

the following

du

where a

(p (x).

called integration under the differential sign.

which were used

of differentials,

b)

(a

0);

b)

xdx =

Using the basic rules and formulas

,053.

to

in

Exam-

are:

dx=^d(ax-\

tegrals:

integral

integral:

of

^d(x

and so on.

integration,

find

the following in-

112

Indefinite Integrals

1140. f-^f-.
sinh x

1143.

H41. f-*_.
J cosh x

1144.

1142

/f

.
'

Find the indefinite

< 1

Ad
46.

,,47.

integrals:

(x 1/5=1? dx.
i

v*

(cothxdx.
J

J sinh x cosh x

1145.

[tanhxdx.

r*

[C/i.

1163.

A \f

J?=CT1

f-^-.
]cos@

*J

" M -j

f^d,.

1165

1148. \xe-**dx.

1151.

,, 67 .

Jl_!d*.

<ioo

--

1168.

i/^c'
V e

,-

jcdx

JV^L
l

J
I

1169.

~ cc^s

sinx
sin

A: -f-

Jx + cos*

1
i

ftan/JC

1166.

U50.

djt

'

A*

cos x

dx.
2

51

f
v

"-^)
v ^

sin SA:

1170.

^tan^-2
>

'

ff 2 -|--^- )-^-.
2x*+l/ 2x*+l
^

,)

1157.

^a

ilax

cosxdx.

7^Ti

1161.

"

M7.:.

!!!_!-.

+ !'

to

fl

_t ^

H76.

K4-tan'x

(<

jsin -Jdx.
(

.rfy.

1174.

Jlan'aArdx

3<

1173.

dAt>

f-i^=.
1160.

I179
1172.

j -j^*
.

ii77. f _

si
J sinajtcosax'

Integration by Substitution

Sec. 2]

113

l/T=3

Sec. 2. Integration by Substitution

1. Change of variable in an indefinite integral. Putting

where

is

new variable and

cp

is

continuously differentiable function, we

will have:

The attempt is made to choose the function q> in such


side of (1) becomes more convenient for integration.
Example 1. Find

Solution.

It

is

natural to put

V~x

1,

whence

A-

way

=/ 2

-}- 1

that

the

right

and dx = 2tdt.

Hencu,

Sometimes substitutions

of the

form

are used.

Suppose we succeeded

in

transforming

the integrand f(x)dx to the form

f(x)dx=g(u)du where u = q>(jc).


t

114

If

[g(u)du

known, that

is

Indefinite Integrals

is,

[Ch. 4

then

we have already made use oi this method in Sec. 1,3.


Examples 2, 3, 4 (Sec. 1) may be solved as follows:

Actually,

Example

2.

= 5#

cfw

2;

Example

3.

= x*;

Example

4.

jc

s
;

du

du

= 5c(x;

_
-~

'

+c_

dx\

xdx = -

2xdx;

= 3x

-du.

dx

x dx

2. Trigonometric
1)

If

~a sin

2)

/;

If

substitutions.

an integral contains the radical ]fa z

x2

the usual thin^

is

to put

whence

an

integral

the

contains

radical

V*

a2

we

put

xawct,

whence
/^x

3)

If

a2

= a tan

an integral contains the radical V'V

^.

+a

2
,

we put* = atan/; whence

It should be noted that trigonometric substitutions


do not always turn
out to be advantageous.
It is sometimes more convenient to make use of hyperbolic substitutions,
which are similar to trigonometric substitutions (see Example 1209).

For more details about trigonometric


Sec. 9.

Example

5.

Find

and hyperbolic substitutions,

dx.

see

Integration by Substitution

Sec. 2]

xlant.

Solution. Put

y~x*+l *

f V~ian 2

1?

dt

"~ f

= In

sin

tan 2

J
2
/

tan

+ sec

r-r

cos 2

cos'

dt

cos !
si "

sin

J
/

f sec

dt

~~ f

cos

dx=

Therefore,

1J5

sin 2

4- cos2

/-cos/

-J
1

<

M-~

+ C = In

cos 2

^/

^
J cos

tan

/ -{-

cos/

~~

2
J sin

V\

-j-tan

tan<
find the following

1191. Applying the indicated substitutions,


integrals:

c)

f
e)

COS

A'

d*

Applying suitable substitutions,


1192.

S*(2x+5)"djc.
1

1193.

('

1194.

+ *..d*.

the following integrals:

find

>97. n-

csinA )"

1198.

l+^A-

f-

J!

JxK2t+l

,, 99<

**

1195. r

Applying trigonometric substitutions,

find

the

tegrals:

,201.

('-=*.
*'
K

,202.

1203.

-=.

1204*.

f
J

iZEl'dx.
^

following

in-

__

U6

1205.

\Ch. 4

Indefinite Integrals

1206*.

(f^+idx
x

f
ff
2
J x y 4

__
x2

1207.
1208. Evaluate the integral

by means

dx

/*(!-*)

x=sin

of the substitution

/.

1209. Find

by applying the hyperbolic substitution x = as\n\\t.


Solution.

We have:

]Aa

+x =
2

]/~a

+a

suih 2

/=a cosh

and

dx=a cosh

/d/.

Whence
\

ya

-\-x

dx=

a cosh t-a cosh

f ctf

~~~2~

Since
__ x

and

we

finally get

where C 1=^C

In a

is

new

arbitrary constant.

1210. Find
C

putting x

= a cosh/.

Sec. 3. Integration by Parts

A formula for integration


on
tiable functions, then
\

by

parts.

udv = uv

If

= <p(*)

vdu.

and u

= i|)(*)are

differen-

Sec. 3\

Integration by Parts

Example

Find

1.

In*, dv

Putting u

H7

x In xdx.

dx

xdx we have da

x*

Whence

~~9

x*

dx

Sometimes, to reduce a given integral to tabular form, one has to apply the
fcrmula of integration by parts several times. In certain cases, integration
by parts yields an equation from which the desired integral is determined.
Example 2. Find
\

We
V

e* cos x dx.

have

e* cos x dx

xd

e* sin x

(sin x)

x sin

dx

AT

x d
(cos x)

e* sin x

sin

-\-

x cos x

e* cos x dx.

Hence,
e* cos x dx

x sm
.v-j-e^cos x

V e

x cos x
dx,

whence
cos v dx --

(sin

x -f- cos

Applying the formula of integration by

.v)

-f C.

parts,

find

the following

integrals:

1211.
^

1212.

\nxdx.

Jarclanjcdx.

1213.

Jarcsin

1214.

A-rfjc.

Jjcsiiucr/.v.

1215.

Jjccos3A'Jx.

1216. Urfjc.

1219*.

1220*.

x sin x cos x d\
a

1222*
$

1223.

(jt

+5x+6)cos

x*

\nxdx.

1224.

Jln'xd*.

1225.

(^djc.

f^d*.
K A

Jx-2-*rfx.

1218**.

1226.

1217.

1221.

JjV'd*.

1227.

1228.

Jjcarctanjcdjc.

Jjcarcsmxdjc.

(x ^2A' + 5)^*dA:.

'-

1229.

1230

'

\n (x

+ V T~x*) dx.

118

Indefinite Integrals

1231

1234

1232.
1233.

[C/i.

eax *

'

JisFr**'235.

mb * dx

$sin(lnx)dx.

Je*sinxdx.
$3*cosjtdx.

Applying various methods,

find the following integrals:

1236. (x*e~* dx.

1246.

V\-x

1237.

\e
2

1238.
1239.

(x

*dx.

1247.

-2x + 3)\nxdx.

1248.

jttan 2*d;t.

^x\n~dx.

1249.

1240.

f^dx.

1250**,

1241.

fllL^d*.

1251*.

cos (In x)dx.

1242.

1243.
1

244.

*n/ir

1245.

{
j

arctan3jcdA:.

jc

(arc tan

( (arc sin

fare

sin

^2

2
A:)
2

jc)

a2)2

2
:

-|

'

1252*.

dx.

1253*.

dx.

1//1

^
j

254*.

-4-

-4^-

x dx.
.

^.

Sec. 4. Standard Integrals Containing a Quadratic Trinomial

1. Integrals of the form


f

mx + n

2-7-7

7dx.

The

principal calculation procedure


the form

ax z

is

to reduce

+ bx + c = a(x-}-k)

the

quadratic trinomial to

(1)

-{-l,

/ are
constants. To perform the transformations in (1), it is
best to take the perfect square out of the quadratic trinomial. The following substitution may also be used:

where k and

If m=0, then, reducing the


quadratic trinomial to the form
the tabular integrals III or IV (see Table).

(1),

we

get

Sec

Standard Integrals Containing a Quadratic Trinomial

4]

Example

119-

1.

dx

-(-*)

H-

g \2
5

-7

Qi

arc
=-o*~7=
-o
2

tan

(-TJ+S '*?

"S?+c.
If m&Q, then from
the numerator
out of the quadratic trinomial

the derivative

-+.

f
J

we can take

a.v

+^

and thus we arrive at the integral discussed above.

Example

f
J

->
;5=^n

2.

,v
d
-

f"

^J

'-x-i

IV
2.

Integrals of the

form

2x-l

are similar to those analyzed above.


0, and VI, if a
lar integral V, if a

>

Example

The

The methods

integral

of calculation

finally reduced

is

< 0.

3.

dx

Example

d*.

f*

dx

4.

^+3
^^2 f
*"
yV+2x + 2
J ^ ^ + 2x4-2

JC
^^

4jt

to

tabu-

J20

[Ch. 4

Indefinite Integrals

3.

of

Integrals

the

form

f
J (mx

+ n)

V^ax'

+ bx + c

By means

of the in-

verse substitution
i

mx+n

these integrals are reduced to integrals of the form

Example

5.

2.

Find
dx

We

Solution.

put

whence

4.

Integrals of the form

]T ax*-\-bx

+ cdv.

By taking

the perfect square

out of the quadratic trinomial, the given integral is reduced


following two basic integrals (sec examples 1252 and 1253):
1)

('

a*

A'

dx

= 4 fa^x* + ^ arc sin


^
2
(a

2)

>

to

+ C;
c/

0);

Vl?

Example

6.

sin

Find the following

1256.

integrals:

S'
'',c
00

xdx

^7^+

one

of the

Integration of Rational Functions

Sec. 5]

oKn
\o\J.

---

<

c r\

<

121

dx

1270.
v2

dy

O *7 1

*~

1272.

'

2* 2

V~2 {-3*
d*
r
I

'

f*
t

1273.

X
1

1-7/1

J
f

/"*l/"o

f~=.

,H4.
1265.

1266.

>1J
-

1267.

xdx

V %

."~

-,-^
/5, -2,

dx.

1277
197 n
1278

dK.

f-1

JYKI

-v

279

x
cJJt

sin

dA

1'-^=.

1268.

t'

T^PTTT^TlTInjfdi

i'

J x

^ _ ilnA._ !n
1

Sec. 5. Integration of Rational Functions

t. The method of undetermined coefficients. Integration of a rational


function, after taking out the whole part, reduces to integration of the proper
rational fraction

where P
rator

and Q

(x)

is

(x)

(A-) are integral


polynomials, and the degree
lower than that of the denominator Q (A-).

If

Q(jr)

= (*

a)*.

of

the

nume-

.(A'-/)\

/ are real
where a,
distinct roots of the polynomial Q (x), and a, ....
.,
K are natural numbers (root multiplicities), then decomposition of (1) into
.

partial fractions

is

justified:

^
calculate the undetermined coefficients A lt A 2t ..., both sides of the
identity (2) are reduced to an integral form, and then the coefficients of
like powers of the variable x are equated (llrst method). These coefficients may likewise be determined by putting [in equation (2) or in an equivalent equation] x equal to suitably chosen numbers (second method).

To

122

Indefinite Integrals

Example

Find

1.

xdx
(*-!)(* +

We

Solution.

[C/t.

"

1)

have:

Whence
t

a) F/rsf

method of determining the

We

coefficients.

the form x^(A-{- B^ x 2 -{-(2A-{- B 2 )x-\-(A


ents of identical powers of x we get:

B2

(x\).

(3)

rewrite identity (3) in


Equating the coeffici-

Whence

=i

= _i

Second method of determining the

b)
<3),

we

,,4.

1=4-4,
Putting x

we

1,

i.e.,

'/ 4

x=\

in identity

get:
1

Further, putting *

Putting

coefficients.

will have:

= 0,

we

B2 =

i.e.,

-2,

/2.

will have:

Hence,

TJ
*

~~
Av

T
t

4-1

\
J Ax

AT

Example

Solution.

2.

Find

We

have:
""
2

x(x
tind
1

When

=A

(*

I)

I)

+ Bx (x

1)

1(JC

1)*

+ Cx.

(4)

advisable to combine the two methods


of determining coefficients. Applying the second method, we put *
in
identity (4). We get 1=4. Then, putting jc=l, we get 1=C. Further, app2
lying the first method, we equate the coefficients of x in identity (4), and
get-

Hence,

solving this example

it

is

= 4 + 0,
4=

1.

fl-=

i.e.,
l,

B=
and

1.

C=l.

In t egration

Sec. 5]

Rational Functions

of

123

Consequently,

/=

f dx

dx

'

the polynomial

If

dx

r- = ln
jc17+ J (*l)

f*

\
J X

'

JC

'

A-

ib of

r-f C.

'

has complex roots a

(x)

In

JC
'

multiplicity k, then

partial fractions of the form


(5)

will enter into the

expansion

Here,

(2).

Bk are undetermined coeflicients which are determined


and A lt B lt
.., A k
by the methods given above For k~\, the fraction (5) is integrated direct,

ly;

k>\,

for

use

is

made

of the reduction

~]

Example

and make the substitution


3.

method;

z
sable to represent the quadratic trinomial x

here,

it

px~{-q in the form

A--J-

z.

j2

is

advi-

first

x-\-~

-f-

Find

Solution. Since
A

then, putting x -\-2---z,


r==

*\._ dz=z

jrc

wo

2
-|

4x

got
2

_j_^

tan ?

Hit i!ini

2. The Ostrogradsky method.

is

(A:)

derivative Q'

and

the greatest

-- -- arc

If

P(x)

where Q,

5-

z=

(A)

has multiple roots, then

"-

A'(v)

common

tan

(.Y)

divisor of the polynomial

(6)

(x)

and

its

(A-);

undetermined coefficients, whose degrees


by unity than those of Q, (A-) and Q 2 (x).
The undetermined coeflicients of the polynomials X (x) and Y (x) are
computed by differentiating the identity (6).
Example 4. Find
(A-)

(x)

are polynomials with

arc, respectively,

less

C
}

dx

U'-

124

[Ch. 4

Indefinite Integrals

Solution.

Ax z + Bx + C

dx

we

Differentiating this identity,

?Dx

-\-Ex

+ F ax
.

get

Dx z +Ex

\=(2Ax-\-B)(x*

3x*(Ax* + Bx

1)

+ C) + (Dx* + Ex + F)(x*

Equating the coefficients of the respective degrees of x,

4 = 0;

D = 0; E

D + 3C = 0;

2fl--=0;

we

+ 24 = 0;

i).

have:

will

+ F==

1;

whence

C = 0; D = 0;
~;
O

5=
and, consequently,

C
9

^ ~
_

To compute the
r

-^

into partial

1)

on the

integral

__ 2

3^ -l

(x

= 0;
dx

3 J x8

right of (7),

F= -4O
1

we decompose

the

fraction

fractions:
1

x8
lhat

is,
1

Putting

#=1, we

= L (x + + 1) + MX (x
2

A:

1)

+N

(A:

(8)

1).

L=-.

get

<j

Equating the coefficients of identical degrees of x on the


,of

we

(8),

find

right

and

or

Therefore,

dx

\yS
J *

^\V

o J x

dx

r
\_

Q\

J j

~~
=ll

]X

11-1
'

and

^^^^^(J^\\ + ^ ln \ x
Find the following

1280

'

^ +^^

integrals:
'

1282

'

mian

^^+ C

'

left

Integrating Certain Irrational Functions

Sec. 6]

1284
' **

1285

'

5 *' + 2
* - 5* + 4* dx*

12Q1
29J
'

f
j (Ii_4jt

125

dx

+ 3)<J

dx
c
J *(*

+ !)*

-~

d*
C

1296.

-r-g^gdx.
^"*
"
I

R V2

C*

Av

C/

rt

^4

.--.

'

1288.

-290.

.Tjpfc

l291 '

J^Sf*.

'292.

J^d*.

1299.
.300.

Applying Ostrogradsky's method,

Applying

l306

different procedures, find

integrals:

the integrals:

'

I3 7 '

l309

find the following

l312

rf *-

'

Sec. 6. Intagrating Certain Irrational Functions

1.

Integrals of the

f.rm
i

/? <s

a rational function and p lt q v p a

q z are whole numbers.

12J

[Ch. 4

Indefinite Integrals

Integrals of form (1) are found by the substitution

where n is the least


Example 1. Find

common

multiple of the numbers

</,,

q2

...

*
dx
a
I

Solution.

The substitution

____dx

2,x

f 2z'dz

J V"5jc=T-- J/2?=l""j

-^ =2

leads

to

an integral

of the

form

r^i
"
2

=2
= (1+

/23T-i)

Find the integrals:


13 15.

f-^^djc.
/*-!
A

1316.

('J^Zdjc.

1321.

J .v-f-2

dx

1322.

7,7==.

1U7.
.

^.---

-. ---('-.
J

/v+l

I-

--^7=
A
(jv

(o

-A)

1-

1323.

v+1

)/(v+l)
1324.

1325.

,320.

2.

Integrals of the form


"

where P n
Put

(x)

is

ai*

- dx,

t,

(2)

+c

a polynomial of degree n

dx

a polynomial of degree (n
with undetermined coeffi1)
number.
The coefficients of the polynomial Q n -i(x) and the number K are found

where Q n _i(x)
cients and A, is

is

by differentiating identity

(3).

Integrating Certain Irrational Functions

Sec. 6]

Example

2.

Whence

Multiplying by V^*
we obtain

+4

and equating the coefficients

of identical

degrees oi

A-,

D-0;

X=_

Hence,
-

3. Integrals

of the

form
dx

i;

a)

V ax

are reduced to integrals of the form (2) by the

They

substitution:

Find the integrals:


1326

1327.

j__^_.
(-f^dx.

1329.

1330.

-^
fv

-^^rfjc.

1331.
FA'

Vl-i

4.

X+l

Integrals of the binomial differentials

(5)

where m, n and p are rational numbers.


Chebyshev's conditions. The integral (5) can be expressed in terms of a
finite combination of elementary functions only in the following three cases:
1) if p is a whole number;
"

2)

=z

s
t

is

if

where

m
3)

if

is

whole number. Here, we make the substitution a + bx n =

the denominator of the fraction p;

+p

is

whole number. Here, use

is

made

of the substitution

128

(Ch. 4

Indefinite Integrals

Example

3.

Find

m=-^;n = -r ;p = -^;

x.

Solution. Here,

we have here Case 2


The substitution

yields *

= (z

I)

\Y dz

(z*

Find the

+ J/T

f (2

1335

),fe=J2ef_

dx

'

^-

133G
0. f
2

*
f_-

,334.

Hence,

1333. r

=2.

integrals:
*

1332.

Therefore,

= 12
where

integrability.

dx=\2z*

-1+1
-T

.v:

i'

337.{

(2

+
,

3
JC

r*

1+

Sec. 7. Integrating Trigonometric Functions

1. Integrals of the form


:

where
1)

m
If

and n are

an odd positive number, then we put

is
'

We

do the same
Example 1.

if

0)

integers.

m^2/j+l
,

'm..

s l(]Zk

( sin 10

x cos " xd ( cos

is

^)

cos 2 x) k cos" xd (cos

( 1

an odd positive number.

x cos 8 * dx

f sin 10 x (1
sin

11

sin

sin x)
13
.v

(sin *)

A:).

Sec.

7]

__

Integrating Trigonometric Functions

the integrand
2) If m and n are even positive numbers, then
formed by means of the formulas
sin

*= y (1

sin JCGOS

Example

=
=

If

3x

3-

cos 6*
dx

cos 12*
n
^

3* dx

--

f /

J \
x

m=

sin

-- -

sin 6x1
p
\

^>

3)

f cos

2.

cos

cos 2*),

= sin
.

,
2

x=

*= y (1 + cos 2*),

3* sin 3*) 2

sin

3x dx

'

f
\

GA:

Zc

sin

(sin 6A:

c \
dx
cos 6^
,

2C
6x cos fi
o^) ax
v

sin 12AC

and

[i

n=

v are integral negative

numbers

of

parity, then

dx
cos

In particular, the following integrals reduce to this case:

Example

3.

-^-= JC sec xd (tan *)= Jf (l+tan*x)


= tan x-p--=-o tan '* + C.

J COS X

d (tan

x)

jc

_1 r ""-

tan 2

5-1900

(I) is trans-

sin 2*.

f (cos
1

129

identical

130

form

4) Integrals of the

(or, respectively, cot*

Example
tan

f tan

5.

P.

= cosec* x

tan*xdx=

\
.

tv

(or

cot w xdx),

where

is

an

in-

1).

l)dx=

6.

tan*xdx =

V tan x (sec*x

tan* x

(sec*x

w xdx

number, are evaluated by the formula

tegral positive

[Ch. 4

Indefinite Integrals

-5

1)

dx=-^-^
~

tanx + x+C.
,

3
J
5) In the general case, integrals 7 m>n of the form (1) are evaluated by
means of reduction formulas that are usually derived by integration by parts.
5

Example

J cos* x

sin

cos

dx

A;

C cos x

2^^"j -^

.
,

cos x

sin

Find the integrals:


1338.
1339.
1

340.

1341.

1352.

Jcos'xdx.

C
\

sin
sin

sin*

-jr

cos

1346.

'T

-~-

dx.

sm*xcos*xdx.
sin

1
A:

cos x dx.

Jcos'Sxdx.

1353.

1343.

345.

-2

x cos x dx.

1342.

COS

sin

Jsin'xdx.

1344.

{;

1354.
J

1355.

JsecMxdx.

1356.

Jtan

1357.

5jcdA:.

jjcot'*d*.

1358.
1359.

1360.

'
COS X

1362.
1363.

1364.

sin'xi/cosxdx.

f
J

V sin x cos*

dx
.

f-==.
J l^tan*

Integrating Trigonometric Functions

Sec. 7]

2.
V

cos

Integrals of the form

mx cos

1)

sin

2) sin

3)

mx

sin

cos

nxdx,

sin

131

rnx

sin

nx dx and

nx dx. In these cases the following formulas are used;

mx cos nx = -~
mx

sin

nx

cosm*cos ^^

Example

7.

= -^

=
sin

-9-

(m + n) x + sin (m

[sin

(m

[cos

9* sin

n)

n) x] ;

cos(m

+ n) x]\

n)*-fcos (m-f/i)

(m

xdx=

8*
-^ [cos

x].

cos

lOjt]

dx=*

Find the integrals:


1365.

1366.

sin

cosSxdx.

sin 10* sin 15

1367.

f cos

1368.

f sin

3.

3jc

~ cos
|-

sin

A:

1370.

d*.

djc,

1371.

~ djc.

1372.

^-

J cos(aA:

sin

a>/

f
sin

(co/ -f-cp)

cos x cos* 3x dx

sin x sin 2* sin

dt.

3* dx.

Integrals of the form


f

where R
1)

1369.

is

/?

cos x) dx,

(sin *,

(2)

a rational function.

By means

of substitution

tan

t,

whence
2*

integrals

of

new variable
Example

form

(2)

are

reduced

t.

8.

Find
f
J

Solution.

f*

Putting

to integrals

__
*E

+ sin x + cos

tan -pr^*'

we

wil1

have

of rational

/.

functions by the

132

we have

2) If

the identity

R(
Ihen

we can

use

[Ch.

Indefinite Integrals

sin*,

the

cos*)

s/?
*

tan

substitution

cos*),

(sin*,

to reduce the

integral (2) to a

rational form.

Here,
sin

jg

COS*=

,-

and

*=arc
Example

tan/, dx

<3 >

Jnn-'-

tan* = f,

sin

dt

dx =

^,

:j

-jz

have

will

dt

dt

note

denominator

that

the

integral

+ C = -7=2 arc tan


F

(3)

it

is

/T tan *) +

evaluated faster
divided by cos 2 *.

is

useful

to apply

C.

the numerator and

if

procedures (see, for

artificial

1379).

Find the
1373

2~)

of the fraction are first

In individual cases,

example,

/"

(/

= -^=arctan
We

Find

9.

Solution. Putting

we

'

1374.

J 3

integrals:

+ 5*cos*

C^

1382 *'

'

*!.

J sin *

+ cos *

cos*

dx
J 3

sin 2

2
J sin *

iooyi*

1384 *1

385.

-|-

P
\

(1

^^.

I39

'

X
r-. dx.
3
cos*)
Sin2x
<*-

COS

jj

p-j-^-rfx
S

'

1389*.

cos 2 *

3sin *cos *

si n

J.

'

dx

1386.

1388

JiHiJT 5sln*cos*
J

100T
1387.

**

1383*. f

10 or

mi-.

+ 5 cos

Is. n

*-6 sin* + 5

^
'

sin*) (3

sin*)

o-.j |;;;;;i::;>.

'

Integration of Hyperbolic Functions

Sec. 8]

133

Sec. 8. Integration of Hyperbolic Functions


Integration of hyperbolic functions is completely analogous to the integration of trigonometric functions.
The following basic formulas should be remembered:
sinh 2

cosh 2 X

1)

2) sinh

3) cosh

*=l;

*=

~ (cosh 2*

x=

Y (cosh 2x+

4) sinh x cosh x

Example

1.

1);

1);

= ~ sinh 2x.

Find
cosh 2 *djt.

We

Solution.

have

f cosh x djc-=r~(cosh 2x + \)dx

Example

2.

( cosh

We

xcU=

jsinh

2x +

Find
\

Solution.

cosh 3 ****.

have

cosh 2 xd (sinh

x)=

(l

+ sinh

Jc)d(sinhA:)

= smh x-\
.

Find the
1391.

integrals:

1397.

Jsinh'jtd*.

1392.

1398.

^cosh'xdx.

1393.

sinh'* cosh*

1394.

1396

Sec.

9.

Integrals

C
2
J sinh

Using
of the

^
A:

cosh 2

1402.

A;

Trigonometric

and

Hyperbolio

Substitutions

for

Finding

Form
(1)

is

a rational function.

134

Indefinite Integrals

[CH. 4

the quadratic trinomial ax* + bx-\-c into a sum or difference


Transfor
Transforming
integral (1) becomes reducible to one of the following types

t
of squares, the

of integrals:

2)

(z,

3)

(z,

The
1) 2
2) 2
3) 2

m* +

Vz 2

z*) dz\

latter integrals

= msin/
= mtan/
= msec/

Example

1.

or z
or 2
or z

dz.

are,

taken

respectively,

by

means

= m tanh
= msinh/,
= m cosh/.

/,

Find
dx

==/.

(x+\Y
Solution.

Putting

We

have

*+l = tan2, we

then have dx
sec 2

dx

= f tan
\

Example

Solution.

Putting

2.

Find

We

have

zdz

= sec*zdz

zsec2~"J

and

~ dz =

f cos 2

of substitutions:

Sec. 11]

we

finally

Using Reduction Formulas

135

have

Find the integrals:


1403.

S/3-2jt-x'd*.

1409.

W-6*-7d*.

"

1404.

1410.

1405.

1406.

Sec. 10. Integration of Various Transcendental Functions

Find the integrals:


J (*

1416.

$*

1417.
1418.

1419.

1420.

+ I)

1415.

cos

2x

1421.

dx.

1422.

SJCC/A;.

x s\nxcos2xdx.

(*-/=
J

1423.

f^lnj^djc.
*

J
2

$e *sin
x

\^e

jcdjt.

1424.

smxsmSxdx.

1425.

x
^ A'e

cos x dx.

1426.

J
J
?

In (x
jc

+ /FT?) djc.

arc cos

(5jc

smxsinhxdx.

Sec. 11. Using Reduction Formulas

Derive the reduction formulas for the following integrals:


1427.

/..Jj-^,,;

1428. /

=$

find /.

and

sln"xdje; find / 4 and

/,.

/,.

136

[Ch. 4

Indefinite Integrals

/w

1430. 7 n

1429.

Sec.

J^h;;

find

(x n e~*dx\

I*

find

and
/ 10

7 4-

12. Miscellaneous Examples on Integration

dx

1448.

xdx
f

JO+jt
x

dx

J *'-2*+2
1433.

xdx

1449.

..

2^ j

j y^i

1450.

dx.

KI-J

+1

dx

2
'

dv

dx

dx

1452.

dx

1453.

dx

P
'

J (x*
r

+ 2)

f-7^
A

'

t/

dx

y l/^v2
*
r

1455.

Jx-2A' +

dx

C__^L_
1457.

'

Jf *

VT^X*

,458.
'

J
5jt

1442.

J
1460.

1443.

-1

1461

dx
-

1444.

IHB^T^

1462.

J/ Jl a!+i)'
(

1446.

ax

'-^-

1463.

1464.
P

1447.

y2

j^===d.

f^
^

dA;

1465.

sin' A;

COS '

cosec'SxdA;.

Miscellaneous Examples on Integration

Sec. 12]

1466.

M67.J.-(f,4
'

l469

dx,

fsinh^l-A

J 2sinx

+ 3cosx-5
"

J 2-+w-*-

f.

,)sinh'*"
dx

1470.

A:

d)c

f*

!468

sinh ^ cosh

484.

1485

137

]>

1488.

C_^_
,.
2e*

2*
J e

d.v

.471.

1489.

"

j sin

2x

sin

jc

dx

1472.

rfx.

j1474.

1476.
1477.
1478.

1479.
1480.

JV? +
A:

a*

sin x dx.

Jxe^djc.

J(x

1493.

J/?Tirfx.

sin*

2>

.j-^z

(sin

C-

>

-s-

1)

src tdn x

dx.

1494.

1495.

(Varc

1496.

J cos(lnx)dA:.

1497.

J (x

1498.

Jxarctan(2x+3)dx.

1499.

fare sin

dv

>-

/\

\nYT=xdx.

x'

10~ *cfx.

1492.

^*V'djt.

r*

1483.

sin*

CJiLdx.

1481.

1491.

aX

<

sinyd.v.

3x) sin 5*

*^i*

cos

"2"^*.

+ cos x)*'

1500.

V^ dx.

rfjc.

Chapter

DEFINITE INTEGRALS

Sec. 1. The Definite Integral as the Limit of a

Sum

1. Integral sum. Let a function f (x) be defined on an interval


and a=xc <Xt <
< x n = b is an arbitrary partition of this interval
n subintervals (Fig. 37). A sum of the form
.

into

(1)

where
/

is
is

= 0,

1,

2,

...

(n

1),

called the integral sum of the function f (x) on [a, b].


the algebraic area of a step-like figure (see Fig. 37).

Geometrically,

S,,

10

Fig. 37

Fig. 38

2. The definite integral. The


number of subdivisions n tends
to zero,

from

is

x=a

limit of the sum S nt provided that the


to infinity, and the largest of them, Ax/,
called the definite integral of the function f (x) within the limits
to *
&; that is,

max

(2)

A*j

-> o

The Definite Integral as the Limit

Sec. 1]

of

Sum

139

it is integrable on [a, b]\


If the function / (x) is continuous on [a, b],
i.e.,
the limit of (2) exists and is independent of the mode of partition of the
interval of integration [a, b] into subintervals and is independent of the
choice of points / in these subintervals. Geometrically, the definite integral
(2) is the algebraic sum of the areas of the figures that make up the curvilinear trapezoid aABb, in which the areas of the parts located above the #-axis
are plus, those below the jc-axis, minus (Fig. 37).

The definitions of integral sum and definite integral are


b.
eralized to the case of an interval [a, b], where a
Example 1. Form the integral sum S n for the function

>

by dividing the interval into n equal parts and chooscoincide with the left end-points of the subintervals
x i+l ]. What is the lim S n equal to?
n -+ CO
9
Ax.
and tc/ J
Solution. Here,
-. Whence
n
n

on the interval
ing
[x it

gen-

naturally

points

[1,10]

that

|/

= 101 =
Hence

(Fig. 38),

S n -58-L.

lim
n

->> oo

Example 2. Find the area bounded by an


and the ordinates * = 0, and x = a (a >

jc-axis,

Partition

Solution.
parts

the

arc of the parabola


0).

base a into n equal

Choosing the value

of

the

tion at the beginning of each subinterval,

func-

we

will

have

The areas of the rectangles are obtained by multiplying each yk

Summing, we

by the base

A*=

(Fig.

39).

get the area of the step-like figure

Using the formula

sum

for the

of the squares of integers,

2>

n(n+\)(2n+\)
6

Fig. 39

= x*

the

140

we

[Ch.

Definite Integrals

find

= g'n(n-l)(2n-l)
6H*

S
and, passing

'to

the limit,

S-

lim
n -*

'

we obtain
>

Sn =

lim
n

-* co

(-D(2-l) = al
6n

>

Evaluate the following definite integrals, regarding them as the


limits of appropriate integral sums:
b

1501.

1502.

1503.

dx.

x*dx.

-2

10

1504.

J(0 t +g*)<tt,

J2*dx.
S

and g are constant.

1505*.

x dx.

1506*. Find the area of a


the hyperbola

curvilinear

by two ordinates: x = a and x = b


1507*. Find

trapezoid

(0<a<&),

bounded

by

and the x-axis.

sintdt.

Sec. 2. Evaluating Definite Integrals by Means of Indefinite Integrals

1. A definite integral with variable upper limit.


continuous on an interval [a, b], then the function

If

a
is

the aritiderivative of the function


F' (x)

= f (x)

2. The Newton-Leibniz formula.


o

f (x)\

that

is,

a<*<6.
F' (*) = /(*)

for
If

th ^n

function

f (t)

is

Sec. 2]

Evaluating Definite Integrals by Indefinite Integrals

The antiderivative F

Example

1.

(x)

is

computed by finding the

indefinite integral

Find the integral

Find

Find the derivatives

of the following functions:

\={\ntdt
1510.

f(jc)=Td/.

1512.

1513. Find the points of the

extremum

of the function

=
j!lild/

in the region

Applying the Newton-Leibniz formula,

l^~-

1515.

find the integrals:

1514.

*>0.

1516.

Jdt.

~*

^-.
*

1517.

J*cos/<#.
'

-.

Using definite integrals, find the limits of the sums:

1520.

lim

n-

Rl

[Ch.

Definite Integrals

142

Evaluate the integrals:


8

2*

J(jt*

+ 3)d*.

1522.

dx

I
Jl

1523.
cos* a da.

1536.
Jl

1524.

2d*.

JK*
2

1537.

sin'cpdcp.

*
]'

1
1539.
Jl

JF

1540.

71

4~
l

iL
8

***

1541.

dx

JF=3F
1831.
1543.
.L

coshA:dA;.

Ins

1532.

\
j

sec a da.

f*

*'

rfjt

5
J cosh !'
In

sinh*xdjc.

Sec. 3]

_ _
Improper Integrals

Sec. 3. Improper

143

Integrals

1. Integrals of unbounded functions. If a function f (x) is not bounded


any neighbourhood of a point c of an interval [a, b] and is continuous
and c<x<>b then by definition we put
for
in

a<*<c

C-B

(f(x)dx =

J
a

lim
C

^a

f(x)dx

+e

lim

f(x)dx.

(1)

-*%+e

are finite, the improper inteIf the limits' on the right side of (1) exist and
a or c
thj
b,
gral is called convergent, otherwise it is divergent. When c
definition is correspondingly simplified.
If there is a continuous function
F (x) on [a, b] such that F'(x) f(x)

when x

then

c (generalized antiderivative),

F(a).

If

a<*<6

|/(*)|<F(x) when

and

(2)

F(*)dx converges, then

the

in-

tegral (1) also converges (comparison test).

and

lim

X-+C

when x-+

then

c,

for

1)

(x)

x m

oo,

oo,

0,

i.

e.,

f(x)~

the integral (1)

If

converges,

the function / (x)

2) for

the

when

continuous

is

^
I

m^>l

then we assume

\f(x)dx=
J

lim
b ->

oo

(3)

\f(x)dx
J

and depending on whether there


the respective integral
Similarly,

is

is a finite limit or not on the


called convergent or divergent.

lim

f(x)dx=

a -^

I'

C
\

m<

integral (1) diverges.


2. Integrals with infinite limits.

<

^A

I/WK^W

(f(x)dx

oo

f(x)dx=

and the

integral

of (3),

oo

and

right

[p(x)dx

lim
a->

oo

converges,

\f(x)dx.
J

then

the

infe-

a
gral (3) converges as well.
If

/W^rO

oo,

then

1)

gral (3) diverges.

and
for

lim

m>

f (x)

xm

= Ajt<x>

the integral

(3)

A^Q,

converges,

i.e.,
2) for

/(x)~-4 when

m<l

the inte-

144

__
Definite Integrals

Example

&_

f 2
J x

[C/i.

1.

lim

f 4?
+ e-*oJ*
"f"*
*2

e -K> J

-i

lim

lim
(I-lU e-*oV
(l-l
e

e->oVe

and the integral diverges.


Example 2.
6

QO

=
Example

3.

lim
lim (*rT^2=
2
&-><
ft-oo J

(arc tan 6

1+^

arc

tan

())

Test the convergence of the probability integral

(4)
*r

Solution.

We

put
C

00

00

e~ x

dx=

<r*

dv+

e~ x dx.
*

The first of the two integrals on the right


2
the second one converges, since e~x <e~*

dx=

not an improper integral, while

is

lim

when

x^\

e~ b

+ e~

and
l

)=e~

hence, the integral (4) converges.


Example 4. Test the following integral for convergence:

w
r
j

^^

J?

'

Solution.

When x -++&>, we have

Since the integral

converges, our integral (5) likewise converges.


Example 5. Test for convergence the elliptic integral

dx

'

Sec. 3]

145

Improper Integrals

Solution. The point of discontinuity of the integrand


the Lagrange formula we get

where

*<*,<!.

Hence,

for

*-+

is

x=l. Applying

we have

/T ^

\\-xj

'

Since the integral


1

v dx

ff
nr^y
converges, the given integral

(6)

converges as well.

Evaluate the improper integrals

(or establish their divergence):

xlnx
a

P AY

1552.

-^.

f1

1560.

I
a

*L
a

oo

1553.

J.

1561.

Jcotjcdx

146

_
00

1562.

kx

\e-

dx

GO

f -arc tan*

Definite Integrals

(*>0).

d*-

1565.

1566

(Ch.

1564.

Test the convergence of the following integrals:

100

1567.

j)

.-

**..,_

/*+2 */*+*

1571.

./*

..

^/l-jc

1570.

1574*. Prove that the Euler integral of the

fiist

kind

(beta-

function)

B(P, q)=*
converges when

p>0

and

q>0.

1575*. Prove that the Euler integral of the second kind (gam-

ma-function)

converges for

p>Q.

Sec. 4. Change of Variable in a Definite Integral

a<x<b

If a function f (x) is continuous over


and *=-q>(0 It a function
continuous together with its derivative cp' (t) over a<^<6, where a=*9(a)
and &=cp(P), and /[<p(01 is defined and continuous on the intervtl

Sec. 4]

of Variable in

Change

a Definite Integral

147

then

Example

1.

Find
x*

We

Solution.

V~a*x*dx

(a

> 0).

put

asin

t\

dt.

Then

P=

arc sin

= arc sin
l

=y

and,
Therefore,

we

consequently,

we

shall

can

take

= arcsinO = 0,

have

IL
a

C x2

VV

x2

=a

a2

dx=\

sin

2
1

a 2 sin 2

J/~a

jl

JL

f sin

cos

d/ =

a cos

dt

IL
t
2

f sin 2/

d/ =

-^-

C
-^-

(1

Jtfl

1576.

Can the substitution

Transform the following


indicated substitutions:

A:

= COS/

definite

be

made

integrals

in the integral

by means

of

the

1577.

f(x)dx,

= arc tan/.

1578

ly=,

1581. For the integral


b

148

__
Definite Integrals

[Ch.

indicate an integral linear substitution

which the limits

as a result of

of integration

would be

and

1,

respectively.

Applying the indicated

evaluate

substitutions,

the

following

integrals:

1583.
a

1584.

x
\Ve -\dx,

n
2

1586.

Evaluate

the

following

integrals

by

means

of

appropriate

substitutions:

1587.

f^=

A~

i5 8 9.

fi^^d*.
*

1590.

'

2
2

1538.

____

f
J 2*4-

%=.

1^3*+

Evaluate the integrals:


a

1591.

C
J

dx
y

xV

x2

1593.

e
+ 5x4-\

tyax

x*dx.

o
271

1594.

fg-f
53 cos x

J
i

1595. Prove that

if

f(x)

is

an even function, then

Integration by Parts

Sec. 5]

But

if

f(x)

1596.

is

Show

an odd function, then

that
00

00

00

~'T=

-00

1597.

Show

149

dx

that
!L
2

arc cos

1598.

Show

that

T
C

f(s\nx)dx =

T
f

Sec. 5. Integration by Parts


functions
If the
interval [a, b], then

(x)

(x) v' (x)

and

dx

Applying the formula

v (x)

are

continuously

= u (x) v(x) for

t;

integration

differentiate on

(*) u' (x) dx.

by

parts,

the

(1)

evaluate the

following integrals:
*L
00

1599.

^xcosxdx.

1603.
oe

1600.

Jlnxd*.

1604.

1601.

J*V*dx.
o

1602.

Je*sinxdx.

1605.
o

e- aJC cos6A:dA:

(fl>0).

__

150

[Ch. 5

Definite Integrals

1606**. Show that for the gamma-function (see


the following reduction formula holds true:

From

+ =

T (n
n\,
\)
that for the integral

this derive that

Show

1607.

=
= [sm
/=
sin" xdx
n

is

1575)

a natural number.

if

Example

cos

71

the reduction formula


n

n ~*

holds true.

Find /, if n is a natural
Devaluate I 9 and 7 10
1608. Applying repeated

number. Using the formula obtained,

integral (see

Example
B(p,

<7)

parts,

evaluate the

where p and 9 are positive


1609*.

by

integration

1574)

integers.

Express the following integral

in

terms of

(beta-

function):

m=
if

sin

and n are nonnegative

x cos

integers.

Sec. 6. Mean-Value Theorem

1. Evaluation of

If

integrals.

f(x)^F(x)

^f(x)dx^^F(

66

f(x) and <p(x) are continuous

where

is

the interval

the smallest
[a,

b]>

for

a<*<6

(1)

and,

besides,

(p(*)^0, then

W ^.

(2)

and

then

X )dx.

mJq>(x)d*<J/(*)9(*)dx<M
a

a<*<&,

If

for

is

the

largest

value of the function /

(x)

on

Mean-Value Theorem

Sec. 6]

In particular,

if

151

<p(#)s=l, then
b

m (ba) <.[f(x)dx*^M

(6

(3>

a).

The inequalities

(2)

and

may

(3)

be replaced,

respectively,

by

their equiva-

lent equalities:
b

and
b

-a),
a

where c and

Example

are certain
1.

numbers lying between a and

b.

Evaluate the integral


5L

M
2

Solution. Since

(Xsin'x^l, we

have

Jl

<

2'

that

i/l
K 2

'

is,

1.57

2. The mean value

</<

of a function.

1.91.

The number
b

is

called the

mean value

of the function / (x)

on the interval

1610*. Determine the signs of the integrals without evaluating

them:

n
b)

152

[Ch. b

Definite Integrals

Determine (without evaluating) which of the following

1611.
integrals

is

greater:
i

V\+x*

a)

dx

or

dx\

x sin x dx

or

e x*dx

or

b) ^

J x sin

x dx\

c)

Find the mean values

of the functions

on the indicated

inter-

vals:

1612. /(x)
1613. f(x)
1614.

=x
=a

0<x<l.

2
,

f(x)=sm

x,

1615.

1616. Prove that (


J
o

dx
lies

1/2+^

Find the exact value

0.70.

between

^2

of this integral.

Evaluate the integrals:


n
4

1617.

+ jfdx.

1620*.
JT

1618

djf

J gqpjj.

2JI

1619.

1622. Integrating by parts, prove that


200JI

10071

43

0.67 and -4=


V^2

The Areas

Sec. 7]

of

153

Plane Figures

Sec. 7. The Areas of Plane Figures

1. Area in rectangular coordinates. If a continuous curve is defined in


f(x) [f(x)^Q], the area of the
rectangular coordinates by the equation y
curvilinear trapezoid bounded by this curve, by two vertical lines at the

a
Fig.

40

a and x
6 and
points x
is given by the formula

by

segment

of

the jc-axis

(Fig. 40),

b
(1)

Example
straight

1.

lines x

Compute

and

.v

the
3,

area bounded by the parabola


and the x-axis (Fig. 41).

y-fi(*)
Fig. 42

Solution. The sought-for area

Fig. 43

is

expressed by the integral

X
= -~
^

the

154

Definite Integrals

Example
the

2.

Evaluate the area

(Ch.

bounded by the curve # = 2

y*

*/

and

/-axis (Fig. 42).

Solution. Here, the roles of the coordinate


sought-for area is expressed by the integral

axes

are changed

and so the

where the limits of integration y l


2 and i/ 2 =l are found as the ordinates
of the points of intersection of the curve with the t/-axis,

a
Fig. 44

In the

curves

Fig. 45

more general case, if the area S is bounded by two continuous


and y = f 2 (x) and by two vertical lines x = a and x = b, where

= M*)

/i(*XM*) wn

a<*<&

(Fig. 43),

we

will then have:

(2)

Example

3.

Evaluate the area 5 contained between the curves

y
<Fig. 44).
Solution. Solving the
Jiniits of integration: x,

If

set
1

=2
of

x 2 and

l
/

equations

=JC 2

(3)

(3)

simultaneously, we find the


of formula (2), we obtain

and # 2 =1. By virtue

the curve is defined by equations in parametric form x


q>(/), y
y(t\
the area of the curvilinear trapezoid bounded by this curve,
by two

then

The Areas

Sec. 7]

a and #=&), and

vertical lines (x
integral

Plane Figures

of

by a segment

155

of the

*-axis

is

expressed

by the

f, are determined from the equations


and 6 = <p( 2 ) [\|?(/)^0 on the interval [/ lt /,]].
Example 4. Find the area of the ellipse (Fig. 45) by using

where
a

and

/,

= (p(/j)

equations

Due

Solution.

quadrant and

we
/

xc

y
l

parametric

= a cos
= bsint.
c

/,

it
is sufficient to compute
the area of a
the result by four. If in the equation jc
aco

symmetry,

multiply

and then x

put x

first

= 0.

to the

then

its

a,

we

get the limits of integration

/,

=
= y and

Therefore,
ji

-i-S

(b

sin

sin /)

dt=ab(

sin

2
/

dt

=^

S nab.
2. The area in polar coordinates.

and, hence,

If a
curve is defined in polar coordinates by the equation r~f (<p), then the area of the sector AOB (Fig. 46),
bounded by an arc of the curve, and by two radius vectors OA and OB,

Fig. 47

Fig. 46

which

correspond

to

the

values

cp,

=a

and

cp,=

p,

is

expressed

by

the

integral

Example

5.

Find

(Fig. 47).

the

area

contained

inside

Bernoulli's

lemniscate

156

__

Definite Integrals

Solution.

By

virtue of the

[Ch. 5

curve we

of the

symmetry

quadrant of the sought-for area:

Whence S = a2

_
determine

first

one

1623. Compute the area bounded by the parabola y


4x
x
and the x-axis.
1624. Compute the area bounded by the curve y
\nx, the
e.
;t-axis and the straight line x
x (x
1625*. Find the area bounded by the curve y
2)
1) (*
and the x-axis.
1626. Find the area bounded by the curve y*
x, the straight
8.
line y=l and the vertical line x
1627. Compute the area bounded by a single half-wave of the
and the Jt-axis.
sinusoidal curve
1628. Compute the area contained between the curve y
ianx,

y=smx

the x-axis and the straight

line

x=~

1629. Find the area contained between the hyperbola xy


m*,
and x 3a (a>0) and the x-axis.
the vertical lines
1630. Find the area contained between the witch of Agnesi

x^a

u= x- + a

and the x-axis.

1631. Compute the area of the figure bounded by the curve


8 and the y-axis.
the straight line y
1632. Find the area bounded by the parabolas y**=2px and

y~x*,
x

= 2py.

1633. Evaluate the area

and the straight line f/ =


1634. Compute the area

bounded by the parabola y = 2x

x*

x.

of a segment cut off by the straight


2
2x from the parabola y = x
1635. Compute the area contained between the parabolas y*=*x*,

line

y=3

*/=Y and
1636.

the straight line y

Compute the area contained

and y = 4

y=

1637.

Agnesi

1638.

between

the

parabolas

|x

Compute the area contained z between the witch

y=

and the

= 2x.

and the parabola

f^.

Compute the area bounded by the curves y**e*>


straight line

x=l.

of

The Areas

Sec. 7\

of

Plane Figures

1639. Find the area of the figure

=
f2

and the straight

__

157

bounded by the hyperbola

linex = 2a.

1640*. Find the entire area bounded by the astroid

1641. Find the area between the catenary

y = a cosh
the (/-axis and the straight line

= ~(e +
2

f/

!)

x
1642. Find the area bounded by the curve a*y*
1643. Compute the area contained within the curve

(a*

2
;c ).

1644. Find the area between the equilateral hyperbola x*


y*
the x-axis and the diameter passing through the point (5,4).

= 9,

1645. Find

the

and the ordinate


Find

1646*.

between

area

x=l

the curve y

(x>l).
bounded

area

the

the

x-axis,
9

the
by
J

asymptote x = 2a (a>0).
1647*. Find the area between the strophoid

and

~i,

cissoid

r/
y

X
= 2ax
o

its

y*

=x

x ~~ a)2

2Jfl^~~

and

asymptote (a>0).

its

1648.
circle

and

jt

the area of the two parts into which


divided by the parabola if =2x.
2
2
the area contained between the circle x
y

the

Compute

r/

=--8 is

1649. Compute
the parabola x*=l2(y
1).
1650. Find the area contained within the astroid
jc

1651. Find
the cycloid

the

= acos

/;

y= b

sin

= 16

/.

bounded by the x-axis and one arc

area

x = a(t
# = a(l

of

sin/),
cos/).

1652. Find the area bounded by one branch of the trochoid

tefr***/and a tangent

to

it

<P<<

at its lower points.

158

(Ch.

Definite Integrals

1653. Find the area bounded by the cardioid


cos2/),

--

sin20.

1654*. Find the area of the loop of the folium of Descartes

*-r+T"

*-

a (1-f coscp).
1655*. Find the entire area of the cardioid r
1656*. Find the area contained between the first and second
turns of Archimedes' spiral, r
acp

(Fig. 48).

1657. Find the area of one of the


leaves of the curve r
acos2cp.
1658. Find the entire area bound2
2
a sin4cp.
ed by the curve r
1659*. Find the area bounded by
the curve r
asin3cp.
1660. Find the area bounded by
Pascal's limagon

Fig. 48

1661.

Find

the

= 2 + cos

area

and the two half-lines

cp.

bounded by the parabola

9=4-

and 9

1662. Fin'd the area of the ellipse


r

= y.
r =
-r
1

+ e cos

1663. Find the area bounded by the curve

=a

sec

(e<l).
v
'

cp

= 2acos3cp

a.
lying outside the circle r
1664*. Find the area bounded by the curve x*

and

+ y* = x* + y*.

Sec. 8. The Arc Length of a Curve

1. The arc length

in rectangular coordinates.

y=f(x) contained between two

The arc length s of


= a and x~b

points with abscissas x

2 8

2 8

curve

is

'
'
Example I. Find the length of the astroid * +/'*-= a
(Fig. 49).
Solution. Differentiating the equation of the astroid, we get

The Arc Length

Sec. 8]

For this reason, we have

of a

Curve

159

for th* arc length of a quarter of the astroid:

Whence s = 6a.
2. The arc

length of a curve represented parametrically. If a curve is


<p(0 and y
represented by equations in parametric form, #
ty(t), then the
arc length s of the curve is

where

and

t2

"

are values of the parameter that correspond to the extremities

of the arc.

49

Fig

Example

2.

Fig. 50

Find the length

one arc

of

x
a(t
[
\ j/=:a(l
Solution.

We

have

of the

cos/)-

cosf) and -^

-^=a(l

= asin/.

/=2a
The limits of integration
of the arc of the cycloid.
If a curve is defined
the arc length s is

^=

and

cycloid (Fig. 50)

sin/),

= 2ji

by the equation

sln~-d/=

correspond

= /(cp)

Therefore,

to

in polar

the

extreme points

coordinates, then

where a and p are the values


the arc.

of

the

polar

angle

at

the extreme points of

160

[Ch.

Definite Integrals

Example

The

3.

Find the

entire curve

is

the

length of

entire

described by a point as

cp

curve

asin-|-

ranges from

(Fig. 51).

to 3ji.

Fig. 51

Solution.

She curve

We

have

r'

= a sin

-^- cos

therefore the entire arc length of

is

8JI

s=

J J/a*

1665.

y*

8JI

= x*

sin

-|

+o

Compute the

cos'

sin*

-f-

-|-

arc

length

of

=a
d<p
the

sin

-f-

rfq>

=^

semicubical

from the coordinate origin to the point x =


1666*. Find the length of the catenary y =

1667.

x=0

to

Compute

B(b,h).
the arc length of

x=l.

the parabola y

curve y

of

the

of

the curve y

*=K8.

*=

1670. Find the


to

1671.

between

length of the curve y

arc

to

= e*

= lnx

= 2}/"x

from

lying between

from x

= /3

= arc sin (e~*)

from

= In secy,

lying

jc=l.

Compute the
t/

and

j/

arc length
-5-

of the

curve x

1672. Find the arc length of the curve x

=1

from the

(6,a) to the point

1668. Find the arc length


the points (0,1) and (l,e).
1669. Find the arc length
to

parabola

4.

acosh-^-

vertex

= e.

= ^-y

-^\ny

from

Volumes

Sec, 9]

of Solids

161

1673. Find the length of the right branch of the tractrix

a+vV-t/

from y = a to

t/=&(0<6<a).

1674. Find the length of the closed part of the- curve

= x(x

3a)

1675. Find the length of the curve


to

t/

= ln ( coth ~-J

from

x=b (0<a<6).
1676*. Find the arc length of the involute of the circle
f

to

= 7\

1677. Find the length of the evolute of the ellipse

1678. Find the length of the curve

x=- a (2 cos/
y = a(2 sin/
1679. Find the length of the

cos 2/)

-y

which

is

cut

off

first

1680. Find the entire length of


1681. Find the arc length of
r = asec

sin 2/).

turn of Archimedes' spiral

the cardioid r
that
part of

by a vertical

line

= a(l + coscp).
the

passing

parabola

through

the pole.
1682. Find the length of the hyperbolic spiral rq>= 1 from the
point (2,'/ 2 ) to the point C/,,2).
ae m v,
1683. Find the arc length of the logarithmic spiral r
r
a.
lying inside the circle

1684. Find

r=l

to

the arc

length

r = 3.

of

the curve

<p

=
-g-

(r

from
j

'

Sec. 9. Volumes of Solids

1. The volume of a solid of revolution. The volumes of solids formed by


the revolution of a curvilinear trapezoid [bounded by the curve y&f (x) fhe
AT- ax is and two
a and x
vertical lines x
b\ about the x- and '(/-axes are
J

1900

162

Definite Integrals

[Cft.

expressed, respectively, by the formulas:


b

Vx =ji

1)

J
a

y*dx\

2)

V Y =2n

xr/dx*).

Example 1. Compute the volumes of solids formed by the revolution of a


bounded by a single lobe of the sinusoidal curve # = sinx and by the
segment O<;*<JT of the x-axis about: a) the x-axis and b) the j/-axis.
figure

Solution.

a)

V^^-ji

b)

Vy=2n

xsinxdx=2jt(

xcosx + sinx)Jc ~2ji t

The volume of a solid formed by revolution about the t/-axis of a figure


bounded by the curve x=g(y), the (/-axis and by two parallel lines y = c and
t/

= d,

may

be determined from the formula

from formula

obtained
x and y.

(1),

given

above, by

interchanging

the

coordinates

If the curve is defined in a different form (parametrically, in polar coordinates, etc.), then in the foregoing formulas we must change the variable of
integration in appropriate fashion.
In the more general case, the volumes of solids formed by the revolution
about the x- and (/-axes of a figure bounded by the curves /!
/! (x) and y 2
f z (x)
a and x
b are, respectively,
[where f\(x)^f z (x)] and the straight lines x
equal to

=
=

and
b

Example
circle x*

2.

+ (y

Find the volume of a torus formed by


2
a 2 (6^a) about the x-axis (Fig. 52).

&)

the rotation of the

*) The solid is formed by the revolution, about the (/-axis, of a curvilinear


a, x
trapezoid bounded by the curve y
b,
f(x) and the straight lines x
and
0. For a volume element we take the volume of that part of the solid
formed by revolving about the 0-axis a rectangle with sides y and dx at a
distance x from the (/-axis, Then the volume element dVy=2nxydx, whence

V K =2JI

xydx.

Volumes

Sec. 9]

Solution.

We

of Solids

163'

have
I/,

=6

I^a

x 9 and y 2

= t>+ Va*x

Therefore,

(the latter integral

is

taken by the substitution

x=asi

-a

x a
Fig

52

The volume of a solid obtained by the rotation, about the polar axis, of a
sector formed by an arc of the curve r
F((p) and by two radius vectors
ipr-=a, <p
P may be computed from the formula

Vp = ~
2

C
JT \

r 8 sin cpd

q>.

This same formula is conveniently used when seeking the volume obtained
by the rotation, about the polar axis, of some closed curve defined in polar
coordinates.

Example

= asin2(p

3.

Determine the volume formed

by the rotation

about the polar axis.

Solution.

= 2.--n\

rsinq>d(p

yJia

sin

.L
2

= ^ Jia
3

sin

4
<p

cos 8

cp

dcp

lOo

jia

8
.

2(p sin q> dcp

of

the curve

164

__

\Ch. 5

Definite Integrals

2. Computing the volumes of solids from known cross-sections. If S S(x)


the cross-sectional area cut off by a plane perpendicular to some straight
line (which we take to be the x-axis) at a point with abscissa *, then the
volume of the solid is
is

where *, and x 2 are the abscissas of the extreme cross-sections of the solid.
Example 4. Determine the volume of a wedge cut off a circular cylinder
by a plane passing through the diameter of the base and inclined to the base
at an angle a. The radius of the base is R (Fig. 53).
Solution. For the *-axis we take th? diameter of the base along which
the cutting plane intersects the base, and for the (/-axis we take the diameter
of the base perpendicular to it. The equation of the circumference of the base
is

*2

+ =R
2

j/

The area

2
.

of the section

ABC

at

a distance

x from the origin

is

S(x) = area A ABC = -^ ABBC = -^yy tana =^- tana.


1

r/

Therefore, the sought-

for

volume

of the

wedge

is

y=

2~

f y2

tanad*=tana

(*

(R

1
*)<& = y tana R

1685. Find the volume of a solid formed by rotation, about


the x-axis, of an area bounded by the x-axis and the parabola
1686. Find the volume of an ellipsoid formed by the rotation
of the ellipse ^r

+| =

1687. Find the

the x-axis', of

volume

about the x-axis.

formed by the rotation, about


an area bounded by the catenary y = acosh
the
of a solid

xa.

and the straight lines


1688. Find the volume of a solid formed by the rotation, about
a
the x-axis, of the curve j/=sin x in the interval between x
xr-axis,

and x = n.
1689. Find the volume of a solid formed by the rotation, about
the x-axis, of an area bounded by the semicubical parabola if = x s
the x-axis, and the straight line x== 1.
1690. Find the volume of a solid formed by the rotation of
the same area (as in Problem 1689) about the {/-axis.
1691. Find-, the volumes of the solids formed by the rotation
of an area bounded by the lines y = e*, x = 0, y =
about: a) the
x-axis and b) the y-axis.
1692. Find the volume of a solid formed by the rotation, about
the t/-axis, of that part of the parabola j/ 2 = 4ax which is cut off
by the straight line x = a.
,

Sec. 9]

_ _
Volumes

of Solids

165

1693. Find the volume of a solid formed by the rotation, about


the straight line x
a, of that part of the parabola y*=4ax which
is cut oft by this line.
1694. Find the volume of a solid formed by the rotation, about
the straight line y
p, of a figure bounded by the parabola

2
t/

= 2p*

and the straight

line

= --

1695. Find the volume of a solid formed by the rotation, about


the x-axis of the area contained between the parabolas y
x*
L

and

y= Y*.

1696. Find the volume of a solid formed by the rotation,


about the x-axis, of a loop of the curve (*
4a)tf =ax(x
3j)

1697. Find the


of the cyssoid y*

volume
A8

= ^ _x

of a solid

about

its

generated

by the rotation

asymptote x = 2a.

1698. Find the volume of a paraboloid of revolution whose


base has radius R and whose altitude is //.
1699. A right parabolic segment whose base is 2a and altitude h
is in rotation about the base. Determine the volume of the resulting solid of revolution (Cavalieri's "lemon").
1700. Show that the volume of a part cut by the plane jc
2a
off a solid formed by the rotation of the equilateral
hyperbola
x*
tf^c? about the *-axis is equal to the volume of a sphere
of radius a.
1701. Find the volume of a solid formed by the rotation of a
bounded by one arc of the cycloid x=-a (/
sin t),
figure
y=^ a (\
cos/) and the x-axis, about: a) the x-axis, b) the y-axis,
and c) the axis of symmetry ot the figure.
1702. Find the volume of a solid formed by the rotation of
acos 8 /, y bsm*t about the //-axis.
the astroid *
1703. Find the volume of a solid obtained by rotating the
cardioid r
a(l -hcostp) about the polar axis.
1704. Find the volume of a solid formed by rotation of the
acos 2 <p about the polar axis.
curve r
1705. Find the volume of an obelisk whose parallel bases are
rectangles with sides A, B arid a, ft, and the altitude is h.
1706. Find the volume of a right elliptic cone whose base is
an ellipse with semi-axes a arid &, and altitude h.
^= a\ which are
1707. On the chords of the astroid *'/
{/'/
to
the
are
constructed
*-axis,
parallel
squares whose sides are
equal to the lengths of the chords and whose planes are perpendicular to the A#-plane. Find tfte volume of the solid formed by
these squares.

166

of

[Ch. 5

Definite Integrals

1708. A circle undergoing deformation


the points of its circumference lies on

moving so that one


the y-axis, the centre

is

describes an ellipse
^^-=1, and the plane of the circle is
perpendicular to the jq/-plane. Find the volume of the solid
generated by the circle.
1709. The plane of a moving triangle remains perpendicular
to the stationary diameter of a circle of radius a. The base of
the triangle is a chord of the circle, while its vertex slides along
a straight line parallel to the stationary diameter at a distance h
from the plane of the circle. Find the volume of the solid (called
a conoid) formed by the motion of this triangle from one end of
the diameter to the other.
1710. Find the volume of the solid bounded by the cylinders
z
2
z*
a*.
a and y
1711. Find the volume of the segment cut off from the ellip"'

+ =

= * by the plane x = a.
paraboloid |21712. Find the volume of the solid bounded by the hyperbo-

tic

loid of

^ -f

one sheet

1713. Find the

^-=1 and

rj

volume

the planes 2
X2

of the ellipsoid
^2

U2

and z

li.

+ ^ + ^2"=

10. The Area of a Surface of Revolution

Sec.

The area

of

formed by the rotation, about


a and x
f(x) between the points x

a surface

arc of the curve y


the formula

the

= b,

is

an
expressed by

x-axis, of

Vl+y'*dx
(ds

is

(1)

the differential ol the arc of the curve).

Zfta
Fig. 54

If

surface

of the curve
cbtained from formula

the equation

$x

is

is

(!)

represented differently, the area of the


by an appropriate change of variables.

Sec.

The Area

10]

of

a Surface of Revolution

Example 1. Find the area of a surface formed by


2
x-axis, of a loop of the curve 9i/
;t(3
x)* (Fig. 54).
Solution. For the upper part of the curve, when

167
rotation, about

= -- (3

mula

x)

y~x. Whence

the differential of the arc

(X*<3,

ds=

X ~^

r_dx.

2
the area of the surface

(1)

the

we have
Fromfor-

= 2n

Example
of

the

2.

\($6

Find the area


x

cycloid

=a

of a surface

s\nt)\

(t

y = a(l

formed by the rotation of one arc


cost) about its axis of symmetry

(Fig. 55).

Solution. The desired surface is formed by rotation of the arc OA about


the straight line AB, the equation of which is x
na. Taking y as the independent variable and noting that the axis of rotation
AB is displaced relative to the #-axis a distance na, we

will

have

Passing to the variable

(na

/,

we obtain

da

n
=

4na

nsin

/sin

y + sinf sin

~-

Fig. 56

dt-j

The dimensions

of a parabolic mirror AOB are indicated


required to find the area of its surface.
1715. Find the area of the surface of a
spindle obtained by
rotation of a lobe of the sinusoidal curve
about the

1714.

in Fig. 56.

It

is

ys'mx

#-axis.

1716. Find the area of the surface formed


a part

of

about the

the

tangential curve

t/

= tan*

by the rotation
from Jt =
to

of

A:==^-,

jc-axis.

1717. Find the area of the surface formed


by rotation, about
the x-axis, of an arc of the curve y
e-* 9 from x
to x

168

\Ch. 5

Definite Integrals

1718. Find the area of the surface (called a catenoid) formed

about the x-axis from


by the rotation of a catenary # = acosh
x = to x = a.
1719. Find the area of the surface of rotation of the astroid
3
x'/'-i

-a

/'/

3 3
'

about the y-axis.

1720. Find the


JC

= -^/

area

the surface of rotation of the curve

of

y=\

from

In (/about the *-axis

to

y = e.

1721*. Find the surface of a torus formed by rotation of the


2
x*
b) ^=a* about the *-axis <b>a).
(y
1722. Find the area of the surface formed by rotation of the

circle

ellipse

^+^ =

about:

1)

the *-axis, 2) the y-axis

(a>6).

1723. Find the area of the surface formed by rotation of one


arc of the cycloid x
cos t) about: a) the
a(t
sin/) and r/
a(l
x-axis, b) the y-axis, c) the tangent to the cycloid at its highest

point.
1724. Find the area of the surface formed by rotation, about
the j^-axis, of the cardioid

x
t/

= a (2 cost
= a(2sin/

cos

2/),

sin2/).

1725. Determine the area of the surface formed by the rotation


2
2
a cos2<p about the polar axis.
1726. Determine the area of the surface formed by the rotation
of the cardioid r
2a (1 +coscp) about the polar axis.

of the lemniscate r

Sec. 11. Moments. Centres of Gravity. Guldin's Theorems

1. Static moment. The static moment relative to the /-axis of a material


A having mass m and at a distance d from tha /-axis is the quantitv

point

Mi~md.
The

static

moment

relative to the /-axis of a system of n material


points
in the plane of the axis and at distances

with masses m,, m 2 ..., m n lying


d lt d 2 ..., d n is the sum
,

M i=

m A2
=

(1)

where the distances of points lying on one side of the /-axis have the
plus
sign, those on the other side have the minus sign. In a similar manner we

define the static moment of a system of points relative to a


plane.
If the masses
continuously fill the line or figure of the x#-plane, then the
static moments
about the x- and /-axes are expressed ^respectivex and
not as the sums (1). For the cases of
ly) as integrals and
geometric figures,
Itie density is considered
equal to unity.

My

Sec.

is

Moments. Centres of Gravity. Guldin's Theorems

II]

In particular:
the arc length,

for

1)

the curve *

= *(s);

we have

y=y(s) whare
t

= V(dx)* + (dy)*

is

M Y =.

(s) ds\

(s)

2) for a

plane figure bounded by th3 curve y


a and y
b we obtain

bounded by the

Find the

static

straight lines:

Solution. Here, y

(2)

= y(x),

ihz Jt-axis and

two

= b II

x\y\dx.

(3)

the x- and

/-axes of a triangle

a
1.

ds

Fig. 58

Example

the differential of the arc);

Fig. 57

vertical lines x

parameter

(ds

the

169

moments about

~-f-^
a
b

l,

= 0, =
//

Applying formula

(Fig. 57)

(3),

we

obtain

ab*

and

2. Moment

of inertia. The moment of inertia, about an /-axis, of a imfe2


point of mass m at a distance d from the /-axis, is the number l t -=-tnd .
of
n
material
of
an
a
The moment of inertia, about
/-axis,
system
points
with masses m lt m 2t ...,
n is the sum
rial

170

__

[Ch. 5

Definite Integrals

where d lf d 2 ..., d n are the distances of the points from the /-axis. In the
case of a continuous mass, we get an appropriate integral in place of a sum.
Example 2. Find the moment of inertia of a triangle with base b and
altitude h about its base.
Solution. For the base of the triangle we take the x-axis, for its altitude,
the y-axis (Fig 58).
Divide the triangle into infinitely narrow horizontal strips of width dy t
which play the role of elementary masses dm. Utilizing the similarity of

we

triangles,

obtain
j
= bL
dm

y
.

dy

and

Whence

3. Centre of gravity. The


figure (arc or area) of mass

coordinates of the centre of gravity of a plane


are computed from the formulas

moments of the mass. In the case of geometnumerically equal to the corresponding arc or area.
For the coordinates of the centre of gravity (x, y) of an arc of the plane
curve y
connecting the points A[a f (a)] and B [6, f (b)],
f (x) (a^x^b),
where

MX

ric figures,

and

My

are the static

the mass

is

we have
B

t>

b
(

-A\yds

2
y 1^1 +(y') dx

'}

The coordinates

dx

of the centre of gravity (x, y) of the curvilinear


trapezoid

x^b, Q^y<^f(x) may

be computed from the formulas

~~

-.(y^dx

^xydx
a

*~~

a_
^

,3

where

S=f y dx

is

the area of the figure.

There are similar formulas for the coordinates of the centre of


gravitv
6
y of
volume.
Example 3. Find the centre of gravity of an arc of the semicircle
,,2^n*- i,i^n\
(pig

59)

Sec

Moments. Centres

11]

We

Solution.

Guldin's Theorems

of Gravity

171

have

and

Whence

f,

'

yds--

xJ ..

,.

yV-.v

^=
y a

Hence,

4. Guldin's theorems.
Theorem 1. The area of a surface obtained by the
plane curve about some axis lying in the. same plane

intersecting

it

circumference of

is

to
circle

equal
the

rotation of an arc of
as the curve and not
the product of the length of the curve by the
described by the centre of gravity of the arc of

the curve.

Theorem 2. The volume of a solid obtained by rotation of a plane figure


about some axis lying in the plane of the figure and not intersecting it is
equal to the product of the area of this figure by the circumference of the
circle described by the centre of gravity of the figure.

Fig. 59

1727. Find the static moments


a segment of the straight line
x
o

lying between the axes.

y
o

about

the

coordinate

axes of

__

172

Definite Integrals

1728. Find the static

about

its

moments

of a rectangle,

[Ch. 6

with sides a and

&,

sides.

1729. Find the static moments, about the x- and t/-axes, and
the coordinates of the centre of gravity of a triangle bounded by
the straight lines x + y
Q.
a, x
Q, and y
1730. Find the static moments, about the x- and (/-axes, and
the coordinates of the centre of gravity of an arc of the astroid

_t_

_i^

-If/*

>

==aT

>

lying in the first quadrant.


1731. Find the static moment of the circle
r

= 2asin<p

about the polar axis.


1732. Find the coordinates of the centre of gravity of an arc
of the catenary

y = a cosh

x=

a to x
a.
Find the centre of gravity of an arc of a circle of radius a
subtending an angle 2a.
1734. Find the coordinates of the centre of gravity of the arc
of one arch of the cycloid

from

1733.

= a(t

sin/); y

= a(\

cos/).

1735. Find the coordinates of the centre of gravity of an area


bounded by the ellipse
2 -|-=l and the coordinate axes (
1736. Find the coordinates
bounded by the curves

of the centre of gravity of

1737. Find the coordinates of the centre of


gravity
bounded by the first arch of the cycloid
x = a(t
sin/), r/ = a(l
cos/)
and the jc-axis.
1738**. Find the centre of gravity of a
hemisphere
lying above the ;q/-plane with centre at the origin.

of

an area

an area

of radius a

1739**. Find the centre of gravity of a homogeneous


right
circular cone with base radius r and altitude h.
1740**. Find the centre of gravity of a homogeneous hemi-

sphere of radius a
origin.

lying

above the jo/-plane with centre

at the

&?c. 12]

Applying Definite Integrals

1741. Find the


its

moment

to Solution of Physical

Problems

173

inertia of a circle of radius a about

of

diarneler.

1742. Find the moments of inertia of a rectangle with sides


a and b about its sides.
1743. Find the moment of inertia of a right parabolic segment
with base 26 and altitude ft about its axis of symmetry.
1744. Find the moments of inertia of the area of the ellipse
2
JC

U2

^ + ^=1

about

its

principal axes.

1745**. Find the polar moment of inertia of a circular ring


and R t (R <RJ> that is, the moment of inertia
with radii
R^
about the axis passing through the centre of the ring and perpen}

dicular to its plane.


1746**. Find the moment of inertia of a homogeneous right
circular cone with base radius R and altitude H about its axis.
1747**. Find the moment of inertia of a homogeneous sphere
about its diameter.
of radius a and of mass
1748. Find the surlace and volume of a torus obtained by
rotating a circle of radius a about an axis lying in its plane
and at a distance b (b>a) from its centre.
1749. a) Determine the position of the centre of gravity of

an arc of the astroid x -\-i/ T = a* lying in the first quadrant.


b) Find the centre of gravity of an area bounded by the curves
2
= 2px and x* = 2py.
t/
17f>0**. a) Find the centre of gravity of a semicircle using
Guldin's theorem.
b) Prove by Guldin's theorem that the centre of gravity of
a triangle is distant from its base by one third of its altitude
Sec.

12. Applying Definite

Integrals to the

Solution of Physical Problems

1. The path traversed by a point. If a point is in motion along some


curve and the absolute value of the velocity o~/(/) is a known function of
the time t, then the path traversed by the point in an interval of time
is
'. *

Example

1.

The velocity

of a point

Find the path


ing

the

during

= 0. 1/

covered by the point


ol motion.

commencement
this interval?

in

is

m/sec.

7=10

the interval of time


sec followis
the mean velocity cf motion

What

174

[Ch. 5

Definite Integrals

Solution.

We

have:
10

t*

.-Jo.irt-o.i

T = 250

metres

and

=-=25
2. The work
of the x-axis,

of a force.
then the work

If

m/sec.

a variable force

X=f(x)

acts in the direction

over an interval [x ly x z ]

of this force

is

A=
2. What work has to be performed to stretch a spring 6 cm, if
kgf stretches it by 1 cm?
Solution, According to Hook's law the force X kgf stretching the spring
kx, where k is a proportionality constant.
by x m is equal to
100v.
we get
and
l kgf,
100 and, hence, X
0.01
Putting x
Whence the sought-for work is

Example

a force of

X=
m

0.08

0.06

A=

X=

100 x dx

= 50 x

= 0. 18

kgm

3. Kinetic energy. The kinetic energy of a material


velocity v is defined as

point of mass

and

mv*

The

mv m

2%

kinetic
...,

mn

energy of a system of n material points with masses


having respective velocities t; lf v 2 ..., v n is equal to
,

To compute the

kinetic energy of a solid, the latter is appropriately partitioned into elementary particles (which play the part of material points); then
by summing the kinetic energies of these particles we get, in the limit, an
integral in place of the sum (1).
Example 3. Find the kinetic energy of a homogeneous circular cylinder
of density 6 with base radius R and altitude h rotating about its axis with
angular velocity CD.
Solution. For the elementary mass dm we take the mass of a hollow
cylinder of altitude h with inner radius r and wall thickness dr (Fig. 60).
We have:

Since the linear velocity of


kinetic energy is

the

mass dm

is

equal

to

t;

/-co,

the elementary

Sec.

12]

Applying Definite Integrals

to

Solution of Physical Problems

175

Whence
2

r 9 dr^=

nco 6/?

fc

4. Pressure of a liquid. To compute the force of liquid pressure we use


Pascal's law, which states that the force of pressure of a liquid on an area S
at a depth of immersion h Is

where y

is

the specific weight of the liquid.

Example 4. Find the force of pressure experienced by


r submerged vertically in water so that its diameter
is

radius

water surface (Fig

semicircle of
with the

flush

61).

Solution, We partition the area of the semicircle into elements


strips
parallel to the surface of the water. The area of one such element (ignoring
higher-order infinitesimals) located at a distance h from the surface is
ds

The pressure experienced by

where y

is

^ 2xdh = 2
this

V'*

element

h2

dli.

is

the specific weight of the water equal to unity.


the entire pressure is

Whence

=2

The velocity of a body thrown vertically upwards with


velocity V Q (air resistance neglected), is given by the

1751.
initial

C
J

ITS

Definite Integrals

\Cfi.

formula

ff'.

where t is the time that elapses and g is the acceleration of gravAt what distance from the initial position will the body
ity.
be in t seconds from the time it is thrown?
1752. The velocity of a body thrown vertically upwards with
initial

velocity

(air

resistance

allowed

for)

is

given

by the

formula

where

t is the time,
g is the acceleration of gravity, and c is
a constant. Find the altitude reached by the body.
1753.
point on the x-axis performs harmonic oscillations

about the coordinate origin;

its

velocity

given by the formula

is

where t is the time and t;


co are constants.
Find the law of oscillation of a point if when / =
it had
an abscissa * = 0. What is the mean value of the absolute magnitude of the velocity of the point during one cycle?
1754. The velocity of motion of a point is v = te~"' Qlt m/sec.
Find the path covered by the point from the commencement of
motion to full stop.
,

1755. A rocket rises vertically upwards. Considering that when


the rocket thrust is constant, the acceleration due to
decreasing

weight of the rocket

increases by the law

find the velocity at any instant of time /,


is zero. Find the altitude reached at lima

/==^~^
if
/

the

(a

initial

ftf

>0),

velocity

r
1756*. Calculate the work that has to be done to
pump the
water out of a vertical cylindrical barrel with base radius R and
altitude H.
1757. Calculate the work that has to be done in order to
pump
the water out of a conical vessel with vertex downwards, the
radius of the base of which is R and the altitude H.
1758. Calculate the work to be done in order to
pump water
out of a semispherical boiler of radius R
10 m.
1759. Calculate the work needed to pump oil out of a tank

through an upper opening (the tank has the shape of a cylinder


with horizontal axis) if the specific weight of the oil is
y, the
length of the tank H and the radius of the base R.
1760**. What work has to be done to raise a body of massm
from the earth's surface (radius R) to an altitude ft? What is
the work

if

the body

is

removed

to infinity?

Sec.

12]

Applying Definite Integrals

What work
*2

=10 cm?
1762**.

and

Problems

177

Two

1761**.
on the

lie

to Solution of Physical

electric charges *
100 CGSE-and e l==200 CGSE
x-axis at points *
and *,
1
cm, respectively.
will be done if the second charge is moved to point

length

p=10kgfcm

cylinder with a movable piston of diameter


/

= 80cm

2
.

is

filled

with

What work must be done

steam

at

to halve the

D = 20 cm
pressure

volume of

steam with temperature kept constant (isoihermic process)?


1763**. Determine the work performed in the adiabatic expan3
of
air
sion
and pressure
(having initial volume u =l
2
8
10 m ?
p _=l kgf/cm ) to volume u,
1764**. A vertical shaft of weight P and
radius a rests on a bearing AB (Fig. 62).
The frictional force between a small part a
of the base of the shaft and the surface of
the support in contact with it is F==fipa,
const is the pressure of the shaft
where p
on the surface of the support referred to
unit area of the support, while pi is the coefficient of friction. Find the work done by the
frictional force during one revolution of the
the

ft

shaft.

1765**. Calculate the kinetic energy of a


and radius R rotating with
angular velocity G> about an axis that passes through its centre
perpendicular to its plane.
1766. Calculate the kinetic energy of a right circular cone of
mass
rotating with angular velocity CD about its axis, if the
radius of the base of the cone is R and the altitude is H.
1767*. What work has to be don? to stop an iron sphere of
2 me'res rotating with angular velocity w = 1,000 rpm
radius R
j
about its diameter? (Specific weight of iron, y = 7.8 s/cm .)
1768. A vertical triangle with base 6 and altitude h is submerged vertex downwards in water so that its base is on the
surface of the water. Find the pressure of the water.
1769. A vertical dam has the shipa of a trapezoid. Calculate
the water pressure on the dam if we know that the upper base
20 m.
a
70 m, the lower base 6=50 m, and the height h
1770. Find the pressure of a liquid, whose specific weight is y.
on a vertical ellipse (with axes 2a and 26) whose centre is submerged in the liquid to a distance h, while the major axis 2a
of the ellipse is parallel to the level of the liquid (h^b).
1771. Find the water pressure on a vertical circular cone
with radius of base R and altitude H submerged in walei vertex
downwards so that its base is on the surface of the water.

disk of mass

178

Definite Integrals

[C/i.

Miscellaneous Problems

1772. Find the mass of a rod of length /


100 cm if the linear
density of the rod at a distance x cm from one of its ends is

= 2 + 0.001 x

g/cm.

1773. According to empirical data the specific thermal capacity


of water at a temperature
is

= 0.9983
What
from

/C (0^/<100)
5.184xlO- + 6.912xlO5

quantity of heat has to be expended to heat


to 100 C?

0C

of water

of
of

1774. The wind exerts a uniform pressure p g/cm on a door


width b cm and height h cm. Find the moment of the pressure
the wind striving to turn the door on its hinges.
1775.

What

is

the

force

of

attraction

of

a material

and mass
on a material point of mass
a straight line with the rod at a distance a from one
length

rod of

lying on
of its ends?

1776**. In the case of steady-state laminar (low of a liquid


through a pipe of circular cross-section of radius a, the velocity
of flow v at a point distant r from the axis of the pipe is given
by the formula

difference at the ends of the pipe, |i is


is the pressure
the coefficient of viscosity, and / is the length of the pipe.
Determine the discharge of liquid Q (that is, the quantity of
liquid flowjng through a cross-section of the pipe in unit time).
1777*. The conditions are the same as in Problem 1776, but
the pipe has a rectangular cross-section, and the base a is great
compared with the altitude 26. Here the rate of flow u at a point
M(x,y) is defined by the formula

where p

Determine the discharge

of

liquid Q.

1778**. In studies of the dynamic qualities of an automobile,


use is frequently made of special types of diagrams: the velocities v are laid off on the Jtr-axis, and the reciprocals of corresponding accelerations a, on the (/-axis. Show that the area S bounded
v 2t and
v l and v
by an arc of this graph, by two ordinates v
by the *-axis is numerically equal to the time needed to increase
the velocity of motion of the automobile from v l to v 2 (accelera-

tion time).

Sec. 12]

Applying Definite Integrals

1779.

downward

of the

horizontal

beam

vertical load

beam, and

of

to Solution of Physical

of length

uniformly

support

Problems

179

/ is in- equilibrium due to


distributed over the length

reactions

A and

fi(yl==5==-y-j

directed vertically upwards. Find the bending moment


x in
a cross-section x, that is, the moment about the point P with
abscissa x of all forces acting on the portion of the beam AP.
1780. A horizontal beam of length / is in equilibrium due to
support reactions A and B and a load distributed along the
kx, where x is the distance
length of the beam with intensity q
from the left support and k is a constant factor. Find the bend-

ing

moment
Note.

Mx

in cross-section x.

The intensity

of

load distribution

is

the

load

(force) referred to

unit length.

1781*. Find the


sinusoidal current

during a cycle

quantity

of heat released

in a conductor

by an alternating

with resistance

/?.

Chapter VI

FUNCTIONS OF SEVERAL VARIABLES

Sec. 1. Faiic Notions

1. The concept of a function of several variables. Functional notation.


variable quantity 2 is called a single-valued function of two variables jc,
y, if to each set of their values (x, //) in a givm range there corresponds a
unique value of z The variables x and y are called arguments or independent
variables. The functional relation is denoted by

Similarly,

we

Fxample

= /(*,

y).

define functions of three or more arguments.


Express the volume of a cone V as a function of

1.

eratrix x and of its base radius y


Solution. From geometry we know that the

where h

is

the altitude of the cone. But h

This

is

volume

**

2
-

of a

cone

its

is

Hence,

the desired functional relation.


of the function z^f(x.y)

The value
point
is

that

(a.b).

is,

gen-

when

x=^-a

and y

at a
bt

denoted by / (a,b) or f (P) Generally speakthe geometric representation of a func-

ing,

tion like

in

(x,y)

nate system X, Y. Z

Example

2.

Find/

is

(2,

Fig. 63

Solution.

Substituting

r=2

and t/=

3,

we

find

rectangular

a surface (Fig.

3)

and/1,

coordi63).
if

Sec.

Basic Notions

1)

Putting

*=1

and replacing y by

we

[SI

have

will

SL\

*'
thai

is,

/(l.

2. Domain

)=f(*.

0).

of definition of a function. By the domain of definition of a


function ?
f(x, y) we understand a set of points (*, r/) in an jq/-plane in
which the given function is defined (that is to say, in which it takes on definite real
values) In the simplest cases, the domain of definition of a function U a finite or infinite part of the jo/-plane bounded by one or several
curves (the boundan, of the domain).

Similarly, for a function of three variables u


f(x, y, z) the
definition of the function is a volume in At/z-space.
Example 3. Find the domain of definition of the function

domain

of

>

<

2
x2
function has real values if 4
or x* + y 2
4.
y
inequality is satisfied hy the coordinates of points lying inside a
circle of radius 2 with centre at the coordinate origin. The domain of definition oi the function is the interior of the circle (Fig 64).

Solution. The

The

latter

Fig. 64

Example

4.

Fiy

Find the domain


z

Solution.

The

first

term

of definition of the function

= arc sin

or
|

function

when

|/

xy

of the function

The second term has

when

65

real

defined for

is

values

.The domain

if

xr/^O,

of

1<~
in

i.e.,

definition

of

or

two

cases:

the

entire

is

shown

in Fig. 65

and includes the boundaries

of the

domain.

Functions

182

of Several Variables

[Ch. 6

3. Level lines and level surfaces of a function. The level line of a function 2
f(x, y) is a line / (*, y)-C (in an *r/-plane) at the points of which
C (usually labelled in
the function takes on one and the same value z

drawings).

The
iace

level surface of a
C, at the
z)

/ (x, y,

function of three arguments u~f(x,


points of which the function takes
value u~C.
Example 5. Construct the
the function z
x*y.
Solution. The equation of

has the form x 2 y


Putting

C = 0,

1784. Find /(1/2, 3), /(I, -1),

__x

2,

~ -j
....

level

lines

of

the level lines

we

get a family

1782. Express the volume V of a


regular tetragonal pyramid as a function of its altitude x and lateral edge y.
1783. Express the lateral surface S
of a regular hexagonal truncated pyramid as a function of the sides x and y
of the bases and the altitude z.

Fig. 66

Find

1,

or y

is a sura constant

z)

on

lines (Fig. 66).

of level

1785

=C

if

x,

f(

f(y,x),

y),

if

f(*,y)

1786. Find the values assumed by the function

at points of the

parabola y

z
,

and construct the graph

function

1787. Find the value of the function


Z
2

at points of the circle x


1788*. Determine f(x),

1789*. Find f(x, y)

if

rrr

+y*=R
if

2
.

of the

Sec.

__
Basic Notions

1]

183

= V7+/(l/x 1). Determine the functions /


y= 1.
Letz = */(-V Determine the functions f and

1790*. Let *
z

if

=x

and

when

1791**.

if

when x=\.
1792. Find and sketch
lowing functions:
a)

b)

z==Y\ x
z=\-\-V

domains

the

of

definition of the fol-

(x

=
2 - x + arc cos
z-l/l-^ +
z = arc sin
X
2 = V 7IT
2 =

c) z

d)

//;

e)
f)
'

g)
h)

1793.

Find the domains

of

the following functions of three

arguments:
a)

u=-\x +

b) u

z\

= ln(xyz):

c)

w=--arc sin jc+arc sin #

d)

u = V\x

+ arc sinz;

z\

1794. Construct the level lines of the given functions and determine the character of the surfaces depicted by these functions:
a)

z^x + y;

b) 2

= ^+y

c) z

= x*-y*;

d) z

2
;

e)

O^

1795. Find the level lines of the


following functions:
a) z
b) 2

= ln(*'+f/);
= ar

d) z

= /(y

a*);

1796. Find the level surfaces of the functions


pendent variables:
x y-\-z;
a) u
*
'
'
b) u =
c) u = J

= +

of three inde-

_ _

[Ch. 6

Functions of Several Variables

184

Sec. 2. Continuity

1. The limit of a function.

= /(*, y)
there
e >
2

is

A number A

is

called the limit of a function

approaches the point P (o.


is a 6 >
such that when
< Q < 6, where Q = |/\x
the distance between P and P', we have the inequality
as

the

point

(x, y)

ft),

if

a)*

for

+ U/

any
6)

y)A\<e.
f(x, y) = A.

/(*,

In this case we write


lim
X-K2
y-*b

2. Continuity and
continuous at a point

points of discontinuity.

(a,

b)

function z=f(x, y)

is

called

if

lim f(x
x-+a

0)

= /(a,

b).

A function that is continuous at all points of a given range is called


continuous over this range
A function /(AT, y) may cease to be continuous either at separate points
(isolated point of discontinuity) or at points that form one or several lines
(lines of discontinuity) or (at times) more complex geometric objects.
Example 1. Find the discontinuities of the function

_*+>
Solution.
zero. But **

The function
-r/

function has for

its

or y

will be meaningless if the denominator becomes


2
x is the equation of a parabola. Hence, the given

discontinuity the parabola y

x2

1797*. Find the following limits of functions:


a)

c)li n

liin(*?^)sinl;

M;

e)lin-L;

1798. Test the following function for continuity:


ft x

/)

= / V\x*
I

if

when * 2 -r-// 2 <l,


when x* 4-//*> 1.

1799. Find points of discontinuity of the


a) z

ln;

c)

e-

functions:

Sec. 3]

Partial Derivatives

1800*.

Show

that the function

SL

when
when

-{
is

185

**
jt

=^=0,

</'

= =

continuous with respect to each of the variables x and y sepabut is not continuous at the point (0, 0) with
respect to

rately,

these variables together.


Sec. 3. Partial Derivatives

t. Definition of a partial derivative.


example, y constant, we get the derivative

If

f(x,y), then assuming, for

which

is called the partial derivative of the function z with


respect to the
variable x. In similar fashion we define and denote the partial derivative of
the function z with respect to the variable y It is obvious that to find partial
derivatives, one can use the ordinary formulas of differentiation.
Example 1. Find the partial derivatives of the function

we

Solution. Regarding y as constant,

dz

get

1
t

dx

tan

cos 2

dz

we

2.

Find the

2.

2*

partial derivatives of the following function of three

= *y z + 2*

3y

+ 2 + 5.

Euler's theorem. A function


if for every real factor k

degree n

2x'

arguments:

Solution.

I/sin
y

have

will

Example

Similarly, holding x constant,

f (x,

y) fs called a

we have

f (kx, ky) --=

ttf

homogeneous function of

the equality
(x,

//)

[Ch. 6

Functions of Several Variables

186

rational integral function will be

and the same degree.


The following relationship holds
of degree n (Euler's theorem):
xfx (x, y)

Find the
1801. z

homogeneous
for a

if

all

homogeneous

its

terms are of one

differentiable function

+ yfy (x, y) = nf (x, y).

partial derivatives of the following functions:


1808. 2
3axy.

1809. 2

1810. 2

1811. 2

1806.

1807. z

= arc tan

1814. Find /;(2,

and fy (2,

1)

1815. Find /;(!, 2, 0),

/i(l, 2, 0),

/(*, y,z)

Verify Euler's theorem


ples 1816 to 1819:
1816. f(x,y)
1817. z

if

1)

xy

f(x,y)

ft

(l, 2, 0)

+L

if

= ln(xy-\-z).

on homogeneous functions

= Ax + 2Bxy-Cy
3

in

Exam-

1818. /

=
J-

1820.

where

2
1821. Calculate

1822.

Show

that

1823.

Show

that

1824.

Show

that

1825.

Show

J826. Find

that

and y
57,

= 2,

if

x+

g+fj+S-0,

g+g+g-1,

-.(,.

y),

it

if

= ln(*'

=
u = (x-y)(y-z)(z- X

if

).

if

-*+J=J.

Sec. 4]

Total Differential of a Function

1827. Find z--^z(x y)

knowing that

(J2

X 2 -4-

= slny

J/
^and z(x, y)

187

when *=1.

Through the point M(l,2, 6) of a surface z = 2x*+y*


drawn planes parallel to the coordinate surfaces XOZ and
YOZ. Determine the angles formed with the coordinate axes by
the tangent lines (to the resulting cross-sections) drawn at their
common point M.
1829. The area of a trapezoid with bases a and b and alti1828.

are

tude h

is

equal

to

S=

/,(fl

+ &) A.

Find

g, g,

and,

using

the drawing, determine their geometrical meaning.


1830*. Show that the function

0,

has partial derivatives fx (x, y) and fy (x, y) at the point (0,0),


although it is discontinuous at this point. Construct the geometric image of this function near the point (0, 0).
Sec. 4. Total Differential of a Function

1. Total

= /(*i

y)

increment of

The

function.

total

increment

of

function

the difference

is

Az-Af (x,

+ Aj/)-f (*,

#)--=/(*+ Ax,

y).

a function. The total (or exact) differential of


a function z
f(x, y) is the principal part of the total increment Az, which
is linear with respect to the increments in the arguments Ax and A//.
The difference between the total increment and the total differential of

2. The

total differential of

is an infinitesimal of higher order compared with Q


At/*.
\^&x*
function definitely has a total differential if its partial derivatives are
continuous. If a function has a total differential, then it is called differenThe differentials of independent variables coincide with their incret table.
ments, that is, dx=kx and dy=ky. The total differential of the function
z
/(x, y) is computed by the 'formula

the function

dz

=^ dX +

d2=

dz

dy

d-y

Similarly, the total differential of a function of three arguments u


is computed from the formula
.

du

Example

1.

= du
-3- dx
.

dx

-j'

du
^- d y
dy
,

du

dz.
+ -rdz
.

For the function


f(x

find the total increment

and the

y)=x* + xyy
total

differential.

=/

(x, y, z)

[Ch. 6

Functions of Several Variables

J88
Solution.

f(x+Ax,

= 2x

+ Ay) = (x -f Ax)

Ax + AA

+x

= [(2x + y) A* + (x

-f (x

+ A*) (y

Ay + y Ax + Ax2 Ay
2y) A^l

-f-

Ay)

2y Ay

+ (A* + AA> Ay

(y -f

Ay

Ay)

2
;

Ay

(2x + y) A* -{-(*
2y) Ay is the total differential of
Here, the expression d/ =
2
2
infinitesimal of higher order
is an
the function, while (A* -f AJC*
)
comared with VAx 2 +Ay 2
compared
Example 2. Find the total differential of the function

AyAy

Solution.

3. Applying
For

sufficiently

Q= y Av 2 -f

Ay

2
,

the total differential of a function to approximate calculations.


small |AA:| and |Ay| and, hence, for sufficiently small
we have for a differentiate function z f(x t y) the approxor

Az^dz

imate equality

dz

3.

Example

The

Solution,

we

altitude of a cone is //
30cm, the radius of the base
the volume of the cone change, if we increase
by

= 10cm. How will


3mm and diminish R
fl

by

The volume

mm?

of the

cone

is

V = -~-nR 2 H. The change


o

in

volume

replace approximately by the differential

AV =^ dV =

-j

n (2RH dR + R* dH) =

= lji(

2.10.30.0.1

+ 100.0.3) =

s 01
Example 4. Compute 1.02
approximately.
Solution. We consider the function z^x^.
considered the increased value of this function
Ay 0.01. The initial value of the function z

lOjiss

31. 4 cm*.

In x
8
Hence, 1.02

01

The desired number may be


when jc=l, y = 3, Ajc = 0.02,
s
l =l,

Ay = 3-1.0.02+

1- In 1-0.01

=0.06.

^ 1+0.06=1.06.
=

1831. For the function

f(x,y)
x*y find the total increment
total differential at the point (1, 2); compare them if
0.1 f Ay
0.2.
a) Ax=l, A//-2;
b) A*
1832. Show that for the functions u and v of several (for

and the

example, two) variables the ordinary rules of differentiation holcb


v

du

udv

Sec. 4}

Find the

total

Total Differential of a Function

differentials of the following functions:

1841.

z- In tan i.
x

1842. Find df(l,


1836. z

1837.

= si

= ln(x*+y*).
= lnl+i.
/*,

1838. z

1844.

1839.

1845.

= arc tan

if

~*'

=
u =
u=

1846. w

-2-+

arctan^.

1847. Find d/

+ arctan-*.
1848.

tj)

1),

1843 - "

2=yx?.

1840. 2

189

(3, 4, 5)

if

One

side of a rectangle is a
10 cm, the other &
24cm.
a diagonal / of the rectangle change if the side a is
increased by 4
and b is shortened by 1

How

will

mm

mm?

the change and compare it with the exact value.


A closed box with outer dimensions
1849.
and 6 cm is made of 2-mm-thick plywood.

Approximate

10 cm, 8 cm,
Approximate the

volume

of material used in making the box.


1850*. The central angle of a circular sector is 80; it is desired
to reduce it by 1. By how much should the radius of the sector
be increased so that the area will remain unchanged, if the
original leng:h of the radius is 20 cm?
1851. Approximate:
2

a)

(1.02)'-

c)

sin32-cos59

calculating

(0.97)

sin 60

b)

(when converting degrees into radius and


three significant figures; round off the

take

last digit).

Show

that the relative error of a product is approximasum of the relative errors of the factors.
1853. Measurements of a triangle
yielded the following
data:
side
a=-100m2m. side
fc
angle

1852.

tely equal to the

ABC

601.

To what

degree

side c?
1854. The oscillation
from the formula

of

period

= 200m3m,

accuracy can

of

we compute the

pendulum

is

computed

190

_ _

[Ch. 6

Functions of Several Variables

where

is

the length of the

of gravity. Find the error,


a
result of small errors A/

1855.

pendulum and g is the acceleration


when determining T, obtained as a

= and Ag = (J in measuring / and


and P (x,
The distance between the points P (*
,

g.

equal to Q, while the angle formed by the vector P P


the x-axis is a. By how much will the angle a change
dx, y
dy)?
point P(P is fixed) moves to P l (x

y)

with

is

if

the

Sec. 5. Differentiation of Composite Functions

1. The case of one independent variable* If z


f(x, y) is a differentiate
function of the arguments x and y, which in turn are differentiate functions
of an independent variable /,

then the derivative of the composite function z


puted from the formula
dz

____i

dt

'"dxdt^dydt

= /[<p(0i

^(01 ma X

be com-

'

In particular, if / coincides with one of the arguments, for instance


then the "total" derivative of the function z with respect to x will be:

x,

dzdy

Example

1.

Find

~,

if

where * = cos/, y

y,

Solution.

e'*+

From formula

^3(-sinO +

Example

2.

* 8X+

(1)

^-2^

Find the partial derivative


z

Solution.

t*.

we have:

= e*y

and the

total derivative

if

where y

jZ^syety.

From formula

(2)

we obtain

2. The case

of several independent variables. If z is a


composite function of
variables, for instance, z
where *=q> (u v)
f(x,y),
y=ty(u t v) (u and v are independent variables), then the partial derivatives z
with respect to u and v are expressed as

several

independent

dz dy
~

Sec. 5]

and

Differentiation of Composite Functions

~~-~

dzdy

dz__dzdx

191

In all the cases considered the following formula holds:


.

dz

dz

-j-

dx

dx

dz

+ dy
5- dy
y

(the invariance property of a total differential).

Example
p

Find ^- and ^du


dv

3.

z=:f(x,

if

x=uv, y=

where

y),

Solution. Applying formulas (3) and

we

(4),

get:

and

Example

Show

4.

that the

function

Solution.

ment

x*

The function

+ y*=t,

<p

(x*

+y

satisfies

the

equation

dx

dy

depends on x and y via the intermediate

cp

argu-

therefore,

dz__dzdt___

and

dzdt

\z

rt

Substituting the partial derivatives into the left-hand side of


tion,

we

get
t/

that

is,

*~^

i)2

the function z satisfies the given equation.

1856. Find

if

where x = e t

1857. Find

y = \nt.

if

= lnsin-=,
f

1858. Find

where x = 3t\ y = yt'+l.

if
j

= xyz,

where

^=/ +l, y=ln^,


I

the

equa-

192

_ _
Functions of Several Variables

1859. Find

if

where x =

r,

I860. Find

if

1861. Find

= uv

1862. Find

where u =

and
2

= arc
~

and

~ and
j
z = f(u, v),
~
ow

and

= arc tan

1865. Find

1866.

Show

= #*.

<f>(x).

if

where a = A:

where x = usmv,

where

= x^/ + ^.

where x =

3*),
cp

sin

ip,

=R

then

1868.

where

Show

=a

if

cos

and

*_a.

if

= /(*.

//

if

1867. Find

v^e

1
t/

if

= /(u),

that

*/

where y =

oy
,

and

and

tan

if

1863. Find

1864. Find

= cos #,

sin x, v

if

= xy

[Ch.

y> *).

that

where

y=

if

/ is a differentiable function,

dz

dz

then

cos

9 cos \|>>

sin 9,

Sec. 6]

1869.

Show

that the function

w = f(u,
where u = x + at, v

= y + bt

dw

dT= a
Show

satisfies the

1871.

,
,

dw

dt

that the function

=J

equation

Show

v),

satisfy the equation

dw
1870.

19S

Derivative in a Given Direction

that the function

equation x-^ + y -^ = j
1872. Show that the function

satisfies the

satisfies the

(A-

The side

^-

+ xy-^ =

increases at the rate


of a rectangle x -^20
the other side f/
30
decreases at 4 m/sec. What
the rate of change of the perimeter and the area of the rect-

1873.

of 5 m/sec,
is

equation

angle?
1874.

What

is

The equations

of

motion

of a material

point are

the rate of recession of this point from the

coordinate

origin?
1875.

Two boats start out from A at one time; one moves


northwards, the other in a northeasterly direction. Their velocities are respectively 20 km/hr and 40 km/hr. At what rate does
the distance between them increase?
Sec. 6.

Derivative in a

Given

Direction

1. The derivative of a function


function z

7- 1900

= /(#,

and

the Gradient

in a given direction.

y) in a given direction /

= PP,

is

of

a Function

The derivative

of a

where f(P) and


If

the function z

where a

(Ch. 6

Functions of Several Variables

194

is

are values of the function at the points P and


differentiate, then the following formula holds:

/ (P,)
is

the angle formed by the vector

with the x-axis (Fig. 67).

Y
e^

P(*,y)

Fig. 67

In similar fashion we define the derivative in a given


function of three arguments u
f(x, y z). In this case

du

du
= du cos a + du
5- cos p + 3- cos v,
dz
dx
dy
,

/ for a

'

dl

direction

(2)

where a, P, Y are ^ ne angles between the direction / and the corresponding


coordinate axes. The directional derivative characterises the rate of change
of the function in the given direction.
2
2
at the point
3</
Example 1. Find the derivative of the function z 2x
a
a
direction
that
120
with
makes
the
in
x-axis.
angle
P(l, 0)
Solution. Find the partial derivatives of the given function and their
values at the point P:

dz
-.

dx

= 4*;
.

(dz\ =4;
A
3\dxjp

dz

Here,

sina

Applying formula

(1),

we

= sin

120

get

The minus

sign indicates that the function diminishes at the given point and
in the given direction.
2. The gradient of a function. The gradient of a function z f(x, ij) is
3 vector whose projections on the coordinate axes are the corresponding par-

Derivative in a Given Direction

Sec. 6]

tial

195

derivatives of the given function:

dz

dz
(3)

The derivative

of the given function in the direction /


the gradient of the function by the following formula:

is

connected

with

~
That is, the derivative in a given direction is equal to the projection of the
gradient of the function on the direction of differentiation.
The gradient of a function at each point is directed along the normal to
the corresponding level line of the function. The direction of the gradient of
the function at a given point is the direction of the maximum rate of increase
of the function at this point, thlft

on

its

is,

when /=grad

z the derivative

-^

takes

greatest value, equal to

In similar fashion
u -=/(*, y, z):

we

define the gradient

du

.
,

du

of

function

.
,

du

of

three

variables,

The gradient of a function of three variables at each point is directed along


the normal to the level surface passing through this point.
Example 2. Find and construct the gradient of the function z~x*y at
the point P(I, 1).

X2

Fig. 68

Solution.

Compute

the partial derivatives and their values at the point P.

dxjpdz
^T"

Hence, grad
7*

= 2t+J

(Fig. 68).

s
\y'*
.

9.2

_ _
Functions of Several Variables

196

1876. Find

the

x*
derivative of the function z
xy2y*
the direction that produces an angle

at the point P(l, 2) in


of 60 with the x-axis.

= x*

1877. Find the derivative of the function z


at the point Af(l, 2) in the direction from

point

tf (4,

[Ch. 6

6).

1
xy*
2x*y
point to the

this

1878. Find the derivative of the function z


lnYx* y*
the point P(l, 1) in the direction of the bisector of the

at
first

quadrantal angle.

1879. Find the derivative of the function u = x*


3yz + 5 at
in the direction that forms identical
the point Af(l, 2,
1)
angles with all the coordinate axes.
1880. Find the derivative of the function u = xy + yz -)- zx at

the point M(2, 1, 3) in the direction from this point to the


point N(S, 5, 15).
eP + e*)
\n (e*
1881. Find the derivative of the function u
at the origin in the direction which forms with the coordinate
axes x, y, z the angles a, p, y, respectively.
1882. The point at which the derivative of a function in any
direction is zero is called the stationary point of this function.
Find the stationary points of the following functions:

a)

z-=x*

= x* + y*=
u
2y*-{ z*xyyz

b) z
c)

1883.

Show

that the derivative of the function z

+ =

any point of the ellipse 2x* y* C* along the


ellipse is equal to zero.
1884. Find grad z at the point (2, 1) if
at

1885. Find grad z at the point

(5,

3)

if

1886. Find grad u at the point


1887. Find the magnitude and

(1,

2,

3),

point

(2,

2,

1)

direction

taken

normal

to the

u=xyz.

if

of

grad

at the

if

1888. Find the angle between the

*=ln-j-

at the points

(1/2,

1/4)

gradients

and 5(1,

1).

of

the

function

Sec. 7]

Higher-Order Derivatives and Differentials

1889. Find the steepest slope of the surface

197

x'
z
4y*
at the point (2, 1, 8).
1890. Construct a vector field of the gradient of the following
functions:

= je-f y:

a) z

= x* + y

c) z

z
\

Sec. 7. Higher-Order Derivatives and Differentials

1. Higher-order partial derivatives. The second partial derivatives of a


function z
/(*, y) are the partial derivatives of its first partial derivatives.
For second derivatives we use the notations

32z

d (dz\

Derivatives of order higher than second are similarly defined and denoted.
If the partial derivatives to be evaluated are continuous, then the result
of repeated differentiation is independent of the order in which the differentiation is performed.
Example 1. Find the second partial derivatives of the function

z~ arc tan

y
Solution. First lind the
dz

first

==

partial

_J_

dz^
dy~~

Now

derivatives:

j__

1_
J
,

differentiate a second time:

^L = d
.

dz z

\-

2 *y

dxdy

We

note that the so-called


so-

different

"mixed" partial

derivative

may

be

found in a

el
way, namely:

d /

dxdy~~dydx

dx(

y*

198

f(x

We

= d(d*z)

d*z

and, generally,
z

function
function:

similarly define the differentials of a function z of order higher than

two, for instance:

If

[Ch. 6

Higher-order differentials. The second differential of a


of this
y) is the differential of the differential (first-order)

2.
z

Functions of Several Variables

= /(x,

differential of

dz = d(d n -

z).

y), where x and y are independent variables, then the second


the function z is computed from the formula

(1)

Generally, the following symbolic formula holds true:

formally expanded by the binomial law.


z
f (x, (/), where the arguments x and y are functions of one or
eral independent variables, then
it

is

If

"- *
If

x and

i/

sev-

+2

are independent variables, then d 2 jt

becomes identical with formula (!)


Example 2. Find the total differentials

= 0,

d 2 y = Q, and

of the first

formula

and second orders

(2)

of

the function
z

Solution. First method.

We

2;t

3xyy

2
.

have

*-<*-*

I*-*.

Therefore,

dz

=
fr

Further we have

~
^-4
djc

whence

it

'

J*!L-_ 3

^l
'

djcd^""

dy*

follows that

Second method. Differentiating we find

Differentiating again and remembering that dx and dy are not


y, we get

x and

= (4dx

3dj/)

dx

(3d*

dependent on

+ 2d(/) dy =4dx*6dx dy2dy

z
.

Higher-Order Derivatives and Differentials

Sec. 7]

if

Find

if

1894. Find

if

1893

= arc tan
A

Find

1895.

xy

if

1896. Find all second partial derivatives of the function

Find

1897.

^~-

dxdy dz

if

u
1898. Find

if

1899. Find

f(0,

0),

= sin (xy).

= (l
1900.

Show

/^(O, 0)

f xt/ (Q, 0),

that

if

= arc sin
1901.

1902*.

Show

Show

that

if

that for the function

if

199

_ _

[Ch. 6

Functions of Several Variables

200

= 0]

[provided that f(0, 0)

we have

rxy(' 0)=-1,
1903

&$"=

Find

where

y(x,

1906.

if

= f(x,

= f(u,

v),

Show

a)*

= arc tan

+ (y

= lny

satisfies the

b)\

(x,

t)

=A

sin (akt

2
d^u_
~~~ a d^u
2

Show

dx

sin

Kx

u(x
//

'

that the function


(x-x

cp)

equation of oscillations of a string


dt*

(where *
conduction

Laplace equation

that the function

1909.

y),

that the function

Show

satisfies the

= <((x,

Laplace equation

= Y(x

1908.

z),

that the function

where

y,

where u

1907.

f(u, v),

y).

Show

satisfies the

0)=+1.

= xy.

where u = x* + y*, v
1904. Find

/;,(0,

'

>

>

'>-

)*

+ (y-y n )*

a are constants)

satisfies

^a'/^-u^U*^

the equation of heat

Higher-Order Derivatives and Differentials

Sec. 7]

Show

1910.

that the function

201

are arbitrary twice differentiable functions, satiscp and \|)


the equation of oscillations of a string

where
fies

Show

1911.

satisfies

that the function

the equation

x
1912.

Show

satisfies the

1913.

+ 2wy dxdy + Jy
,

5-,2
dv

r-

x-,
dy*

= n0.

that the function

equation

Show

-7

'

d su

that the function z

= f[x + y(y)]

satisfies the

equa-

= u(x,

which

tion
d22

dz

dx dx dy

1914.

Find u-^u(x, y)

dzd 2z
2
dy dx

'

if

dTSy^^
1915.

the form of the function u

Determine

satisfies the

equation

1916. Find d*z

if

1917. Find d u

if
11

1918. Find d*z

if
- rn
\l^

1919.

(f}
\^/F

Find dz and d*z


z

=u

whpr^
vviitivx

/
t>

-r

y2

j\r

-1- /y* .
|

if

where u

=~

= xy.

y),

_ _

202

Functions of Several Variables

1920. Find d'z

= ax,

where u

v),

= by^

= ye*.

if

= f(u,

1922. Find d*z

if

z~f(u,
1921. Find d*z

[C/i.

where u = xey

v),

if

= e x cos y.

1923. Find the third differential of the function

= * cos y + y sin x.

Determine all third partial derivatives.


1924. Find df(l 2) and d*f(l, 2) if
9

= x + xy + y*
2

f(x, y)

4\nx

\Q\ny.

1925. Find d /(0, 0,0)

f(x

Sec. 8.

y,

=x
z)

if

Integration of Total Differentials

t. The condition for a total differential. For an expression P (x, y)dx-}~


Q(* y)dy> where the functions P (x, y) and Q (x, y) are continuous in a

simply connected region


(in D) the total
ficient that

together with their first partial derivatives, to be


some function u (x, y), it is necessary and suf-

differential ol

aq^ap
~~
dx

Example

is

a total

t.

Make

sure that the expression

differential of

some function, and

Solution. In the given case,

1,

'

dy

find that function.

P = 2x + y Q
t

x+2y.

Therefore,

,5 = --

and, hence,

where u
It

is

is

the desired function.

given

that

-=

But on the other hand


and
Finally

we have

2jt

+ #;

therefore,

= x + y' (y) = x + 2y, whence

q>' (y)

= 2y,

(p(f/)

Sec.

ti\

2. The

203

case of three variables. Similarly, the expression

P(x,

where P

Integration of Total Differentials

z)dx

y,

+ Q(x,

z)dy

y,

+ R(x

z)dz,

y,

R(x, y, z) are, together with their first partial


functions of the variables x, y and 2, is the total
differential of some function u (x y, z) if and only if the following conditions
(x, y, z),

Q(x,

y, z),

continuous

derivatives,

are fulfilled:

dQ^W dR^dQ dP^dR


'

dx

Example

2.

Be sure

'

dy

dy

dz

dx

dz

that the expression

the total differential of some function, and find that function.


f
2
3jc
Solution. Here, p
1,
3x, R
2yz+\. We establish
30
the fact thai
is

dQ
dx

Q=z +

= dP = O

dy

dR = dQ
-r

dz

dy

dP
c.

rr

= OR =
r

V.

dx

dz

and, hence,

~
where u

We

is

the sought-for function.

have

hence,

u=
On

(3x

+ 3y

\)dx

= x* + 3xy

+ <p(y,

z).

the other hand,

du
-

~
= dtp

dz

whence

y^

=z

and ~P

dz

= 2f/z+l.

two variables q>(#, 2)


tion for total differential
find q>:

of

whose
is

The problem reduces


partial derivatives are

fulfilled.

We

that

is,

y(y,

= ^2 + 2 + C,
2

e)

And

finally,

to finding the function

known and

the condi-

_ _
Functions of Several Variables

204

Having convinced

are

total

differentials

[C/i.

yourself that the expressions given below


of certain functions, find these functions.

1926.

ydx + xdy.

1927.

(cosx+3x*y)dx + (x'y

1928-

1930.
1931.

-dx^dy.
y
x
dx +
*

dy.

1932. Determine the constants a and 6 in such a


the expression
2
z
2
(ax + 2xy + y ) dx-(x + 2xy + by*) dy

should

be

total

differential of

some function

z,

manner

and

find

that

that

function.

Convince yourself that the expressions given below are


some functions and find these functions.

total

differentials of

1933.
2

+ 3z)dx+(4xy + 2y
+ 8xy* + 2) dx +

z)dy

+ (3x

l)dy

(x

1934. (3x
2y*
1935. (2xyz3y*z

2)dz.

xdx + ydy + zdz


1938*. Given the projections of a force on the coordinate axes

v _.

v_

to

where A, is a constant. What must the coefficient K be for the force


to have a potential?
1939. What condition must the function f(x, y) satisfy for the
expression
f(x,

y)(dx

+ dy)

to be a total differential?

1940. Find the function u

if

du = f(xy) (ydx + xdy).

Sec. 9]

Differentiation of Implicit Functions

Sec. 9. Differentiation of Implicit Functions

205

1. The case of one independent variable.

If the equation f(x, y)


0, where
differentiate function of the variables x and y, defines y as a
function of x, then the derivative of this implicitly defined function, provided

that

is

y)

(*

y) ?= 0,

f' (x,
y

may

be found from the formula

dy
dx

f'
x

(**y)

f'y(x,y)'

found

are

derivatives

Higher-order

by successive differentiation

formula

of

a)
Example

1.

Find

dx

-~

and

Solution. Denoting the

if

dx 2

left-hand

side

of this

equation by

f (x, y),

we

find

the partial derivatives

f'u

(x

y)-=3(x

Whence, applying formula

+y

(1),

z 2

2y

we

get

find the second derivative, differentiate with respect to x the first derivative \vhich we have found, taking into consideration the fact that y is a functiun of x'

To

x -~
dx

y
J

dx

(x,

dx\

y J

y
J

y J

'

if

2. The case of several independent variables. Similarly,


0, where F (x, y, z) is a differentiate function
y, z)

x,

y and

z,

Fz

(x

y,

z)

if

of

the equation
the variables

defines z as a function of the independent variables x and y and


then the partial derivatives of this implicitly represented
0,
function can, generally speaking, be found from the formulas
t

'

dK

F'z

(x,

dlJ

y, z)

F'g

(x, y,

z)

another way of finding the derivatives of the function


ating the equation F (x, y, z)=0, we find

Here

is

dF

Whence

it

is

_,

dF

dF

rt

possible to determine dz, and, therefore,


dz
TT-

dx

dz

and 3dy

z:

different^

_ _
Example

we

[Ch. 6

Functions of Several Variables

206

Find

2.

and

-T-

if

j-

Solution. First method. Denoting the left side of this equation


find the partial derivatives

F'x

(x, y,

z)

= 2x,

Applying formulas
dz

d*~

F'x(x>
F'z (x

(2),

y. *)

F'

y (x, y, z)

we

40-z+l, F z

(x,

y,

2)

^
~"

2)

(x,

//,

z),

= 6z-0.

get

2x

~~
y,

by

Jy(*.

dz^
dy~

'

6*

z)

(x>

!/,

4//

62

z)

Second method. Differentiating the given equation, we obtain


2x dx

4f/

Whence we determine dz

dy

+ 6zdz

that

is,

zdy + dy = 0.

y dz

the total differential of the implicit func-

tion:
\

z}dy

4//

~ dy we
Comparing with the formula dz = -Q- dx -\,

2x

dz

3. A system

6z

dy

F(x,
G(x,

and v as functions

y,

62

system of two equations

of implicit functions. If a

\
defines u

\4yz

dz
'

dx

see that

i>)

y, u,

o)

= 0,
=

of the variables

x and y and the Jacobian

dF_ dF_

D(F, G)
D(u, v)'

dudv

dGdG
du dv

then the differentials of these functions (and hence their partial derivatives
as well) may be found from the following set of equations
.
dF
dF
dF
dx ^
dv =0,
^+ -z- dyy + -^~ du ^
+ ^dx
dv

'dF

,,

dy

Example

3.

3dx

-r~

dy

^du
dG
-^--3y
dv
du
*

/Q

'

The equations

define u and v as functions of x and ^


w; find

and
-,,dx
dx
dy
dy

rr?

Differentiation of Implicit Functions

Sec. 9]

Solution.

First

207

method. Differentiating both equations with respect to #

we obtain
du

dv

whence
__

?ff

dx~~
Similarly

we

u ~^~y
x
y

dv __ u-\-x
dx~~ x y

'

'

find

dy~~

du

+y

dy~~~ x

we

Second method. By differentiation


differentials of all four variables:

x du

dv
'

two equations that connect the

find

du 4- dv = dx + dy,
+ u dx + y dv + v dy =4).

Solving this system for the differentials du and dv,

we obtain

xy
Whence

_
dx

'

(}y__-f-x

dx~~xy

t)f/

dv

v -}-x
'

'

dy~~x

4. Parametric
bles x and y

is

representation of a function. If a function


represented parametrically by the equations
z

= z(u,

of the

varia-

v)

and

then the differential


equations

be found from the following system

may

of this function

of

dx

dx

Knowing
dz
^~=pr and 3-

dz

dx

the differential

dy

~^.
^

dz^p

du

dx
dv

dz

dz

xdu

dv

5~du-\--5- dv,

d*

+ qdy,

dv.

we

find the partial derivatives

_ _

208

[Ch. 6

Functions of Several Variables

Example

The function

4.

arguments x and y

of the

is

defined by the

equations
_.

dz

dz

Find ^- and 3ox

dy

we

Solution. First method. By differentiation


connect the differentials of all five variables:

dx

From

the

first

= du + dv

find three equations that

two equations we determine du and dv:


20 dx

"^

dy

2u dx

dy

'

'

2(v

2(u

u)

w)

Substituting into the third equation the values of du and


have:
d
dy

rfy

just found,

we

= 6wu (u
'

Whence
3-

3au,

3-

^jc

dt/

=TT
2

(w-fy).
v
;

Second method. From the third given equation we can find

+ 3^;
*=3Jf
dx
^
dx
Differentiate the
respect to y:

two equations

first

the

first

system we

dy

we

da

dv
a'

dy

u
v

find

= _l__

dy~~2(u
Substituting the expressions

__

dx~ u

dv_

v)'

and

^
(5)
'

with respect to x and then with

dy

dx~~ v
the second system

first

dy

find
du___

From

= 3'Jf + 3t,'f!.

dx

From

f
dy

dy~~2(vu)'
into formula (5),

we obtain

Sec. 9]

209

Differentiation of Implicit Functions

1941. Let y be a [unction of x defined by the equation


y*

Find
ma
*

dy
dhj
di' d?

1942. y

Show

that

and
ana
is

dx"'

a function defined

^=

1943. Find

and explain the

%
^

1944. Find

by the equation

if

y=\+y x

and

if

result obtained.

A;

and
(g)
(g)
\ax J K~\
\ax jx=i

1945. Find

if

Taking advantage of the results obtained, show approximately


the portions of the given curve in the neighbourhood of the point

*=1.
1946.

The function y
In

dx

1947.

and

Find

dx

c,.
j
Flnd

5i

and

and

-1-

2//

variables x and y

and

^
dy

3;q/z

2r/

is

defined by

+ 3 = 0.

%-

~
dx

A:

Find

if

dx z

^2

1949. Find

1950.

arc tan

function z of the

x
,

|?T7 =

dx*

1948. The
the equation

^2

defined by the equation

is

cosy

The function

and

for the
-j-

if

-|-

y cos 2

+zcosx= 1.

defined by the equation

is

x*

+y

system

of

xy

= 0.

values *==

1,

//

= 0,

= l.

__

210

dz
1QR1
1951. KinA

Fmd,

1952. /(*, y,z)


1953.

equation

Functions of Several Variables

l>

= 0.

= (p(x, y),
= 0.
ty(x,
2

f/)

d* 2

&*

dz

1955. 2

is

if
if

where y
Find ~.

is

function of x

defined by the

if

a function of the variables x and # defined

equation
2x*

[Ch. 6

Show

2
Find dz and d z,

1954.

d*Z
,

__

+ 2y*

-4-

8xz

z*

+8-

by the

Find dz and d z for the values x = 2, f/^=0, 2^=1.


2
1. What are the
1956. Find dz and d 2, if In z=jc + i/ 4-2
first- and second-order derivatives of the function 2?
1957. Let the function 2 be defined by the equation
2

where

1958.

an arbitrary differentiate

is

<p

Show

constants.

Show

is

satisfies the

02,

bz)

1960.

y=

xq> (z)

f(y y)
,

Show

&, c are

Q,

an arbitrary differentiate function of two arguments,


equation

a-+b - =
1959.

a,

that the function 2 defined by the equation

F(x
where F

and

function

that

o|>

= 0.

that

(2)

Show

the

that

function

satisfies the

xfx +y~ = z.
2

defined

the

by

equation

equation

d*zf(

1961. The functions y and 2 of the independent variable x are


2
2
2
2
2
2
defined by a system of equations *
2 ^0, x
32
4.
2#

+#

_ _

Sec. 10]

Change

2H

of Variables

1962. The functions y and z of the independent variable x are


defined by the following system of equations:

Find dy,

dz, d*y, d*z.


1963. The functions u and v of the independent variables x and y
are defined implicitly by the system of equations

Calculate
du

du

d2u

d*u

dzu
2

~dy

for x=--Q,

'

dv
dv
dx' d~y

62 v
'

dz v

d*v

~d\*'

dxdy

dy*

1.

1964. The functions u and v of the independent variables


and y are defined implicitly by the system of equations

Find du, dv, d*u,

d*v.

The functions u and

v of the variables x and


defined implicitly by the system of equations

1965.

.v

= (p(w,

v),

y are

//=-i|)(w, v).

(b
du
du du
51' cty' d~x> fy'

^ and g? x = M cos u, y w sin u, 2 =


x = u + v, y = u
and ~
v,z = uv.
b) Find
=
c) Find dz,
where r and
are functions of the variables
1967. e = F(r,
1966. a)

Find

if

if

\l

jc

cp)

<p

x and y defined by the system of equations

Find

ax

and

dy

1968. Regarding z as a function of x

x = a cos

Sec.

q)

cos

i|),

and

y, find

y = b sincpcosij), z

and ~,

if

10. Change of Variables

When

changing

variables in differential

them should be expressed

in

expressions,

the derivatives in

terms of other derivatives by the rules of

entiation of a composite function.

differ-

[Ch. 6

Functions of Several Variables

212

\. Change of variables in expressions containing ordinary


Example 1. Transform the equation
x2 y

dx
putting

derivatives.

y-

Solution. Express the derivatives of y with respect to x in terms of the


derivatives of y with respect to /. We have

dx

dy

dy

dt

dt

dx

dt

dt

'

t*

dt

Substituting

the

expressions

equation and replacing x by

the

of
-r-

we

derivatives

found

just

the given

into

get

or

dt*

Example

2.

ruij -"'

Transform the equation


x

g + ^y_g =0

taking y for the argument and x for the function.


Solution. Express the derivatives of y with respect to x in
derivatives of x with respect to y.

terms of the

dx^Jx''
dy

"~

'

dx*

dx{ dx
\

dyl dx \dx~~
\ dy J

fdx\
(Ty)

dx

'

fdx\*
(dy)

dy

Substituting these expressions of the derivatives into the given equation,


will

have

d*x
dj

1
'

dy

\dy

L=o

dx

dy

'

we

Sec.

or,

10]

_ _

213

of Variables

Change

finally,

Example

3.

Transform the equation

~xy'

dx

by passing to the polar coordinates


,v=rcoscp,
Solution. Considering

dx = cos

cp

f/

= rsinq>.

as a function of

dr

cp,

d// = sin

r sin cp dcp,

(1)

from formula
dr

cp

+ r cos

cp

(1)

we have

dcp,

whence
dr
.

d//_sm cp dr -f
cos cp
3J
T dr

cos

dcp __
~~

cp

sin cp
r

r sin cp
T d(pr

cos

cp

3- -f

r cos cp
^

_rcos

d<P

-- r sin

dr
coscp

or,

--

dr
cp

Putting into the given equation the expressions for


sin

cos (p
Y

dcp

cpH-

rcoscp

r sin q)

x,

rsm

//,

and

-^

we

will

have

cp

rsincp*

cp

after simplifications,

Change of variables in expressions containing


Example 4. Take the equation of oscillations of

and change

it

new independent

to the

partial derivatives.
a string

variables a and p, where

a = .v

at,

Solution. Let us express the partial derivatives of u with respect to x and t


terms of the partial derivatives of u with respect to a and p. Applying
the formulas for differentiating a composite function

in

du___du

du__du da

du dp

da,

di^dadT^d^dT

du dp

^dadx^dfidi

we get
du

du

^T-=^(
da

d/

^'^

du__du

a)

du

+
,

du
-35"

dp

a (du
is

\dp

^'

_^_,^f

du\
3~

day

_ _

214

Functions of Several Variables

..

[C/i.

same formulas:

Differentiate again using the

dt~~dadt

dt 2 ~~dt

d2u

d 2 u\,

2
(d u

d*u

dz u

dx*~dx\dx)

da\dx

_d

d'u

d*u

/)ft 8

'

rtn

'

'

/)ft

/}R2

we

Substituting into the ^iven equation,


2

/d a

d2 u

-=

for

5.

2
Transform the equation x 2
^- + y

the

new independent
*

d*u\

= 0.

Example

d2 u

Jd u

d u\

d u

have

will
2

=z

taking

and

variables,

-g-

for
z

,v,

the

new

function.

and
Solution. Let us express the partial derivatives
ypartial

derivatives

^~

and

To do

On

dx
v

dy

oy_

-^^,

dx

dz
^

the other hand,

dw

dw =

:;

du

du

dw
~\

dv

dv

Therefore,

dw
-3-

du

du

dw

dx

dv =
+ -5dv
x
,

dz

=
2

or

dw

i^ w_f^x__m dy\^,^x

&*_

Whence

dw

dw\

and, consequently,

dz^_
dx~~

dw ~~ 1 dw \
J
z
x*dv J
~~du
\x

terms of the

differentiate the given relation-

this,

ships between the old and new variables:

~ in

dw

Sec.

Change

10]

215

of Variables

and
dz __
z^

dw

2
dy~~y dv

'

Substituting these expressions into the given equation,

we

get

or

Transform the equation

1969.

x j--

putting x=--e*.
1970. Transform the equation

putting

A:

cos/.

rm
1971

69

Transform the following equations, taking y as the ar-

gument:

The tangent of the angle [A formed by the tangent line


and the radius vector OM of the point of tangency (Fig. 69)

1972.

MT
is

expressed as follows:
tan u=^

216

_ _
Functions of Several Variables

Transform

this

by

expression

to

passing

1973. Express, in the polar coordinates x


the formula of the curvature of the curve

1974.

[C/i.

polar

= r cos

coordinates:

q>,

y=

r sin

cp,

Transform the following equation to new independent

variables u and v:
dz

dz

if

u = x

v =x +y
2

*-*3&

'

2
.

1975. Transform
variables u and v:

the

following
dz

if

=n

equation

new independent

dz

to

= x, 0=-p

11

1976. Transform the Laplace equation


<Fu

dM_
n
~-"

dx*~*'dy

to the polar coordinates


A:

= rcoscp,

= rsm<p.

1977. Transform the equation

^2_ y ^__
~ U>
dx*
2

ar/

u=*xy and

putting

1978. Transform the equation


dz

dz

yTx- K jy=(y-^ z

>

by introducing new independent variables

and the new function w=\nz


(x + y).
1979. Transform the equation
dx*

taking u

>=

= x + y,

for the

new

=^

for

function.

the

new independent

variables

and

Sec. 11]

The Tangent Plane and the Normal

to

a Surface

217

1980. Transform the equation

putting u

= x+y,

=x

w = xyz,

y,

where

w=w(u,

v).

11. The Tangent Plane and the Normal to a Surface

Sec.

1. The equations of a tangent plane and a normal for the case of explicrepresentation of a surface. The tangent plane to a surface at a point
to
(point of tangency) is a plane in which lie all the tangents at the point
various curves drawn on the surface through this point.
The normal to the surface is the perpendicular to the tangent plane at the
point of tangency
If the equation of a surface, in a rectangular coordinate system, is given
in explicit form, z
function, then
f (x, y), where f (x, y) is a differentiate
z ) of the surface is
the equation of the tangent plane at the point
(x
f/
it

z-*o=/i(*
z

0o)(X-*o)

+ /i(*o,

)0

-0o).

and X, K, Z are the current coordinates

Here,
t/ )
f (x
the tangent plane.
The equations of the normal are of the form

where

F,

.Y,

Example
the surface z

1.

(i)
of

the point of

are the current coordinates of the point of the normal.


Write the equations of the tangent plane and the normal

at the point

M (2,

the

of

=v
'

dx

to

1,1).

Solution. Let us lind the partial derivatives


their values at the point

<k

(fo\
'

given

function and

=2

\dxjM

*~
Whence, applying formulas
or 2x-|-2f/

10

= ^-i- =

-, which

2. Equations
it

representation
implicitly,

and F

(X Q

which

t/

is

(1)
is

and

(2),

we

will

the equation

have

of the

=2(*

2)

+2

tangent plane and

(r/-|- 1)

==

the equation of the normal.

of the tangent plane and the normal for the case of implicof a surface. When the equation of a surface is represented

= 0,

the corresponding equations will have the form

_ _
Functions of Several Variables

218

which

[Ch. 6

the equation of the tangent plane, and

is

XXQ
F'X

(**,

Yy

_
*

ZZQ

Z 0)
F'z (*0. J/0.

(*0> 00. *0)

Fy

which are the equations of the normal.


the equations of
Example 2. Write
s
8

the tangent plane and the normal to


a.
z
a at a point for which x
the surface 3;q/z
0, z/
Solution. Find the z-coordinate of the point of tangency, putting x
0,
a 8 whence z
a. Thus, the
z*
a into the equation of the surface:
.j/

point of tangency

is

Denoting by

M (0,

and

Applying formulas

(3)

+ a=:0,

a).

the left-hand side of the


their values at the point Af:

which

and

we

(4),

~r~

'

wn

equation,

~2 5

the

Qj/

cn are

e q ua ^ions of

^ ne

/?sina,

of

paraboloid

the normal.

and

revolution

= x*+y

the

the equaindicated

at

the

point

cone

b) to the
c) to

the

points:
a) to
'

find

get

1981. Write the equation of the tangent plane


tions of the normal to the following surfaces at

0-

we

the equation of the tangent plane,

is

or

(x, y, z)

partial derivatives

or ^-(-z

a,

the

= at the
x*+y* + z = 2Rz

^ + -^

y-

sphere

point
at

the

(4, 3, 4);

point

(ffcosa,

/?).

1982. At

what point

of the ellipsoid
~2

y2
_4.
""
^ f_4.^ __
f.2

c2

does the normal to it form equal angles with the coordinate axes?
1983. Planes perpendicular to the A:- and #-axes are drawn
z*
169.
through the point
(3, 4, 12) of the sphere x*
y*
Write the equation of the plane passing through the tangents to
the obtained sections at their common point M.
1984. Show that the equation of the tangent plane to the
central surface (of order two)

ax

+ by

-\-cz

=k

+ =

Sec. 11]

The Tangent Plane and the Normal

M (x

at the point

Draw

to

the

Draw

1986.

which cuts

off

219

a Surface

has the form

surface x
x
to
the
4</4 6z
parallel
plane
1985.

to

+ 2tf + 3z = 21

= 0.

^a+fi + 'T^l

to the ellipsoid

tangent

planes

tangent

plane

on the coordinate axes.


equal segments
2
2

z*
1987. On the surface je
2*^=0 find points at which
the tangent planes are parallel to the coordinate planes.
s
1988. Prove that the tangent planes to the surface jq/z
a
of
the
tetrahedron of constant volume with the planes
form
coordinates.
r
f
1989. Show that the tangent planes to the surface }/ x-\-\/ y

+y

=m

-\

is

\/

z^Ya

cut

on the coordinate axes, segments whose sum

off,

constant.

Show

1990.

that the cone

^-i-f!

= -^

and the sphere

are tangent at the points (0,


b,c).
1991. The angle between the tangent planes drawn to given
surfaces at a point under consideration is called the angle between
two surfaces at the point of their intersection.
At what angle does the cylinder x*-\-y*
R* and the sphere

(x-R)

!-</'+z

-#

intersect at the point

Affy,

^-^,

OJ?

1992. Surfaces are called orthogonal if they intersect at right


angles at each point of the line of their intersection.
Show that the surfaces x*+y* z* r t (sphere), y xiany
2
(**
(plane), and z
y*)ian*-ty (cone), which are the coordinate
surfaces of the spherical coordinates r, cp, tj?, are mutually ortho-

gonal.
1993.

z^xf(~
*
\

Show
)

+ =

that all the planes tangent lo the conical surface

at the point

(,v

//

),

where x

+ 0,

pass through

the coordinate origin.


1994*. Find the projections of the ellipsoid
/'

+z

on the coordinate planes.


1995. Prove that the normal
revolution z

= /(/^ +

f/

(/'

xy

at

+ 0)

1=0
any point

of

the

intersect the axis

of

surface of
rotation.

220

Functions of Several Variables

[C/i.

Sec. 12. Taylor's Formula for a Function of Several Variables

of all orders
f (x, y) have continuous partial derivatives
(rc+l)th inclusive in the neighbourhood of a point (a, b). Then
Taylor's formula will hold in the neighbourhood under consideration:

Let a function

up

to the

-a) + f'y (a, b)(y-b)]

where

In other notation,
)

+ -Jy

[/tfX

if

...+[^^
or

-j

df

(x, y)

2
rf

/ (x, y)

The particular case of formula (1), when a b Q, is called Maclaunn's


formula.
Similar formulas hold for functions of three and a larger number of
variables.
A3
Example.' Find the increment obtained by the function f (x, y)
when passing from the values
1 to the values *,-- -{-//,
1, y

*=

Solution. The desired increment may be found by applying formula


calculate the successive partial derivatives and their values at

First

given point

(1, 2):

= 3- 1+3.2=9,
=
/;il,2)=-6.4 + 3.1 -21
fxx (\, 2) = 6-l=6
/;

(1,2)

^(1,2)= -12.2= -24,

(2).

the

Taylor's Formula for a Function of Several Variables

Sec. 12]

are

All subsequent derivatives


into formula (2), we obtain:

[/i

-64-2/z k- 3-f

1996.

powers

Expand
h and

of

2
fc

f (x i
fe

+ ^y

24))

ft,

zero.

identically

[/i

+ k)

+ 3/z

fc.

in a series

of

221

these

Putting

3/ife

-0-f *'(

positive

results

12)]

integral

if

*
1997. Expand the function f (x, y)
6x
2xy
3y*
4 by Taylor's formula in the neighbourhood of the point
2/y

(-2,

1).

1998. Find the increment received by the function


^-x*y when passing from the values x=l, {/=! to

4*

tegral

the

yz

neighbourhood

of

(1, 1, 1).

Expand

powers

of

f (x
/?,

k,

/ (x, y, z)

2001.

f(x, y, z)

z-l-4 by Taylor's formula in

3#

the point
2000.

= Jc'-fy + 2* +2xy
1

Expand the function

1999.

f(x,y)~

=--=

ft,

/)

in a series

of

positive in-

if

/,

z-|

fr,

//-J

and

// 4 z

2A:e

2jr//

Expand the following function

2yz.

Maclaurin's series

in a

Maclaurin's series

in

up to terms of the third order inclusive:


/(.Y,

2002.

//)

= ?* sin//.

Expand the following function

up to terms

of order four inclusive:


/ (x, //)

= cos x cos y.

2003. Expand the following function in a Taylor's series in


the neighbourhood of the point (1, 1) up to terms of order two
inclusive:
/(*.

{/)

= {/*

2004. Expand the following function in a Taylor's series in


the neighbourhood of the point (1,
1) up to terms of order
three inclusive:

_ _

222

[Ch. 6

Functions of Several Variables

2005. Derive approximate formulas (accurate to second -order


terms in a and P) for the expressions

if

and |p| are small compared with unity.


2006*. Using Taylor's formulas up to second-order

|a|

terms,

approximate

^O98;

1/T03;

a)

2 01
-

b) (0.95)

an implicit function of x and y defined by the


+ y = 0, which takes on the value z= 1 for x= 1
equation
and y=l. Write several terms of the expansion of the function
z in increasing powers of the differences
and y
1.
2007. z
z

is

2xz

x\

Sec.

13. The

Extremum

of

a Function

of Several

Variables

1. Definition of an extremum of a function. We say that a function


f(x,y) has a maximum (minimum) f (a, b) at the point P (a, b), if for all
points P' (x, y) different from P in a sufficiently small neighbourhood of P
the inequality /(a, b) > f(x, y) [or, accordingly, /(a, b) < f (x y)] is fulfilled.
The generic term for maximum and minimum of a function is extremum.
In similar fashion we define the extremum of a function of three or more
t

variables.

2. Necessary conditions for an extremum. The points at which a differentiate function f (x, y) may attain an extremum (so-called stationary points)
are found by solving the following system of equations:

0)-0,

t'
x (x.

f't/

(x

y)-Q

(1)

(necessary conditions for an

extremum). System (I) is equivalent to a single


0.
In the general case, at the point of the extremum
equation, df(x, #)
P (a, b), the function f (x, y), or df (a, ft) = 0, or df (a, b) does not exist.
3. Sufficient conditions for an extremum. Let P (a, b) be a stationary
point of the function f(x, y), that is, df (a, &)- 0. Then: a) if d*f (a b) <
for dx z
dy*>Q then /(a, b) is the maximum of the function f(x, //); b) if
d z f(a, ft)>0 for d* 2 -}- di/ 2
then /(a, b) is the minimum of the function
0,
2
/(* 0); c ) if d /(a, ft) changes sign, then f (a, b) is not an extremum of /(v, //).
t

>

The foregoing conditions are equivalent

= f'y (a,

ft)

-0

Then: I) if
P(a, ft), namely
(or

C>0);

2)

A=f xx

and

if

then the question


(which is to say,

>

0,

(a,

then

maximum,

<

0,

then

ft),

B~fxy

the
if

an extremum

(or
is

to the

ft),

<

following: let

C = /^(ci,

has

function

A<

there

(a,

an

0),

f[ (a,

We

ft).

form

extremum at
a minimum,

and

no extremum

at

(a

b)----

the

the point
if

>

A==0.
remains open

ft);

3)

if

of the function at P (a, ft)


requires further investigation).
4. The case of a function of many variables. For a function of three or
more variables, the necessary conditions for the existence of an extremum
of
it

Sec.

13]

The Extremum of a Function

of Several Variables

223

are similar to conditions (1), while the sufficient conditions are analogous to
the conditions a), b), and c) 3.
Example 1. Test the following function for an extremum:

Solution. Find the partial derivatives and form a system of equations

(1):

or
r

**

\ xy

we

Solving the system

Let us find

(2,

P 4 (_2,-1).

P,(-l,-2);

1);

second derivatives

tiie

d2 z
adx 2

=c

6.v,

d2z
3 T-

dxdy

For the pomt

= 6, A^=4C

= 36

<

144

0.

6r/,

for

c
= 6x

dy

each stationary point.

=6. B =

A = (g}
2

\dx Jp

d*z
T-22

= ry

B2

and form the discriminant A=^/4C


1)

get four stationary points:

Pt

P,(l,2);

+ *_5-0,
2 = 0.

=12, C=(g) =
(fL\
\dy J p,
\dxdyjp,
2

Thus, there

2) For the point P 2 4 --12, B^6, C-12;


the function has a minimum. This minimum
function for A -2, y~\'
:

no extremum at the point P,.

is

= 144

is

36

equal

> 0,

>

/I

0.

the value

to

3012^28.
6; A = 36
^-6,
12, C^
^- 12, B = 6, C= 12; A = 144

At P 2
of

the

3) For the point P 9


no extremum.
4) For the point P 4

At the point

P4

fi---

the function has a

maximum

equal to

2 ma x

144

36

< 0.

>

0,

There

< 0.

6-f-30-{-

4- 12 ---28
5*.

of

Conditional extremum. In the simplest case, the conditional extremum

a function /(A,

attained
<|)(jr,

on

//)

is

maximum

condition

that

its

or

//),

minimum

this function which is


related by the equation
the conditional extremum of a funcwe form the so-called Lagra<ige
i/)

arguments

(coupling equation). To find


given the relationship q> (A-,

w)

tion /(A-,

the

of

are

function
F(A-,

where X

is

y)-=f(

an undetermined

multiplier, and

we seek

the ordinary

of this auxiliary function. The necessary conditions for the


to a system of three equations:

with three unknowns


to

x,

t/,

X,

determine these unknowns.

from which

it

is,

generally

extremum

extremum reduce

speaking,

possible

_ _

[Ch. 6

Functions of Several Variables

224

The question of the existence and character of a conditional extremum is


solved on the basis of a study of the sign of the second differential of the
Lagrange function:
-

yi
for the given

system

dx 2

dxdy

of values of x, y,

dy

h obtained from

(2)

or the condition

that dx and dy are related by the equation

<

Q, and a
Namely, the function / (x y) has a conditional maximum, if d*F
conditional minimum, if d 2 F
0. As a particular case, if the discriminant A
of the function F (x, y) at a stationary point is positive, then at this point
there is a conditional maximum of the function / (x, y), if A
(or C
0),
and a conditional minimum, if A
0)
(or C
In similar fashion we find the conditional extremum of a function of
three or more variables provided there is one or several coupling equations
(the number of which, however, must be less than the number of the variables)
Here, we have to introduce into the Lagrange function as many undetermined
multipliers factors as there are coupling equations.
Example 2. Find the extremum of the function
t

>

<

>

>

z=:6

4*

<

3y

provided the variables x and y satisfy the equation

x*-\-y*=\
Solution. Geometrically, the problem reduces to finding the greatest and
6
4.v
plane z
3y for points of its
intersection with the cylinder ji 2 -f// 2 =l
We form the Lagrange function
least values of the e-coordinate of the

F(x, y)--=6

We

have

T=~

following system

=
***-

+ 2>jr,

4x-3f/-l-M*
3

+ 2X#.

The

of equations:

:
i

Solving this system

we

find
i

^-"2"'

'-~5~'

and
2

____^_
~"
2

___

^~""5"

Since

dx 2
it

follows that

-*"'

dxdy

=0,

-|-{/

1).

necessary conditions yield the

Sec.

The Extremutn

13]

54

~__

and

-=-

3
= --,
o

Function of Several Variables

thend 2 /7 >0,

and,

225

the function

4^3

consequently,
5

-- and f/
-=O
D
and, consequently, the function at this point has a conditional

has a conditional
z

F <Q,
maximum.

then d

of a

minimum

at this point.

If

K-

-^
Z

Thus,

= 6 + i5 + -=ll,

z max

6. Greatest and smallest values of a function.


rentiable in a limited closed region attains
its greatest (smallest) value either at a sta-

function

that

is

diffe-

tionary point or at a point of the boundary


of the region.

Example

3.

Determine the greatest and

smallest values of the

function

in the region

A'<0, [/<0, x + y^z

Solution. The indicated region is a


angle (Fig. 70).
1) Let us find the stationary points:
I

z'
K

7
{J

~ 2xy 1=0,
^ 2y x -}-1^0;
\-

whence x-=
1, //-=
At A1 the value
to test for

tri-

70

and we get the point


1
the function ZM

1;

of

It

1,

is

not absolutely necessary

an cxtrcmum

1)

Let us investigate the function on the boundaries of the region.


we have 2 [/ 2 -f-f/, and the problem reduces to seeking the
When A
greatest and smallest values of this function of one argument on the interval
2)

3^//^0.

Investigating,

(2 sm ) v _

(3,0);

When

find

get

(Zsm) y =*

x-[-y

that

(2g r ) x=0

T" at thc
3

A-

P oint

we

find

that

(z &m ) x

,,^^^~r

we

(~ V 2

will

we

the

at

point

3);

(0,

l^)

x z -\-x. Similarly,

3 or //-=

=6

at the point (0,

When //~0 we
point

we

at the point

'

(2g r ) v=0

=6

at

the

have
/

that

find

= 3A

3
-^

-f-9A'-j-6.

3 \
~

Similarly

(2 gr )

+ =

3
metres coincides with (z gr ) x =o anc (^r).jf=o- ^ n * ne straight line jc
^
we could test the Function for a conditional extremum without reducing to
a function of one argument.
3) Correlating all the values obtained of the function z, we conclude
at the stationary
1
that z gr
6 at the points (0,
3) and (3, 0); z sm
^

point

8-1900

M.

226

_ _

[Ch. 6

Functions of Several Variables

maximum and minimum

Test for

the following

functions of

two variables:

2013.

= (x \)*+2y*.
= (x I) 2y*.
2x y.
z = x* + xy + tf
=
z
*y (6 xy)(x>0
2 = x +y
2x*+4xy 2y
z =x

2014.

z=l

2008. z

2009. z
2010.
2011.
2012.

=
2 =

2015. z
20,6.

Find the extrema


2
2017. a = x

of the
2

+ +z

2018.

xy

^++

Find the extrema

functions

following

f/

+x

of

three variables:

2z.

-(^>0, y>0, z>0).

of the following implicitly represented func-

tions:

2019*. x*
2020. x

+ y* + z* 2x+4y 62 11=0.
y*3x + 4y + z* + z 8 = 0.

Determine the conditional extrema


2021. z

= xy

for

+ 2y
z = x* + if

2022. z^=x

for

2023.

for

2027.

= cos + cos
W=
2y + 2z
^=^ +
+2
u = xtfz*

2028.

u^xyz

2024. z
2025.

2026.

A:

-=

A:

f/

f/

for//
for

for

of the following functions:

A:
2

A:

+
-

= ~-.

for .v-f-y

+ z=

12(jc>0,

provided x-(~j/+e=5,

xy+yz+zx=8.

2029. Prove the inequality

if

x^zQ
Hint: Seek the

maximum

of the

function u

= xyz

provided

Finding the Greatest and Smallest Values of Functions

Sec. 14]

in

227

z=l+x + 2y

2030. Determine the greatest value of the function


the regions: a) x>0, y^*0, x
y^l\ b)

2031. Determine the greatest and smallest values of the func2


2
#*
tions a) z
y in the region x?+y *^l.
x*y and b) z
2032. Determine the greatest and smallest values of the func-

tion

= sinx-h sin y-f- sin (x + y)

in

the

O^jt^-2.-

region

2033. Determine the greatest and smallest values of the func1


x'
tion z
3xy in the region
y*

0^x^2,

14. Finding the Greatest and Smallest Values of Functions

Sec.

Example t. It is required to break up a positive number a into three


nonnegative numbers so that their product should be the greatest possible.
Solution. Let the desired numbers be x, y, a
x
We seek the maxi//.

mum

the function / (x, y)


xy(a
to the problem, the
closed triangle x^O, y^zQ, x
of

According

+ y^a

x
y).
function

/ (x,

is

y)

considered inside a

(Fig. 71).

Fig. 71

Solving the system of equations

f'(x,

we

will

have the unique stationary point

(-TTI

triangle. Let us test the sufficiency conditions.

>

(/)

20,

T
We

(x, y)=a2x2y,
f
IXIf

or

* ne

have
f"
yy'(x,

/)

of

the

_ _

228

[Ch. 6

Functions of Several Variables

Consequently,

And

so

at

f-g-,

~)

the function reaches a

the contour of the triangle, this

maximum

to say that the product will be greatest,


a3
greatest value is equal to
is

maximum.

Since f(x, */)=0 on

will be the greatest value,


if

=y=a

//

which

= --, and

the

-^=-

Note

The ploblem can

also be solved

extremum, by seeking the maximum


that x

+ y + z = a.

2034.

From among

all

of the

by the methods of a conditional


function u
xyz on the condition

rectangular

parallelepipeds

with

given volume V, find the one whose total surface is the least.
2035. For what dimensions does an open rectangular bathtub
given capacity V have the smallest surface?
2036. Of all triangles of a given perimeter 2p, find the one
that has the greatest area.
2037. Find a rectangular parallelepiped of a given surface S

of a

with greatest volume.


2038. Represent a positive number a in the form of a product of
four positive factors which have the least possible sum.
on an x^- plane, the sum of
2039. Find a point
(x, y),
the squares of the distances of which from three straight lines
= Q, f/ = 0, x y+l=0) is the least possible.
(x
2040. Find a triangle of a given perimeter 2p, which, upon
being revolved about one of its sides, generates a solid of
greatest volume.
2041. Given in a plane are three material points P, (x lf # ),
P*( x z' #2)* ^i(*> #3) w ith masses m lf m 2 m 3 For what position
of the point P(x, y) will the quadratic moment (the moment of
inertia) of the given system of points relative to the point P
2
P2 P2
P 3 P 2 ) be the least?
(i.e., the sum m.P.P
2
3
2042. Draw a plane through the point
c) to form
(a, b
a tetrahedron of least volume with the planes of the coordinates.
2043. Inscribe in an ellipsoid a rectangular parallelepiped of
greatest volume.
2044. Determine the outer dimensions of an open box with a
given wall thickness 8 and capacity (internal) V so that the
smallest quantity of material is used to make it.

+m

+m

Sec.

14]

Finding the Greatest and Smallest Values

of

229

Functions

2045. At what point of the ellipse


w2

r2

-Ta2

does the tangent line to


angle of smallest area?
2046*. Find the axes

2047.

Inscribe in
surface.

'

form with the coordinate axes a

it

of

5x

+ TT-1
b
tri-

the ellipse

2
-f-

8xy + 5tf

given

= 9.

sphere

cylinder

having

the

greatest total

2048. The beds of two rivers (in a certain region) approxi2 = 0.


y
mately represent a parabola y = x* and a straight line x
It is required to connect these rivers by a straight canal of least
length. Through what points will it pass?
2049. Find the shortest distance from
to the straight line
x __
~~
1

~~
3

the

point

M(l,

2,

3)

2
'

2050*. The points A and B are situated in different optical


media separated by a straight line (Fig. 72). The velocity of

*1

Fii<.

72

Fig. 73

light in the first medium is v l9 in the second, v 2


Applying the
Fermat principle, according to which a light ray is propagated
along a line AMD which requires the least time to cover, derive
the law of refraction of light rays.
2051. Using the Fermat principle, derive the law of reflection
of a light ray from a plane in a homogeneous medium (Fig. 73).
2052*. If a current / Hows in an electric circuit containing a
.

resistance

/?,

proportional

then the quantity of


2

to /

/?.

heat

released in unit time is


/ into

Determine how to divide the current

230

_ _

currents / t

#i

[Ch. 6

Functions of Several Variables

72 ,

so

#a

R*>
possible?

/,

by means

that

the

of three wires,

generation of

whose resistances are

heat would be the

least

Sec. 15. Singular Points of Plane Curves

1. Definition of a singular point.

is

f(x, #)
at once:

2. Basic

called a singular point

if

At

point Af (x c y Q ) of a plane curve


coordinates satisfy three equations
,

its

M (*

let

the

a node (double point) (Fig. 75);


is either a cusp of the first kind (Fig. 76) or of
c) if
second kind (Fig. 77), or an isolated point, or a tacnode (Fig. 78).

the

types of singular points.

a singular point

),

second derivatives

be not

all

equal to zero and

then:
a)

b)

if
if

A>0,
A<0,

A = 0,

then
then
then

M
M
M

is

A = /1C-B 2

an isolated point (Fig.

74);

is

Fig. 74

Fig. 75

When solving the problems of this section it is always necessary to draw


the curves.
z
x* has a node if a
ax*
0; an
Example 1. Show that the curve y
isolated point if a
0.
0; a cusp of the first kind if a
2
z
Solution. Here, f (x, y)z=zax
Let us find the partial derivatives and equate them to zero:

<

+ x'y

fx

(x,

t/)

>

Singular Points of Plane Curves

Sec. 15]

system has two solutions:

This

coordinates of the point

Hence, there

is

do not

0(0, 0) and
the

satisfy

unique singular point

(0,

231

equation

--~a

Oj,'btit

the

0).

Fig. 78

Fig. 77

Fig. 76

of the given curve.

Let us find the second derivatives and their values at the point 0:
,

4=20,
0=0,

a =0
fa>0

Pig.

Fig. 80

79

Fig. 81

Hence,
if

a>0,

if

if

2
{/

=x

< 0,

a--^0,
8

or

#=

then
then
then

A<0
A

>

Aj^O.
y
which

Y^\

about the x-axis,


first kind (Fig. 81).

is a node (Fig. 79);


and the point
is an isolated point (Fig. 80);
and
The equation of the curve in this case

exists only
is a tangent.

when Jc^O;

the curve

Hence, the point

is

is

will

be

symmetric

a cusp of the

232

Functions of Several Variables

Determine the character

the

of

[Ch. 6

singular points of the follo-

wing curves:
2053. y*
2054. (y

x* -\-x\

2055.

=*
ay=aV-x'.

2056.

jtyjt

2057. x*

+y

f/^O.

3axy = Q (folium

2061.

of Descartes).

= (cissoid).
= a*(x y (lemniscate).
+
= (a x)x (strophoid).
+
= ftV (a>0, 6>0)
(x* + y*)(x
a)*
3

2058. y*(a
x)
2
2059. (x*
y*)
2060. (a
x)y*

jc

(conchoid).

Consider three cases:


1)

a>6,

2)

= 6,

a<6.

3)

2062. Determine the change in character of the singular point


2
curve y
(x
b) (x
a)(x
c) depending on the values of

of the
a,

6,

c(a<ft<c

are real).

Sec. 16. Envelope

t. Definition of an envelope.

The envelope of a family of plane curves


curve (or a set of several curves) which is tangent to all lines of the
given family, and at each point is tangent to some line of the given family.
2. Equations of an envelope. If a family of curves
is

f(x

dependent on

a)=0

y,

a single variable parameter


of the latter are found

metric equations

a has an envelope, then the parafrom the system of equations

= 0,
a) = o.

f(x. y, a)

&(*.

y.

Eliminating the parameter a from the system


the form

D(x,

0)

(1)

(1),

we

= 0.

get an equation of
(2)

It should be pointed out


that the formally obtained curve (2) (the *>ocalled "discriminant curve") may contain, in addition to an
envelope (if
there is one), a locus of singular points of the given f amily, which locus ts
not part of the envelope of this family.
When solving the problems of this section it is advisable to make

drawings.

Example. Find the envelope

*cosa+f/sina

of

the family of curves

= 0(p = const,

p>0).

Sec. 16]

233

Envelope

Solution. The given family of curves depends on the parameter a.


the system of equations (1):

J
\

Solving the
envelope

system

for

*cosa + y sin a
x

sin

x and y

a+y
t

cos

p = 0,

= 0.

we obtain parametric equations

= pcosa,

r/

of the

Squaring both equations and adding, we eliminate the parameter

frig.

Form

a:

82

Thus, the envelope of this family of straight lines is a circle of radius p


with centre at the origin. This particular family of straight lines is a family
of tangent lines to this circle (Fig. 82).

2063. Find the envelope of the family of circles

2064. Find the envelope of the family of straight lines

(k is a variable parameter).

2065. Find the envelope of a family of circles of the same


radius R whose centres lie on the jc-axis.
2066. Find a curve which forms an envelope of a section
of length / when its end-points slide along the coordinate axes.
2067. Find the envelope of a family of straight lines that
form with the coordinate axes a triangle of constant area S.
2068. Find the envelope of ellipses of constant area S whose
axes of symmetry coincide.

Functions of Several Variables

234

2069. Investigate the character

the "discriminant curves"


a constant parameter):

of

of families of the following lines (C

+ x) (y

d) strophoids (a

is

cubic parabolas y
(x
C)
2
b) semicubical parabolas t/
(x
2
C)
(x
c) Neile parabolas y*
a)

[Ch. 6

C)*;

=*

C)

x).

(a

Fig. 83

2070.

The equation

trajectory of a shell fired from a


v
at an angle a to the horizon

of the

with initial velocity


point
resistance
(air
disregarded) is

Taking the angle a as the parameter, find the envelope of all


trajectories bf the shell located in one and the same vertical
plane ("safety parabola' ) (Fig. 83).
1

17. Arc Length of a Space Curve

Sec.

The
dinates

where

differential of
equal to

an arc

of a

space curve in rectangular Cartesian coor-

is

current coordinates of a point of the curve.

z are the

x, y,

If

are parametric equations of the space curve, then the arc length of a section
of

it

from

to

t2

/a

f
j

is

1/7 <M + (dy\* + ( dz


y
(-dr
2

(-df)

The Vector Function

Sec. 18]

of a Scalar

235

Argument

In Problems 2071-2076 find the arc length of the curve:

2071. x

t,

y=/

2-~-'

from

to

= 2.

= 2 cos y = 2 sin z = -|- from == to =


= *' cos y = e* sin z = e from = to arbitrary t
from JC==0 to x==6
y = 4~' 2 = 4~
* = 3(/, 2jcy = 92
from the point (0, 0, 0) to M (3, 3, 2).
= aarcsin~, z = -|-ln ^j from the point 0(0,0,0)

2072. x

/,

t,

it.

2073.

A:

2074.

/,

/,

2075.
2076.

f/

M(*

to the point

j/

).

2077. The position of a point for any time


by the equations

Find the mean velocity


Sec.

of

(f>0)

motion between times

is

defined

and ^=10.

18. The Vector Function of a Scalar Argument

1. The derivative of the vector function of a scalar argument. The vector


a (0 may be defined by specifying three scalar functions ax (t)
a (t) and a z (t) which are its projections on the coordinate axes:
function a

The derivative of the vector function a-=a(t) with respect to the scalar
argument t is a new vector function defined by the equality

da
The modulus

a(t

+ M)-a(t)_da x (t)

day (0

da f

(t)

of the derivative of the vector function is

da
dt

The end-point

of the variable of the radius vector

r=r(/) describes

in

space

the curve

r=x(t)l+y(t)J+*(t)*.
which

is

called the hodograph of the vector r.

The derivative

-~

is

a vector, tangent to the

hodograph at the corre-

sponding point; here,

dr
[

where

s is

dt

[_
\

ds

dt

'

the arc length of the hodograph reckoned from

For example,

Up

some

initial point.

236

__
If

the

parameter

Functions of Several Variables

the time, then -jT

is

tf

is

[Ch. 6

the velocity vector of the

=w

is * ne acceleration vector of the


and
JTS
"^T
of
vector
the
r.
extremity
2. Basic rules for differentiating the vector function of a scalar argument.

extremity

of the vector r,

= m-~-,

2)

-77-

(ma)

3)

-77-

(cpa)==-~-a

4,

7)

where

+ (p-^-

is

a constant scalar;

where q>(0

a scalar function of

is

<*,_..+..:

a-~=0,

if
|

|-= const.

Example 1. The radius vector


time defined by the equation

r=i
Determine the trajectory
Solution.

From

Eliminating the

(1)

time

of

of

moving point

of

motion, the velocity and acceleration.

t,

we

find that the trajectory of


1

~~

z
'

differentiating,

we

find the velocity

and the acceleration

The magnitude

any instant

(1)

4"" 3
(1),

at

we have:

From equation

is

4t*j+3t*k.

line:

We

/;

of the velocity

note that the acceleration

\*L

\dt*

is

is

constant and

= ]/(_ 8) +6
2

is

=iO.

motion

is

a straight

Sec.

18]

Show

2078.

where r

The Vector Function

the

that

of

a Scalar Argument

vector

and r 2 are radius vectors

= (/* 2
equation
two given points,

rr,

of

237

r,)
is

/,

the

of a straight line.
2079. Determine which lines are hodographs of the following
vector functions:

equation

a)

= at -f c\
=
a/ +
r
r

c)

b)

ft/;

d)

= a cos -f b sin
r = a cosh -f 6 sinh

t\

/,

a, ft, and c are constant vectors; the vectors a and b


are perpendicular to each other.
2080. Find the derivative vector-function of the function
a(t)a (/), where a(t) is a scalar function, while a(/)
a(t)
is a unit
vector, for cases when the vector a(t) varies: 1) in
length only, 2) in direction only, 3) in length and in direction
(general case). Interpret geometrically the results obtained.
2081. Using the rules of differentiating a vector functisn with
respect to a scalar argument, derive a formula for differentiating
a mixed product of three vector functions a, 6, and c.
2082. Find the derivative, with respect to the parameter t,
of the volume of a parallelepiped constructed on three vectors:

where

a = i + tj+t z k,

2083.

The equation

of

motion

is

= 3/cos/~j-4/sinf,

is
the time. Determine the trajectory of motion, the
the acceleration. Construct the trajectory of motion
and
velocity
and the vectors of velocity and acceleration for times, / = 0,

where

2084. The equation of motion

is

Determine the trajectory of motion, the velocity and the accelWhat are the magnitudes of velocity and acceleration
and / =
and what directions have they for time =
-y?
2085. The equation of motion is

eration.

r= i cos a cos at -\-jsmacostot + k sincof,


where a and o are constants and / is the time. Determine the
trajectory of motion and the magnitudes and directions o! the
velocity and the acceleration.

Functions of Several Variables

238

2086. The equation of motion

of

shell

\Ch. 6

air re-

(neglecting

is

sistance)

where V Q {V OX v oy v oz } is the initial velocity. Find the velocity


and the acceleration at any instant of time.
2087. Prove that if a point is in motion along the parabola
,

z/=

manner

in such a

on the x-axis remains constant

that

the

of

projection

[-= const],

velocity

then the accelera-

tion remains constant as well.

2088. A point lying on the thread


into a beam describes the spiral

of

screw being screwed

where 9 is the turning angle of the screw, a is the radius of the


screw, and h is the height of rise in a rotation of one radian.
Determine the velocity of the point.
2089. Find the velocity of a point on the circumference of a
wheel of radius a rotating with constant angular velocity co so
that its centre moves in a straight line with constant velocity V Q
.

Sec. 19. The Natural Trihedron of a Space Curve

At any nonsingular point

(;c,

//,

sible to construct a natural trihedron

z) of a

space curve r

consisting

of

three

r(t)

is

it

pos-

mutually perpen-

dicular planes (Fig. 84):

M A4jA4

containing the vectors

1)

osculating plane

2)

normal plane

3)

rectifying plane MMjAIj, which

MM*M

S,

which

is

perpendicular to
is

At the intersection we obtain three straight lines;


1) the tangent
2) the principal normal MAf 2
of which are defined by the appropriate vectors:
1)

2)
3)

T=-rr

(the vector of the tangent l'me)\

^-gfx^p

NBXT (the

the vect

f tne

binormal);

vector of the principal normal)]

The corresponding unit vectors

and

-r-^

the vector

JV

3)

and

at

perpendicular to the first

MM^

all

TT

two planes.

the binomial

MM

9t

The Natural Trihedron

Sec.

19]

may

be computed from the formulas

dr

t= dF

of

a Space Curve

239

ds

dt
ds

If X
K, Z are the current coordinates of the point of the tangent, then
the equations of the tangent have the form
f

X-x = Y

y==

'

3C

(I)

'2

if

Normal
Rectifying

plane

plane

Osculating
plane

Fig. 84

where
of

Tx

--

Tv=

the line and the plane

T 2 = -~

we

from the condition

get an equation

of the

2)

of

perpendicularity

normal plane:

= 0.

T x Ty 7\ by B x

(2)

Bv

B z and A^.

(1) and (2), we replace


get the equations of the binomial and the principal normal and,
respectively, the osculating plane and the rectifying plane.
Example t. Find the basic unit vectors T, v and p of the curve
If

in

Wy,

equations

^V^,

at the point

at this point.

Solution.

and

= 1.

Write the equations


mal

we

We

have

of

the tangent, the principal normal and the binor-

[Ch. 6

Functions of Several Variables

240

Whence, when

1,

we

get
dt

dt

2 3
^= 026
1

dt
i

>'

662
Consequently,
T

Since for

= <+2/+3A

P=

3/-ay+*

?=1 we have *=1, y=l, 2=1,

'

-1U-8/+9*

follows that

it

"~

""

are the equations of the tangent,

x\_y
~~

l_z
~~
3

are the equations of the binomial and

*-l

z-1

y-1

11

are the equations of the principal normal.


If a space curve is represented as an intersection of
F(JC, y, z)

then in place

of

the vectors

-^-

= 0,
and

G(x,
TT-Z

y, 2)

two surfaces

= 0,

we can take

the vectors dr{dx, dy,


dz}

and d 2 r {d*x, d*y, d z z}; and one of the variables x, y, z may be considered
independent and we can put its second differential equal to zero.
Example 2. Write the equation of the osculating plane of the circle
*2
at its point

M(l,

1,

+ + z = 6,
2

J/

+ y + z^Q

(3)

2).

Solution. Differentiating the system (3) and considering x an independent


we will have

variable,

x dx

+ y dy + z dz

--=

0,

and
dx*

+ dy + y d*y + dz + z d*z .= 0,
1

d 2 (/
Putting

x=l, y=\>

z~2,

we

get

The Natural Trihedron of a Space Curve

Sec. 19]

Hence, the osculating plane

dx

{dx,

241

defined by the vectors

is

and

0}

|o,

yd*

jdx*\

or
1,

{1,

and

0}

1,

{0,

1}.

Whence the normal vector of the osculating plane


J

B=

-1
and, therefore,

its

equation

is

lj

is

-l(x-l
that

as

is,

it

should be, since our curve

is

located in this plane.

2090. Find the basic unit vectors

x^l
at the point

= -g-

T, v,

y=sin/,

cosf,

p of the curve
z

2091. Find the unit vectors of the tangent and the


of the conic spiral

principal

normal

at an arbitrary point. Determine the angles that these lines


with the z-axis.
2092. Find the basic unit vectors r, v, p of the curve

x*,

make

= 2x

2.
at the point x
2093. For the screw line

= asmt,

= bt

write the equations of the straight lines that form a natural


trihedron at an arbitrary point of the line. Determine the direction cosines of the tangent line and the principal normal.
2094. Write the equations of the planes that form the natural
trihedron of the curve
x*

one

-1- 1/

+ * = 6,
2

if

-1-

-4

points M(l, 1, 2).


the equations ot the tangent line, the normal
z
t*
the
and
t*,
/,
osculating
plane of the curve *
plane
y
at the point
(2, 4, 8).

at

of

2095.

its

Form

_ _

[Ch. 6

Functions of Several Variables

242

2096.

Form

and binormal

the equations of the tangent, principal normal,


an arbitrary point of the curve

at

Find the points at which the tangent to


to the plane x
10 = 0.
3y-\- 2z

this curve is parallel

2097. Form equations of the tangent, the osculating


the principal normal and the binormal of the curve

at the point /
at this point.

= 2.

Compute the

direction cosines of the binormal

2098. Write the equations of the tangent and


plane to the following curves:
a)

x = R cos 2

b)

z=x*+y*, x = y
*

C)

/,

y = R sin /cos/,

+ y + z = 25,
2

= Rsmt

for

x+z=5

2/3,

at the point (2,

normal
Find the equation
y = x at the coordinate origin.
2100. Find the equation of the osculating plane

normal

the

?-;

at the point (1,1, 2);

2099

z=x

plane,

of

3).

plane to the curve

the

if,

*,

(/

= <>-',

2101. Find
2

*
2
b) *
2

a)

c)

JC

to the

curve

= ty2

0.
at the point /
the equations of the osculating plane to the curves:

+y + 2 = 9, x y = 3 at the point (2, 1, 2);


= 4y, x' = 24z at the point (6, 9, 9);
+ z = a y fz = 6 at any point of the curve (x
2

QJ

y ot z

).

Form

the equations of the osculating plane, the principal


normal and the binormal to the curve

2102.

y
2103.

Form

= x,

=z

at the point (1,

1,

1).

the equations of the osculating plane, the princi-

normal and the binormal to the conical screw-line A; = /COS/,


j/=/sin/, z~bt at the origin. Find the unit vectors of the
tangent, the principal normal, and the binormal at the origin.
pal

Sec. 20. Curvature and Torsion of a Space Curve

1. Curvature.

By

the curvature of a curve at a point

number
*

/(

lim
AS-+O

JL,
As

we mean the

Curvature and Torsion

Sec. 20]

of

a Space Curve

243

the angle of turn of the tangent line (angle of continence) on a


curve MN, As is the arc length of this segment of the curve.
segment
R is called the radius of curvature. If a curve is defined by the equation
r=r(s), where s is the arc length, then

where

(p

is

of the

For the case

we have

parametric representation of the curve

of a general

(1)

2. Torsion. By
mean the number

torsion (second curvature) of a curve

at

point

we

-1
r-l-ihn
As-*o As
Q

the angle of turn of the binormal (angle of contingence of the


where
second kind) on the segment of the curve MN. The quantity Q is called the
radius of torsion or the radius of second curvature. If r=r(s), then
is

drd 2 rd sr
ds ds 2 ds 3

dp_

ds

where the minus sign

is

direction, and the plus


If /=/(/), where t

taken when the vectors

when

sign,

and v have

the

an arbitrary parameter, then

is

dr d 2r dV
d/d?" d7
-

(2)

~d,

dt

Example

1.

Find the curvature and the torsion of the screw-line


r

Solution.

We

same

not the same.

= i a cos

-\-j

a sin

+ k bt

+ja cos t + kb

(a

> 0).

have

~= _/ a

sin

d r

==

a cos

= _/ a sin/

/a

sin

/,

cos/.

Ja

Whence
k

J
a sin
a cos

a cost b
-a sin

ab sin

jab cos

+ a*k

Hence, on the basis

formulas

of

JDL

R ~(

we

(2),

+ 6i)

"fl'

/.

get

VaT+b*^
i

fl

and

(1)

+a

and
1

a*b

a 2 (a 2

+ b*)~~ a + b
2

'

Thus, for a screw-line, the curvature and torsion are constants.


3
Frenet formulas:

dr

^v
'

7s~"~R

2104. Prove

___
~~

5s

iP

^P_

'

ds~~~~o"'

~~~~R~T~~Q

the curvature at all points of a line is


a straight line.
2105. Prove that if the torsion at all points of a curve is zero,
then the curve is a plane curve.
2106. Prove that the curve
that

zero, then the line

if

is

x=\+3t + 2t
is

2
,

= 22t + 5t\ z=lt*

plane curve; find the plane in which it lies.


Compute the curvature of the following curves:

2107.
a)

x = cost, y = s'mt,
2
2
2
z = l, y
-|
//

b) x*

2108.
curves:
a)

b)

= cosh

= e'cos/,

at the point /
0;
at the point (1, 1,

Compute the curvature and

y = e sint, z
x^acosht, y asiuht.

je

-2x + z = Q

1).

torsion at any point of the

e*\

= at

(hyperbolic screw-line).

2109. Find the radii of curvature and torsion at an arbitrary


point (x, y, z) of the curves:
2

x
9
b) x

a)

=
=

2110. Prove that the tangential and normal


are expressed by the formulas
acceleration
v2
dv
VOif
T
<MJ
V
V,

WT-^T,

Wv-

components

of

where v is the velocity, R is the radius of curvature of the


T and v are unit vectors of the tangent and principal
trajectory,
the
to
curve.
normal

Sec. 20]

2111.

la cost

tion w.
2112.

Curvature and Torsion of a Space Curve

+ja

sin

in

is

point
t

+ btk

The equation

Determine,

at

trajectory and
acceleration.

times
2)

the

of

24S

uniform motion along a screw-line r =*


with velocity v. Compute its accelera-

motion

is

and /=!: 1) the curvature of the


tangential and normal components of the

Chapter VII

MULTIPLE AND LINE INTEGRALS

Sec. 1. The Double Integral

in

Rectangular Coordinates

1. Direct computation of double integrals. The double integral of a continuous function f (x, y) over a bounded closed region S is the limit of the
corresponding two-dimensional integral sum

f (x,

y)dx dy =

lim
max A*i max Ar// -

(1)

&y k = yk+l yk and the sum is extended over those


xg,
and k for which the points (*/, y k ) belong to S.
2. Setting up the limits of integration in a double integral. We distinguish two basic types of region of integration.

where A*

values of

x,

= Xf +l

o
Fig. 85

1)

The region

of integration

Fig. 86

(Fig. 85)

is

bounded on the

left

and

right

while the vaFig. 85). In the region S, the variable x varies from x l to x
riable y (for x constant) varies from
9, (x) to j/ 2
q> 2 (x). The integral (1) ma>

^=

The Double Integral

Sec. 1]

247

in Rectangular Coordinates

be computed by reducing to an iterated integral by the formula


*a

<Pa (X)

f(x, y)dy,

J
(S)

<PaU>

where x

is

held constant

when

calculating

/(x, y) dy.

2) The region of integration S is bounded from below and from above


by the straight lines y y and y yt(y z >yi), and from the left and the
(y)],
right by the continuous curves x = ^> (y) (AB) and x = 2 (y) (CD) [t|? 2 (y)
each of which intersects the parallel y = Y (y*
Y y t) at only one point
l

(Fig. 86).
As before,

^^

i|?

^ <

we have
$2

Vl

JJ/(*.

y)dxdy=\dy

(S)

here, in the integral

j/i

f(x, y) dx

we

(U)

f (x.

J
i|?,

y)dx

((/)

consider y constant.

If the region of integration does not belong to any of the above-discussed


types, then an attempt is made to break it up into parts, each of which does
belong to one of these two types.
Example 1. Evaluate the integral

Solution.

Example

2.

Determine the limits

of integration of the integral

(x,
(S)

y)dxdy

248

_ _
Multiple and Line Integrals

[Ch. 7
2

*
1
the region of integration 5 (Fig. 87) is bounded by the hyperbola y
2 and x
2 (we have in view the region constraight lines *
taining the coordinate origin).
Solution. The region of integration ABCD (Fig. 87) is bounded by the
2 and x
2 and by two branches of the hyperbola
straight lines
if

and by two

x-

y=yT+*
that

is,

it

belongs to the

first

and

type.

We

(/--1
have:

dx

f(x, y)dy.

Evaluate the following iterated integrals:

21

2113.

2114

35

\dy\(jf + 2y)dx.

2117.

^pTv?31

2118.

Jdy

rrfr.

dcp

j
a

sin

<p

Jt_

2115

X 2dU

COS

(p

'

00
Write the equations of curves bounding regions over which the
following dduble integrals are extended, and draw these regions:
2

2121.

2-t/

Jrf/

f(x, y)dx.

X+9

2122.

JdxJ
IX
4

2125.

f(x, y)dy.

10-y

dy
o

JdxJ/(*. y)dy.
K25-JC-

2123.

2X

2124.

00
^dx

f(x, ^)dx.

2126.

f(x, y)dy.

X+2

d*

-ix*

f(x, y)dy.

Set up the limits of integration in one order and then in the


other in the double integral

JJ/(*. y)dxdy
(S)

for the indicated regions S.

Sec.

The Double Integral

1]

2127.

C(0,

is

a rectangle

S
S

is

a triangle

is

in Rectangular Coordinates

with vertices 0(0,

0),

249*

4(2,0), 5(2,

1),

1).

2128.
2129.

C(0,

with vertices 0(0,

4(1,

0),

trapezoid with vertices 0(0,

0),

0),

(2, 0),

5(1,
5(1,

1).

1),

1).

2130.

C(2,

7),

2131.
0),

(0,

parallelogram with

is

D(l,

5).

is

whose

circular sector
arc end-points are

vertices

045
A

4(1,

2),

5(2,

4),

with centre at the point


and 5 f - 1, 1) (Fig. 88).

(1, 1)

Fig

89

right parabolic segment 405 bounded by the


a segment of the straight line 54 connecting
and
parabola
the points 5(~-l, 2) and 4(1, 2) (Fig. 89).
2133. S is a circular ring bounded by circles with radii r=l
and /?-=2 and with common centre 0(0, 0).
2134. S is bounded by the hyperbola if
x?
\ and the circle

2132.

is

504

^9

(the region containing the origin is meant).


2135. Set up the limits of integration in the double
//

(x,

integral

y)dxdy

(S)

if

the region S

b) *
c)

h//

defined by the inequalities

is

<a

e)

|-

//*

*;

Change the order

//<x<y4
0<*/

2a;

:a.

of integration in the following

double integrals:

\2X

2136.

f(x, y)dy.

2137.

JdJ/(x.

y)dy.

Multiple and Line Integrals

250

2138.

\dx

f(x, y)dy.

2141.
\

d*

dy

f(x

y)dx.

VTZx^tf

2139.

[Ch. 7

f(x, y)dy.

2142.

\dy

V*-*

$/(*,
yi
~2

20

2140.

Jdx

f(x

y)dy.

RVT
A

2143.

-V

d*J/(x,

sin

2144.

\dx

t/)dy+

f(x. y)dy.

/)d/.

J/(jc,

Evaluate the following double integrals:


(
where S is a triangle with vertices 0(0,
[ xdxdy

2145.

0),

(S)

A(\,

1),

and B(0,

1).

A(2,0)X
Fig. 90

2146.

^xdxdy,

where the region

Fig. 91

of integration

Sis bounded

(S)

by the straight line passing through the points A (2, 0), fi(0, 2)
of a circle with centre at the point C(0, 1), and

and by the arc


radius

(Fig. 90).

The Double Integral

Sec. 1]

2147.

Vr a
,

JJ

in Rectangular Coordinates

===, where S
x*y*

is

251

a r
part of a circle of radius

(S)

a with centre at 0(0,


2148.

lying in the

first

where S

y dx dy

V**

$ $

0)

is

quadrant.
triangle with

vertices

(S)

0(0,0),
2149.

-1), and

(I,

fl(l,

1).

y*dxdy, where S

\j !/"*#

triangle with vertices

is

(S)

0(0,

4(10,

0),

rr
2150.

ed

J J
(S)

1),

and

fl(l,

1).

dxdy, where S

ey

by the

parabola

y*

is

=x

a curvilinear triangle

and

OAB

bound-

x = Q, (/=!

the straight lines

(Fig. 91).

2151.

ff^Ti,
-

the parabola
2152.
extend:

y=7f and

Compute the

is

a parabolic segment

the straight line y

integrals

\dx

[
j

= x.

_rt

tfsmxdy;
c)

$ <ty
/

bounded by

and draw the regions over which they

i-f-cosjc

at

a)

where S

$/

x* sin'

"

b)
COS*

When solving Problems 2153 to 2157


the drawings first.
2153. Evaluate the double integral

it

is

abvisable to

make

(5)

if

line

is

a region

x = p.

bounded by the parabola

y*

= 2px and

the straight

2154*. Evaluate the double integral

^xydxdy,
extended over the region S, which
2
2
and an upper semicircle (x
2)
#

is

+ =

bounded
1.

by

the #-axis

_ _
Multiple and Line Integrals

252

(Ch. 7

2155. Evaluate the double integral

dxdy

/25=5

(S)

where S

the area of a circle of radius a,


to
the
coordinate axes and lies in the
gent
2156*. Evaluate the double integral
is

which
first

circle

is

tan-

quadrant.

^ydxdy,
(S)

where the region S

is

bounded by the axis

of abscissas

and an

of the cycloid

= R(t
y = R(\

sin/),

cos/).

2157. Evaluate the double integral

(S)

in which the region of integration


nate axes and an arc of the astroid

2158. Find the

region

mean value

is

bounded by the coordi-

of the function f(x, y)=--xy

in the

SJO^Jt^l, 0<y<l}.

Hint.

The mean value

of a function

f(x, y)

the region

in

is

the

number

2159. Find the mean value of the square of the distance of


2
2
from the coordia point
(x, y) of the circle (x
nate origin.

Sec.

2.

Change

^R

af+y

of

Variables in a Double Integral

1. Double integral in polar coordinates. In a double integral, when passing


from rectangular coordinates (x, y) to polar coordinates (r, cp), which are
connected with rectangular coordinates by the relations
#

we have

= /'Cos(p,

r sin

ip,

the formula

^{
(S)

f (*>

y)dxdy=((
(S)

(r

cos

q>,

r sin

cp)

r dr

e/cp,

(1)

Sec. 2]

Change

of Variables in a Double Integral

253

a
integration (S) is bounded by the half-lines r
the curves r
and r r 2 ((p), where r l (q>)
r,(cp)
r z f<P) [ r i ( fP)^ r 2( (P)l are single-valued functions on the interval
then the double integral may be evaluated by the formula
If

the

region

r=^p(a<P)

of

and

and
and

a^rp^p,

C f

r) r

(q),

dr

dcp

/-J

(S)

<<p)

\ dcp

r) r rfr,

(cp,

r, (cp)

r 2 (cp)

where F

(cp,

In evaluating the

r sin (p).

r)~/(rcos(p,

integral

((p,

r)rrfr

'i (<p)

we hold

the quantity (p constant.


the region of integration does not belong to one of the kinds that has
been examined, it is broken up into parts, each of which is a region of a
If

given type.

2. Double
if

integral in curvilinear coordinates.

the more general case,

In

double integral

in the

(x,

y)dxdy

from the variables x, y to the variables u, v, which


it is required to pass
are connected with x, y by the continuous and differentiate relationships
A-^cp(w, i),

between the points


region

of

the

of

t|?(K,

I/

v)

both directions, continuous) correspondence

that establish a one-to-one (and, in

and the points

region S of the .v//-plane


if the Jacobian

the

of

some

UV- plane, and

*^

D(u
retains a constant

dx dy
du da

y)

V'

dx dy
dv dv

sign in the region 5, then the formula

(^

y) dx

dy

I [cp (u,

v),

(w,

v)

du du

(6)

holds true

The limits of the new integral are determined from general rules on the
basis of the type of region S'
Example 1. In passing to polar coordinates, evaluate

where the region 5

is

origin (Fig 92).


Solution. Putting x
J/'l

radius

a circle of

rcoscp,

if^

Y"\

//

(r

/?

rsincp,
cos

cp)

we

with centre

at

the coordinate

obtain:

(r sin cp)

= ^1

r8

Multiple and Line Integrals

254

Since the coordinate r in the region


to 2jt, it follows that
varies from

varies

2Jt

ff

Pass to

to

for

any

q>,

and

q>

y\x*
coordinates r and cp and set up the limits of
to the new variables in the following

polar

with

integration

from

(Ch. 7

respect

integrals:

2160.

2162.

K,

2161.

y)dy.

Jd*$/(]/J

JJ/(x, y)dxdy,
\-'/

where S

a triangle

is

bounded by the straight

lines

(/

= #, y~

x,

</=!.

2163.

-i

2164.

^f(x, y)dxdy, where S

is

bounded by the lemniscate

(S)

Fig. 92

2165. Passing to polar coordinates,

calculate the double inte

gral
(S)

where S

C(f,0)

is

a semicircle

(Fig. 93).

of

diameter a with centre at the poin

Sec. 2]

Change of Variables in a Double Integral

255

2166. Passing to polar coordinates, evaluate the double integral

(S)

extended over a region bounded by the circle jc*


y* = 2ax.
2167. Passing to polar coordinates, evaluate the double

in-

tegral

(S)

integration S is a semicircle of radius a with


centre at the coordinate origin and lying above the #-axis.
r
2168. Evaluate the double integral of a function f(r, <p)
over a region bounded by the cardioid r
a(\ +coscp) and the
a. (This is a region that does not contain a pole.)
circle r
2169. Passing to polar coordinates, evaluate

where the region

of

V a*-x*

Jdx

Vx' +

tfdy.

2170. Passing to polar coordinates, evaluate

(S)

where the region S

is

loop of the lemniscate

2171*. Evaluate the double integral

ll

T/^R;

(S)

extended over the region S bounded by the ellipse

x
x*
-j

+ ^^
u

passing to generalized polar coordinates:


X

^wo

2172**. Transform
c

p*

\dx\f (x y)dy
t

(0<a<p

ax

and c:>0) by introducing new variables u

256

__

Multiple and Line Integrals

2173*.

= x + y,

Change the variables u

= xy

[Ch. 7

in the integral

\dx\f (x,y)dy.
2174**. Evaluate the double integral

(S)

where S

is

bounded by the curve

a region

-_ k

b2

Hint.

Make

~~fi 2

'

the substitution
br sin

cp.

The area

of

>,

Sec. 3. Computing Areas

1. Area

in rectangular coordinates.

a plane region S

is

(S)
If

the region

defined

is

by the

a^x^b,

inequalities

q>

(x)

^y ^

\|)

(x)

then
b

op

\dx

X)

is

cp

dy.

(x)

2. Area in polar coordinates. If a region S in polar coordinates r and


defined by the inequalities a^cp^p, / (cp)^/'
(q>), then

<F

= ffrdcpdr=

(S)

6/9

q>

P)

/-dr.

/(q

2175. Construct regions whose areas are expressed by the in-

tegrals
x+2

a))

dx
J

d;
dy;

b)

dy

d*.
J

Evaluate these areas and change the order of integration.


2176. Construct regions whose areas are expressed dy the
tegrals
ji

arc tan

a)

]
n_

T
Compute

d(f

sec

<p

rdr\

b)

"T"
these areas.

a(i+coscp)

^9

\
a

in-

Sec

_ _
Computing Areas

3]

2177.

Compute

the area bounded by the straight lines

257

y,

2178. Compute the area lying above the x-axis and bounded
3a.
4ax, and the straight line x-[-y
by this axis, the parabola y*

2179*.

Compute

the area bounded by the ellipse

2180. Find the area

bounded by the parabolas

f
z
y =10x4- 25 and y

6x +

9.

2181. Passing to polar coordinates, find the area bounded by


the lines
2
2
z
Q.
x
x -[-y*
4x,
2x,
x,
y
y

-y =

2182. Find the area bounded by the straight line r cosq)=l


circle r~2. (The area is not to contain a pole.)
2183. Find the area bounded by the curves

and the

= a(14coscp)

and

acosq>(a>Q).

bounded by the

2184. Find the area


(

~*~

V 4

line

(/!V__
" ~ **_//?
"9 J

'

4"

2185*. Find the area bounded by the ellipse

(x2y

!-3)

4 (3* -\-4y-

1)

100.

2186. Find the area of a curvilinear quadrangle bounded by


2
z
the arcs of the parabolas x --=ay x*
a,x,
y
by y*
$x(Q<.

<a<b, 0<a<p).
Hint. Introduce the

new variables u and


x2

= uy,

if

u,

and put

vx.

2187. Find the area of a curvilinear quadrangle bounded by


the arcs of the curves if=ax, if
a, xy
bx, xy
$(Q<.a<.b,

0<a<p).
Hint. Introduce the

new variables u and


u,

-1900

v,

y*~vx.

and put

(Ch. 7

Multiple and Line Integrals

258

Sec. 4. Ccmputing Volumes

The volume V

a cyltndroid bounded above by a continuous surface


of
low by the ph-ne 2
0, and on the sides by a right cylindrical
surface, which cuts out of the ju/-plane a region S (Fig. 94), is equal to
*

= /(*,

y),

be

2188. Use a double integral to express the volume of a pyravertices 0(0,


0)",
A(\, 0, 0), fl(l, 1,0) and C(0, 0, 1)
ol
the
limits
Set
integration.
95).
up
(Fig.

mid wiih

C(0,0,1)

Fig. 94

Fig. 95

In Problems 2189 to 2192 sketch the solid


expressed by the given double integral:
2189.

f
J

dx

f (1
J

Z-X

2190.

y)dy.

2193. Sketch the solid


V a*

tegral

dx

- x*

YC?

2191.

value

is

x)dy.

2192.

whose volume

tfy* dy\

whose volume

is

expressed by the in-

reason geometrically to find the

of this integral.
2184. Find the volume of a solid bounded by the elliptical
2
x + y=\, and the coordi2x* -f f/
1, the plane
paraboloid z
nate planes.
215. A solid is bounded by a hyperbolic paraboloid z x* tf
and the planes (/ 0, e 0, x=l. Compute its volume.

Sec. 5]

2196.

planes

*/

solid

= 0,

is

= 0,

th

Computing

Area* of Surfaces

2
bounded by the cylinder x

y = x.

Compute

its

=a

+-z

259

and the

volume.

Find the volumes bounded by the following surfaces:

= y\ x* -\-y* --='*, 2 = 0.
=
= 2J/T, * + 2 = 6, 2 = 0.
y"x,
y
=
x* +y\ y = x
z
//=!, 2 = 0.
x -}-*H-2 = a, 3*4-*/ = a, ~x4-tj=^a,
+ -1, y = *' = 0, 2 = 0.
= 2ax, 2 = a*, 2 = p* (a > p).
x
if

2197. az
2198.

2199.

2200.
2201.

2202.

f/

*/

= 0,

= 0.

-h

In Problems 2203 to 2211 use polar and generalized polar


coordinates.
2203. Find the entire volume enclosed between the cylinder
2
2
2
2
a
x 2 -\-y
a and the hyperboloid x -f- if
2*
2204. Find the entire volume contained between the cone
2
2
2
z* =
a
2
and the hyperboloid x 2 -f- if
2(* +if)
2
x + /* f
2205. Find the volume bounded by the surfaces 2a2

**

4-//

=a

= 0.

volume

2206. Determine the

of the ellipsoid

2207. Find the volume of a solid bounded by the paraboloid


2
2
2
-= x 2 H- // 2 and the
3a
-f- 2
(The volume lying
sphere * 4inside the paraboloid is meant.)
2208. Compute the volume of a solid bounded by the jq/-plane t
2
2
2
2
the cyl inder x -*- y
2a^, and the cone x -f if = 2
2209. Compute the volume of a solid bounded by the jq/-plane,
2
2
2
the surface 2 =-ae~ <*'J4 " >, and the cylinder x + y = /?
2210. Compute the volume of a solid bounded by the *f/-plane,

202

the paraboloid

=^+

=
and the cylinder
^-h| 2

2211. In what ratio does the hyperboloid


2
2
2
divide the volume of the sphere x 4- t^
2212*. Find the volume of a solid bounded
4-

2~.

jc

^3a

-h//

= a*

by the surfaces

Sec. 5. Computing the Areas of Surfaces

The area o of a smooth single-valued surface z


on the jci/-plane is the region S, is equal to

-JJ

= f(x

y),

whose projection

Multiple and Line Integrals

260

[Ch. 7

%+j+=

2213. Find the area of that part of the plane

lies between the coordinate planes.


2214. Find the area of that part of the surface of the cylin2
2
der x 4 y
R 2 (z^O) which lies between the planes z mxand

which

=
= nx(m>n>Q).
2215*.
2

Compute
2
2

=z

the

area of that

part

which is situated
y
bounded by the plane y -\-z-a.

cone x

2216.

Compute
2
2

y
cylinder 2
2
2
2
x
y +z =-a
2217. Compute

the area

=ax

of that

which

part

of

out

of

cut

is

of the

in the

first

the
it

surface of the
octant and is

surface of the
by the sphere

sphere x

-\-

the area

=a

of that

part

of the
y2

cut out by the surface


-^

surface

of

the

..2

+ ^=1.

2218. Compute the area of that part of the surface of the


2
2
ax
paraboloid y + z = 2ax which lies between the cylinder tf
and the plane x= a.
2219. Compute the area of that part of the surface of the
2
2
2ax which lies between the xy-plane and the
cylinder x + y

cone x

-\-y

=z

2
.

2220*. Compute the area of that part of the surface ol the


2
2
= 2ax.
cone x 2 y 2
which lies inside the cylinder x 2
z
f/
2221*. Prove that the areas of the parts of the surfaces of the
2
2
z
2
2az arid x
2az cut out by the cylinparaboloids x + y
y
2
2
der x -\-tf
are
size.
of
R
equivalent
2222*. A sphere of radius a is cut by two circular cylinders
whose base diameters are equal to the radius of the sphere and
which are tangent to each other along one of the diameters of the
sphere. Find the volume and the area of the surface of the remaining part of the sphere.
2223*
An opening with square base whose side is equal
to a is cut out of a sphere of radius a. The axis of the opening
coincides with the diameter of the sphere. Find the area of the
surface of the sphere cut out by the opening.
of
2224*. Compute the area
that
part of the helicoid

= carctan

ders

X
2

-i-y

which

=a

lies

in

the

first

octant

between the cylin-

and

Sec. 6. Applications of the Double

Integral

in

Mechanics

1. The mass and static moments ot a lamina. If S is a region in an


jq/-plane occupied by a lamina, and Q (x, y) is the surface density of the
lamina at the point (x, y), then the mass
of the lamina and its static

Sec. 6]

Applications of the Double Integral in Mechanics

moments
x and
double integrals
A4

MY

relative to the

--=

(x, y)

dx

and t/-axes are

x-

Mx= J

dy,

jj

(S)

yQ

expressed

261

by the

dx dy,

(x, y)

(S)

M Y =^

J*e(x,

y)dxdy.

(1)

(5)
If

the lamina

homogeneous, then Q

is

2. The coordinates

centre of gravity of a lamina, then

My
_i_

where

is

tive to the

formulas

(1)

const.

(x, y)

of the centre of gravity of a lamina.

-M
~_
//

M x My

coordinate axes (see 1).


we can put Q=l.

the

3. The moments
lamina relative

of

to the x-

/X=

S S

and

t/-axes are,

y'Q (x, y) dx dy,

of

its

is

moments

The

moments

static

rela-

homogeneous, then in
of inertia 01 a

respectively, equal to

/r=

*2 Q

J J

(*. y)

*x dy.

(2)

(S)

(S)

The moment

are

lamina

a lamina.

of

inertia

the

'

the mass of the lamina and


If

is

(x, y)

x
d

'

If

inertia of a lamina relative to the origin

is

(3)

Putting Q(X, //)-=! in formulas


inertia of a plane iigure.

2225. Find the

mass

(2)

and

(3),

of a circular

we

get the geometric

lamina

of radius

proportional to the distance of a point


density
and is equal to 6 at the edge of the lamina.
is

2226.

OB = a

A lamina has the


OA = b, and its

shape of a right

moments

if

of

the

from the centre

triangle

with

legs

and

density at any point is equal to the


distance of the point from the leg 0/4. Find the static moments
of the lamina relative to the legs 0/4 and OB.
2227. Compute the coordinates of the centre of gravity of the
sin*
area OmAnO (Fig. 96), which is bounded by the curve //
and the straight line OA that passes through the coordinate origin

and the vertex

(-^

Ij

of a sine curve.

2228. Find the coordinates of the centre of gravity of an area

bounded by the cardioid r = a(\ + cosij)).


2229. Find the coordinates of the centre

of gravity of a circular sector of radius a with angle at the vertex 2a (Fig. 97).
2230. Compute the coordinates of the centre of gravity of an
area bounded by the parabolas //
4.x f 4 and if
2x4-4.
2231. Compute the moment of inertia of a triangle bounded
2 relative to the #-axis.
2, y
2, #
by the straight lines x
y

[Ch. 7

Multiple and Line Integrals

262

2232. Find the


ters

d and

moment

D(d<D):

a)

of inertia of an annulus with diamerelative to its centre, and b) relative to

diameter.
2233. Compute the moment of inertia of a
relative to the axis passing through its vertex
the plane of the square.
2234*. C)mpute the moment of inertia of
the parabola if = ax by the straight line x
its

square with side a


perpendicularly to

segment cut

=a

straight line

//

relative

to

oil

the

a.

Fig. 96

2235*. Compute the moment of inertia of an area bounded by


4 and the straight line x-\-y
the hyperbola xy
5 relative to
the straight line x
y.
2236*. In a square lamina with side a, the density is proportional to the distance from one of its vertices. C:>mpute the moment of inertia of the lamina relative to the side that passes
through this vertex.
2237. Find the moment of in?rtia of the cardioid r
a(l
cos<p)
relative to the pole.
2238. Compute the moment of inertia of the area of the lem2
2
niscate r ^-2a cos2cp relative to the axis perpendicular to its
in
the
pole.
plane
2239*. Compute the moment of inertia of a homogeneous lamina
bounded by one arc of the cycloid x^a(t
sin/), y
a(\
cos/)
and the x-axis, relative to the x-axis.

=
=

Sec. 7. Triple Integrals

1. Triple integrals in rectangular coordinates. The triple integral of the


/(*, y, <) extended over the region V is the limit of the corresponding threefold iterated sum:
function

62

= lim

max
max
max

\x\
A'/j

- o
~> o

Az/c -> o

2 2 2^
f

fa

(*t> &/>

'

Sec. 7]

2o3

Triple Integrals

Evaluation of a triple integral reduces to th? successive computation


ordinary (onefold iterated) integrals or to the computation
double and one single integral.
three

Example

Z d*d/dz,
JJ*y
V

/=.$
where the region V

We

is

defined by the inequalities

have
X

2
'</

Example

one

Compute

1.

Solution.

of the

of

7T

dy

Evaluate

2.

AT

extended over the volume of the ellipsoid

//
--

-|-

-j-

Solution.
a

x2

dxdy

dz

x2

dA-

J J

-a

(V)

S yz

is

We

^ + -^^1 --

the area of the ellipse

S vvzz -=Jib I/

x*S
^

yz

dx

-a

(5^.)

M^

\\here

dydz=

^^

j^2

JC

= cons ^

an(^

=*n
\-V
r
a

l-~-c

a-

therefore finally get

-a

(V)

2. Change

of variables in a triple integral.

0.

If

in tha triple integral

2)dxdydz

from the variables AT, //, z to the variables


it is required to pass
u, ay,
which are connected with *, //, z bv the relations x
^(u v, w), y = ty(u,v,w),
z = X(". u ^)i where the functions cp, i|), x are:
1) continuous together with their paitial first derivatives;
,

2)

in one-to-one (ind,

tween the points

some region

in

both directions, continuju?) correspondence beV oi xf/z-space and the points of

of the region of integration

of l/l/l^-space;

[Ch. 7

Multiple and Line Integrals

264
3) the functional

of these functions

determinant (Jacobian)

dx dx

dx

~dv

dw

dii

D (u,

v,

dy
dy dy
da dv dw

w)

dz

dz

dz

dw

da dv

we can make

region V, then

retains a constant sign in the

use

the for-

of

mula

=
$ J

\f(x,y,2)dxdydz

(V)
\

IT ( w

\ f

L?

w )>

>

ty( u

>

w )<

In particular,
1) for cylindrical coordinates

X
get

cp,

rcosrp,

spherical coordinates

the radius vector) (Fig. 99),

we have

r,

//

h (Fig. 98), where


z^-//,

rsinrp,

r\

2) for
r

du dv dw.

Fig. 99

Fig. 98

we

Example

3.

= r cos

cos

i|)

cp,

ap,

r (<p

is

the

longitude,

q),

\\>

the

latitude,

where

cos 9,

f/

= /-cosi|3 sin

sin

v|\

i[).

Passing to spherical coordinates, compute

JSJ
(V)

where V

is a sphere of radius R.
Solution. For a sphere, the ranges of the spherical
tude), \|) (latitude), and r (radius vector) will be

coordinates

fp

(longi-

Sec

We

265

Triple Integrals

7\

therefore have

f f f

Vx

-\-y

3. Applications
sional A'//z-space

-\-z*dxdydz=\

of

dcp

dty f r A'COS \|?dr

The volume

triple integrals.

= Ji# 4

of a region of

three-dimen-

is

<n

The mass

of a solid

occupying the region V

-----

C f f

z)

is

(V)

where \(x,y,z) is the density of the body at the point (*,//,*).


The slal/c moments of the body relative to the coordinate planes are

M yy =
"(V)'

Myz =

(*

(*

I
'

)}
0')"

MZX

~-

'

(A

f/

2)

f/

dx dy dz.

(V)

The coordinates

gravity are

of the centre of

_
J ~~

'

'

Al

then we can put Y (*, y> z) =


If the solid is homogeneous,
mulas for the coordinates of the centre of gravity.
The moments of inertia relative to the coordinate axes are

T-

J J $

(y*

+ **) Y (*,

U, 2)

dx dy

in

the

for-

dz;

(V)

(V)
l

=J

J J

(*

(V)

Putting
of inertia

of

Y(*0

^i^21

in

* nese

formulas,

we

get the

the body.

A. Evaluating triple integrals


Set up the limits of integration in the triple integral
J J
(V)

for

the indicated regions V.

^f(x,y,

z)dxdydz

geometric

moments

266

Multifile

2240.

is

a tetrahedron
Jr

2241.

is

2242*.

2243.

is

+ ^fl

J/

= 0,

= 0.

= 0,

= /f.

volume bounded by the surfaces

000

2245.

= 0,

cone bounded by the surfaces

Compute the following


2244.

bounded by the surfaces

a cylinder

is

bounded by the planes

yJrZ =\,

JC'

[Ch. 7

and Lire Integrals

integrals:

+ +-2+1

djt

dy

2246

fd*

00

('

1-X

2247.

rfy

.1

.)

dx

1-JC-t/

dy

xyzdz.

J
o

2248. Evaluate

d* dy dz
'

1)3

where V is the region ol integration


planes and the plane x-\~y-[z\.
2249. Evaluate

bounded

by the coordinate

r r r
(V}

where V (the region


2
paiaboloid

2cu^x

of
2

-\-y

integration) is the
and the sphere X?

common

of the

part

+ y* + 2 ^3a

2
.

2250. Evaluate

(V)

where
spheres

V (region
x

+ y*

\-z*

of

integration)

^R'

and

is

the

+ \f + z

common

^ 2Rz

part

of

the

gee.

__
Triple Integrals

7]

267

2251. Evaluate

^zdxdydz,
(V)

where V

is

volume bounded by the plane


j. +

half of the ellipsoid

-+

and the upper

-J.==l.

2252. Evaluate

(V)

where

Xz
is

2
IJ

+ "^r

the interior of the ellipsoid


~^r

2253. Evaluate

where
2

(jc

region

(the
2

hi/

of

is

integration)

and the plane

bounded

the

by

cone.

= h.

2254. Passing to cylindrical coordinates, evaluate

where V
2

jc

the surfaces x*
is a region bounded by
a
-=z and containing the point (0,0, R).
-|
//
2255. Evaluate
2

21

jc

transforming it to cylindrical coordinates.


Evaluate

first

225(5.

di/

dz,

transforming it to cylindrical coordinates.


2257. Evaluate

first

VR*-X*

Vfla-jir*-0a
J

\dx
-/?
first

transforming

-/f^TJa
it

dy

(A:

to spherical coordinates.

+y*

-\-

z*

= 2Rz

[Ch. 7

Multiple and Line Integrals

268

2258. Passing to spherical

evaluate the integral

coordinates,

(V)

where V
B.

is

the interior of the sphere x -\-y

Computing volumes by means

2259. Use a triple integral


bounded by the surfaces

to

+z

triple integrals

of

compute the volume

of a solid

2260**. Compute the volume of that part of the cylinder


2
2
x 2 -f tf = 2ax which is contained between the paraboloid* + y = 2az
and the xy-plane.
2261*. Compute the volume of a solid bounded by the sphere
2
2
2
2
2
2
x +y +z =a and the cone z ---x + /y" (external to the cone).
2262*. Compute the volume of a solid bounded by the sphere
2
2
z
x 2 +y +z = 4 and the paraboloid x +if=-3z (internal to the
paraboloid).
2263. Compute the volume of a solid bounded by the xy-plane,
z
2
2
2
z
2
a (internal
ax and the sphere x
the cylinder x
-f- z
to the cylinder).
2264. Compute the volume of a solid bounded by the paraboloid

+y =

+y

+ -~ = 2 -i

and the plane

C. Applications
to

of

triple integrals

mechanics and physics

2265. Find the mass

0<z<c,

<*/<??,

Fig.

and the plane

(l(x,

y,

z)

point

the den-

if

y,

(x,

= x-\-y-\-z.

z)

is

2266. Out of an octant of the


2
2
2
2
c
z
x .> 0,
x
y
sphere
cut a solid OABC
*/^0,
bounded by the coordinate planes

+ +
z^O

100

-+- =

sity

the

of a rec-

Q^x^a,

tangular parallelepiped
at

x--=a.

(a

c,

&<c)

(Fig.

100).

<

Find the mass

of this body if the density at each point (x, y, z) is equal to


the 0-coordinate of the point.
2267*. In a solid which has the shape of a hemisphere
22*0, the density varies in proportion to the

Improper Integrals Dependent on a Parameter

Sec. 8]

260

distance of the point from the centre. Find the centre of gravity
of the solid.
2268. Find the centre of gravity of a solid bounded by the
2
2
4x and the plane x=2.
paraboloid // +2z
2269*. Find the moment of inertia of a circular cylinder,
whose altitude is h and the radius of the base is a, relative to
the axis which serves as the diameter of the base of the cylinder.
2270*. Find
the
moment of inertia of a circular con^
base,
a, and
(altitude, /i, radius of
density Q) relative to
the diameter of the base.
2271**. Find the force of attraction exerted by a homogeneous
cone of altitude h and vertex angle a (in axial cross-section) on
a material point containing unit mas^ and located at its vertex.
2272**. Show that the force of attraction exerted by a homogeneous sphere on an external material point does not change if
the entire mass of the sphere is concentrated at its centre.

on a Parameter.

Sec. 8. Improper Integrals Dependent

Improper Multiple Integrals


1. Differentiation with respect to a parameter. In the case of certain
imposed on the functions / (.v, a), f'a (x, a) and on the corresponding improper integrals \vc have the Leibniz rule

restrictions

(.v,

a) dx

Example

fa

(A-,

a) dx.

differentiating with respect to a parameter, evaluate

By

1.

'i

~~

>

dx

(a

> 0,

> 0).

Solution. Let

Then

~
da

Whence F
equation.

(a,

We

p)

have

Whence C(p)

2a

Ina + C(p). To

0=

= -^-lnp.

In P

Hence,

find

+ C(P).

"2a*

C(p), we put a =

in the latter

Multiple and Line Integrals

270

2. Improper double and


a) An infinite region. If
region 5, then we put

triple integrals.
a function f (x, y) is continuous in

y)

\{f(x,
U

[Ch. 7

dx dy

lim

"S\

( f (x, y)

an unbounded

dx dy,

(1)

where a is a finite region lying entirely within S, where a -+ S signifies that


we expand the region o by an arbitrary law so that any point of 5 should
enter it and remain in it. If there is a limit on the right and if it does not
depend on the choice of the region o then the corresponding improper inte,

gral

is

called convergent

otherwise it is divergent.
is
nonnegative [f (x,
//)

If the inU'grand / (,v,


vergence of an inirioper integral

is

it

y)^Q], then

on

the right of (1) lo exist at least for


the region 5.
b) A discontinuous function. If a function / (x, y)
uous in a bounded closed region S, except the point
(*,

for

the con-

and sufficient for the limit


one system of regions o that exhaust

necej-sary

y)dxdy=\\m

is

everywhere contin-

(a,

b),

then we put

f(x,y)dxdy.

(2)

where S 6 is a region obtained from S by eliminating a small region of dia


meter e that contains the roint P. If (2) has a limit that does not depend
on the tyre of small regions eliminated from 5, the improper integral under
is called convergent, otherwise it is divergent.
/(A, //)^>0, then the limit on the ripht of (2) is not dependent on the
type of regions eliminated from S; for instance, such regions may be circles

consideration
If

with centre at P.

of radius

The concept
of

of

improper double integrals

is

readily extended to the case

triple integrals.

Example

2.

Test for convergence

dxdy

3)

is}

where S

is the entire j/-plane.


Solution. Let a be a circle of radius
origin. Passing to polar coordinates for

If

p<

then

then

1,

lim

lim 7 (a)
O -* S

7(o)=

= lim
Q

/(a)

=00 and

with

p^

1,

centre

at

the

coordinate

we have

the integral diverges. But

if

p>

1,

~f QC

and

the

integral

converges.

For

p=l

we

have

Sec. 8]

_
Q

2JT

Improper Integrals Dependent on a Parameter

(a)= Cdcpf -f^l-==jiln(l+Q


r
*
J

lim/(a)

);

~r

Q -*

oo

for

p>

= oo,

that

is,

271

the

integral

diverges.

Thus, 'the integral

2273. Find

(3)

/' (*),

converges

1.

if

2274. Prove that the function


+

00

*f'

2
\
J ^

,2

(l/

dz

2)

QC

satisfies

the Laplace equation

*u
dx*

is

'

~c)y*

2275. The Laplace transformation


defined by the formula

Find
d)

d
~U
+
^ *-Q

F(p),

if:

= cos p/.
/(/)

a)

/(O-l;

b)

(p)

/(/)=e;

2276. Taking advantage of the (ormula

compute the

integral
\

xn

2277*. Using the formula

evaluate the integral

for the

\nxdx.

c)

function /(/)

/(/)

= sinp/;

272

Multiple and Line Integrals

Applying differentiation with respect

__

\Ch. 7

to a parameter,

evaluate

the following integrals:


GO

2278.

"

00

2279

rno^
228

"-""*

~**~ e ~'X

ax

arc tan

dx

>

0,

>

0).

smrnxdx (a>0, p>0).


.

dx

e-<"d*

2282.

(a

(o^O).

Evaluate the following improper integrals:


GO

QC

2283.
x_

2284.

2285.

\dy\ev

dx.

\\

c) c/

*,

^ 4 ~r

where S

is

a region defined

by the inequali-

(5)

ties

#^5

2287.

1,

The

Euler-Poisson

integral

defined

by

the

formula

00

may

also be written in the

uate / by multiplying
coordinates.
2288. Evaluate
CO

form

I=\c-^dy.

Eval-

these formulas and then passing to polar

00

GO

dz

273

Line Integrals

See. 9]

Test for convergence the improper double integrals:


2

2289**.

where S

55 \nVx*-\-y dxdy,

is

circle

2
f/

<K

(S)

2290.

where S

TTiriva

is

defined by

region

the ine-

(6')

quality x

2291 *.

2292.

tf^\

-|

$,

("exterior" of the circle).


.

J_

d xd

CCr
JJJ

(x T~

where S

^\
\

a square

is

where V

is

<

A;
|

\y\^l.

defined

region

by

the

(V)

inequality x

-\-

if

-\-

("exterior" of a sphere).

Sec. 9. Line Integrals

1. Line integrals of the (Irst type. Let / (x, y) be a continuous function


= (p (A-) [a<: \ <;&] be the equation of some smooth curve C.
*/
Let us construct a system of points M, (A-,-, //,) (/~0, 1, 2, .... n) that
break up the curve C into elementary arcs Al _,M = As and let us form the
and

integral

max

2^

sum S n

As/ -^ 0,

is

AV

^ s r ^ ne

#/)

'

im it of this sum,

when n

-* oo

and

called a line integral of the first type


n

lim

"->>ac
(cfs

is

2=

/(xf,

r/,-)

As/=

/ (x,

i/)

ds

the arc differential) and

is

evaluated from the formula

b
'

f (A

!/) rfs

C
In

the

case
-

'

"

of
rt

],

J
a

f(x

parametric

we have

cp (,v))

y~\

+ (q>'

representation

of

(A-))

the

c/.vr.

curve

C:

.Y

= q>(/),

Also considered are line integrals of the first type of functions of three
variables f (x, y, z) taken along a space curve. These integrals are evaluated
in like fashion A line integral of the lirsl type does not depend on the direction of the path of integration; if the integrand / is interpreted as a linear
density of Hie curve of integration C, then this integral represents the mass
of the curve C.

274

Multiple and Line Integrals

Example

1.

Evaluate the

[Ch. 7

line integral

where C is the contour of the triangle ABO with vertices A (1,


and 0(0, 0) (Fig 101).
Solution. Here, the equation AB is (/=l
x, the equation
and the equation OA is */ = 0. We therefore have

AB

2. Line
uous

BO

0),

OB

B
is

(0,

1),

= 0,

OA

integrals of the second type. If P (x, y) and Q (x, y) are continand f/ <p(A) is a smooth curve C that runs from a to b as

functions

A
Fig.

x varies, then the corresponding

101

line integral

of the

second type

is

expressed

as follows:

(x,

y)dx + Q

(x,

y)dy =

[P

(x,

cp

(*))

<p'

(x)

(x, q> (x))\ dx.

In the more general case when the curve C is represented parametrically:


y
ty(t), where / vanes from a to 0, we have

= <P(0,

y)

Similar formulas hold


space curve.

dy

for

'

[P

(<p

Une

(0,

integral of

(9(0,

the

second

type

(OJ

taken

over a

A line integral of the second type changes sign when the direction of the
path of integration is reversed. This integral may he interpreted mechanically
P (x, y), Q(x, y) } along the
as the work of an appropriate variable force
{
curve of integration C
Example 2. Evaluate the line integral
Cyi

____

____

C is the upper half


clockwise.
Solution. We have

where

2
y dx

275

Line Inlegtah

Sec. 9]

+ x* dy =

[b

sin

the

of

t-(a sin

*--=a cost,

ellipse

/)

+a

-b cos /] dt

ab* f sin 8

dt

+ a*b

jt

3. The case

of a total differential.

traversed

sin*

oo

cos 2

=b

dt

= -t a&.

?t

the

If

C cos 8

of

integrand

line integral

second type is a total differential of some single-valued function


U^--U(x, */), that is, P (x, y)dx-\-Q(x, y)dy-=dU(x, y], then this line integral
is not dependent on the path of integration and we have the Newton-Leibniz
formula
(x 2 yj

of

the

P(x, y)dx

where

is

In particular,

if

(x lt

y)dy = U(x 2

+ Q(x,

the initial and (* 2 #,) is the


the contour of integration C
,

yJ

terminal
is

(x l9

point
then

closed,

(1)

</,),

the

of

path

(2)

of integration C is contained entirely within some


S and 2) the functions P(r, (/) and Q (x, y) together
with tlvir partial dcnviitives of the first order are continuous in 5, then a
is the
necessary and sufficient coiditioi foi th? existence of the function
identical" fulJilment (in S) of the equality
If

1)

the

contour

simnlv-connectH

reaio.i

2_^
dX

(3)
(d)

~dy

If conditions one and two are not ful(see integration of total differentials)
does not guarantee the existence of a
jillcd, the presence of condition (3)

single-valued lunction U, and formulas (1) and (2) may prove wrong (see
Problem 23:2) We give a method of finding a function U (x, u) from its
total diflerential based on the use of line integrals (which is yet another
method of integrating a total differential). For the contour of integration C
let us take a broken line P P
(Fig 102), where P (.v
y ) is a fixed [oint
and
0,
y and dy
(x, y) is a variable point. Then along P P, we have y
We get:
we have dx
and along P t

(x

y)-U

II

(J

(x

)=

(x,

y)

dx

+ Q (x,

y)

dy =
y
(x.

Similarly, integrating with respect to

P C P 2 M, we have
x

U(x.

y)-U(*v

*/

)-$Q(* y)dy+^P(x,
,

Vo

x*

y)dx.

y)dy.

[Ch. 7

Multiple and Line Integrals

276

+ 2y) dx+ (2x 6y) dy = dU.


= O, y = Q. Then

Find U.

Example 3. (4x
Solution. Let A-O
U(x,

y)

or

+ 2y) dx + C=
U

where C

(0,

0)

is

-i-c,

3//

an arbitrary constant.

Uo

--4/I

XQ

102

Fig.

4. Green's formula for a plane. If C is the boundary of a region S and


the functions P (x, y) and Q (x, y) are continuous together with their firstorder partial derivatives in the closed region S-j-C, then Green's formula holds:

(S)

here

t'^e

remain

circulation about the contour

chosen so that the region S should

is

to the left.

5. Applications

of line integrals.

(p

An
y

area

dx=

bounded by the closed contour C

(p

is

xdy

(the direction of circulation of the contour is chosen counterclockwise).


The fo lowing formula for area is more convenient for application:

2)

The work
(x,

X=

X(x, y,
of a force, having projections
accordingly, the work of a force field),

z) (or,

2),

Y=Y

(x, y,

along a path

z),
is

Sec. 9]

__

277

Line Integrals

expressed by the integral

A= ^Xdx + Ydy + Zdz.


c

If the force has a potential, i.e., if there exists a function


potential function or a force function) such that

(a

dV
dx

=A

dU

dU
j

TT

-7

*j)

//2,

where

(v l5

(*J,

'/J.

(*i,

l/ lt

Xdx + Ydy + Zdz^

J
(*|. {/,,

dU

the initial and (x 2

Zj) is

= U(x

is

tli2

2293.

xyds, where

2t

y tt

z 2 )-t/ (x lt y,

z } ),

terminal point of the path.

A. Line Integrals of the First

Evaluate the following

equal to

2-J

z2)

</,,

is

Z.)

Zi)

f/ 1?

(x, y, z)

dz

dy

then the work, irrespective of the shape of the path C,


(*.

U=U

Type

line integrals:

the contour of the square

is

x\

y\

=a

(fl>0).
s

2294.

--^=

,.
2
A:

if

|-

connecting the points 0(0,

^xyds, where

2295.

where

a segment of the straight line

is

\-

and A

0)

(I,

a quarter

is

2).

of the

lying

the

in

2296.

first

fi>

quadrant.

where C

ifds,

ellipse ^i

is

the

first

arc of the cycloid x

=a

(t

sin /)>

cos

(1

/).

}x

2297.

+y

ds,

where

circle

----

a (cos

2298.

^ (x

(-/sin/), y
2

2
-\-

d$,

is

an arc

= a(smt

where C

is

of the involute of

the

tcost) I0^/=^2ji].

an arc

of the logarithmic spi-

c
ral

r^ae m v(m>Q)
2299.

niscate r

2300.

+ y)
JU
c
= a cos2<p.
rfs

where C

is

(0, a)

to the point

0(

oo, 0).

the right-hand loop of the lem-

(A:

\-y)ds,

c
q/2

from the point

>

where

is

an

arc

of

the

curve

JK

=^

Multiple and Line Integrals

278

2301.

%2

(*

~T~

# = acos/ y = as\nt,
f

2302.

J t/"2#

is

the

where

is

turn of the screw-line

first

= bt.

-I-

where

*2

[Ch. 7

z ds y

the

circle

+ y +z = a
2

2
,

x=y.
2303*. Find the area of the

cylinder y

surface

lateral

bounded by the planes

the parabolic

of

= 0, x~0, 2 = x, = 6.
//

ae cos?,
2304. Find the arc length of the conic screw-line C x
sin /, z
ae from the point 0(0, 0, 0) to the point A (a, 0, a).
2305. Determine the mass of the contour of the ellipse
1

y = ae*
2
2-

yi
-T2-

= l,

if

the linear density ot

it

at each

point

M (x,

y) is

equal to \y\.
2306. Find the mass of the first turn of the screw-line A; = a cos/,
y = asmt, zbt, if the density at each point is equal to the
radius vector of this point.
2307. Determine the coordinates of the centre of gravity of
a half-arc of the cycloid

x = a(t

sin

/),

y = a(\

cost)

[0</<ji].

2308. Find the moment of inertia, about the z-axis, of the


turn of the screw-line x = a cos/, !/
bt.
asin/, z
2309. With what force will a mass
distributed with uni2
2
form density over the circle x -f y 2
a
z
0, act on a mass m
located at the point A (0, 0, &)?

first

B. Line Integrals of the Second

Type

Evaluate the following line integrals:


2310.

J (x

2xy)dx-\- (2xy+tf)dy, where

AB
2
parabola y = x from the point
2311.

(2a

y)dx

-xdy,

^4(1,

where C

to

1)
is

an

AB

is

the

point

arc

an arc

of

of the

B(2,
the

4).
first

arch of the cycloid


x
a(t
a(l
sin/), #
cos/)
which arc runs in the direction of increasing parameter

/.

taken along different paths emanating


J 2xydxx dy
OA
from the coordinate origin 0(0, 0) and terminating at the point
2312.

(2,
ia)

1)

(Fig. .103):

the straight line

Om A\

Lim

Sec. 9]

parabola OnA,

b) the

279

Integrals

the axis oi

symmetry

of

which

is

the

of

which

is

the

(/-axis;

c)

the parabola

the axis of

OpA,

symmetry

x-axis;

broken line

d) the
e)

OBA\
OCA.

the broken line


2

2313.

2xydx ^x dy

as in

Problem 2312.

OA

(x+u)dx

(x

x*

+ y*

i/)dy

counterclockwise.

Fig.

^tfdx + x*dy, where C

2315.

103

the upper half of the ellipse

is

x^acost, y =
2316.

bs\r\t traced clockwise.

cosydxsmxdy

taken along the segment

bisector of the second quadrantal


point A is 2 and the ordinate of
d

x!f(l'

2317.

dll}

^*

(f

where C

angle,

is

is

if

the

AB

of the

abscissa oi the

2.

the right-hand

loop

ol

the

lemmscate
co$2(p traced counterclockwise.
2318. Evaluate the line integrals with respect to expressionswhich are total differentials:
r*

(a. 4)

8)

(2.

a)

rt

xdy -\-ydx,

(2,

f
(1.

c)

i)

$
(0, 0)

1)

1)

ydx
d)

b)

(O r

2)

(i.

xdx + ydy,

2)

xdy
,

(along a path

that

does

not

intersect the

Multiple and Line Integrals

280

(x, y)

dx + dy
x ~ry

f
J

e)

f)

p a th

ong a

^at

[C/t.

not

(joes

intersect

the

= 0),

straight line A;-f-y


(* 2

(a i

___

J/ 2 )

q>

J
(*;, </0

2319. Find the antiderivative functions


evaluate the integrals:

integrands and

0)

(3,

a)

of the

(x<

+ 4xy') dx 4

(6x

5#

dy

(-2, -1)
o)

(i,

* J ~~~ y
b)

!
x

-1)

(0,

straight line y
(,

C)

1)

y)

(the integration path does not intersect the

= x),

(x+Wx + ydy

(the integration

j
a.

i)

the straight line y


A\

(0,

\V

X
!

x*

0)

2320.

+ y*

path does not intersect

x),

yA
J

fix

\}^xZ + y

fat

Compute
/=.

xdx + yfy

taken clockwise along the quarter of the

X*

ellipse -5

the first quadrant.


2321. Show that if f(u) is a continuous
closed piecewise-smooth contour, then

+ ^=!

that

lies in

function and

2322. Find the antiderivative function


a)

b)
c)

du = (2x
du = (3* 2
du =

d)' dtt

if:

+ 3y)dx + (3x4y)dy\

2xy + y

2
)

dx

(x

2xy

+ 3y*) dy\

is

Sec

281

Line Integrals

9]

Evaluate the
curves:
2323.

line

taken along the following space

integrals

z)dx+(zx)dy + (x

^ (y

where

tj)dz,

is

turn

c
of the screw- line
i

= asin/,

corresponding to the variation


2324.

of

the parameter

ydx + zdy + xdz, where

(p

c
I

;t

/y

is

from

to

the circle

= /?cosacos/,
= /?cosa sin
= ^sina (a = const),
/,

traced in the direction of increasing parameter.

2325.

xydx + yzdy

+ zxdz,

where

OA

is

an

arc

of

the

OA
circle

situated on the side of the A'Z-plane where //>0.


2326. Evaluate the line integrals of the total
(,

a)

4.

zdz,

xdx-\-ydy

]
(1,

0,

-3)

(i.

i.

i)

(3,

4.

5)

b)

differentials:

8)

//

dx f 2 x dy
-

xd\

\-

xy dz

\-\idij-\-zdz

rA'.|.y"

+2

'

(0, 0, 0)

-M
^v
d)

yte + mv + 'y**

f
J
(i.

xyz
,

(the

integration

path

is

situated

in the first octant).

C. Green's

Formula

2327. Using Green's formula, transform the line integral


/

\f*

-\

if

dx + y [xy +

In

(jc

where the contour

C bounds

the region S.

K?T?)]

dy,

_ _
Multiple and Line Integrals

282

[C/t.

2328. Applying Green's formula, evaluate


/

2 (x?

if)

-t-

dx +

(x

+ y)

dy,

where C

is the contour of a triangle (traced in the positive direcverlices at the points A (I, 1), fl(2, 2) and C(l, 3).
with
tion)
Verify the result obiained by computing the integral directly.
2329. Applying Green's formula, evaluate the integral

x*y dx

+ xif dy,

where C
2330.

chord

is

Find

the circle x*

is

+ if = R*

traced counterclockwise.

A parabola AmB, whose axis is the #-axis and whose


AnB, is drawn through the points A (1,0) and 8(2,3).
(x

+ y)dx(x

y)dy directly and by applying Green's

AmBnA
formula.
2331. Find

e* y [y*

dx

(1 -f xtj)dy\,

\-

if

the points

A and B

AmB
on the #-axis, while the area, bounded by the integration
path AmB and the segment AB, is equal to S.
2332*. Evaluate
Consider two cases:
lie

^ifc^f.

when
when

the origin is outside the contour C,


the contour encircles the origin n times.
b)
2333**. Show that if C is a closed curve, then

a)

where

s is the arc length and n is the outer normal.


2334. Applying Green's formula, find the value of the integral

= (j)[xcos(X, n)+ysm(X,

n)]ds,

where ds

is the differential of the arc


the contour C.
2335*. Evaluate the integral

taken along the contour of a

(1,

is

traced counterclockwise.

0).

fl(0

1),

C(-l,

0)

and n

is

the outer normal to

square with vertices at the points


1), provided the contour

and>(0,

2S&

Line Integrals

Sec. 9]

D. Applications of the Line Integral

Evaluate the areas of figures bounded by the following curves:


a cos/, y
bs\nt.
2336. The ellipse x
acos 3 /, #-=asin 8 /.
2337. The astroid jt
x a (2 cos/
cos 2/),
2338. The cardioid
y a (2 sin/

=
=

sin 20-

2339*.

of

loop

the

folium

(a>0).

Descartes x*

+if

3zxy

=Q

= axy.

2340. The curve (x + y)*


2341*. A circle of radius
fixed circle of radius

integer, find the area


by some point of the

of r

of

(cardioid).
A circle of

2342*.

R and

is

rolling without

outside

sliding along a
n
is an
Assuming that

it.

bounded by the curve (epicycloid) described


moving circle. Analyze the particular case
radius

a fixed circle of radius

R and

is

rolling

inside

without sliding along


D

Assuming that

it.

is

an

integer, find the area bounded by the curve (hypocycloid) described by some point of the moving circle. Analyze the particular
r>

when

case

=j

(astroid).

2343. A field is generated by a force of constant magnitude F


Find the work that the field does
in the positive jt-direelion
when a material point traces clockwise a quarter of the circle
2
2
x -^-y ^=R lying in the first quadrant.
2344. Find the work done by the force of gravity when
a material point of mass m is moved ironi position A (JC P // l? zjz 2 ) (the z-axis is directed vertically upto position B (x 2
// 2
,

wards).
2345. Find the work done by an elastic force directed towards
the coordinate origin if the magnitude of the force is proportional to the distance of the point fiom the origin and if the point
of application of the force traces counterclockwise a quarter of
the ellipse
2346.

^s4-^i=l
Find

the

lying in the
potential

first

and determine the work done by the


= 0, K:=0. Z-=rng (force
a) X
rial

is

point

B(*
b)

Uv

moved

from

quadrant.

of a force R {X, Y, Z\
force over a given path if:
of gravity) and the mate-

function

position

y l9

zj

where

jx

(x lt

to

position

*i)'-

x=

?,

K=-^. Z=-*

= const

and

Yx* 4 if f (Newton attractive force) and the material point


moves from position A (a, b, c) to infinity;

-\-

_ _
Multiple and Line Integrals

284

X=

c)

force),

x* 4-

Sec.

and

k*x, Y
k*y, Z
the initial point of the

+ z = /?
2

2
,

[Ch. 7

where k

k*z,

= const

(elastic

path is located on the sphere


while the terminal point is located on the sphere

10. Surface Integrals

1. Surface integral of the first type. Let


function and z=-cp(*, y) a smooth surface S.

The

surface integral of the first type

'(x,

//,

z)dS=
n

is

f (x,

//,

2)

be a continuous

the limit of the integral

sum

lim
-> 00

fas

.3

is the area of the


/th element of the surface S, the point (*/, y lt
belongs to this element, and the maximum diameter of elements of partition tends to zero.
The value of this integral is not dependent on the choice of side of the
surface S over which the integration is performed.
If a projection a of the surface S on the jo/-plane is single-valued, that
to the z-axis intersects the surface S at only
is, every straight line parallel
one point, then the appropriate surface integral of the first type may be
calculated from the formula

where AS/
z/)

(*' y)

dx

dlJ-

(a)

Example

where S

is

1.

Compute

the surface integral

the surf ace of the cube

0<Jt<l,

<//<!,

Let us compute the sum of the surface integrals over the


the cube (z=l) and over the lower edge of the cube (z
0):

00

2. Surface

integral

is

of

00

00

The desired surface

upper edge

obviously

three

times

greater

and

equal to

If P
P(x, //, z), Q
Q (*, y, z),
continuous functions and S + is a side of the smooth surface S characterized by the direction of the normal n
{cos a, cos p, cos Y}. t'hen
ihe corresponding surface integral of the second type is expressed as follows:

R = R(x,

integral of the second type.

y, z) are

P dy dz + Q

dz

dx+ R

dx

dy=

f f

(P cos a

-f

cos p

+ R cos Y) dS.

Sec. 10]

285

Surface Integrals

When we pass to the other side, S~, of the surface, this integral reverses sign.
then the direcIf the surface 5 is represented implicitly, F (x, y, z)
0,
tion cosines of the normal of this surface are determined from the formulas

1
OF
COSBQ^
COSa==-pr^-,
D dx

dF

-FT^
D dy

COS VT

OF

-rr- -T

dz

where

and the choice

should

of sign before the radical


of the surface S.

with the side

be

brought into agreement

P (.v, //, z), Q Q (x, //, z),


If
the functions P
continuously differentiable and C is a closed contour bounding a two-sided surface S, we then have the Stokes' formula
3.

formula.

Stokes'

R = R(x,

y, z) are

(j)

dQ\
= rr\fdR
3-- 5- 1
dz
\

JJ l\dy
5

c s

Id?

dR\
Tdx

+
-3
^\dt

fi
cos a
l

+
^
,

dP\

fdQ

T-

3-2-

\dx

dy J

cos v
y

dS,
|

where cos a, cos p, cosy are the direction cosines of the normal to the surface S, and the direction of the normal is defined so that on the side of the
normal the contour S is traced counterclockwise (in a rigiit-handed coordinate
system).

Evaluate the following surface integrals of the


2347.

$$ (*

tf)dS, where S

first

the sphere x +//

is

type:
2

-{-z

= a*.

2348.

5$

Vx

s.

-\-tfdS

where 5

is

the

surface

lateral

the

of

+ g_?6 i= =0 [O^z^bl

cone

Evaluate the following surface


2349.

\ \

integrals

of

the second type:

yz dydz -\-xzdz dx-\- xydxdy, where 5

is

the

external

side of the surface of a tetrahedron

=Q

2350.

= 0, x+y + z = a.

Nzdxdy, where S

is

bounded by the planes

the external side of the ellipsoid

x dydz-\-y*dzdx + z*dxdy,

2351.
o

where S

is

+? =
2

the
2

a (z
side of the surface of the hemisphere
+//
2352. Find the mass ot the surface of the cube
Os^z
1, if the surface density at the point
Is equal to xyz.

O^y^l,

<

0,

A:

external

^0).

O^x^l,

M (x,

y, z)

_ _
Multiple and Line Integrals

286

[C/i.

2353. Determine the coordinates of the centre of gravity of a

2
2
homogeneous parabolic envelope az^je + #

Find

2354.

the

moment

(0<z<a).

inertia of a part

of

=V

the lateral

of

< <

x 2 -f y 2 [0
z
about the z-axis.
surface of the cone z
ft]
2355. Applying Stokes* formula, transform the integrals:
2

(x

a)

- yz) dx + (y

zx)

dy +

(z

xy) dz\

c
b)

(j)

ydx-\-zdy + xdz.

Applying Stokes' formula, find the given integrals and verify


the results by direct calculations:
2356.

(f (y

+ z)dx + (z + x)dy + (x + y)dz,

where

the circle

is

2357.

x)dy +

z)dx^(z

(y

JC

-|-//

(x

y)dz, where

is

the ellipse

X + 2=l.

=1,

+ (x-{-y)dy + (x + y + z)dz, where C is the curve


= acos/, z = a (sin + cos/) [0 </^2nj.
0sin/,
2359. $ y dx-{ z dy + x dz, where ABC A is the contour of
ABCA

2358. ()xdx
c

//

/IflC with vertices

2360. In what case


/

(a, 0, 0),
is

C (0,

(0, a, 0),

0, a).

the line integral

= $ Pdx + Qdy + Rdz


c

over any closed contour

equal to zero?

11. The Ostrogradsky-Gauss Formula

Sec.

If

=Q

thMr

is

closed

R=

(A% y, z),
first partial

smooth surface bounding the volume V, and P

(x, y, z),

are functions that are continuous together with


derivatives in the closed region V then we have the Ostro/? (v,

(/,

e)

gradsky- Gauss formula

where crsa, cos p,


surface S

cosy are the direction cosines

of the outer normal to the

Applying the Ostrogradsky-Gauss formula, transform the following surface integrals over the closed surfaces S bounding the

>ec.

11]

The Ostrogradsky-Gauss Formula

/olume V(t*osa, cosp,


lormal to the surface
2361. JJ

287

cosy are direction cosines of the outer


S).

xydxdy+yzdydz -\-zxdzdx.

2362.

J J
6

x 2 dy dz +

y* dz

dx

+z

d* dy.

2363.

Using the Ostrogradsky-Gauss formula, compute the following


surface integrals:

2o65.

x*dydz + y*dzdx + z*dxdy, where S

the external

is

side of the surface of the

2366.
of a

\ \

cube

O^x^a, O^r/^n, O^z^a.

xdydz + ydzdx + zdxdy, where 5

the external side

is

pyramid bounded by the surfaces x + y-{-z

= a,

= Q,y = Q,

z=-0.
2367.

x*

dydz-\-if dzdx

= z* dxdy,

where 5

the

is

external

side of the sphere x

2368

~a
cos p + 2
2

^(jc cosa-t y

//
2

-|-z

cos

y)

d5,

where S

is

the exter-

nal total surface of the cone

2369. Prove that


direction, then

if

is

closed

surface

and

is

any

fixed

where n

is the outer normal to the surface S.


2370. Prove that the volume of the solid
surface S is equal to

=
-3

V bounded by

the

where cose, cosp, cosy are the direction cosines


normal to the surface S.

of

the outer

Multiple and Line Integrals

288

[Ch. 7

Sec. 12. Fundamentals of Field Theory

1. Scalar and vector


of the point

surfaces f

t/,

(x,

fields.

= /(/>) = /(*,
= C, where
y, z)

z),

C=

defined by the scalar function


is a point
of space. The
const, are called level surfaces of the scalar
scalar field

where

is

(x, y, z)

field.

A vector field is defined by the vector function of the point a a(P)~


a(r), where P is a point of space and r=xi-\-yj+zk is the radius vector
of the point P. In coordinate form, a
a v j-\-a z k, where a x ~a x (x, y, z),
ax i
a
and a z a z (x, //, z) are projections of the vector a on the
y
ay(x, y, z),
coordinate axes. The vector lines (force lines, flow lines) of a vector field are
found from the following system of differential equations

~~~'

dx__dy _dz

scalar or vector

'ry;
-

that

field

where

"

V = ^3-+y^- + ^y

is

not depend on the time


is called nonstationary.

does

t
time,
depends on the
t. The
Gradient.
Th vector

it

if

stationary;
~

it

Hamiltonian operator

the

or

(del,

called

is

nabla),

is

called the gradient of the field U


f (P) at the given point P (ci. Ch. VI, Sec. 6).
The gradient is in the direction of the normal n to the level surface at the
point P and in the direction of increasing function U, and has length equal
to

dn~~
If

the direction

is

\dx

given

by the unit vector

=
(the derivative of the function
3. Divergence and rotation.

+ ay j+a z k

the scalar

is

The rotation

in

cos

- cos p +

a+

the direction

The divergence

diva-^ +

(curl) of a vector field

{cos a, cos p, cos Y},

/).

of a vector field

is

the integral

an dS =

a (P)

~ a^i

\~

+ ^^Va.
a (P) = ax + ay j + az k

4. Flux of a vector. The flux of a vector field a(P)


in a direction defined by the unit vector of the normal
S

cosy

is

the

vector

da
y

da z

to the surface

then

an dS

\
\

(ax cos

a -|- a

cos p

through a surfaces
ujcosa, cos p, COSY}

+ az cos Y) dS.

If S is a closed surface bounding a volume V, and n is a unit vector of the


outer normal to the surface S, then the Ostrogradsky -Gauss formula holds,

which

289

Fundamentals of Field Theory

Sec. 12]
in vector

form

is
ff

(\

r\

r*

div

5. Circulation of a vector, the work


a along the curve C is defined by

vector

f a

dr =

a s ds

a dx dy

dz.

The

of a Held.

integral of the

line

the formula

a x dx

-f-

a dy -f a z dz
y

(0

and represents the work done by the field a along


projection of the vector a on the tangent to C).

curve

the

(a s is

the

If C is closed, then the line integral (1) is called the


circulation of the
vector field a around the contour C.
If the closed curve C bounds a two-sided surfaces, then Stokes' formula
holds, which in vector form has the form

adr=

f f

rotadS,

/i

where n is the vector of the normal to the surface S; the direction of the
vector should be chosen so that for an observer looking in the direction of n
the circulation of the contour C should be counterclockwise in a right-handed
coordinate system.

6.
tial

Potential

and solenoidal

fields.

The

vector

iield

a(r)

is

called poten-

if

U,

where Uf(r) is a scalar function (the potential of the field).


For the potentiality of a field a, given in a simply-connected domain,
it
be non rotational, that is, rota = 0. In
it is necessary and sufficient that
that case there exists a potential U defined by the equation

dU ~a x dx-}- a v dy -f- a 2 dz.


If

the potential

is a

single-valued function, then

a dr

(B)

U (A);

AB
in

particular, the circulation of the vector

a = 0;

If

is

equal to zero:

m adr=Q.

vector field a (r) is called solenoidal if at each point of the field div
in this case the flux of the vector through any closed surface is zero.
the field is at the same time potential and solenoidal, then div (grad U)=.Q

and the potential function

"Er++5-<>.

is

or

harmonic;

AU=0

'

that

where

is,

it

satisfies the

Laplace

A= *'=;+>+> isthe

Laplacian operator

2371. Determine the level surfaces of the scalar field

where

U = F(Q),
10-1900

\fx*+y*-\-z*.

where

What

will the level surfaces be of a field

290

_ _
Multiple and Line Integrals

[Ch. 7

2372. Determine the level surfaces ot the scalar field

U = arc sin

2373. Show that straight lines parallel to a vector c are the


vector lines of a vector field a(P)
c where c is a constant
vector.
2374. Find the vector lines of the field a
CD*// 4 CDJC/, where CD
is a constant.
2375. Derive the formulas:

a)

+ C K) = C
gTad(UV) = Ugrad
grad (t/ = 26/ grad
grad(C

i/

gradf/

+C

gradV, where C,

and

C2

are constants;
b)

c)
j\

d)

V grad(/ U
AfU\ =-*
grad

e)

grad

(^ J

<p ((/)

cp' (t7)

grad U.

2376. Find the magnitude and the direction of the gradient


3
x*
z
of the field U
if
3xyz at the point A (2, 1, 1). Deterthe
mine at what points
gradient of the field is perpendicular to
the z-axis and at what points it is equal to zero.
2377. Evaluate grad f/, if U is equal, respectively, to: a) r,

b)

r\

c)

+ +

d) /(r)(r

= /?+^qr?).

2378. Find the gradient of the scalar field U


cr, where c is
What will the level surfaces be of this field,
and what will their position be relative to the vector c?

a constant vector.

2379. Find the derivative of the function

given point P(x, y,


this point. 'In

z)

U = ^ + y^ + ~ala

in the direction of the radius vector r of


will this derivative be equal to the

what case

magnitude of the gradient?


2380. Find the derivative

of

the

function

U=

in the di-

rection of /{cosa, cosp, cosy}. In what case will this derivative


be equal to zero?
2381. Derive the formulas:
a)

div^aj + C^^Cjdivaj + Cjjdivajj, where C and C


l

constants;
b) div (i/c)
c)

= grad /, where c is
div((/a) = grad U-a+ (/diva.

2382. Evaluate

di

2383. Find div

where

for the

central

are

a constant vector;

vector

field

a(P)

= /(r)~

Sec.

Fundamentals of Field Theory

12]

291

2384. Derive the formulas:

+ C a = C rota + C rota where C and


rot(/c) = grad U-c, where c is a constant vector;
rot (Ua) = grad U a + U rot a.
rot(C a l

a)

2)

are

constants;
b)
c)

2385.

Evaluate the divergence and the rotation of the vector


respectively, equal to: a) r\ b) re and c) f(r)c, where c

if

is

a constant vector.

is,

2386. Find the divergence and rotation of the field of linear


velocities of the points of a solid rotating counterclockwise with
constant angular velocity o> about the z-axis.
2387.' Evaluate the rotation of a field of linear velocities
co r
of the points of a body rotating with constant angular

velocity <o about some axis passing through the coordinate origin.
2388. Evaluate the divergence and rotation of the gradient of
the scalar field U.
2389. Prove that div(rota)
0.
2390. Using the Ostrogradsky-Qauss theorem, prove that the
flux of the vector a
r through a closed surface bounding an
arbitrary volume v is equal to three times the volume.
2391. Find the flux of the vector r through the total surface
2
of the cylinder # 2
</? 2 0<e<//.
//
2392. Find the flux of the vector a
x*i
y*j+z*k through:

a) the lateral surface of the cone


0<e<//; b) the
total surface of the cone.
2393*. Evaluate the divergence and the flux of an attractive

j<f^,

force

F=

of a

mass w, located

point of

at the coordinate

through an arbitrary closed surface surrounding this point.


2394. Evaluate the line integral of a vector r around one
turn of the screw-line * = /?cos/; y = Rsmt\ z = lif from / =

origin,

to

2n.
2395. Using Stokes' theorem, evaluate the circulation of the
vector a
zk along the circumference x* + if
x*tfi
R 2 z=-0,
2
2
jt
taking the hemisphere z
J/"/?
if for the surface.
2396. Show that if a force F is central, that is, it is directed
/

+j+

towards a fixed point

and depends only on the distance r from


where
f(r) is a single-valued continuous
point:
function, then the field is a potential field. Find the potential U

F = f(r)r,

this

of the field.

2397. Find the potential


by a material point of mass
nates:

a=

equation
10*

~r. Show

of a gravitational field generated


located at the origin of coordi-

that the potential

satisfies the

Laplace

_ _
Multiple and Line Integrals

292

(Ch. 7

2398. Find out whether the given vector field has a potential
find
if the potential exists:

and

a)

b)
c)

a
a
a=

2399. Prove that the central space field


lenoidal only
is

U,

when

= ~,
f(r)

2400. Will the vector field


a constant vector)?

where k

is

a = r(cxr)

= /(r)rwill

be so-

constant.

be solenoidal (where c

VIII

Chapter

SERIES

Sec.

1.

Number Series

1. Fundamental concepts. A number series


00

a,+a t +...+a +...=


tt

is

called convergent

if

its

has a finite limit as n

>

(1)

tt

sum

The quantity

oo.

S=

lim S n
n -+ oo

is

then called the sum

number

of the series, while the

is

partial

2a
n-\

called the remainder of the series.

If

Sn

the limit lim


n -*

does not exist (or

is

QO

infinite), the series is then called divergent.


If a series converges, then lim a n
Q (necessary condition for convergence).

n-*oo

The converse

not true.
For convergence of the series (1) it is necessary and sufficient that
such that for n
any positive number e it be possible to choose an
and for any positive p the following inequality is fulfilled:
is

(Cauchifs test).
The convergence or divergence
subtract a finite

number

of

2.

Tests of convergence

a)

Comparison

test

*!

I.

of

series

is

not

violated

if

for

>N

we add

or

terms.

its

and divergence

<a,,<6 n

If

+ *,+ ..

of positive series.
after a certain n
n

+*,!+. ..^

and the

series

(2)

converges, then the series (1) also converges. If the series ( J) diverges, then
(2) diverges as well.
It is convenient, for purposes
of comparing series, to take a geometric
progression:
00

2 aq
n=o

n
(a

0),

294

[Ch. 8

Series

which converges

which

is

|^|<land

for

\q\^\, and

diverges for

the harmonic series

divergent series.

Example

The

1.

series

++

converges, since here


a

J_

"~n-2 n

2*'

while the geometric progression


1

n=i

whose ratio

Example

is

<7

The

2.

converges.

series

diverges, since
of

b)

its

In

ln_3

ln_2

general term

is

/i

greater than

the

term

corresponding

the harmonic series (which diverges).

Comparison

test

II.

there

If

exists a finite

and

nonzero

limit

lim ?
y. b
n

n if

a n -^b n ), then the

(in

particular,

the

same time.
Example 3. The

series (1)

and

series

diverges, since

1"

whereas a series with general

Example

4.

The

term

n J

diverges.

series

_J_

_J

converges, since

while a series with general

term

converges.

(2)

converge or diverge

at

Sec. 1]

c)

__
Number

D'Alembert's

Let

test.

>

an

295

Series

certain

(after

n)

and

there be

let

a limit

-l =

lim
n

Then the
it

not

is

<

series (1) converges

if q
the series

known whether

Example

5.

an

-> GO

q.

and

1,

diverges

if

>

If

1.

<7

1,

then

convergent or not.

is

Test the convergence of the series

1+1+1+
2
2
'

22

Solution. Here,

and

?-H =

lim

Hence, the given series converges.


d) Cauchy's test. Let of^^O (after a certain n) and
lim
n

Then

(1)

converges

if

q<\,

let

there

-> OD

and diverges

if

q>\. When q=l

remains open.
e) Cauchy's integral test. If a n
f(n), where the function f (x)
rnonotomcally decreasing and continuous for jc^a^l, the series

of the convergence of the series

be a limif

n/~ =

the question
is

(1)

positive,
and the

integral
00

"

/ (x)

dx

converge or diverge at the same time.

By means

converges
of

series

if

of the integral test

p>

may

1,

be

and diverges
tested

by

it

may

be proved that the

The convergence of a large number


if p<^\.
comparing with the corresponding Dirichlet

series (3)

Example

6.

Test the following series for convergence

--^"

-U--L + -L+
Solution.

We

*
1

r "'
i

have
1
~

Dirichlet series

__1_1

J^
4/i

'

__

296

[Ch. 8

Series

Since the Dirichlet series converges for p


2, it follows that on the basis of
test II we can say that the given series likewise converges.
3. Tests for convergence of alternating series. If a series

comparison

+ |fll+...,

l+.-.
composed

values of the

of the absolute

(4)

terms of the series

converges,
But if (1) con-

(1),

then (1) also converges and is called absolutely convergent.


verges and (4) diverges, then the series (1) is called conditionally (not absolutely) convergent.
For investigating the absolute convergence of the series (1), we can make
use [for the series (4)] of the familiar convergence tests of positive series.
For instance, (1) converges absolutely if

<

lim

-> oo

In the general case, the divergence of

gence of

But

(4).

n
(4)

?2_J

lim

if

GO

/KI<

lim

or

0-n

>

(1)

or

not

does

1.

from

follow

lim
/\a n
n -> oo K

\>

1,

the

diver-

then not only does

diverge but the series (1) does also.


Leibniz test If for the alternating series

*!-*
conditions

the following

are

(*^0)

+*3- **+
fulfilled:

1)

(5)

^ b ^b

bl

^.

(5)

bn

lim

2)

then

oc

converges.

In this case, for the

remainder

of

Rn

the series

the evaluation

holds.

Example

7.

Test for convergence the series

Solution. Let us

form a series of

the

absolute

values

o!

the

terms of

this series:

Since

lim

lim
I

oo

the series converges absolutely.

Example

8.

The

series

converges, since the conditions of the Leibniz


converges conditionally, since the series
14-

diverges (harmonic series).

-*

4-

4-4

test are fulfilled.

This series

__

Sec

Number

/\

Series

297

Note. For the convergence of an alternating series it is not sufficient that


general term should tend to zero. The Leibniz test only states that an
alternating series converges if the absolute value of its general term tends
to zero monotonically. Thus, for example, the series
its

diverges despite the fact that its general term tends to zero (here, of course,
the monotonic variation of the absolute value of the general term has been

S 2k = S'k + S"k

violated). Indeed, here,

and

S k = cc(S k

lim
k -

Sk

lim

limit
fe

exists

is

a partial

and

sum

finite (S k

is

where

of the
a

is

harmonic

partial

sum

series),

whereas the

o f the convergent geo-

-*

metric progression), hence,

S 2fe =oo.

lim
k

-> 00

On

the other hand, the Leibniz test is not necessary for the convergence
series: an alternating series may converge if the absolute
value of its general term tends to zero in nonmonotonic fashion
Thus, the series

of

an alternating

.22

~3 a

42

~~'""(2n

1)'

converges (and it converges absolutely), although the Leibniz test is not fulfilled: though the absolute value of the general term of the series tends to
zero, it does not do so monotonically.
4. Series with complex terms A series with the general term c n a n

00
z

-]-ib n (i

1)

converges

and only

if,

if,

the

series

with

real

terms

an

n=i
00

and

2&,, conver g e
n

a*

same time;

the

SC

n=i

=S=
n

The series (6) definitely converges


series

whose terms are the moduli


5. Operations on series.
a)

that

is,

convergent series

and

of the

may

8
i

be

+'2Xn =

(6)

is

called absolutely convergent,

terms of the series


multiplied

+a

(6),

+...+fl n + ...=S

if

the

converges.

termwise by

if

a,

then

in this case

any

number

fc;

__

298

Series

b)

sum

the

By

two convergent

(difference) of
fli

[Ch.

series

+ flt+-..+fl n +...=5
+ *,+ .. .+*+...=$,

(7)

If

(8)

*!

we mean

a series

+ (a

bl)

(a,

c)

The product

of

b z)

If

verges
d)

terms
verges

cn

the series

= a,bn + a A-I +

+ (a n

(7)

and

bn )

=S,

S2

the series

is

(8)

+ c>+...+cn +...
+0M = 2

cl

where

*>

(9)

the series (7) and (8) converge absolutely, then the series (9) also conabsolutely and has a sum equal to S S 2
If a series converges absolutely, its sum remains unchanged when the
of the series are rearranged. This property is absent if the series conconditionally.
.

Write the simplest formula of the


the indicated terms:

term

/ith

of the series

using

2401.

1+1+J + I+...

2404.

i+4 + j+^+...

2402.

1+1+1+1+"-

2405.

2403.

1+1 + 4 + 4+..
+ +
+ ~4

2406.

4+*.+ ^+...
f+{ + + +

2407.

2409.

2410.

1+

H-

- 14 ~
1

+...

-I-

In Problems 2411-2415 it is required to write the first 4 or


5 terms of the series on the basis of the known general term a n
.

2411. a n

2412.

2413

|q^.

2414. a n

2415
\*
din*. a

t^.
flrt

Test the following series for convergence by applying the comparison tests (or the necessary condition):

2416.

+1

+ ...+(

1)-'

...

Number

2418.

+ +

0419

J___1_
444^
^
4 ^ 6

2422.

++

2421.

'

'

4^

-I-

'

2i

'

'

'

'

...

-= + -=-f -=
+
^3-4

... 4'

1/2-3

02

2423. 2

299

Series

]fn

(n

+ \)

O"

OJ

+ + y+...
i-

+=;-+..

-'

4 J/3

3 V'2

Using d'Alembert's

(n

test

lest,

the

gence:
2427.

98
-*'

-l=
2
1
!

1-5

''-}
^ 1-5-9
-

"

following

2 "~'

+ 4 4-^- +...-'

1-1)

'

'

'

1-5- 9. ..(4/i-3)

Test for convergence, using Cauchy's test:

Test for convergence the positive series:

2431.

1+1 + 1+...+1+...
1 + 1 + 1+ ..+
_
1

2432.

((H

2433

r4+4T7 + 7no+

)2

r/lo/

^TtjO

+...

+(3n-2M3n+l

24.i+f -^+... +sSFI+


2435.

...

^ + 1- + ^+...+^+...
^

.'^

J_
i

_X.

9
^

j_

__________
l

bones

for

conver-

300

Series

2439.

-+ +

+...

2440.

1+

+...

[Ch.

++...

2441>
2442.

1++.+
1,
4

1-3.5

-3,

1-jT

...

4^M2~*"

1.3.5.. .(2/1-1)
'

""

4-8-12...4n

2445.
""

1000. 1002. 1004... (998


1-4-7. ..(3n-2)

2.5-8...(6 n
!. 5.9...

2447

-7)(6g-4)

+2n)

"

(8_H)(8n -7)

...

I,

'- 4

1 '

4'9

^--^ 1.3.5.7.9
....^ '""

2449

2450.

2451.

1-4-9.

..

""*

'

1-3-5-7- 9... (4n

arcsin.

2455.

sin.

2456.

3)

!,

i=J

2452
152.

Ulnfl+l).
^
\

n J

2457.

,.
~

^-

ri'lnn-ln In n

00

2453.

yin^i.

2458.
/J=2

2454

E -'

2459

STTOTJ

Number

Sec. 1]

2464.

Series

cos-2-)
^

2469. Prove that the series


1)

converges for arbitrary q

p>l,
p<l,

if

and

for

?>1,

if

p=l;
p=l.

and for </<!, if


2) diverges for arbitrary q, if
Test for convergence the following alternating series. For convergent series, test for absolute and conditional convergence.
i

(_n-i

2470.

l-

2471.

2472.

l_ + _... +

2473.

l_

2474

+ -....ui--f...

-- - + /2
K3

..

+ 3_... + (^S +

"

'

...

^- -. +(-!)-

2475.

__

2476.

-- -- -- ---^--h...+

+ _. ..+(_!)
^

7=4

2|^2

.^+...

7=4

3^3

4^41

n+\

{)
(n

947Q
"'

l_k!-J-lL_
7

7.9~t"7.9.11

-i

"TV

ii-'
U

\)

1-4. 7... (3/1-2)


7.9-11. ..(2/1
5)'

302

[Ch. 8

Series

na

sin

2481.

2482.

(_l)"l^,

ly-'tan

n^i

Vn

2483. Convince yourself that the d'Alembert test for converdecide


ide the quesl
question of the convergence of the

gence does not


series

2a

where

>

whereas by means of the Cauchy

test it is possible to establish


that this series converges.
2484*. Convince yourself that the Leibniz test cannot be
applied to the alternating series a) to d). Find out which of
these series diverge, which converge conditionally and which con-

verge absolutely:
a)'

___

_J

_1_

'

^\
d)

i_

+y
*~

+ 21

.j;
r

3F

"

"L

32

h 5

3?+

__u
"

'

'

'

3*

_
_
T _l + T T + TT T +...
!

Test the following series with complex terms for convergence:


CO

00

^2+#.

2485.

2488.

00

2486,

00

(2 '

X" 7

1=1

V.

1)
''.

2489.
=i

__

Sec.

Number

1]

Series

{/

= X-?

r(2-Q + 1"
l

'

[n(3-20-3ij

fa
2493. Between the curves

and
y=-^
X

and

303

the right

to

intersection are constructed segments parallel


an equal distance from each other. Will the sum
the lengths of these segments be finite?
2494. Will the sum of the lengths of the segments mentioned

of their point of
to the t/-axis at

of

Problem 2493 be

in

curve y

=
X

curve \)^-\
x

finite if the

is

by the

replaced

?
00

Form

2495.

Does

sum

the

^-^ and

the series

of

sum converge?

this

00

2496.

Form

the

anc*

* es *

and

difference

2497.

]T

Does

the

formed

series

by

the

subtracting

series

00

TT

divergent

series

QC

the

for convergence.

'*

rt

of

sef i es

-f

21
n

converge?

sum converges while

2498. Choose two series such that their


their difference diverges.

CC

2499.

Does

Form

the product

of the

QC

V
~nVn

series

and V.OTTM2

tt

this product converge?

2500.

Form

the series
(l

+1 + 1+

..

+ J~-f-

V.

Does

this series converge?

2501. Given the series

1+1 1+

...+

(
-

^+...

Estimate

the error committed when replacing the sum of this series with
the sum of the first four terms, the sum of the first five terms.
What can you say about the signs of these errors?
2502*. Estimate the error due to replacing the sum of the
series

y+
by the sum

2!

(2")

of its first

+ ~3\('2J + " + 5H2")


n terms.

+"

_ __

304

[Ch. 8

Series

2503. Estimate

the

due to replacing the sum

error

the

of

series

by the

sum

JL -L-L-L-l.

4-

^2M3M

of its first

4-

"+/i!^"'

n terms. In particular, estimate the accu-

racy of such an approximation for n=10.


2504**. Estimate the error due to replacing

the

sum

of the

series
1

sum

+ 22 + 32+

+^i+

'

n terms. In particular, estimate the accuan approximation for n= 1,000.


2505**. Estimate the error due to replacing the sum of the

by the

of its first

racy of such
series
1

by the

sum

+ 2(i)'43()V.
of

its

first

n terms.
00

Zl

_ I)""

does one have

n=i

take to compute
decimals?

to

2507.

sum

its

How many

two decimal

to

terms of

the series
(2/1

have to take to compute

its

sum

to

to

places?

-MIS*

three

does one

two decimal places?

to three?

to four?

2508*. Find

the

2509. Find the

sum

sum

the

of

series

-L +

gL + jL

..

of the series

Sec. 2. Functional Series

1. Region of convergence. The

set of values of the

argument x

for

which

the functional series


/.<*)

converges

is

+ /.(*)+.. .+M*)+...

called the region of convergence of this series.

S(*)

=
n

where S n

(x)

vergence,

is

of the series.

fl

(x)

+f

called the

lim

Sn

(1)

The function

(x),

-* QO

(x)+ ...+f n (x), and x belongs to


sum of the series; R n (x) = S(x)S n

the
(x) is

region of conthe remainder

Sec. 2]

In

_ _

305

Functional Series

the

cases,

simplest

it

(1), to

convergence of a series
holding x constant.

is
sufficient, when determining the region of
apply to this series certain convergence tests,

Diverges

,%f?

-3-101

Determine the region

1.

104

Fit*.

Example

Diverges

of

of the series

convergence

x+\

" (x+\y ~ (X+\Y


"
~ (*+iy
TT" 2-2* "~T^r t""'~ n- 2" +
1

'

we

Solution. Denoting by u n the general term of the series,

lim

M -*|

\U n

Using d'Alembert's

we

get the

we have

'<!

that

l,

is,

harmonic
the

*'* *
lim I'+H
Jx+l\
2
n^*2 n + (n+\) |jc|

series

that

series

+ -~

(c n

and a are

gence)

a
|

+ c, (A-a) + c

real

<

1< x <

or

To +...,

converges (conditionally).
Thus, the scries converges when
2. Power series. For any power
c

3<x<l;

if

is,

+ TT+-Q-+..
o
2

2,

test)

have

'

assert that the series converges (and converges

oo<^<

if

will

we can

test,

if

absolutely),

11

(*

-t

oo

the

series diverges,
104).

(Fig.

When x=l

which diverges, and when

which

(in

if

x=

with the Leibniz

accord

3^*<1.
series

fl)'+

+c n (x

B
fl)

(3)

numbers) there exists an interval (the interval of convercentre at the point x


a, with in which the series (3)

R with

>

a
R the series diverges. In special cases, the
converges absolutely; for \x
and oo. At the end-points of
radius of convergence R may also be equal to
a
the interval of convergence x
R, the power series may either converge
or diverge. The interval of convergence is ordinarily determined with the
help of the d'Alembert or Cauchy tests, by applying them to a series, the
terms of which are the absolute values of the terms of the given series (3).
Applying to the series of absolute values
\

the convergence tests of d'Alembert and Cauchy, we get, respectively, for the
radius of convergence of the power series (3), the formulas

=
lim
n -

rt*

and

# = Hm
-

"/|c,,|

too

'

However, one must be very careful in using them because the limits on the
right frequently do not exist. For example, if an infinitude of coefficients cn

306

__
Series

\Ch. 8

vanishes [as a particular instance, this occurs if the series contains terms
with only even or only odd powers of (x a)], one cannot use these formulas.
It is then advisable, when determining the interval of convergence, to apply
the d'Alembert or Cauchy tests directly, as was done when we investigated
the series (2), without resorting to general formulas for the radius of convergence.
z

If

= x + ty

is

complex variable, then

z-Zo)
(cn = a n + ib n

for the

series

power
n

+...+C n (Z-Z Q +...

(4)

there exists a certain circle (circle of convergence)


z
z
inside which the series converges
z |>fl the series diverges. At points lying on the cirabsolutely; for Iz
cumference of tne circle of convergence, the series (4) may both converge and
diverge. It is customary to determine the circle of convergence by means of
the d'Alembert or Cauchy tests applied to the scries
-z

|z

<R

Jt

-f/f/

with centre at the point

whose terms are absolute values of the terms of the given series. Thus, for
example, by means of the d'Alembert test it is easy to see that the circle of
convergence of the series
""

""

2-2 2

1-2

~~"'~~

3.2 s

~""'

n .2*

is determined by the inequality |z-f 1 |<2 [it is sufficient to repeat the calculations carried out on page 305 which served to determine the interval of
convergence of the series (2), only here x is replaced by z]. The centre of
the circle of convergence lies at the point z
1, while the radius R of this
circle (the radius of convergence) is equal to 2.
3. Uniform convergence. The functional series (1) converges uniformly on
some interval if, no matter what e
0, it is possible to find an N such that
does not depend on x and that when
for all x of the given interval
we have the inequality R n (x)
where R n (x) is the remainder of the
e,

>

given series.
If
\fn( x )\*f* c n

cn

converges,

(rt=l,

then

the

2,

<

...)

n>N

a^x^b

when

functional

series

(I)

and

number

the

converges

series

on the interval

n-\

absolutely and uniformly (Weierstrass' test).


series (3) converges absolutely and uniformly on any interval
lying within its interval of convergence. The power series (3) may be termwise differentiated and integrated within its interval of convergence (for
fl| <RY, that is, if
|x
[a,

The power

*-) +

+c n (xa)+

then

for

-a)+. ..+ncn (xa)*- +.


cQ

dx+

any x of the interval of convergence of the series

d (xa) dx+

c2

(xa) dx+

cn

=/(*),
(3),

(5)

we have

=f (x),

(xa) n dx+...=

(6)

Sec. 2]

Functional Series

307

number # also belongs to the interval of convergence of. the series (3)J.
Here, the series (6) and (7) have the same interval of convergence as the

[the

series

(3).

Find the region

of

convergence

of the series:

2510.

(-D-'.

2511.
n-

n-

+I

2512.

-jbn

l)"

M=

25.9.

252

2-(jzrk-

n-i

00

O(T

3D

M
x
l^A-

v~^ sin (9n

^'oMI^fl

v^

2/1-4^'*'

>

O^vQI

n=i

ri

1
*

=o
nr

2514.

25.5**.

"

2 sin

X^i.
/l

2523.

2516.

2517.

2522

-J-

(-l)"

fI

V^

/I=l

*- nMn *.

2524*.

2525.

2-p-

Find the interval of convergence of the power series and test


the convergence at the end-points of the interval of convergence:
2531.

2532.
30

2528.

yin

X ^rr.
n-1

2533.

2529.

n=i

2530.

2534.

'

-_^".

2535.

308

Series

[Ch.

CO

OCQA
25<JO.

V>,
^

00

V""

s
;-=
\2rt-f-ly

r* .

0**1
>
2551.
^-

7^
n

2n4

n=i

CO

+ 5)* n ~

^-rr
4

CO

2537.
CO

2553.

(-!)
n=i

2 "'
V
X
(3

,n
2539.

^-'
2554.

2541.

^1

a=i

2556.

2542**.

2^

nl

xnl

n=i

2557.

+l

21
/I

l)"

>

(AT-2)"
/j"

2544*.

2V=1

2558.

2545.

l(

2(-l)"" -^|r.
n

2559*.

n=o
X(A;

2549.

2 ^S^

2562.

Y.

(3ft

n=

2550.

V n"

(*

3)".

2563.

2)".

Y(

- 2)

>"
.^-.
!
(n+l) 2"+
l

1)"

309

Functional Series

Sec. 2]

Determine the circle


2564.

of

convergence:

V W.

V <=gC

2566.

Aarf

^arf

fl.

n=i

n=o

00

2565.
2568.

2567

(1-M0 2 ".

'

'

+ 2i) + (1 + 20 (3 -h 20 2 +

(1

+ (l+
2569.

+ r=-, + (l-lHI-20

+ (1-00-20. ..(l-/ii)
9*70
257 2571. Proceeding from the definition of uniform
prove that the series

convergence,

..+*"+...

does not converge uniformly in the interval (


1, 1), but converges uniformly on any subinterval within this interval.

sum

Solution. Using the formula for the


get, for

|jc|<

of a

geometric

progression,

we

1,

Within the interval (


1, 1) let us take a submterval [
1+ct, 1
a], where
a is an arbitrarily small positive number. In this subinterval |jc|s^l a,
|1

xl^a

and, consequently,
I

*,.(*)

<

_ at"*

/I
(

1
.

;>

To prove the uniform convergence of the given series over the subintervai
it is possible to choose
a, 1
a], it must be shown that for any e >
an N dependent only on e such that for any n > N we will have the ine-

[1+

quality

Rn

(x)

<e

Taking any e>0,

is,

Thus>

puttin g

a)

n>N

\R n (x)\

is

indeed

<

+ >
1

[since ln(l

ea

ln

ln(ea)_

when

let

a)<ln(ea), that

(n+l)ln(l
In (1

x of the subinterval under consideration.


(1
a)"* 1
w+l <ea,
us require that e; whence (i
a)

for all

Ns=s

ln(l
less than

\
a)
e

1,

we

are

a)<0] and

convinced that

x of the subinterval
the given series on any sub-

for

all

1+a, 1 aj and the uniform convergence of


interval within the interval (
1, 1) is thus proved.
As for the entire interval (
1, 1), it contains points that are arbitrarily

(-

close to Jt=i,

and since

Hm R n (x) = \im
X-+1

X-+1

-- =00,

J/I+
.

no mattei

how

large n

is,

310

_ __
Series

[C/i.

points x will be found for which R n (x) is greater than any arbitrarily large
N we
number Hence, it is impossible to choose an N such that for n
e at all points of the interval (
would have the inequality R n (x)
1, 1),
and this means that the convergence of the series in the interval (
1, 1)
is not uniform.

>

<

2572. Using the definition of uniform convergence, prove that:


a) the series

converges uniformly in any

finite

interval;

b) the series
x*

converges

(-1.

throughout

uniformly

the

of

interval

convergence

i);

c) the series

1+F + F+- -+;?+...


in

converges uniformly
positive number;

the

interval

(1-1-8,

where 8

co)

is

any

d) the series
2

+ (x*- *) +

jc

(x

8
jc ) -f-

(jc

x 2n+2 )

+ (x "
2

-h

1,
converges not only within the interval (
1), but at the extremities of this interval, however the convergence of the series in

(1,

1) is nonuniform.
Prove the uniform convergence

of the functional series in the

indicated intervals:
00

2573

on the inlerval

I"

'

!]

on

2574.

over *he en tire number scale.

2^ ~~2~
00
I

2575.

Applying

sums

)"~

T=

termwise

on the interval

differentiation

[0,

and

integration,

of the series:
Y2

V3

y.3

y5

\n

+ ^ + ^+. ..+?-+...
x
x -*- + --...+(-l)'-*L + .,
, + + +... + _ + ...

2576. x

2577.

2578

yLfl

1].

find

the

__
Taylor's Series

Sec. 3]

v8

2579.

+ -' .- +(-!)"+2x + 3x* + ... 4

l-3*

-[5x

2582. 1.242-3A;

+---

...
2

...+(/+

3-4A: 4-

Find the sums of the


2583

r 2 "" 1

*-

2580.
2581.

311

!)*""'

series:

+++ ++

^ + l+!+...+^L i+...

2586.

/r

Sec. 3. Taylor's Series

1. Expanding a function in a power series. If a function f (x) can be


of the point a, in a series of
expanded, in some neighbourhood
a, then this series (called Taylor's series) is of the form
powers of A:

\xa\<R

(I)

When
holds

the Taylor scries is also called a Maclaurin's series. Equation (I)


the remainder term (or simply remainder) of the

\xa\<R

when

if

Taylor series

as n

oo.

To evaluate
fin (x)

the remainder, one can

make

<*-

f(n

l)
l

fl

(Lagrant*e's form).

Example

f
if

We

X ) =,

smh

even, and

is

x,

(H)

0<0<1

where

(2)

find

f" (x)

)l

the function f (x)


cosh x in a series of powers of x.
the derivatives of the given function f (x)
cosh x,
sinh x, ...; generally, f (n) (x)
cosh x, f" (.v)
cosh x,
sinn A, if n is odd. Putting a
0, we get /(0)
(A-)
1,

Expand

1.

Solution.

fl

use of the formula

=
=

/'(0)=0, T(0)-l, /'"(0) = 0, ...; generally, / >(0)


")=-0 if n is odd. Whence, from (1), we have:
(II

1,

if

is

even, and

(3)

To determine
d'Alembert

the

test.

interval

We

of

convergence

of

the

series

have

lini

n->

oo

(2/1

(3)

we apply the

312

[Ch. 8

Series

for any*. Hence, the series converges


remainder term, in accord with formula

.cosh 9*,

R n (x)=

>9>

it

sinh9*,

=^e'*',

X I"-*'**
and therefore \R n (x)\^

if

is

odd, and

if

is

even.

e1

series

with the general term

in

with

accord

the

necessary

Y\ n

^
\

made immediately evident with

any x

(this is
therefore,

|sinh9*|

test);

QO<JC<OO. The

follows that

converges for
d'Alembert's
convergence,

interval

has the form:

n 1
A +

R n (x)=
Since

the

in
(2),

the help of

condition

for

n+l

lim

and consequently lim #(*) = ()

for

any

x.

This signifies that the

sum

of the

/2->00

series (3) for

any x

is

indeed equal to cosh*.

2. Techniques employed for expanding in power


Making use of the principal expansions
I.

= !++*!+.

series.

(_oo<*<oo),

..+fj+...

II.

III.

IV.

and also the formula for the sum of a geometric progression, it is possible,
in many cases, simply to obtain the expansion of a given function in a power series, without having to investigate the remainder term. It is sometimes
advisable to make use of termwise differentiation or integration when expanding a function in a series. When expanding rational functions in power
series it is advisable to decompose these functions into partial fractions.

On

x=

the boundaries of the interval of convergence (i.e., when


1
it converges absoas follows: for
m
1
it
and
lutely on both boundaries; for
diverges when
for
1
it
l;
diverges on both bounconditionally converges when x
*)

and x=l) the expansion IV behaves

daries.

>

>

m^

m^O

x--\

313

Taylor's Series

Sec. 3]

Example

Expand

2.

in

powers

of x *) the function

Solution. Decomposing the function

into

fractions,

partial

we

will have

Since

(4)

and
2

V^

^^ ... _

^^

\\j
1

n<l n x. n
fi

A>

(5)

n =o
it

follows that

we

finally get

(6)
/i

n=Q

=o

The geometric progressions


and

of

|*|<y;

hence,

3. Taylor's series
two variables /(x,

point

(a,

b)

(4)

n
n-Qv

and

formula

(5)

converge,

(6)

holds

when

respectively,
for

|x|<-j

|x|

i.e.,

= = 0,
is

when

for a function of two variables. Expanding a function


into a Taylor's series in the neighbourhood of a
//)

has the form

...

fc
If a
notation

<

the

Taylor

series

is

then called

Maclaunn's

series.

(7)

Here the

as follows:
,

y)

(X

x=a
!l=t>
~

d*f(x,y)

a)

dx*

+2

*)

dy*

~W

and so

Here and henceforward we mean "in positive integral powers".

forth.

314

(Ch. 8

Series

The expansion

occurs

(7)

if

the remainder term of the series

fe=l

as n

* oo.

The remainder term may be represented

in the

form

n+i

where

0<0<

1.

Expand the indicated functions in positive integral powers


of *, find the intervals of convergence of the resulting series and
investigate the behaviour of their remainders:
2587. a

2589. cos(*
2
2590. sin *.

(a>0).

9*a sin
cin^r-i-M
* -r--r
Jooo.

,-,>*

+ a).

/^

of the principal expansions I-V and a geometric


the expansion, in powers of *, of the following
write
progression,
functions, and indicate the intervals of convergence of the scries:

Making use

2592.

2593.

X2

*7
~4X

2594. xe~*

2598. cos

7^rJT2
5

*.

2599. sin 3*

,.

+ * cos 3*.

2600.

2595. e \

2601.

2596. sinh*.

2602. In

*
.

2597. cos2*.

2603. In

(1

,v

-f*

2*

).

the

Applying
expand
following functions in
powers of *, and indicate the intervals in which these expansions
differentiation,

occur:

2604.

(l+*)ln(l+*).

2606. arc sin*.

_
+*

2607. In (x
V\
).
various
the
functions
in
techniques, expand
given
Applying
powers of * and indicate the intervals in which these expansions
occur:

2605. arc tan*

2608. sin *cos *.


2609.
2610.
2611.

+O~

(1

2613

'

2614

cosh8

*'

Sec

315

Taylor's Series

3}

2615. ln(x

+ 3x + 2).

in/i_L.^/*v
+ *)**
rMl

2618.
J.

2616.

f'Jl

dx.

2619.
2617.

^L_

"-* z ^"

*4

Write the first three nonzero terms of the expansion of the


following functions in powers of x:
2620. tan*.
2623. sec*.
2621. tanh*.
2624. In cos*.
2622. e^ x
2625. <?*sin*.
2626*. Show that for computing the length ot an ellipse it is
possible to make use of the approximate formula
.

where

is

2627.

the

eccentricity

and 2a

is

the

major axis

of

the

ellipse.

tenary
J line

heavy string hangs, under its own weight, in a ca= acosh


and H is the horizontal
where a =
//
J
,

tension of the string, while q


that for small *, to the order
string hangs in a

2628.

powers

-f

2*

5*

in

series

of

of *-| 4.

2629.

powers

function

the

y^a

parabola

Expand

the weight of unit length. Show


4
of *
it
may be taken that the

is

of

4*

f(*)-5*

3* +2.

in a series of

Expand f(x+h)

h
in a

powers of *
series of powers of *

in a

series of

2630.

Expand In*

2631.

Expand

2632.

Expand

Xz

in a series of

powers

of

1.

1.

*+l.

2633.

2634.
2635.

Expand

X2

~Y~

oX

in a series of
~\~ JL

Expand

Expand

e* in a

4jc

2636. Expand ]/* in


2637.

Expand cos*

2638.

2
Expand cos *

2639*.

Expand In*

of

powers

*-f4.

powers of x
series of powers of * + 2.
a series of powers of *
4.
7

in a series of

in a series of

in a series of

powers

of

powers

of

*
j

in a series of

powers

_^

of

^^.

T-J

|-2.

316

[Ch. 8

Series

2640.

-*

Expand

in a series of

of

powers

2641.

What

is

the

the

of

magnitude

error

we put appro-

if

ximately

^2 + 1 + 1 + 1?
To what degree of accuracy
if we make use of the series

2642.
ber

we

will

by taking the sum

of

its

five

first

2643*. Calculate the number


ing the function arc sin A: in a
ple 2606).
2644. How many terms do

-*
O

x=l?

to three decimals

we have

num-

..,

terms when

series of

cosAr=l

calculate the

powers

of

by expand(see

Exam-

to take of the series

|j+...,

in order to calculate cos 18

2645.

How many

to three decimal places?


terms do we have to take of the series

+...,

to calculate sin 15 to four decimal places?


2646. How many terms of the series

have to be taken to
2647.

How many

find the

number

to four

decimal places?

terms of the series


In

do we have

to take to calculate

cimals?

In 2

to

two decimals? to 3 de-

__

2648. Calculate \/7 to two decimals by expanding the funcx in a series of powers of x.
tion l/S
2649. Find out the origin of the approximate formula

\/ (f-{-x&a

a = 5,

+ ~-

(a>0), evaluate

it

by means

and estimate the_ error.

2650. Calculate J/I9 to three decimals.

of

Y^3

putting

317

Taylor's Series

Sec. 3]

2651. For what values of x does the approximate formula


2
JC

yield an error not exceeding 0.01? 0.001? 0.0001?


2652. For what values of x does the approximate formula
sin

X&X

yield an error that does not exceed 0.01? 0.001?


'/*

2653. Evaluate

to four decimals.

y^-dx
1

^e~* dx

2654. Evaluate

to four decimals.

2655. Evaluate

l/^ccosxdx to three decimals.

2656. Evaluate

dx to three decimals.

2657. Evaluate

j^l+^'dx

decimals.

to four

1/9

2658. Evaluate

^yxe*dx

to three decimals.

2659. Expand the function cos(x


y) in a series of powers
x and y, find the region of convergence of the resulting series
and investigate the remainder.
Wiite the expansions, in powers of x and y, of the following
functions and indicate the regions of convergence of the series:

of

2663*.

2660. sin x> sin y.


2
2661. sin(* +y*).

664 *

\n(lxy+xy).
arctan

j/_

2662*.

\=*!!.
\+xy

2665. f(x, y)

wers of

ft

2666.
function

*:=

4-

ft,

2667.
2668.

~*y

and
/(*,

= ax*-\-2bxy + cy*.

Expand f(x+h,y-\k)in po-

k.

y)

= x*

2y*

when passing from

+ 3xy.

Find

the values

the

#=1,

increment
f/

=2

to the

of

this

values

y=2 + k.
Expand
Expand

2.
the function e x+y in powers of x -2 and y
the function sin(x+y) in powers of x and

__
Series

318

[C/i.

Write the first three or four terms of a power-series expansion


x and y of the functions:
x
2669. e cosy.

in

2670.

*'-

(H-*)

Sec. 4. Fourier Series

let

1. Dirichlet's theorem.
conditions in an interval
1)

is

We

uniformly bounded;

is

say that a function

satisfies the
the function

f (x)

b)

if,

in this interval,

that

is

\f(x)\^M when

(a,

<x<

b,

Dirich-

where

constant;
2)
of

all

has no more than

them

the function

are

a
the

of

number

finite

kind

first

has a finite limit on

f (x)

of

points

of

[i.e.,

at

each

the

0)=

left f (g

discontinuity and
g
discontinuity
Urn f (I
e) and a
~"

finite limit

on the right /(-{-0)= lim /(

+ e)

(e>0)J;

e ->o

has no more than a finite number of points of strict extrenium.


Dirichlet's theorem asserts that a function /(*), which in the interval
of this interval at
ji, Ji) satisfies the Dirichlet conditions at any point x
(
which /(x) is continuous, may be expanded in a trigonometric Fourier series:
3)

f(x)=?+ a, cos x + b

sin

+a

cos 2x-\-b 2 sin

2*+

+a n cos nx +

+ b n sinnx+...,
where the Fourier

a n and b n are calculated from the formulas

coefficients

ji

=
JT

J\

(1)

ji

f(x)cosnxdx(n = Q,

1,

...);&=

2,

JI

-n

-jt

If x is
a point' of discontinuity, belonging to the interval (
of a
jt, n),
function f (,v), then the sum of the Fourier series S (x) is equal to the arithmetical mean of the left and right limits of the function:

SM = ~
At the end-points

of the interval

2. Incomplete Fourier
s=/(jc)], then in

formula

series.

If

n and X = K,

a function

/ (*)

is

even

(1)

6 rt

-0

(w

2,

...)

and
ji

/"=^

= 0,

1,2,

...).

[i.

e.,

/(-

x)

a function

If

319

Fourier Series

Sec. 4]

odd

is

/ (x)

/(

[i.e.,

and

x)

an

then

/ (*)],

=Q

t/i

= 0,

2 ...)

i,

IT

bn

=~

nx dx

/ (x) sin

J\

JT

(n

2,

. .

.).

function specified in an interval (0, n) may, at our discretion, be contior an odd function; hence,
in the interval (
Jt, 0) either as an even
it may be
expanded in the interval (0, Ji) in an incomplete Fourier series
of sines or of cosines of multiple arcs.
3. Fourier series of a period 21. If a function f (x) satisfies the Dirichlet
the discontinuities
conditions in some interval (
/,
/) of length 2/, then at
of the function belonging to this interval the following expansion holds:

nued

r,

=a +
y

ju

/(*)

fli

cos
-J-

2nx

juc

+ &! stay +a
,

cos
j-

2nx

+ b sm+..,
.

nnx

nnx

where

-M

-- dx

flllX

f (x)

cos

= 0,
/-v

(/i

1,

2,

...),
(2)

6n

= -L

\f(x)sln^-dx(n=\,

'

2,

-i

At the points

of discontinuity of the function f (x) and at the end-points


of the interval, the sum of the Fourier series is defined in a manner
similar to that which we have in the expansion in the interval (
Jt, n).
In the case of an expansion of the function / (x) in a Fourier series in

x-l

an arbitrary interval
formulas

(2)

(a,

a-f-2/) of

length

the

2/,

limits

of

integration

in

should be replaced respectively by a and a-|-2/

functions in a Fourier
determine the sum of the series

the following

in the
the points
of discontinuity and at the end-points of the interval (x
ic,
X
function itself and of the sum
JT), construct the graph of the
of the corresponding series [outside the interval (
JT, ji) as well]:
Q
x
when ~~ n

Expand

interval

ji,

ji),

series

at

< ^

Consider the special case when


9fi79
2672.

xr

Consider
c)

fl

= 0,

the

1;

2673. f(x)
2674. f(x)
2675. f(x)
2679.

=x
= eax
= slnax.

(0,

=~

bx when
special cases:
0.
d) a=l, 6
2

Expand

the interval

ct

>

a)

= b = l\

2676. / (x)
2677. f(x)
2678. f (x)

the function f(x)

2ji).

1,

b)

1,

6=1;

= cos ax.
= smhax.
= cosh ax.

-jr-

in

a Fourier

series

in

320

Series

2680.

in the interval

number

the function f(x)

Expand

=~

in sines of

(Ch. 8

multiple arcs

Use the expansion obtained

n).

(0,

__
to

sum

the

series:

Take the functions indicated below and expand them, in the


interval (0, jt), into incomplete Fourier series: a) of sines of
multiple arcs, b) of cosines of multiple arcs. Sketch graphs of
the functions and graphs of the sums of the corresponding seiies
in their domains of definition.
x. Find the sum of the following series by means
2681. f(x)
of the

expansion obtained:

I+P+P+...
Find the sums of the following number series
2682. f(x) = x
by means of the expansion obtained:
2

1)

2683. f(x)

+7)2+ 32+

= eax

2684.

2^"

32

42

"!"

when

/(*)=

when

-^

x when
x when

ji

Expand the following


sines of multiple arcs:

2687. f(x)
2688.

2)

(x
=

0<;c^~
*

functions, in the

when

0<#^^-,

when

~n < x < n.

x(n

interval

(0,

K),

in

x).

= sin-|.
/(A;)

Expand the following functions, in the interval


sines of multiple arcs:
u
when

(0,

Ji),

in co-

Sec. 4\

__

321

Fourier Series

2690.

when
2691. / (x)

= x sin x.
cos x

when

cos x

when

< x^ ~

2693. Using the expansions oi the functions x and x* in the


in cosines of multiple arcs (see Problems 2681 and
it)
2682), prove the equality
interval (0,

2694**. Prove that

?L
Ji,

and
in

x}

the function /(x)

if

x\

f(~

then

its

n) represents an expansion

if

the function f(x)

the interval

Ji,

odd

is

is

it

Ji)

even and we have

is

Fourier series in
in cosines of

arid

/fy-f *) =/

expanded

in

the

interval

odd multiple

(y~*)

sines

of

arcs,

then

odd mul-

tiple arcs.

Expand the following functions in


cated intervals:
2695.
2696.
2697.
2698.

f(x)

= \x\

f(x)=*2x
f(x)

Fourier series in the indi-

!<*<!).

(0<JC<

1).

= e* (/<*</).

f(jc)=10

Jt

(5<jc<15).

Expand the following functions, in the indicated intervals,


in incomplete Fourier series: a) in sines of multiple arcs, and
b) in cosines of multiple arcs:

(0<*<

2699. /(*)=!
*
2700. /(*)
2701.

1).

(Q<x<l).

2702.

/M-{ 2 _-; ^n

2703.

Expand the following function

T
(3

'

in

cosines

\
:

when -|<jr<2,
3_j^ when 2<jc<3.
1

11-1900

of

multiple

Chapter IX

DIFFERENTIAL EQUATIONS

Sec. 1. Verifying Solutions. Forming


Curves. Initial Conditions

1. Basic concepts. An equation


F(x, y,

where y

Differential

Equations

of

Families of

of the type
(n)

y'

t/)

= 0,

(1)

y(x) is the sought-for function, is called a differential equation of


order n. The function y
y(x), which converts equation (1) into an identity,
is called the solution of the equation,
while the graph of this function is
called an integral curve. If the solution is represented implicitly, O(A, f/)~0,
then it is usually called an integral
Example 1. Check that the function t/ sinjt is a solution of the equation

Solution.

We

have:

and, consequently,
/"

+y

sin x -f sin x

The integral
*/,

Cp

....

Cw

0.

(2)

of the differential equation (1), which contains n independent arbitrary constants C t ..., C n and is equivalent (in the given region) to equation (1), is
called the general integral of this equation (in the respective region). By assigning definite values to the constants C,, ..., C n in (2), we get particular
,

integrals.
if we have a
family of curves
C n from the system of equations

Conversely,
eters Cj

= 0,

dx

-0.

....

(2)

and eliminate

the param-

*-0,

dx n

we, generally speaking, get a differential equation of type (1) whose general
integral in the corresponding region is the relation (2).
Example 2. Find the differential equation of the family of parabolas

y^C^x-CJ.
Solution. Differentiating equation
/'

= 2C

(*

Eliminating the parameters C, and


the desired differential equation

C 2)

(3)

(3)

twice,

and

C 2 from

(/"

we

get:

= 2C,.

equations

(4)
(3)

and

(4),

we obtain

Sec

It

is

Verifying Solutions

function

verify that tha

to

easy

323

converts

(3)

this

equation

identity.

2. Initial conditions.
differential equation
y

desired particular solution y

for the

If

= f(x,

(n)

y, y'

into

= y(x)

(n ~ l)

an

of a

(5)

the initial conditions

and we know the general solution

are given

y = <V(x,
then the

arbitrary constants

from the system

of

Cn

(5)

are determined

(if

this

which y(0)

Find the curve

y'(0)
have:

l.

We

Solution.

of the

family

in

formulas

(6)

2.

i/

Putting #

possible)

C lt .... C B ),
c ..... c ).
<?*(*<>.

*,

for

is

-cp(*o,

</o=

3.

C, .....

...,

equation

C n ),

equations
</

Example

C lt

of

'=

and

(6)

1=0,

we obtain

(7),

2-C

+ ^,

(?)

2C 2

whence
(:,=(),

and, hence,

C,=

= e~.

Determine whether the indicated functions are solutions of the


given differential equations:
2704. *//'== 2#,
2705. y
2706.

2707.
2708.

2709.
2710.

t2

(jc

= 5x*.

t/

= x*
h

//)

djc -h

JK

dy

= 0,

= ^=^

+ = 0, = 3sinjc 4 cos*.
~? +
= 0, x = C, cos <o/ 4- C sin CD/.
2j/'+0 = 0; a) y = xe* b) y = x
#"
//

(/

^/

a) jc

-(X.
y^*
Show

X/
2

2711. (x

-f

e.

that for the given


relations are integrals:

11*

//

//

differential

equations

the

indicated

_ _

324

Differential Equations

2712. (x
2713. (xy-x)y"

Form

differential

C lt

+ xy' + yy'
equations

Q,
2y'
y
\n(xy).
of the given families

(C,
C,, C, are arbitrary constants):
Cx.
2714. y

=
=

272l

9718
ii&.
971Q
2719.

n *.=

9799
*722.

fo' ? 3re P arameters >-

yr

2724.

2720.

& M-- = 2 + Ce"^.

2726.

Form

thej

+ay

0,- y .)'-2p*

-*
y

*-.

curves

of

a parameter).

</'

#//-plane.

'

+ = <:'.

2717. *'

2715. y
Cx*.
2716. *>=-.2C*.

[Ch. 9

</

2725.

differential

=
=

equation of

all straight lines in

the

2727. Form the differential


vertical axis in the ^y-plane.
2728. Form the differential

equation of
equation

of

all

with

parabolas
circles

all

in

the

xy-plane.

For the given families of curves find


the given initial conditions:
2729. x*y*
5.
C, 0(0)
2730.
2731. y
C sin
2732. y^C.e-x

=
y=(C^
=

the

lines

that

satisfy

= 0,

Sec. 2. First-Order Differential Equations

P. Types of first-order differential equations. A differential equation


the first order in an unknown function yt solved for the derivative y'
is
the form
t

y'

where f(x, y)
consider the

= f(*.

of
of

(i)

</).

the given function. In certain cases it is convenient to


variable x as the sought-for function, and to write (1) in the
is

form

x'=e(*,y)>

$--.

(i')

where

gfr

Taking into account that


(1)

and

(!')

may

0'

and

*'

j^

the

differential

equations

be written in the symmetric form

P(x, y)dx+Q(x, t/)<ty=0,

where

and Q

(2)

y) are knowri functions.


By solutions to (2) we mean functions of the form t/ cp(jc)
ihat satisfy this equation. The general integral of equations (1)
(x, y)

(x,

or

and

x=ty(y)
(I'),

or

equation

where

325

First-Order Differential Equations

Sec. 21

is

(2),

is

form

of the

an arbitrary constant.

2. Direction

The

field.

set of directions

tana = /(x,

y)

called a direction field of the differential equation (1) and is ordinarily


depicted by means of short lines or arrows inclined at an angle a.
Curves f(x, y)
k, at the points of which the inclination of the field
has a constant value, equal to k, are called isoclines. By constructing the
isoclines and direction field, it is possible, in the simplest cases, to give a

is

Fig

rough sketch of the

field of

intagral

105

curves, regarding

the latter

which at each point have the given direction of the field.


Example 1. Using the method of isoclines, construct the

as

field of

curves
integral

curves of the equation


y'=*x.
Solution.
rection field,

The family

By constructing the isoclines x~k (straight lines) and the diwe obtain approximately the field of integral curves (Fig. 105).

of parabolas

the general solution.


Using the method of isoclines, make approximate constructions of fields
of integral curves for the indicated differential equations:
is

2733. y'

2734.

</'

=
=

x.

-f-

2735. y'=l-ftf

2736.

y'=

2737. y'

8
.

326

(Ch. 9

Differential Equations

3. Cauchy's theorem.
b < y < B}

If

U\a<x<A,

a function / (#, y) is continuous in some region


in this region has a
bounded derivative

and

(** y)> tnen through each point (*


y Q ) that belongs to U there passes one
and only one integral curve y y(x) of the equation (1) [cp (* ) #ol4. Euler's broken-line method. For an approximate construction of the

f'y

(*
integral curve of equation (1) passing through a given point
replace the curve by a broken line with vertices M,-(x/, #/), where

2.

(i'0,

Using Euler's method

find (/(I),

if

y(0)=l

(/i

1.

for the

We

t/

),

we

(one step of the process),

/i

/)

Example

2,

...).

equation

'

-0.1).

construct the table:

Thus,

/(!)=

1.248.

For

the

sake

of

comparison,

the

exact

value

is

T ss 1.284

Using Euler's method, find the particular solutions to the


given differential equations for the indicated values of x:
2738. y' = y, y(0)=l; find y(\) (A-0.1).
2739.

y'-x + y, /(!)-

2740. ^'
2741.

= -X_,

t/(0)

1;

find y(2),

= 2;

-, y(Q)=l;

find </(!)
find y(l)

= 0.1).
= 0.1).
(A
= 0.2).

(A

(/t

Differential Equations with Variables Separable

Sec. 3]

Sec.

First-Order

3.

Differential

Equations

with

327

Variables

Separable.

Orthogonal Trajectories

1. First-order equations with variables separable.


is a first-order equation of the type

An equation with

variables

separable

X (x) Y

(y)

Dividing both sides of equation


-

= f(x)dx

y'

= f(x)g(y}

dx

+ X, (x) Y, (y) dy = Q

(1)

(i)

(!')

by g(y) and multiplying

Whence, by integrating, we

get

the general

by

dx,

we

get

integral of equa-

tion (1) in the form

Similarly, dividing both sides of equation (!') by X, (x)


we get the general integral of (!') in the form

(y)

and integrating,

then the function y


is
ti Q
(r/ )=0,
yQ we have
directly evident) a solution of equation (1) Similarly, the straight
a and
will be the integral curves of equation (!'), if a and b
are, respectively, the roots of the equations X, (*)() and Y (*/)
0, by the
ieft sides of which we had to divide the initial equation.
Example 1. Solve the equation
If

for

also (as
lines x

some value y

is

y-b

'

3>

In particular, find the solution that satisfies the initial conditions

Solution. Equation

(3)

be written in the torm

may

dx~~

Whence, separating variables, we have

and, consequently,
In
|

where the arbitrary constant

In

x\

taken
antilogarithms we get the general solution
In C,

is

+ ln C
in

|t

logarithmic form. After taking

f
where

C=

When

C,.

dividi
dividing
by y
ila
contained in the formula

we could' lose
= 0.(4) for C
'

the solution

=0. but

the

latter

is

328

(Ch. 9

Differential Equations

Utilizing the given initial


sired particular solution is

conditions,

we

get

C = 2;

hence, the de-

and,

J2

y ~~

'

Certain differential equations that reduce to


2
separable. Differential equations of the form

with

equations

variables

reduce to equations of the form (1) by means of the substitution u


is the new sough t-for function
3
Orthogonal trajectories are curves that intersect the lines of the given
a
right angle. If F (x, y, #')
(x, y> ort=0 ia is a parameter) at
family
is the difierential equation of the family, then

where u

is

the differential equation of the orthogonal trajectories.


Example 2. Find the orthogonal trajectories of the family of ellipses

Solution Differentiating the equation (5),


tion of the family

Whence, replacing

if

by

^7,

we

find

the

duerential

106

get

the

differential

equation

orthogonal trajectories
~~

~~

integrating,

we have

*x*

(family

'

it'

of

equa-

0.

(/'

Fig.

we

parabolas) (Fig. 106).

of

the

329

Differential Equations with Variables Separable

Sec. 3\

4. Forming differential equations. When forming differential equations in


geometrical problems, we can frequently make use of the geometrical meaning
of the derivative as the tangent of an angle formed by the tangent line to
the curve in the pos'tive x-direction. In many cases this makes it possible
straightway to establish a relationship between the ordinate y of the desired
curve, its abscissa x, and the tangent of the angle of the tangent line (/',
that is to say, to obtain the difleiential equation. In other instances (see
Problems 2783, 2890, 2895), use is made of the geometrical significance of
the definite integral as the area of a curvilinear
trapezoid or the length of
an arc. In this case, by hypothesis we have a simple integral equation
(since the desired function is under the sign of the integral); however, we
can readily pass to a differential equation by differentiating both sides.
Example 3. Find a curve passing through the point (3,2) for which the
segment of any tangent line contained between the coordinate axes is divided in half at the point of tangency.
Solution. Let
(x,y) be the mid-point of the tangent line AB. which by
hypothesis is the point of tangency (the points A and B are points of intersection of the tangent line with the y- and *-axes). It is given that OA
2y
and OB Zx. The slope of the tangent to the curve at
(x, y) is

OA

dy_
dx~
This

is

y
x

OB~

'

the differential equation of the sought-for curve. Transforming,


d\

we

get

dy _ ~

~x

~T~~y~

and, consequently,
In

\nx-\-\ny
Utilizing the initial condition,
curve is the hyperbola xy
6.

Cor xy

C.

we determine C = 3-2

Solve the differential equations:


2
2
2742. tan A: sin y d* + cos x cot ydy

6.

Hence,

the

desired

= Q.

2743. xy'~ //
{/'.
2744. xyy' =-.
= a(l
2745. //
jq/'
x
fan ydx
2746. 3c
2747. y' tan *
//.

\x\
=

+*V). x

+ (l

sec*

ydy = Q.

Find the particular solutions of equations that


indicated initial conditions:
x
0.
2748. (1 +e ) y y' = e*\ //= 1 when jt
!
when x
2749. (xy* + x) dx-\-(x* yy)dy=* 0; //
2750. r/'sin x

= y\ny\ y~l

when

=
=
*=
-|.

satisfy

= Q.

Solve the differential equations by changing the variables:


2751. y' = (x+y)**

2752. i/
(8*42//+l)'.
2753. (2x + 3{/
l)dx-{ (4x
2754. (2x
y)dx (4x 2y

+ fo/
-

5) dij

= 0.

the

_ _

330

Differential Equations

In

Examples 2755 and 2756, pass

[C/i.

to polar coordinates:

2755.

2756.
2757*. Find a curve whose segment of the tangent is equal
to the distance of the point of tangency from the origin.
2758. Find the curve whose segment of the normal at any
point of a curve lying between the coordinate axes is divided in

two

at this point.

2759. Find a curve whose subtangent is of constant length a.


2760. Find a curve which has a subtangent twice the abscissa
of the point of tangency.
2761*. Find a curve whose abscissa of the centre of gravity
of an area bounded by the coordinate axes, by this curve and
the ordinate of any of its points is equal to 3/4 the abscissa of
this point.

2762. Find the equation of a curve that passes through the


point (3,1), for which the segment of the tangent between the
point of tangency and the *-axis is divided in half at the point
of intersection with the y-axis.
2763. Find the equation of a curve which passes through the
point (2,0), if the segment of the tangent to the curve between
the point of tangency and the t/-axis is of constant length 2.
Find the orthogonal trajectories of the given families of curves (a is a parameter), construct the families and their orthogonal trajectories.
2

2764. x
2
2765. t/

+ y =a
= ffx.
2

2766. xy
2767. (x

Sec. 4. First-Order Homogeneous Differential

1. Homogeneous equations.

= a.
2

a)

t-f/

=a*.

Equations

differential equation

P(x y)dx+Q(x,y)dy =
t

(1)

is called homogeneous, if P (AT, y) and


Q (x, y) are homogeneous functions of
the same degree. Equation (1) may be reduced to the form

and by means
it

is

of the substitution y
xu, where u is a new unknown function,
transformed to an equation with variables separable. We can also apply

the substitution

Example

1.

x-yu.
Find the general solution

to the equation

Sec. 4]

First-Order Homogeneous Differential Equations

Solution. Put y

we

Integrating,

get w

ux',

then

-f xu'

In In

that reduce to

or

whence
x In In

2. Equations

= eu + u

331

homogeneous equations.

If

'

and 6
I

#2^2

Ue

0,

then, putting into equation (2) x

+ a,

j/

= t;-fp,

where

the constants

a and P

are found from the following system of equations,

+ c, = 0,

a 2a

+ b$ + c = 0,
t

we

get a homogeneous differential equation in the variables u and v. If


u, we get an equation with variables
0, then, putting in (2) a,x 4- b^y
separable.

Integrate the differential equations:

2768. 0'
2769.

=1

277

1.

y^-^.

(x-y)ydx-x*dy = Q.

find the family


2771. For the equation (x +y*) dx
2xydy
of integral curves, and also indicate the curves that pass through
the points (4,0) andj_l,l), respectively.
2772.

=
+

2773. xdy
-\-ifdx.
ydx Vx*
2
2
0.
2774. (4x* + 3xy
jf) dy
f/ ) dx + (4y
3jvy
1
2775. Find the particular solution of the equation (x
=
x
2.
2xydy Q, provided that r/=l when
Solve the equations:
2776. (2x

9777 f/./
2/77.

3y*)dx+

2779. Find the equation of a curve that passes through the


point (1,0) and has the property that the segment cut off b\ the
tangent line on the r/-axis is equal to the radius vector of the
point of tangency.
2780**. What shape should the reflector of a search light
have so that the rays from a point source of light are reflected
as a parallel

beam?

532

_ _

[Ch. 9

Differential Equations

2781. Find the equation of a curve whose subtangent is equal


mean of the coordinates of the point of tangency.
2782. Find the equation of a curve for which the segment
cut off on the y-axls by the normal at any point of the curve
is equal to the distance of this point from the origin.
2783*. Find the equation of a curve for which the area contained between the #-axis, the curve and two ordinates, one of
which is a constant and the other a variable, is equal to the
ratio of the cube of the variable ordinate to the appropriate
to the arithmetic

abscissa.
off

2784. Find a curve for which the segment on the y-axis cut
by any tangent line is equal to the abscissa of the point of

tangency.
Sec. 5. First-Order Linear Differential

Equations.

Bernoulli's Equation

1. Linear equations. A

differential equation of the

)-y^Q

form
(1)

(x)

of

degree one in y and y is called linear.


If a function Q(jt)=~0, then equation
).y

takes the form

(1)

=Q

(2)

and is called a homogeneous linear differential equation. In this case, the


variables may be separated, and we get the general solution of (2) in the
form

y = C-e

P P(X) dx

(3)

To solve the inhomogeneous linear equation (1), we apply a method that


called variation of parameters, which consists in first finding the general
solution of the respective homogeneous linear equation, that is, relationship (3). Then, assuming here that C is a function of x, we seek the solution
of the inhomogeneous equation (1) in the form of (3). To do this, we put into
(1) y and y' which are found from (3), and then from the differential equation thus obtained we determine the function C(x). We thus get the general
solution of the inhomogeneous equation (1) in the form
is

^/x -f
J
= C(x).e
Example

I.

Solve the equation


y'

Solution.

Solving

it

we

tan **/-}- cos

x.

The corresponding homogeneous equation


get:

C
^- r
*^I
1

(4)
is

333

Bernoulli's Equation

Sec. 5]

Considering

and differentiating, we

as a function of x,

dC
Putting y and y' into

dC

cos*

djc

we

(4),

A
dx

cos x

sin

fi'nd;

~
'

/,x>o2
2
x
cos

get:

sin*

cos 2 *

cos*

dC
-r-=

or

'
,

dx

whence

Ccos 2 *d*==i-* +

Hence, the general solution

of

equation

j
(4)

has the form

COS*
In solving the linear equation

we can

(1)

make

also

use of the substitu-

tion

uv

where u and v are functions of

x.

we require that
'

(1) will

have the form

+ v'u^Q(x).

\-P(x)u]v

[u
If

(5)

Then equation

(6)

+ P(jc)M = 0,

(7)

then from (7) we find M, and from (6) we find u; hence, from (5)
2\ Bernoulli's equation. A first order equation of the form
y'

+ P (<) y ^ Q

(x)

we

find y.

y\

where a 7=0 and a 7= 1, is called Bernoulli's equation


""*.
near equation
by means of the substitution z
the
or
substitution
the
y = uv
apply directly

It

r/

It

reduced to

is

li-

also possible to
method
of
variais

tion of parameters.

Example

2.

Solve the equation


y'

Solution. This

is

y+*

VH-

Bernoulli's equation. Putting

y=^u>v,

we

ijet

u'v

+ v'u

To determine

uv

+x

the function u

y"uv

we

or

u'

~w-0
x

we have
u

Putting this expression into (8),

u
j

-f

v'u

require that the relation


u'

be fulfilled, whence

we

get

x*.

=x

V^taT.

(8)

334

_ _
Differential Equations

whence we

[Cfi.

find v:

and, consequently, the general solution

is

obtained

in

the form

Find the general integrals of the equations:


2785.

ax

-=*.
x
+ = x*.

2786.

2787*. (\
2788. y*dx(2xy

Find the particular solutions that satisfy

the

indicated

con-

ditions:

+y

2789. X y'

e*

= Q\

y = b when x

1-- *

2790. y'

j-2-7

2791. y'

yianx =

= 0;

cos x

*/

y=

= a.

when x-0.

when

jt

= 0.

Find the general solutions of the equations:


2792. *l
dx

+ JLx =
'

Xyy*

2793. 2xy
x
2794.

0dx +

+x

2795. 3xdy--=y(l
sin A:
3y* smx)dx.
2796. Given three particular solutions y,

equation. Prove that the expression

What

^^

y lt y 2 of a linear
remains unchanged for

the geometrical significance of this result?


2797. Find the curves for which the area of a triangle formed
by the *-axis, a tangent line and the radius vector of the point
of tangency is constant.
2798. Find the equation of a curve, a segment of which, cul
off on the x-axis by a tangent line, is
equal to the square of the
ordinate of the point of tangency.
2799. Find the equation of a curve, a segment of which, cut
off on the y-axis by a tangent line,
is equal to
the subnormal.
2800. Find the equation of a curve, a segment of which, cut
off on the y-axis by a tangent line,
is proportional to the
square
of the ordinate of the point of tangency.

any

x.

is

Exact Differential Equations. Integrating Factor

Sec. 6]

2801. Find the equation of the curve for which


of the tangent is equal to the distance of the point
tion of this tangent with the x-axis from the point

335

the

segment

of

intersec-

M (0,a).

Sec. 6. Exact Differential Equations.


Integrating Factor

1. Exact

differential equations. If for the differential equation

P(x.y)
the equality

dU

is

:p =-^-

fulfilled,

dx+Q(x

y)dy = Q

then equation

(1)

may

(1)

be written

in

the

and is then called an exact differential equation. The gen= C. The function U (x, y) is detereral integral of equation
(1) is U (x, y)
mined by the technique given in Ch. VI, Sec. 8, or from the formula
form

(see

(x,

t/)

Ch, VII, Sec.

Example

1.

9).

Find the general integral


2

(3x

Solution.

This

+ 6Af/

2
)

an exact

is

5= i2 X y and,

dx

of the differential

equation

+ (6x*y + 4y') dy = 0.

differential

since

equation,

hence, the equation

is

of

the form

-J
K/

= 0.

Here,

and

whence

U=

(3^

+ 6xy*) dx +

q>

(y)

= x> + 3*V +

q> (y).

= x y + 4y* (by
y -f
(y)
=
and
hypothesis); from this we get
y* + C*. We finally get
(y)
f/(r, t/)-=
-f-3xV + +C consequently, x -f3^V + / = C is the sought-for
Differentiating

with respect

to y,

q>

jt

we

6jc

cp'

-y

= 4(/*
'(//)

find

q>

.V
.

C(

general integral of the equation.


2. Integrating factor. If the left side of equation (l)is not a total (exact)
differential and the conditions of the Cauchy theorem are fulfilled, then there
exists a function U,
U.(A', y) (integrating factor) such that

\i(Pdx+ Qdy) = dU.


Whence

it

is

found that the function

The integrating

factor

u,

is

u,

satisfies the

(2)

equation

readily found in two cases:

336

Differential Equations

Example
Solution.

"

and

Solve

2.

equation

Here P = 2*y + rV-f-^

_
d#

or

dx

+ x*y + ~

2xy

'

hence>

r dx

cty

^^^ +
r

Q=x*+y*

"

Since
it

the

[C/t.

'

ft=

Q
x jB,
dx

follows that
1
fdP
^
JS
----dQ\
and
dx=dx
Q.\dy
dxj

-r

rffi

Multiplying the equation by

which

is

^.

\\

\i

= e*

an exact differential equation.

In

11
*

= ^,

11 = ^.
r

we obtain

Integrating

it,

we

get

the

general

integral

Find the general integrals of the equations:


2802 (x
+ 2z/)dy = 0.
t/)d^+(^
1
2803. (V + 1/
2x) dx + 2xydy = Q.
3
8
2804. (jc
S^ 8 + 2) dx (3x'y y ) d(/ = 0.

2805.

^-^=='

2806.

2807. Find the particular integral of the equation

which

satisfies the initial condition #(0)


2.
Solve the equations that admit of an integrating factor of the
form fi = M*) or M'
H'(y)

+ y*)dx2xydy = 0.
4-xy)dxxdy = Q.

2808. (x
2809. y(l
2810.

281

1.

(jc

cos

t/

y sin

r/) rfi/

+ (x sin

*/ -(

//

cos y) dx

= 0.

First-Order Differential Equation? not Solved for Derivative

Sec. 7]

Sec.
for

7.

337

Equations not Solved

First-Order Differential

the Derivative

1. First-order differential equations of higher powers.


F(x,

which

for example
two equations:

is

y,

two

of degree

!/')

in y',

y'=ti(x,y)>

If

an equation

= 0,

(I)

the.i

</'=/ 2

by solving

(1)

for y'

<*.</)

we

get
(2)

Thus, generally speaking, through each point


(xQt (/ ) of some region
of a plane there pass two integral curves. The general integral of equation
(1) then, generally speaking, has the form

0(AM/,
where

and

C^OM*,

C) 0> 2

{/,

C)

(x. y,

= 0,

(3)

the general integrals of equations (2).


2 are
Besides, there may be a singular integral for equation (1). Geometrically,
a singular integral is the envelope of a family of curves (3) and may be obtained by eliminating C from the system of equations
tl> 1

<I>

OJx,
or

by eliminating p =

t/'

0,

C)

= 0,

from the system

F(x

y,

(*, y,

0>c

F'

p)-0,

of

C)

(4)

equations

(x,y,p)=0.

(5)

We

note that the curves defined by the equations (4) or (5) are not
always solutions of equation (1); therefore, in each case, a check is necessary.
Example 1. Find the general and singular integrals of the equation
2
A'//'

Solution. Solving for y'

2A'j/'

-f

Q.

we have two homogeneous

equations:

do fined in the region


x

the general integrals of which are

or

Multiplying,

we

get the general integral of the given equation


(2*

+y

C)

2
(A-

+ xy) -

or
(a

we

(It

family of parabolas).
Differentiating the general
find the singular integral

may

integral with respect

be verified that y-(-jc=0

to

C and

eliminating C,

+ Jt-O.
is

the solution of this equation.)

Differential

Equations

(Ch. 9

by differentiating
possible to find the singular integral
with respect to p and eliminating p.
2. Solving a differential equation by introducing a parameter. If a firstorder differential equation is of the form
It

is

+ 2xp

also

#=

then the variables y and x


1

from the

be determined

may

system of equations

dq>

where p = t/' plays the part of a parameter.


Similarly, if y = ty( x #') tnen x and y are determined
>

from the system

of equations

Example

Find the general and singular integrals

2.

of the

equation

y=y' -xy'+^.
Solution.

we

Making the substitution t/'=p,

rewrite the equation in the

form

Diffeientiating with respect to x and considering


ft

dp

dp

p a function of

x,

we have

p-*p-p-*fx +*
or

-p(2p

x)**(2p

x),

or-j-

into the original equation,

l.

we have

Integrating

we

get p

= x + C.

Substituting

the general solution


or

</

=y

Differentiating the general solution with respect to C and eliminating C, we


x^
x*
obtain the singular solution: */
-r-. (It
may be verified that */ -r- is the

solution of the given equation.)


If we equate to zero the factor 2p
x which was cancelled out, we get
x
x2
into
the
which is the
and, putting p
given equation, we get y=-j
t

pay

same singular

solution.

Find the

general

(In Problems
curves.)

2812.

2812

and singular integrals


and 2813 construct the

(/''-^'-M^O.
2

2813. 4y'

9JC-0.

of

the

field

equations:
of

integral

Sec

8]

The Lagrange and Clairaut Equations

Q.
2814. yy'*(xy+l)y' + x
2
2815. yij'
2xy'+y Q.
2816. Find the integral curves

339

that pass through the point

0,

the

of

equation

y'*

+y* =

Introducing the parameter y' = p, solve the equations:


2820. 4y = x*+y'\
2817. x=smy'+lny'.
2818. y
2819.

= y'*e>".
= y'*

/2

,*

9 91

'

Sec. 8. The Lagrange and Clairaut Equations

1. Lagrange's equation. An equation

of

the form

= *<P(P) + 1|>(P),
where p

= tf

(1)

called Lagrange's equation Equation (1) is reduced to a linear


equation in x by differentiation and taking into consideration that dy^pdx:
is

(p)] dp.
If

p^q>(p),

then from (1) and

(2)

we

(2)

get the general solution in parametric

form:

parameter and f(p)^ g(p) are certain known functions. Besides,


a singular solution that is found in the usual way.
2. C'lairaut's equation. If in equation (l)p^<Mp), then we get

where p
there

is

may

be

raut' s equation

^(C) (a family of .straight lines).


general solution is of the form y-Cx
is also a particular solution
(envelope) that results by eliminating the
parameter p from the system of equations
Its

There

x~

lJ:=

Example. Solve the equation


</~2t/'A'

Solution. Putting
ing

dy by pdx, we

y'^p we

+ i.

have //^2pv

(3)

get

p dx = 2p dx +

2.v

or

Solving this lineai equation, we will have

*=l

dp

different! a Ting

and replac-

__

340

__

Differential Equations

^_

[C/t.

Hence, the general integral will be

=l(l

To

find the singular integral,

we form

f/-2px +
in the usual

the system

l,

= 2*-

way. Whence

*=

and, consequently,

Putting y into (3) we are convinced that the function obtained Is not
a solution and, therefore, equation (3) does not have a singular integral.

Solve the Lagrange equations:


2822.

y-'
=

2824. y

= (1 +y'

+ r=FT

2823. ,
,'
Find the general and singular integrals of the
tions and construct the field of integral curves:
2826. y
xy' +y'*.
2827. y
xy'+y'.
r

2828.

2829.

by

=
=
= Xy' + V \-\-(y')
y = xy' +
j,.

ij

Clairaut equa-

2830. Firid the curve for which the area of a triangle formed
a tangent at any point and by the coordinate axes is con-

stant.

2831. Find the curve it the distance of a given point to any


tangent to this curve is constant.
2832. Find the curve for which the segment of any of its
tangents lying between the coordinate axes has constant length /,
Sec. 9. Miscellaneous Exercises on First-Order Differential Equations

2833. Determine the types of differential equations


cate methods for their solution:

=
=
2ju/ + *';
y'

b) (*-*/)//'
c)

d) y'

(f/-

</';

8)

and indi-

Miscellaneous Exercises on First-Order Differential Equations

Sec. 9]

= (* 4+ y sin (x*
xy) y' = #';

i)

</'

j)

x cos f/'

k)

+ 2xy')dx +
+ (y* + 3*y dy = 0;
m) (x' 3xy) dx + (* + 3)dy = 0;

1)

</)';

1 ;

{/'

341

(x'

n)

Solve the equations:


2834. a) (x

ycos^

b) jcln

2835.

dy

xrfx=(^

y'^dy.

2836. (2xy*y) dx-\-xdy=-Q.


2837. xy' -\- y -^ xtf \nx.
2838. y
xy'
2839. f/
or/'
jc#'
2840. ^(
2841. (1

=
=

2842.

+
+

y'-y^^l.

2843. y^-((/'-l 2xe>)y'.


2844. //'-|-//cosAr=sin.tcosx.
2848. (x'y
2849.

x*

-\-y-l) dx

2845

'

2846

'

(l-^

^'-T-^ =

2847. y' (xcosy-'r a sin

2</)

= 1.

(xy + 2x3yQ)dy = Q.

=
y'

2850. ./"

d.v

= (x*y f 2)

<///.

+ y dy J-| dJf = 0.
286
=*
X
2862.
*.
= cosA:.
f/y'-hf/

2852. 2dx

2853..

i ,'

2854.

2863.

2855. xdy-\ ydx^ifdx.


!.
2856. //' (j: |- sin //)

2857.

H? = -P + P

/
ff

= 2^'H-l/l

/
//'==

j (1 +lny

2864. (2e

d*- (x

2859. x

*/" -h

Zxyy'

2860.

^^H-\t

ln^).

dy
x
<je

y')

^-

+
2/'

+y

0.

= 0.

2865.
2868.

dx =

^ dydx =

/'

=2 f

(;(/'

4
f

2858. ^'

(-y'

v 4- 2

y-1 J

-1- 1 )

2867. a(xt}'
2868. xdy- y dx

= y* dx.

342

[Ch. 9

Differential Equations

2869. (x*

)/

At,

dy +

(x*

+ 3xy V^-\) dx = 0.

2870.

2871. J/oM1
dy+ (x y
2872. xyy'*
(x* +y*)y'

= xy' +
(3* + 2;q/
= 3p

2873. y
2874.
2875.

Ya*

+x

dx = Q.

y'Jdx

+ f*

2xy

3y*)dy=--Q.

-t-4*/

2</p|
Find solutions to the equations

for the indicated

initial

con-

ditions:

2876.

#=
y'=^\
y

2881.

jc=l.

= \\ y=\ for *=1.


= 0.
y = 2; y = 2 for
=
=
=
for
0.
^
+ !) !;
= cos
= for * = 0.
#' +
for x = 0.
x*\ y =
2y =
y'
=
=
for * = 0.
y'+y 2x\ y

2877. e*- y'


2878. cot ;q/'
2879. e^(^
2880.

for

*/

jt

-f

A;;

{/

2882.
for x = 0.
2883. xy'=y\ a) //==! for jc=l; b) y
for x-0.
2884. 2xy'
y\ a) y=l for jc=l; b) y
z
2
for x-0; b)y=l forjc=-0;
2885. 2xyy'-\-x
0; a)
#
= Q for je= 1.
c) y
2886. Find the curve passing through the point (0, 1), for
which the subtangent is equal to the sum of the cooidinates of
the point' of tangency.
2887. Find a curve if we know that the sum of the segments*
cut off on the coordinate axes by a tangent to it is constant and
\

=
=

y-0

equal to 2a.
2888. The

sum of the lengths of the normal and subnormal


equal to unity. Find the equation of the curve if it is known
that the curve passes through the coordinate origin.
2889*. Find a curve whose angle formed by a tangent and the
radius vector of the point of tangency is constant.
2890. Find a curve knowing that the area contained between
the coordinate axes, this curve and the ordinate of any point on
it is equal to the cube of the ordinate.
2891. Find a curve knowing that the area of a sector bounded by the polar axis, by this curve and by the radius vector
of any
point of it is proportional to the cube of this radius
is

vector.

2892. Find a curve, the segment of which, cut off by the


tangent on the x-axis, is equal to the length of the tangent.

Miscellaneous Exercises on First-Order Differential Equations

Sec. 9]

2893. Find

the curve,

between the

contained

which the segment

of

axes

coordinate

of the

divided

is

343

tangent

into half by

the parabola if =--2x.


2894. Find the curve whose normal at any point of it is
equal to the distance of this point from the origin.
2895*. The area bounded by a curve, the coordinate axes,
and the ordinate of some point of the curve is equal to the
length of the corresponding arc of the curve. Find the equation
of this curve if it is known that the latter passes through the

point (0, 1).


2896. Find the curve for which the area of a triangle formed
vector of the point of
by the x-axis, a tangent, and the radius
2
is
to
a
constant
and
tangency
equal
2897. Find the curve if we know that the mid-point of the
segment cut off on the x-axis by a tangent and a normal to the
curve is a constant point (a, 0).
.

When forming

first-order differential equations, particularly in phvsical


frequently advisable to apply the so-called method of differenwhich consists in the fact that approximate relationships between
tials,
infinitesimal
h.crements of the desired quantities (these relationships are
accurate to infinitesimals of higher order) are replaced by the corresponding
relationships between their differentials. This does not affect the result.
Problem. A tank contains 100 litres of an aqueous solution containing
10 kg of salt. Water is entering the tank at the rate of 3 litres per minute,
and the mixture is flowing out at 2 litres per minute. The concentration is
maintained uniform by stirring. How much salt will the tank contain at the
end of one hour?
Solution. The concentration c of a substance is the quantity of it in
unit volume. If the concentration is uniform, then the quantity of substance in volume V is cV.
Let the quantity of salt in the tank at the end of t minutes be x kg.
The quantity of solution in the tank at that instant will be 100
/ litres,

problems,

is

it

and, consequently, the concentration

During time

c=

kg per

QQ

litre.

solution flows out of the tank (the


solution contains 2cdt kg of salt). Therefore, a change of dx in the quantity
of salt in the tank is given by the relationship

This
ing,

is

2dt litres

dt,

the

of

the sought -for differential equation. Separating variables

we obtain

ln* =

21n(100+0 + lnC
C

or

(100-M)

C=
x

The constant C
100,000.

100,000
-

At

is

the
.

..

found from the


expiration of

ft
f
=^ 3.9 kilograms of

and integrat-

salt.

fact

one

'

that

xvh^n f
0,
the tank

hour,

10,

will

that

is,

contain

_ _

344

[Ch. 9

Differential Equations

2898*. Prove that for a heavy liquid rotating about a vertical


axis the free surface has the form of a paraboloid of revolution.
2899*. Find the relationship between the air pressure and the
2
altitude if it is known that the pressure is 1 kgf on 1 cm at
2
at an altitude of 500 metres.
sea level and 0.92 kgf on 1 cm
2900*. According to Hooke's law an elastic band of length
/ increases
in length klF(k
const) due to a tensile force F.
By how much will the band increase in length due to its weight
if the band is
suspended at one end? (The initial length of
the band is /.)
2901. Solve the same problem for a weight P suspended from
the end of the band.
When solving Problems 2902 and 2903, make use of Newton's
law, by which the rate of cooling of a body is proportional to the
difference of temperatures of the body and the ambient medium.
2902. Find the relationship between the temperature T and
the time f if a body, heated to T degrees, is brought into a room

at constant

temperature

(a degrees).

2903. During what time will a body heated to 100 cool off
and during the first
to 30
if the temperature of the room is 20
20 minutes the body cooled to 60?
2904. The retarding action of friction on a disk rotating in
a liquid is proportional to the angular velocity of rotation. Find
the relationship between the angular velocity and time if it is
known that the disk began rotating at 100 rpm and after one

minute was rotating at 60 rpm.


2905*. The rate of disintegration of radium is proportional
to the quantity of radium present. Radium disintegrates by one
half in 1600 years. Find the percentage of radium that has disintegrated after 100 years.
2906*. The rate of outflow of water from an aperture at
a vertical distance h from the free surface is defined by the

formula

where c0.6 and g is the acceleration of gravity.


During what period of time will the water filling
spherical boiler of diameter 2 metres flow out of
cular opening of radius 0.1
in the bottom.

it

a hemithrough a cir-

2907*. The quantity of light absorbed in passing through


a thin layer of water is proportional to the quantity of incident
light and to the thickness of the layer. If one half of the original
quantity of light is absorbed in passing through a three-metrethick layer of water, what part of this quantity will reach a depth
of 30 metres?

Sec.

345

Higher-Order Differential Equations

10]

2908*. The air

resistance to a body falling with a parachute


to the square of the rate of fall. Find the limit-

is

proportional
ing velocity of
2909*. The
is covered with

descent.

bottom of a tank with a capacity of 300 litres


a mixture of salt and some insoluble substance.
Assuming that the rate at which the salt dissolves is proportional to Ihe difference between the concentration at the given time
and the concentration of a saturated solution (1 kg of salt per 3
litres of water) and that the given quantity of pure water dissolves 1/3 kg of salt in 1 minute, find the quantity of salt in solution at the expiration of one hour.
2910*. The electromotive force e in a circuit with current i,
resistance /? and self-induction L is made up of the voltage drop

Rl and the electromotive force of self-induction


the current

and

Sec.

10.

time

at

= 0.

when

if

e^Esmat

(E

and

L^.

Determine

are constants)

o>

Higher-Order Differentia) Equations

1. The case

of direct integration.

If

then

Miles

2. Cases of reduction of order.


contain y explicitly, for instance,

then, assuming y'

we

p,

I.

If

differential

equation does not

get an equation ot an order one unit lower;

F(x, p

Example

I)

p')-0.

Find the particular solution

of

the equation

that satisfies the conditions

^
Solution. Putting

Solving

we

get

the

= 0,

f/'

#'=p, we have

latter

equation as

when

/ = p',
a

linear

= 0.
whence

equation

in

the

function p,

346

Differential Equations

From

the fact that

y'=p =

[Ch

when x = 0, we have

=^

0,

i.e.,

Cj==0.

Hence,

or

___ ____
2

dx~~

'

whence, integrating once again, w? obtain

Putting

y~

solution

is

when x

0,

we

find

C 2 = 0. Hence,
*2

y^
2)

If

y'=p,

we

p-?->

y"

get

desired

particular

differential equation does not contain

then, putting

the

x explicitly,

an equation

of

an

for

instance,

order

one unit

lower:

Example

Find the particular solution

2.

provided that

/=!,

Solution. Put y'

We

#' =
=p
t

when
then

From

it,

we

the fact that f/'=p

equation

= 0.

tf^p-- and

have obtained an equation

the argument). Solving

of the

our equation becomes

of the Bernoulli

type in p (y

is

considered

find

when r/=l, we have C 1 =

1.

Hence,

P=y Vy^
or

Integrating,

we have
arc cos

Putting

y=l

and *=0, we obtain C 2 = 0,

=C

2,

whence

= cosx

or t/==secx.

Sec.

__

347

Higher-Order Differential Equations

10]

Solve the following equations:


291

2912.
2913.

/=1

2920. yy"

2921.

</"=-|T.

y"=\-y'\

2914. xy"

i/'

yif-y'

2922.

= 0.

= jfy' + y".

/"=

2923. (x

-f-

+ y') = 0.

(I

-p

!)</"

(x-[- 2) y'

+
2915.

// = {/".

*f/"

2925

y'+T^" =^"-

'

'

+ t/")=a/".

2918.

t/'(l

2919.

xy + x0' =

</"+ 1=0.

= 0.

= r/'ln^.
*

2924.

= 0.

2916. <//4- f/"


s
2917. (1 I- X ) </"

+ x+
2

1+x

2926. xy"
/"=
2927. y""-f y"*=l.
solutions for the indicated initial con-

l.

Find the particular


ditions:

2928.

(1

2*/'

-M')z/"
l

2929.

= 0;

l+t/'
2j/t/";
r/=l,
2930. ytf + i/"=*y"\
y=*l,
2931. xy"
0,
t/
t/'
y';

Find the general integrals


2932.
2933.
2934.

2935.

for

for

\
JC

x=

0.

= 0.

following equations:

f/

+ y" =

l;

i/V=l;

l,

xif=V\

(/'

= i.
= 0.
for

forjt

0=1, y'=\
for x=l; y=l
y=
Jt

-\-y'

for

A:

=e

s
.

+ -^'y' = 2 + \nx;y = -^, y' = tor


= -i, y'-l for jc= 1.
= 2 for x = 0.
y'y'' + y'(y\)=*Q; = 2,
=
=
for ^ = 0.
3/Y {/-(-i/" -F 1; y= 2, y'
=
=
foi x = 0.
0; (/=!,
-t-y"-2//''
= and = for * =
^j/' + /'*
yy" = 0; j/=l for

2939. y"(\-\-\nx)

2941.

= 0.

satisfy the indicated conditions:

2940.

forx=l.

for

of the

=3

y"-yy"=ify'.
J
= 0.
f/i/"-hf/' -y'lnj/

2937. yy"
2938.

j/'

yy^Vy^ify'-y'if.
yy' = y" +

Find solutions that


2936.

= 0,
=
y'
=
y'

j/

/-n-ln;

</'

(/

<

2942.
2943.

2944.

/'

J/

-|-

jc

1,

348

(Ch. 9

Differential Equations

2949.

= Q\ y = 0, y' = 2 for = 2.
= 0; y=l, #' = 2 for * = 0.
2jr-3/ = 40 t/= 1, -0 for = 0.
= 0; #=1, y' = for * = 0.
2yy"-\-y*
I/"-*/' -*/; y=-l, y' = l for x=l.

2950.

/ + e>y-2^' = 0;

2945. 2y'
(y'
2946. y'y*+yif

*/'

2947.

A:

Gx)-y"
I

2948.

jc

*/'

*/'*

//-I,

y'

=e

for

^--

= 0; = 0, 0' = for*=l.
=
(H-00')/ (l+0'")0'; 0=1, 0'=1 for x =
(*+l)0" + x0 = 0'; 0=-2, 0'=4 forx=l.

2951.

H-//y"-f

2952.

y'

ff

2953.

Solve the equations:


2954. y'
2955. 0'

=
=

/2

2956.

y"

= 4y

//

2957. yy' y" --= y'*


y"* Choose the integral curve passing through
x
Q.
the point (0, 0) and tangent, at it, to the straight line y
2958. Find the curves of constant radius of curvature.
2959. Find a curve whose radius of curvature is proportional
to the cube of the normal.
2960. Find a curve whose radius of curvature is equal to the
.

+ =

normal.
2961. Find a curve

whose radius

of curvature

is

double the

normal.
2962. Find the curves whose projection of the radius of curvature on the //-axis is a constant.
2963. Find the equation of the cable of a suspension bridge
on the assumption that the load is distributed uniformly along
the projection of the cable on a horizontal straight line. The
weight of the cable is neglected.
2964*. Find the position of equilibrium of a flexible nontensile thread, the ends of which are attached at two points and
which has a constant load q (including the weight of the thread)
per unit length.

2965*. A heavy body with no initial velocity is sliding along


an inclined plane. Find the law of motion if the angle of inclination is a, and the coefficient of friction is p,.
(Hint.
plane.)

The

2966*.
is

frictional force is

We may

consider

to the

that

proportional
square
if the initial velocity

motion

where

ji/V,

^V is the force of reaction of the

the

of the
is

zero..

air resistance

velocity.

in free fall

Find the law

of

Sec. 11]

Linear Differential Equations

349

2967*. A motor-boat weighing 300 kgf is in rectilinear motion


with initial velocity 66 m/sec. The resistance of the water is proportional to the velocity and is 10 kgf at 1 metre/sec. How long
will it be before the velocity becomes 8 m/sec?
Sec. 11. Linear Differential Equations

1. Homogeneous

equations.

The

functions 0i
q>i(x), f/ 2
if there are constants
C,

==(P/iW are called linearly dependent


not alt equal to zero, such that
!/

=
f

q> 2 (A:)

C lf

...,

...

Cn

otherwise, these functions are called linearly independent.


The general solution of a homogeneous linear differential equation

+ P, (x) e/<"- +
=
coefficients P,-(x)
'>

//<>

th continuous

!,

(/

where

are linearly
...,/
(/,, y tt
(fundamental system of solutions).

2. Inhomogcneous

+ P n (x) y =
....

2,

independent

The general

equations.

is

n)

(1)

of the

solutions

form

of

equation

(1)

solution of an inhomogeneous

linear differential equation


(2)

with continuous coefficients

where
and Y

P,-

and the right side

(x)

has the form

f (x)

the general solution of the corresponding homogeneous equation (1)


particular solution of the given inhomogeneous equation (2).
If the fundamental system of solutions (/,, y % ..... y n of the homogeneous
equation (1) is known, then the general solution of the corresponding inhomogeneous equation (2) may be found from the formula
f/

is

is

=C

(x) y,

where the functions Cj(x)


ing system of equations:
'[ (*)

</i

+ C'

(*'

+C

(*)y*

:;w</;

!,

2,

(x)

y2

+C n (x) ya

.... n) are

+ C'n (x)

determined from the follow-

/,

= 0,

+...+c;(o;=o.
0)

(the method of variation of parameters).


Example. Solve the equation
(4)

_ _

350

Differential Equations

[C/i.

Solution. Solving the homogeneous equation

*ir+if'=o

we

get
(5)

Hence,

it

be taken that

may

yl

and the solution

of

equation

Forming the system


the equation

is

(4)

(4)

and

(3)

= \nx
may

taking

t/"+~ =

and # 2 =

be sought in the form

into account

that the reduced form of

we obtain

jt,

Whence

and

>l

and, consequently,

where A and B are arbitrary constants.

2968. Test the following systems

of

functions for linear rela-

tionships:

a) x,
b)

ing

x\

1,

d) x,

x+1,

x +2;

2969.

Form

a linear

a)
b)
c)

d)

g)

h)

x'\

e *, e**\

sin *, cos A:, 1;


2
2
sin x, cos *, 1.

homogeneous

differential

equation, know-

fundamental system of equations:

= sin x,

y 2 = cos x\
==xe*\
y
2
y^e*.
2
y^x* # 2 = *

e*

f)

c) 0,

its

x\

e) *,

1;

2x

<

f/,

= ^x

y*

= ^x

sin ^,

f/ 8

= ^ cos

A:.

2970. Knowing the fundamental system


homogeneous differential equation

find its particular solution

y that

of solutions of a linear

satisfies the

initial

conditions

Linear Differential Equations with Constant Coefficients

Sec. 12]

351

2971*. Solve the equation

its

knowing

particular solution y

-^.

2972. Solve the equation

x*(\nx-l)y"-xy' 4-y = 0,
particular solution y =x.

knowing its
By the method of variation of parameters, solve the following
inhomogeneous linear equations.
}

2973. **(/"
3;t
xy'
2
2974*. x*y" + xy'
y=x
2975. y'" -f (/'-sec A:.
Sec.

12. Linear Differential

Equations of Second Order

with Constant Coefficients

t. Homogeneous equations. A second-order linear equation with constant


and q without the right side is of the form

coeflicients p

U)
k

If

and

fc

are roots of the characteristic equation


.Q

then the general solution


three ways:
1)

2)
3)

of

equation

(/-CVV + C.e*-* 6, and


= *,;
y-eV(C, + CV)
*,
-^(^cnspx-HC^mpA)
if

kz

(1)

is

(2)

\vritten in one

are real and

/?,

of the following

k.\

if

(/

2. Inhomogeneous

equations.

if

*,=a + p

and * a -=a

The general solution

of a

pi (p 76 0).

linear inhomoge-

neous differential equation


y"

+ py' + w=fW

(3)

be written in the form of a sum:

may

where y

the general solution

of the corresponding equation (I) without


determined from formulas (1) to (3), and Y is a particular
solution of the given equation (3).
The function Y may be found by the method of undetermined coefficients
in the following simple cases:
a
e *P n (x), where P n (x) is a polynomial of degree n.
1. f (x)
If a is not a root of the characteristic
equation (2), that is, (p (a) 96 0,
then we put Y
e**Q n (x) where Q n (x) is a polynomial of degree n with
undetermined coeflicients.
If a is a root of
the characteristic equation (2), that is, <p(a)
0, then
r ax
ritfht

is

and

side

Y=x

2.

Qn

/ (*)

\\here r is the multiplicity of the root


*
[P n (*) cos bx+ Q m (x) sin bx\.

(x) t

a(r=l

or r

= 2).

352

__

[Ch. 9

Differential Equations

If

cp(a

^ 0,

bi)

then we put

Y = e ax

[S N (x) cos bx

+ T N (x) sin bx],

where S^(x) and Tu(x)


But if cp(a
&/) = 0, then

are polynomials of degree

K = x re ax
where
r=l).
is

fc

the

is

[Stf (x) cos 6*

+ T N (x) sin to]

the roots a

of

multiplicity

N-max

bi

{n,

m},

second-order

(for

equations,

In the general case, the method of variation of parameters (see Sec. 11)
used to solve equation (3).
2
Example 1. Find the general solution of the equation 2y" y' y 4xe *.
2
&
l=u has roots fc,~l and
Solution. The characteristic equation 2&

The general solution

(first

type)

homogeneous equation

e* + C 2e
The right side of the given equation is/ (x) =
Y e zx (Ax + B), since n=l and /=0. Difleren(x). Hence,
twice and putting the derivatives into the given equation, we
is

==C

f/

=4xe zx =t ax P n
tiating
obtain:

the corresponding

of

%,** (4 A X

+ 45 + 4^) __ e i* (2Ax + 25 + A)

2
e *

(Ax H- B)

4xe zx

zx

and equating the coefficients of identical powers of x arid


Cancelling out e
the absolute terms on the left and right of the equality, we have bA=4 and
4
28
744-5fl

= 0,

whence 4

and 5

-=-

2A
Thus, K^

oH

-g-*

anc^

-.

Jo

ne gei] eral solution of the given equation

Find the general

+ =

is

solution of the equation y*


xe*
y
2y
2
characteristic equation k
has a double root
2/f-f- 1
x
ft=l The ri^ht side of the equation is o! the form f(x)xe
here, 0=1
and n=-l. The particular solution is Y =x*e* (Ax
B), since a coincides \nHth
2.
the double root k=-\ and, consequently, r
Diilerentiating Y twice, substituting into the equation, and equating the

Example

2.

Solution. The

coefficients,

we obtain

/l

fl

equation will be written in the

= 0,

Hence, the general solution of the given

form
*

Find the general solution of the equation */*-f y=


2
The characteristic equation
and
-j-l=r() has roots /?,
The general solution ot the corresponding homogeneous equation
3, where a~0 and P = l| be*

Example
fc

3,

Solution.
i.

will |see

The right side

fe

is

of

the form

/'

Sec.

Linear Differential Equations with Constant Coefficients

12]

0, 6=1, P n (jc)=0,
particular solution Y,

where a

#=1,

Q OT

= *.

(*)

To

353

this side there corresponds the

= 0,

fc=l, r=l).
twice and substituting into the equation, we equate the
coefficients of both sides in cos*, xcosx, sin*, and xsmx. We then get four
44 = 1, from which we deter0,
0, 4C = 0, -25 + 20
equations 2A + 2D
1
1
X2
X
mine A
cos * -f- -j- sin *.
0, C
0, D
Therefore,
(here,

Differentiating

=
=

3. The

is

= C, cos + C

jc

x2
2

sin

is

principle of superposition of
the sum of several functions

and K/(/

l,

then the

sum

3,

y'+py'+w^-fiW
y
is

the solution of equation

A:

sin *.

-7-

solutions.

If

the right side of equa-

corresponding solutions of the equations

are the

.., n)

cos

-j-

tion (3)

2,

44

K=

The general solution

(<

i.

2 ..... n).

= Y + Y +...+Y n
n

(3).

Find the general solutions of the equations:


2982. y" + 2y'
2976. tf
5y'6y Q.
0.
2983.
2977. if
4y'
9y
2984. y" + ky
2978.
0.
2985.
2979. iT
y
0.
2980. ^_2i/ +2j/
2981.
+ 40' +130 0.

yy'^Q.
=
+

y=

=
=

Find the particular solutions that satisfy the indicated conditions:

2987. y"5tj'-\-4y
2988. y"+ 3tf' +20

2989. 0"
2990. 0^
2991.

= Q\
= 0;
= 0,

y=5

= 8 for * =
for jc0.
y=5
0' =
= 2 for x = 0.
t

y'

1,

+ 40 = 0;
+ 20' = 0; 0=1, 0'=0 for ^ =
= a, 0' = for x = 0.
f

/=;

2992. 0"
2993. 0"

+ 30'=0; =
= 0; =
+
f

ji

for x
for

jc

=
=

and
and

=
=

for

x=l.

type of particulai solutions


inhomogeneous equations:
2994.

a)

Indicate

0"-40 = A:V

b) 0"

12-1900

90

the

x
;

= cos 2x\

= 3.

for

for

the given

_ _

354

Differential Equations

[Ch. 9

= sin 2x + e**\
x
d) y" + 2y' + 2y = e sir\x\
- 5</' + 60 = (x* + l)e* + xe**\
e)
y"2y' + 5# = xe* cos 2x #V sin 2x.

c) y"

4y' -f 4y

</"

f)

Find the general solutions of the equations:

=
/ ^ +# = +
^ + + = ^.
+
/

2995. y"
jc
4i/'
4(/
2996.
*'
6.
2997.
2^'
^
2998.
8^ 7^=14.
2999. y"y^e*.
3000.
3001.
3002.
3003.
f
3004. t/'
3005. y"
2y'-{ 5y
3006. Find the solution of the equation y"
0.
satisfies the conditions y=l, */'= 1 for x

+ 4y =

sin x that

Solve the equations:


W2y

3007.
2)

^-2

+ x=/l
o)

Consider

sinp/.

the

cases:

1)

p-co.
3008.
3009.
3010.
3011.
3012.
3013.
3014.
3015.
3016.

3017.

/
t/"

y"
y"

2y'

y"

2(/'

y"-\-y'

8y = e
= 5x + 2e x

^_y'^2;c-~l

8cos2*.
.

3e

x
.

+ 2r/ + y = ^ + ^ x x
2y + lQy=s\r\3xi-e
/_4f/' + 4f/ = 2^ +
if
3y' =
/

{/"

f/"

3018.
3019. Find the solution to the equation
that satisfies the conditions /=-, f/ /==l for ^

Solve the equations:


3020. (/"
3021. j/"
3022.
3023. tf
3024.
3025.

y = 2xsmx.
2x
4(/==g

sin2A:.

/ H- 9y = 2x sin

2y'=e

= 0,

tx

o>;

Sec.

Linear Differential Equations with Constant Coefficients

12]

=
+
=
=

355

3026. tf2y'
3y x(l+e' ).
3x 2xe*.
3027. y"
2y'
x
3028. */"
4y'+4y xe*
2xe-'*
3029. y"
3y
2y'
(x^ l)e*.
3030*. y"
y 2x cos x cos 2x.
3031. t/'
2y 2xe*(cosx
sinx).

+
+ =
=

Applying the method

variation

of

following equations:
3032. y" + y
ianx.
3033.

3034.
3035.

=
y" + y = cot x.
= -.
y"
2y'
y" + 2y' + y =

3036.

of

parameters, solve

<

the

3037.
3038. a)

-(-*/

</"

b) y"

y = tanh

A:.

x
2y = 4x*e \

3039. Two identical loads are suspended from the end of a


spring. Find the equation of motion that will be performed by
one of these loads if the other falls.
Solution. Let the increase in the length of the spring under the action
in a state of rest be a and the mass of the load, m. Denote by x
the coordinate of the load reckoned vertically from the position of equilibrium in the case of a single load. Then
of

one load

where, obviously, k
tion

is

x=C

and

d*
-77
Tt

=
:

cos

when

I/
/

= 0;

and, consequently,
- 1

+C

I/

sin

whence C

=a

t.

7*7*

The

^ ~" ^ *
initial

The

general

conditions yield

solu-

Jt

=u

and C a = 0; and so

3040*. The force stretching a spring is proportional to the


increase in its length and is equal to 1 kgf when the length
increases by 1 cm. A load weighing 2 kgf is suspended from the
spring. Find the period of oscillatory motion of the load if it
is pulled downwards slightly and then released.
3041*. A load weighing P
4 kgf is suspended from a spring
and increases the length of the spring by 1 cm. Find the law
of motion of the load if the upper end of the spring performs
a vertical harmonic oscillation (/
2sin30/ cm and if at the
initial instant the load was at rest (resistance of the medium is

neglected).
12*

356

_ _

(Ch. 9

Differential Equations

is attracted by each of two


3042. A material point of mass
centres with a force proportional to the distance (the constant
of proportionality is k). Find the law of motion of the point
knowing that the distance between the centres is 26, at the initial instant the point was located on the line connecting the
centres (at a distance c from its midpoint) and had a velocity

of zero.

3043. A chain of length 6 metres is sliding from a support


without friction. If the motion begins when 1 m of the chain
is hanging from the support, how long will it take for the entire
chain to slide down?
3044*. A long narrow tube is revolving with constant angular
velocity o> about a vertical axis perpendicular to it. A ball inside the tube is sliding along it without friction. Find the law
of motion of the ball relative to the tube, considering that
a) at the initial instant the ball was at a distance a from

the axis of rotation; the initial velocity of the ball was zero;
b) at the initial instant the ball was located on the axis of
rotation and had an initial velocity v 9
.

Sec.

13.

Linear

Differential

Equations

of

Order

Higher

than

Two

with

Constant Coefficients

1. Homogeneous equations. The fundamental system of solutions y lt


Un f a homogeneous linear equation with constant coefficients
n
y<
+ a iy n + + an _,y' +a n y =
(1)

#t

>

<

is

constructed on the basis of the character of the roots of the

characteristic

equation
Q.

(2)

1) if k is a real root of the equation (2) of multiplicity m, then to


this root there correspond
linearly independent solutions of equation (1):

Namely,

2)

then

if

to

the

p/ is a pair of complex roots of equation (2) of multiplicity


latter there correspond 2m linearly independent solutions

equation (1):
*x
y l e cos PX, #,

= e*x sin px,

yt

= xe* x cos PX,

2. Inhotnogeneous equations.
equation

y4

= xe* x sin PX,

m,
of

...

particular solution of the inhomogeneous


(3)

is

sought on the basis of rules 2

and 3

of Sec.

12.

Sec.

Euler's Equations

14]

__

357

Find the general solutions of the equations:


3058. y /
13</"+12</'=0.

3045. y'"

3048. y' v
2J/"
3t"
3049. y'"
3050. y'v + 4y

= 0.

'

w'v' + ^O.
'

*054.

w' 1

sss-

^T

'

306 >- y' v


3062 y'"iJ

ai/ = b.

3060.

3053.

n(n-l) ,.

= x'

MS,---..

SB:

3067. Find the particular solution of the equation

y'"+2y"-{-2y'+y = x
that satisfies the initial conditions y (0)
14.

Sec.

Euler's

= y' (0) = y" (0)=-=0.

Equations

linear equation of the form


n

y^f(x)

(I)

a, b, A ..... <4,,_,, A n are constants, is called Enter's equation.


Let us introduce a new independent variable /, putting

where

ax

+ b-^e

1
.

Then

and

Euler's

is

equation

transformed

coefficients.
z

Fxample 1. Solve the equation x y"


Solution. Putting x^e*. we get

into

linear equation

+ xy' +{/ =

1.

==e

Tr

~Sx

dx~z==e

(~di*~~~di)'

Consequently, the given equatioi takes on the form

whence

= C, cos + C, sin +
y = C, cos (In *) + C a sin (In x) +
y

or

1.

with constant

358

_ _

[Ch. 9

Differential Equations

For the homogeneous Euler equation

the solution

(2)

be sought in the form

may

</

= **.

(3)

found from (3), we get a characteristic equaexponent k.


If k is a real root of the characteristic equation of multiplicity m then to it
correspond m linearly independent solutions
m- 1
k
2
Putting into (2)
tion from which

y'

(/,

...,

we can

(n}

find the

0i
If

= **.
a

is

p*

correspond

2m

0,

= **-ln*.

*/,

= ** (In*)

..., y m

y 4 =*Mnx.sin(plnx) .....

2.

We

into the given


characteristic equation

Solving

find

y^C
Solve the equations:

3069.

3070.

*'g + 3*| + = 0.
*V xy' 3(/ = 0.
xY-|-x(/'4-4(/ = 0.
<,

3071.
3072.

3073. tT

3074.
3075.

3076.

there

cos (p

+4l/=0.

equation, after

Hence, the general solution will be

3068.

it

= *a In x cos (P In *),

^m-i^*" (In*)*"

3Xy'

we

to

put

Substituting

it

Solve the equation

X *y''
Solution.

(\nx)

m, then

pair of complex roots of multiplicity


linearly independent solutions
(/ 8

Example

=x

^y_

46-1-4

= 0.

cancelling

out x k

we

get

the

Sec.

15]

Systems

of Differential

359

Equations

3077. Find the particular solution of the equation

+ y = 2x
conditions y = 0,

*V
that satisfies the initial

Sec.

xy'

15. Systems of Differential

Method

*/'

Equations

To find the solution, for


first-order differential equations, that

of elimination.

system of two
form

them with respect

the desired functions,


have, for example,

we

first

equation of the system

(1)

to x.

Determining z from the

instance, of
is,

We

solved for the derivatives

of

when *=1.

of

normal

system

differentiate

of

the

one of

and substituting the

value found,
A.. \

(3)

into equation (2), we get a


tion u. Solving it, we find

equation with one unknown func-

second-order

(4)

where C, and C 2 are arbitrary constants. Substituting function (4) into formula (3), we determine the function z without new integrations. The set of
formulas (3) and (4), where y is replaced by \|>, yields the general solution
of the system (1).

Example. Solve the system

+'-'-T*
Solution.

We

differentiate the first equation

with respect to

x:

^ + 2^dx^+ 4^-4.
dx*^

From

the

first

from the second

equation

we

will

dx

we determine

have -^
ax

-5-

&

**

-T-

+*+

-;

l+4x
-75-

&

dy

TT"T
ax

2y

and then
j

Put ting

and j- into the equation obtained after differentiation, we arrive at a secondorder equation in one unknown y:

360

[Ch. 9

Differential Equations

Solving

it

we

find:

and then

We

do likewise

can

in

the case

of

system with a larger number of

equations.

Solve the systems:


dy

3078.

=z

JJ

3085.
dx
r/

= 0, z =

when

je

= 0.

when

= 0.

3079.
3086.

3080.

dz

= 0, y=\

dT
dx

3081.

4/_//
3087.

dy
dz

3088*.
dx

xy~
c)

3082.

da

__

dy __dz_
x-\-y~~~ z
dy __

yz zx

'

dz

xif

isolate the integral curve passing through the point (1, 1,


2).
dy

3089.

3083.

3084.

dz

3090.

.Sec.

Integration of Differential Equations by Power Series

16}

361

3091**. A shell leaves a gun with initial velocity u at an


angle a to the horizon. Find the equation of motion if we take
the air resistance as proportional to the velocity.
3092*. A material point is attracted by a centre
with a
force proportional to the distance. The motion begins from point A
at a distance a from the centre with initial velocity
perpendicular to OA. Find the trajectory.
by Means

Sec. 16. Integration of Differential Equations

of

Power Series

not possible to integrate a differential equation with the help of


elementary functions, then in some cases its solution may be sought in the
If

is

it

form of a power

series:
00

'(*
y=2
n=

n
*o)

0)

The

undetermined

series (1) into the


of the binomial x

are found by putting the


coefficients c n (n
\, 2, ...)
equation and equating the coefficients of identical powers
x on the left-hand and right-hand sides of the resulting

equation.

We

in the

can also seek the solution of the equation

form

of the

Taylor's series

y(*)

^ ^^ (*-*)"

(3)

and the subsequent derivatives y (n) (x )


where y(x Q) = y Q
t/ )
y' (x ) = f (*
are
found
successively
by differentiating equation (2) and by
(n- 2, 3, ...)
,

putting X Q in place of x

Example

Solution.

1.

Find the solution

We

put

of the equation

whence, differentiating, we get


y"

= 2.\Ct + 3.2c x+...+n(n-\)cn x n -* + (n+l


s

+ In + 2)(rt-t-l) <:+,*"+...
Substituting y and y" into the given equation,

Collecting

powers

together, on

of x

the

left

of this

and equating to zero the

we

arrive at the identity

equation, the terms with identical


of these powers, we will

coefiicients

362

Differential Equations

[Ch.

have

cs

^~

and so

forth.

Generally,

3.4-6. 7-...
(k

=1,2,

3,

...).

Consequently,

(X*

c =f/
and c = y'Q
Applying d'Alembert's

where
for

< x < + oo

Example

2.

3.4-6.?..

.-3* (3fc+

1)

+ '" )*

(4)

oo

X^

X1

+ 3T4 + 3.4.6.7 + '" +


test,

it

is

readily

seen

that series (4) converges

Find the solution

of the

equation

y'

Solution.

We

We

put

= + = Differentiating equation y' = x + y, we succes/=! + ^=1 + = 2, y'"=y\ /o" = 2, etc. Consequently,

have y =\, i^

sively find

l.

(/',

For the example

at

hand, this solution

*-l
The procedure

may

x) or

be written in final form as

= 2e*

x.

similar for differential equations of higher orders. Testing the resulting series for convergence is, generally speaking, complicated
and is not obligatory when solving the problems of this section.
is

With the help of power series, find the solutions of the equations for the indicated initial conditions.
In Examples 3097, 3098,
3101, test the solutions
3099,
obtained for convergence.
2
3093. y'
y x
y
3094. y'
x
1; y
2y

= +
= 2 for * = 0.
= +
= y for x=l.
= y for x==a
0' = / + *;
= for * = 0.
y' = x*
y*\ #
for
(1
x)y' = l+x
y\ y =
\

3095.
3096.

3097.

f/

Problems on Fourier's Method

Sec. 17]

363

= for x = 0.
=
for * = 0.
0\ 0=1, #'
+ = 0; y=l, */'=0 for =
+ = 0; 0=1, 0'=0 for x =
= 0; * = a; ~ = for = 0.

= 0;

3098*.
3099.
3100*.

/'

3101*.

= 0,

y'

Jt

*/

*/

3102.

Sec. 17. Problems on Fourier's Method

To

find the solutions of a linear

partial differential equation

homogeneous

by Fourier's method, first seek the particular solutions of this special-type


equation, each of which represents the product of functions that are dependent
on one argument only. In the simplest case, there is an infinite set of such
solutions M (tt=l, 2,...), which are linearly independent among themselves
in any finite number and which satisfy the given boundary conditions. The
desired solution u is represented in the form of a series arranged according
rt

to these particular solutions:

u=

Cn u n

(1)

C n which remain undetermined

The coefficients
conditions.

Problem. A transversal displacement u


u(x
with abscissa x satisfies, at time *, the equation

from the

are found
t)

initial

of the points of a string

_
~~ a
where a
string).

-?

(T Q

the

is

Find the form

tensile

of the

and Q

force

string

(2)

dx*

dt*

at

time

the

is
if

its

linear

ends

x=

density

of the

and * = / are

2
Fig.

fixed

and

=~*

at the initial

(/

Solution.

x) (Fig.
It

that satisfies the

is

instant,

107) and

required

its

= 0,

107
the string had

the form of a parabola

points had zero velocity.

to find the

solution

= u(x,

t)

of

equation

(2)

boundary conditions
a(0,

0-0,

!!(/,

0=0

(3)

_ _

364

\Ch. 9

Differential Equations

and the

initial conditions

(*,0)

We

^ *(/-*),

;<*, 0)

= 0.

(4)

seek the nonzero solutions of equation (2) of the special form

=X

u
Putting

this expression into

(x)T(t).

equation

and separating the variables, we get

(2)

ZL(0--*1W
a 2 7tO~~

(5)'
l

'

(x)

Since the variables x and t are independent, equation (5) is possible only
the general quantity of relation (5) is constant
Denoting this constant
X 2 we find two ordinary differential equations:

when
by

=Q

)*-T(t)

and X"

(x)

+ K*X

(x)

= 0.

Solving these equations, we get

= A cos a\t + B sin aKt,


(*) = C cos Kx + D sin X*.

(t)

X
where A, B, C,

From

are

X(0) =

cannot be equal to

this reason,

XA

= -p

we do
To

Let

constants.

arbitrary

we have

(3)

(since

sinX/

For

condition

where k

is

and

zero

at

an integer.

determine the constants.


and
X(/) = 0; hence, C =
the same time as C is zero).
us

It

will readily be seen that

not lose generality by taking for k only positive values


every value h k there corresponds a particular solution

kan

kan

Ak
(, cos-j-

whose sum obviously

equation

A k and B k

choose the constants


satisfy the initial conditions

knx

kant\

knx

-7- + ^sm-y-J sm-j-

cos

satisfies

We

and the
so that the

(2)

boundary conditions

sum

of the series

Since

(4).

CD

du

& =2*
it

kan (

~\~ A

follows that, by putting

= 0,

sm

kant

we obtain

kant\

-p + ^cos-y-J

and
0)

2, 3,...).

(3).

kant

tj sin-y-

Ihat satisfies the boundary conditions


We construct the series

(k=\

kan

sin

knx

-^

(3).

should

Sec.

17]

Problems on Fourier's Method

Hence, to determine the


a Fourier series,
.

du

..

function

A k and B^

coefficients

is

it

365

necessary to expand in

= ^-

sines only, the function u(x, 0)

in

_
x

(lx)

and

the

^ ^n

(x, 0)'

0.

at

From

familiar formulas (Ch. VIII, Sec. 4,3)

we have

/
ft

if

&

is

Ak

odd, and

if

&

is

knx

32h

even;
/

kast _

The sought-for solution

=
*

will

jix

fA
Osin
,

d*

T- J

A
= 0,

be
cos
/

sin

3103*.
its

ends,

sin

At the initial instant


x=
and * = /, had

and the points

form of the string


3104*. At the
string

0<jt</

at

A
= 0.

fl*

time

initial

the

form

string, attached at
of the sine curve

had zero velocity. Find the

it

t.

time

= 0,

of

(2/i-f \)jtx

= 0,

receive a velocity -~

the

1.

points of

a straight

Find the form

of the

l are
and x
string at time t if the ends of the string #
fixed (see Problem 3103).
3105*. A string of length /=100 cm and attached at its ends,
and *=-/, is pulled out to a distance A
2 cm at point
x

*=;50 cm

then released without any


impulse. Determine the shape of the string at any time /.
3106*. In
longitudinal vibrations of a thin homogeneous
and rectilinear rod, whose axis coincides with the jr-axis, the
of a cross-section of the rod with
t)
displacement u = u(x
abscissa x satisfies, at time /, the equation
at

the

initial

time,

and

is

c^u_
~~ a 2

d/ 2
f
where a

<Pu
dx*

(E

is

Young's modulus and Q

is

the density of the

rod). Determine the longitudinal vibrations of an elastic horiand pulled


zontal rod of length /= 100 cm fixed at the end *
back at the end *=100 by A/
l cm, and then released without

impulse.

366

_ _
'

Differential Equations

[C/i.

3107*. For a rectilinear homogeneous rod whose axis coincides


with the Jt-axis, the temperature u = u(x, t) in a cross-section with
abscissa x at time /, in the absence of sources of heat, satisfies
the equation of heat conduction
di

where a is a constant.
for any time t in a rod

Determine the temperature distribution


of length

100

cm

if

temperature distribution
u(x, 0)

= 0.01

A;

(100

x).

we know

the initial

Chapter

APPROXIMATE CALCULATIONS

Sec.

1.

Operations on Approximate Numbers

1. Absolute

The absolute error of an approximate number a which


number A is the absolute value of the difference between
them. The number A, which satisfies the inequality
error.

replaces the exact

is called the limiting


the limits a

absolute

A^/l^a + A

error.

or,

more

The exact number A


A
briefly, A=a

is

located within

2. Relative error. By the relative error of an approximate number a


0) we understand the ratio of the absolute
replacing an exact number A (A
error of the number a to the exact number A. The number 6, which satisfies

>

the inequality

\*-l ^*
-,

(2)

is

called the

of the

limiting relative error

actual practice

A^a,

we

number

often take the

number

approximate
6=

for

a.

Since in

the

limiting

relative error.

3. Number of correct decimals. We say that a positive approximate


number a written in the form of a decimal expansion has n correct decimal
places in a narrow sense if the absolute error of this number does not exceed
half unit of the nth decimal place. In this case,
take, for the limiting relative error, the number

one

where k

is

the

first

significant
/

is

known

that

6^

I
,

i
,

places

definitely

has

narrow

77:

we can

number

a.

And

conversely,

it

if

then the number a has n correct decimal

1)

n correct

the absolute error


of the last decimal

If

unit

the

\10/
meaning

2(k-\in

>

of the

digit
\-i

when n

of

decimals

the
in

word.

the

In

particular,

narrow meaning

if

number a

the

"if I To)
approximate number a does not exceed a
for example, are numbers resulting
place (such,

of

an

from measurements made to a definite accuracy), then it is said that all


decimal places of this approximate number are correct in a broad sense. If
there is a larger number of significant digits in the approximate number,
the latter (if it is the final result of calculations) is ordinarily rounded off
so that all the remaining digits are correct in the narrow or broad sense.

[Ch. 10

Approximate Calculations

368
Henceforth, we shall
not otherwise
(if

assume that

all
digits in the initial data are
the narrow sense. The results of intermediate calculations may contain one or two reserve digits.
We note that the examples of this section are, as a rule, the results of
final calculations, and for this reason the answers to them are given as
approximate numbers with only correct decimals.
4. Addition and subtraction of approximate numbers. The limiting absolute error ot an algebraic sum of several numbers is equal to the sum of
the limiting absolute errors of these numbers. Therefore, in order to have,
in the sum of a small number of approximate numbers (all decimal places
of which are correct), only correct digits (at least in the broad sense), all
summands should be put into the form of that summand which has the
smallest number of decimal places, and in each summand a reserve digit
should be retained. Then add the resulting numbers as exact numbers, and
round off the sum by one decimal place
If we have to add approximate numbers that have not been rounded off,
they should be rounded off and one or two reserve digits should be retained.
Then be guided by the foregoing rule of addition while retaining the appropriate extra digits in the sum up to the end of the calculations.
Example 1. 215.21 -f- 14.182 -f 21 .4-215.2(1)
14.1(8)4-21 4-= 250.8.
The relative error of a sum of positive terms lies between the least and
greatest relative errors of these terms.
The relative error of a difference is not amenable to simple counting.
Particularly unfavourable in this sense is the difference of two close numbers.
Example 2. In subtracting the approximate numbers 6 135 and 6.131 to
four correct decimal places, we get the difference
004. The limiting relative

correct

in

stated)

0.001
error

6=

is

.J.

+1

0/XM

=-4 = 0.25.
j

..

,t

0.004

Hence,

not one of the

decimals

it
difference is correct. Therefore,
is
always advisable to avoid
subtracting close approximate numbers and to transform the given expression,
if need be, so that this undesirable operation is omitted.
5. Multiplication and division of approximate numbers. The limiting
relative error of a product and a quotient of approximate numbers is equal
lo the sum of the limiting relative errors of these numbers Proceeding from
Ihis and applying the rule for the number of correct decimals (3), we retain
in the answer only a definite number of decimals
Example 3. The product of the approximate numbers 25.3-4.12=104.236.
Assuming that all decimals of the factors are correct, we find that the
limiting relative error of the product is

of

the

Whence
result,

if

number

the
it

is

correctly, 25

final,

mth power

of

^0.01 +^0.01=^0.003.
decimals of the product is three and the
be written as follows: 25.3-4 12=104, or more

correct

should

3-4.12= 104 2

6. Powers and
of the

of

0.3.

roots of approximate numbers.

an approximate number a

relative error of this number


The limiting relative error of
Is

the

th

the

mth root

is

of

The limiting relative error


equal to the m-fold limiting

an approximate number a

part of the limiting relative error of the

number

a.

7. Calculating the error of the result of various operations on approximate numbers. If Aa lf ... Aa,, are the limiting absolute errors of the appro,

Sec.

Operations on Approximate Numbers

1]

xitnate

numbers a t

an

then the limiting absolute error

S = /(a t,

may

369

AS

of the

resuU

..., a n )

be evaluated approximately from the formula

The limiting relative

error

is

then equal to

,**-

*Ji/r

".++
=

Example 4. Evaluate S ln (10.3+ V^4. 4 ); the approximate numbers


10.3 and 4.4 are correct to one decimal place.
Solution. Let us first compute the limiting absolute error AS in the
1
r
A& \
ln (a+ V ~b),
AS
Aa
We have
general form: S

Aa=A6=i=2Q; 1^4.4
the approximate
is

then equal to

2.0976...;

number y

is

'

^SOTT^JQ

^TT;

we

leave

equal to

we can

be sure

2.1,

=-

since the relative

-r=

error of

the absolute error

tne first decimal place. Hence,

U UUi>
0005
-

20

'

"2"

Thus, two decimal places will be correct.


Now let us do the calculations with one reserve decimal:
2.517.
log (10. 3+ |/4 4) =5= log 12 4-1.093, In (10 3+ V 4.4)=^ 1.093-2.303
And we pet the answer: 2 52
8. Establishing admissible errors of approximate numbers for a given
error in the result of operations on them. Applying the formulas of 7 for
the quantities AS or 6S given us and considering all particular differentials

U-M

quantities \-~Ac** or the M


equal, we calculate the
\dak
\da k
\f\
absolute errors Aa lt ...
Aa^, ... o the approximate numbers a t

admissible

..

,a n

...

that enter into the operations (the principle of equal effects).


It should be pointed out that sometimes when calculating the admissible
errors of the arguments of a function it is not advantageous to use the
principle of equal effects, since the latter may make demands that are
practically unfulfilable In these cases it is advisable to make a reasonable
redistribution of errors (if this is possible) so that the overall total error does
not exceed a specified quantity. Thus, strictly speaking, the problem thus

posed

is

indeterminate.

Example

5.

The volume

off a circular cylinder

(equal

to

=~2 R*

2R)
tan a.

at

an

by

angle

To what

of a

segment", that is, *a solid cut


passing through the diameter of the base
to the base, is computed from the formula
"cylindrical

plane

degree

of

accuracy

should

we measure

the radius

_ _

[Ch. 10

Approximate Calculations

370

#=s:60 cm and the angle of inclination a so that the volume of the cylindrical
segment is found to an accuracy up to 1%?
Solution. If AV, A/? and Aa are the limiting absolute errors of the
quantities V, R and a, then the limiting relative error of the volume V that

we

are calculating

is

3A/?

2Act

R "Nn^a

...

We

assume

3A

-^ < m
1

and

_<2Aa

100"

Whence

cm
-=1
60

sin

mm;

2a

Thus, we ensure the desired accuracy in the answer to


and the angle of inclination a to 9'.
Ihe radius to 1

mm

1%

if

we measure

3108. Measurements yielded the following approximate numbers


that are correct in the broad meaning to the number of decimal
places indicated:

1207'14"; b) 38.5 cm; c) 62.215 kg.


their absolute and relative errors.
3109. Compute the absolute and relative errors of the following approximate numbers which are correct in the narrow sense
to the decimal places indicated:
a)

Compute

a) 241.7; b) 0.035; c) 3.14.


3110. Determine the number of correct (in the narrow sense)
decimals and write the approximate numbers:
a) 48.361 for an accuracy of 1%;
b) 14.9360 for. an accuracy of l%\
c) 592.8 for an accuracy of 2%.
3111. Add the approximate numbers, which are correct to the
indicated decimals:
3.10
0.5;
a) 25.386 + 0.49
2
41.72
0.09;
b) 1.2-10
3.124.
c) 38.1+2.0
3112. Subtract the approximate numbers, which are correct
to the indicated decimals:

a)

148.1-63.871;

b)

29.7211.25;

c)

34.22-34.21.

3113*. Find the difference of the areas of two squares whose


measured sides are 15.28 cm and 15.22 cm (accurate to 0.05 mm).
3114. Find the product of the approximate numbers, which
are correct to the indicated decimals:
a) 3.49-8.6; b) 25.1-1.743; c) 0.02-16.5. Indicate the possible
limits of the results.

Operations on Approximate Numbers

Sec. /]

371

3115. The sides of a rectangle are 4.02 and 4.96


(accurate
cm). Compute the area of the rectangle.
3116. Find the quotient of the approximate numbers, which
are correct to the indicated decimals:

to

a)

5.684

5.032; b) 0.144

1.2; c)

216:4.

3117. The legs of a right triangle are 12.10 cm and 25.21 cm


(accurate to 0.01 cm). Compute the tangent of the angle opposite
the first leg.
3118. Compute the indicated powers of the approximate
numbers (the bases are correct to the indicated decimals):
a) 0.4158';

b) 65.2'; c)

3119. The

side

of

1.5

2
.

square

is

45.3

cm

(accurate to

mm).

Find the area.


3120.

Compute the values

of

the

roots

(the

radicands are

correct to the indicated decimals):


a)

1/27715; b) j/65^2

c)

KsTT.

3121. The radii of the bases and the generatrix of a truncated


0.01 cm; r= 17.31 cm
23.64 cm
0.01 cm; /
cone are #
3.14. Use these data to compute the
0.01 cm; ji
10.21 cm
total surface of the truncated cone. Evaluate the absolute and

relative errors of the result.

3122. The hypotenuse of a right triangle is 15.4 cm


0.1 cm;
0.1 cm. To what degree of accuracy
one of the legs is 6.8 cm
can we determine the second leg and the adjacent acute angle?
Find their values.
3123. Calculate the specific weight of aluminium if an aluminium cylinder of diameter 2 cm and altitude 11 cm weighs
93.4 gm. The relative error in measuring the lengths is 0.01,
while the relative error in weighing is 0.001.

3124.
to 221

current if the electromotive force is equal


and the resistance is 809 ohms
1
ohm.
of oscillation of a pendulum of length / is

Compute the

volts

volt

3125. The period


equal to

where g is the acceleration of gravity. To what degree of accuracy


do we have to measure the length of the pendulum, whose period
is close to 2 sec, in order to obtain its oscillation period with a
relative error of 0.5%? How accurate must the numbers n and g

be taken?
3126. It is required to measure, to within 1%, the lateral
surface of a truncated cone whose base radii are 2 m and 1 m,
and the generatrix is 5 m (approximately). To what degree of

372

_ _
Approximate Calculations

[Ch. 1Q

accuracy do we have to measure the radii and the generatrix and


how many decimal places do we have to take the number n?
3127. To determine Young's modulus for the bending of a
rod of rectangular cross-section we use the formula

to

*P ~~ 1
4

'

flL
'

d*bs

where / is the rod length, b and d are the basis and altitude ol
the cross-section of the rod, s is the sag, and P the load. To
of accuracy do we have to measure the length / and
the sag s so that the error E should not exceed 5.5%, provided
that the load P is known to 0.1%, and the quantities d and 6
cm?
are known to an accuracy of 1%,
cm,

what degree

/50

s2.5

Sec. 2. Interpolation of Functions

1. Newton's interpolation formula. Let * x lt ... xn be the tabular values of an argument, the difference of which h
0,l
Ax; (Ax/
*,-+,
x/; i
., y n are the correspond..., n
1) is constant (table interval) and #
y lt
ing values of the function y Then the value of the function y for an intermediate value of the argument x is approximately given by Newton's interpolation formula
,

where

o=

and A#

"7

= #,

A2

t/

= A#,

A#

...

are successive finite

the polynomial
diilerences of the function y. \\hen x
0, 1, ..., n),
x/ U
0, 1,
., n). As
(1) takes on, accordingly, the tabular values y { (/
particular cases of Newton's formula we obtain: for n=l, linear inter potation;
the
of
To
use
Newton's
for /i
2, quadratic interpolation.
simplify
formula,
it is advisab'ie first to set up a table of finite differences.
If y
f (x) is a polynomial of degree n, then

& n y.

= const

and A" +1 f//=^0

and, hence, formula (1) is exact


In the general case, if / (x) has a continuous derivative f ln + l} (x\ on the
x lt ..., x n and x, then the error
interval [a, b], which includes the points *
,

of

formula

(1)

is

| is some intermediate value between *;(/


practical use, the following approximate formula

where

=
is

I,
..., n) and
more convenient:

(),

x.

For

Interpolation of Functions

Sec, 2]

373

If the number n may be any number, then it is best to choose it so that


within the limits of the given accuracy; in other
the difference A" + {/
w
words, the differences A # should be constant to within the given places of
1

^0

decimals

Example
sin

1.

Find

sin

2615'

the

using

tabular

data

sin

26

= 0,43837,

27 -0.45399, sin 28 -0.46947.


Solution. We set up the table

Here,

/i

= 60',

26

15'

q--

Applying formula
have

sin

that

2615' = 0.43837

26

_!_
'

60'

(1)

and using the

0.01562

Let us evaluate the error R 2


if */
sun, then |*/ (/l '|*^l,

first

horizontal line of the table,

(0.00014) = 0.44229.

Using formula

we

will

we

(2)

and taking into account

have:

Thus, all the decimals of sin 2615' are correct.


Using Newton's formula, it is alsc possible, from a given intermediate
value of the function //, to find the corresponding value of the argument x
(inverse interpolation). To do this, first determine the corresponding value q
by the method of successive approximation, putting

and
?C'H

n\

2!

(i-O,

1,

2,

...).

Here, for q we take the common value (to the given accuracy!)
m + >. Whence x jt
cessive approximations q (m}
<;<
-{-<//!.
Example 2. Using the table

approximate the root

of

the equation

smhx

of

two sue*

374

Approximate Calculations
Solution. Taking y

10

we have

4.457,

,_5-4.457
~
1.009

We

(Ch

0.543
5

^i.oog

0.565-0.435

0.220

1.009

=0.538;

= 0.538 + 0.027 = 0.565;


:0, 538 + 0. 027 = 0. 565.

can thus take


*

= 2. 2 + 0. 565- 0.2 = 2. 2 + 0. 113 = 2. 313.

2. Lagrange's interpolation formula. In the general case, a polynomial of


0, 1, .... n), is given
degree n, which for *=*/ takes on given values yf (/
by the Lagrange interpolation formula

*!> (x

(x

__

x2)

(x

*o) (*

xn )

*,).

XQ) (x

(x

* 2)

(x

xn )

Xk ^) (X

.(X

'

'

'

'

"

'

3128. Given a table of the values of x and y:

Set up a table of the finite differences of the function y.


3129. Set lip a table of differences of the function y
x*
f
1
for the values *=1, 3, 5, 7, 9, 11. Make sure that
JC
5jc
all the finite differences of order 3 are equal.
3130*. Utilizing the constancy of fourth-order differences, set
10*' +2**
3jt
up a table of differences of the function y x*
for integral values of x lying in the range l^jt^lO.
3131. Given the table

log

1=0.000,

log 2 -0.301,
0.477,
log 3

=
=
log 4
0.602,
log 5 = 0.699.
Use

linear interpolation to
and log 4. 6.

log 3.1,

compute the numbers: log

1.7,

Iog2.5,

Interpolation of Functions

Sec. 2]

375

3132. Given the table


sin 10

sin 11
sin 12

= 0.1736,
= 0.1908,
= 0.2079,

sin 13

sin 14
sin 15

= 0.2250,
= 0.2419,
= 0.2588.
=

in the table by computing (with Newton's formula, for


2)
the values of the sine every half degree.
3133. Form Newton's interpolation polynomial for a function
represented by the table
Fill

3134*. Form Newton's interpolation polynomial for a function


represented by the table

Find y

for

3135.

= 5. 5.

function

Form Lagrange's
y

for

JC

= 0.

For what x will y =


is given by the table

interpolation polynomial and find the

value of

3136. Experiment has yielded the contraction of a spring (x


as a function of the load (P kg) carried by the spring:

mm)

Find the load that yields a contraction of the spring by

mm.

14

376

(Ch. 10

Approximate Calculations

3137. Given a table of the quantities x and y

0.5 and for x


2: a)
the values of y for x
of linear interpolation; b) by Lagrange's formula.

Compute

by means

Sec. 3. Computing the Real Roots of Equations

1. Establishing initial approximations of roots.


roots of a given equation

The approximation

/(*)=0

of the

(i)

consists of two stages: 1) separating the roots, that is, establishing the intervals (as small as possible) within which lies one and only one root of equation (1); 2) computing the roots to a given degree of accuracy
on an interval [a, b] and
If a function /(A) is defined and continuous
of equation (1).
/(a)./(6)<0, then on [a, b] there is at least one root
or /' (x)
This root will definitely be the only one if /' (x)
when

<

>

a<x<b.

it is advisable to use millimetre


In approximating the root
paper and
construct a graph of the function y = f(x). The abscissas of the points of
intersection of the graph with the x-axis are the rools of the equation /(x)
0.
It is sometimes convenient to replace the given equation with an
equivalent
as the absequation (p (#)
if (A). Then the roots of the equation are found
cissas of points of intersection of the graohs y
q)(x) and y
ty (x).
2. The rule of proportionate parts (chord method). If on an interval [a, b]
of the equation f (A)
there is a unique root
O, where the function f (x)
is continuous on
b], then by replacing the curve y
f(x) by a chord
[a,
passing through the points [a, f (a)] and [b, f (b)], we obtain the first
approximation of the root

/~\
(2)

To obtain

we apply formula (2) to that one of


ends of which the function f (x) has
values of opposite sign. The succeeding approximations are constructed in the
same manner. The sequence of numbers cn (n=l, 2,
converges to the
.)
a

the intervals

second approximation c 2
[a,

c,]

or

[c,,

b]

at the

root

that

is,

Generally speaking, we should continue to calculate the approximations c,,


until the decimals retained in the answer cease to change (in accord
.,
with the specified degree of accuracy!); for intermediate calculations, take
one or two reserve decimals This is a general remark.
If
the function / (x) has a nonzero continuous derivative /' (x) on the
interval [a, b], then to evaluate the absolute error of the approximate root

c2 ,

crt ,

we can make

where

377

Computing the Real Roots of Equations

Sec. 3]

u=

use of the formula

min

<x<

(/' (x)

|.

and f" (x) ^


for
3. Newton's method (method of tangents). If /' (x)
a^x^b, where f(a)f(b)<0, f(a)f'(a)>Q, then the successive approximaof an equation f(x)=Q are computed
tions * (rt = 0, 1, 2, ...) to the root
rt

from the formulas


(3)

Under the given assumptions,

lim *
n -> oo

To evaluate

where u,= min


a

we can

the errors

<

1,

2,

...)

is

mono-

use the formula

\f'(x)\.

xQ

a=

<b

For practical purposes

where

sequence x n (n

the

tonic and

.,

= a,

is

it

more convenient

= *_,

which yield

the

a/^.,)

i/i

to use the simpler

same accuracy

l,

2,

formulas

...),

(3')

as formulas (3).

&.
f(b)f"(b)>Q, then in formulas (3) and (3') we should put x
4. Iterative method. Let the yiven equation be reduced to the form
If

= <PM,

(4)

where \y' (x)\*^r< 1 (r is constant) for a ^x^b. Proceeding from the iniwinch belongs 4o the interval [a, b], we build a sequence of
tial value *
numbers x lt x 2 ... according to the following law:
,

If

a<jc

rt

<6

(n

l,

2,

...),

then the limit

is the on/r/ roo/ of equation (4) on the interval [a, 6]; that is, xn are successive approximations to the root |.
The evaluation of the absolute error of the nth approximation to x n is
given by the formula

it

IS

_ xn
y

^^s

xn + l
j

Therefore,

if

xn and x n + l coincide

error for x n will be

-r

to

_r

xn

within

E,

then

the

absolute

limiting

In order to transform equation f(x)

Q to

(4),

we

replace the latter with

an equivalent equation

where the number

A,

is

chosen so that the function

-7-

[x

Kf

(x)]

f
1

Kf

(jc)

.378

__

Approximate Calculations

should be small in absolute value in the neighbourhood of the point


example, we can put 1
X/'(x )=0].
Example 1. Reduce the equation 2x Inx 4=0 to the form (4)
initial approximation to the root * = 2.5.
Solution. Here, /(x)

lent

equation
values of A,;

|l_ A,( 2\

The

=x

this

X ]
initial

= 2x

In *

\nx

k(2x

number

U = 2.5 =0,
equation

/'(jc)

and

4)

close

is

that

4;

is,

10

[Cft.

= 2-~. We

(for

for

the

write the equiva-

take 0.5 as one of the suitable


of
the
to
root
the
equation

r^O.6.
.6

close to

reduced to the form

is

=x

Inx

0.5 (2x

4)

or

Example
equation that
of
(5)

2.

Compute, to two decimal


between 2 and 3.

places, the root \ of the preceeding

lies

Computing the root by th


Example 1, putting x ? = 2.5.

iterative method. We make use of the result


We carry out the calculations using formulas

with one reserve decimal.

*f

=2+

*,

= 2 + 4-

And so 1^:2 45 (we can


Become fixed)
Let us npw evaluate the

Considering that

all

^2. 448,

In 2.450

*4 = 2

(p(*)

y In2.458ss2.450,

+ ~ln2. 448=^2. 448.


since

stop

here

error.

Here,

= 2 + ~lnx

approximations

to

and
xn

the

third

decimal

place

has

<P'(*)=^..

lie

in the

interval

[2.4, 2.5],

we

get

Hence, the limiting absolute error


remark made above,

in the

approximation to x 9

is,

by virtue

of the

A==

^-^-=0.0012 ^=0.001.

Thus, the exact root g of the equation


2 447

we can

lies

within the limits

< g < 2.449;

take g^2.45, and all the decimals


be correct in the narrow sense.

of

this

approximate number will

Sec. 3]

Computing the Real Roots

of

Calculating the root by Newton's method.

<x<3

Wee

We

Here,

>

On the interval 2
we have:
/(3)f (3)>0. Hence, the conditions

Equations

/' (x)
of 3 for

> 0; /(2)f(3)<0;
= 3 fare(x)fulfilled.

and
*

take

carry out the calculations using formulas

jtj^S

=2
4)

In 3

0.6(2-3
0.6(2-2 4592
0.6(2-2.4481
0.6(2-2 4477

= 2. 4592
= 2.4481
*,
* 4 = 2. 4477
*,

(3')

with two reserve decimals:

4592;

In 2

4592

In 2. 4481

In2. 4477

= 2 4481;
= 2. 4477;
4) = 2 4475.
4)

4)

At this stage we stop the calculations, since the third decimal place
2. 45.
We omit the
does not change any more. The answer is: the root,
evaluation o! the error;
5. The case of a system of two equations. Let it be required to calculate the real roots of a system of two equations in two unknowns (to a given
degree of accuracy):

f(*.

0=0,

there be an initial approximation to one of the solutions (|, r\) of


system Jt x ot y = y Q
This initial approximation may be obtained, for example, graphically,
by plotting (in the same Cartesian coordinate system) the curves f(x, #)
and by determining the coordinates of the points of interand tp (x, #)

and

let

this

section of these curves.


a)

Newton's method. Let us suppose that the functional determinant


dtf.jp)

d(x,y)

xx

Then by Newdoes not vanish near the initial approximation


y
Q1 y
ton's method the first approximate solution to the system (6) has the form
where a P O are the solution of the system of two.
a
x
f/
x,

= +

j/,

= +P

linear equations

The second approximation

is

obtained in the very same way:

= *!+<*!,
*,
where a p $ v are the solution

Similarly

we obtain

of the

^1

= ^1+ Pi,

system of linear equations

the third and succeeding approximations.

380

_ _
Approximate Calculations

We

b) Iterative method.

the system of equations

(6),

[Ch

10

can also apply the iterative method to solving


by transforming this system to an equivalent one
* F(x.y).

=
\ y (D(*,
(

and assuming thai

|F>.y)| + |0;<x 0)|<r<l;

(f)

y)

\F'y (x, y)\

\<S> v (x.

#|<r<

(8)

in some two-dimensional neighbourhood U of the initial approximation (* y Q )


which neighbourhood also contains the exact solution (, rj) of the system.
The sequence of approximations (x n y n ) ( = 1, 2, ...), which converges
to the solution ol the system (7) or, what is the same thing, to the solution
,

x>f

(6),

is

constructed according to the following law:

= F(x
If

all

(*, y n ) belong

to

yz )

t,

then

*/,-c

lim
n

lim

-+ oo

y n =i\.

oo

The following technique is advised for transforming the system of equa*


tions (6) to (7) with condition (8) observed. We consider the system of
equations
a/

which

is

equivalent to

(x.

provided

(6)

a.

that

Rewrite

0.

it

in the

form

Y.

y)^F(x

Choose the parameters


functions F(v, y)

-jnd

a,

p,

O (x,

y, & such
y} will be

that

the

y),

derivatives of the
zero in the initial
approximate solutions

partial

equal or close to

in other words, we find


approximation;
system of equations

a,

Y.

fi,

o as

of the

,v

t/

= 0,

Condition (8) will be observed in such a choice of parameters a, P, Y


on the assumption that the partial derivatives of the functions / (x,
y) and
<p(A, y) do not vary very rapidly in the neighbourhood of the initial approximation (X Q // ).
Example 3. Reduce to the form (7) the system of equations
,

given the initial approximation to the root *

= 0.8,

= 0.55.

Sec. 3]

Solution.

fv (x*

the Real Roots of Equations

Computing

Here,

= 1.1;

<P*

y)-=x*

f (x t

(*<>.

= 1-92,
0o)

+ y*\

= 1.6,

y )
equivalent to the initial one)
,

is

a(* + // -l) + p(*'-i/)-0,


2
2
\ Yl* + -0-i-a(*'-</)=0
2

J
in the

i/ c )

1.

<fy(*

Write down the system (that


2

381

y)=x'-y- fx (x

q> (x,

__

/la, pi

VI Y. *l

form

For suitable numerical values of


system of equations
1

Y anc* ^ choose the solution

a, p,

of

the

+ 1.60+1.920 = 0,
l.la

p-0,

+ 1.925 = 0,
+ 1. lY 6 = 0;
1.6 Y

i.

y^

we put a =^0.3, p ^r 0.3,


Then the system of equations

e.,

which

=x
y=y
x

0.3(^

2
2

0.5,

b^QA.

1)

0.3(A-

2
r/

0.5 (* -{-#

1)

y),

+ 04 (x*y)

equivalent to the initial system, has the form (7); and in a sufficiently small neighbourhood of the point (x Qt y Q ) condition (8) will be fulfilled.
is

Isolate the real roots of the equations by trial and error,


of the rule of proportional parts compute them to

by means

and
two

decimal places.
s
x
1-0.
3138. X
4
*
05* 1.55 = 0.
3139.
+
8
4* -1--0.
3140. x
Proceeding from the graphically found initial approximations,
use Newton's method to compute the real roots of the equations
to two decimal places:

3141.

JC

3142. 2x

-\

_2jc
In*

5-0.
4

= 0.

3143. 2
3144.

= 4x.

logjc=y.

Utilizing the graphically found initial approximations, use the


iterative method to compute the real roots of the equations to

two decimal places:


8
3147. jc
x
3145. x'-5.*M 0.1=0.
2 = 0.
=
cos*.
3146. 4*
Find graphically the initial approximations and compute the
real roots of the equations and systems to two decimals:
3151. x- In*
14 = 0.
3* -(-1=0.
3148. A:'
8
2
8
=
x
3152.
2* + 3*
5
0.
3149. *
+3* 0.5 = 0.
2
4
3153. 4*
7sin* = 0.
3150. * +*
2*
2 = 0.

382

[Ch. 10

Approximate Calculations

3154. x*
3155. e*
'

+ 2x
+ e- *

= 0.
= 0.

"

~~

" Lff

x*

157

+y

'

= 0,

\Qgx-l =0.

~~

3156.
3158.

Compute

the equation tan

3159.

jc

to three decimals the smallest positive root of

jc.

Compute the

roots of the equation x-tanh

x=l

to four

the

integral

decimal places.
Sec. 4. Numerical Integration of Functions

1. Trapezoidal formula. For the approximate evaluation

of

If (x)
[a,

is

a function

continuous on

into n equal parts

b]

= x + ih
Xi

a,
let

xn

with an absolute error

of

we divide the

[a, b]]

and choose the interval

= b,

of

interval of integration
calculations

h==

be the abscissas of the partition points, and


//
/(*/) be the corresponding values of the integrand
f(x). Then the trapezoidal formula yields
y

Let

(x

0,

2,

1,

..., n)

where M 2 = max f (x) when


To attain* the specified accuracy
|

is

That

h must be of the order


value so that

is,

when evaluating

the integral,

the

in-

found from the inequality

terval h

of

V^e

The value

of

h obtained

is

rounded

off to the smaller

a==
~~ n
h

be an integer; this is what gives us the number of partitions n.


Having established h and n from (1), we compute the integral by taking the
values of the integrand with one or two reserve decimal places.
2. Simpson's formula (parabolic formula). If n is an even number, then

should

in the notation of

Simpson's formula

h
Q-

K#o + #n)

+ 4 (y\ + #3+.. +yn -\) +


+0n-2)l

(3)

Numerical Integration

Sec. 4]

of Functions

383

holds with an absolute error of


(4)

M 4 = max

where

when

!
|

(x)

To ensure

the specified accuracy e when evaluating the


interval of calculations h is determined from the inequality

integral,

the

(5)

That

the interval h

is,

to the smaller value so

is

of the order

that n

J/JF,
is

The number h

is

rounded

off

an even integer.

Remark. Since, generally speaking, it is difficult to determine the interand the number n associated with it from the inequalities (2) and (5),
practical work h is determined in the form of a rough estimate. Then,

val h
in

after the result is obtained, the number n is doubled; that is, h is halved.
one to the number of decimal
If the new result coincides with the earlier
places that we retain, then the calculations are stopped, otherwise the pro-

cedure

is

repeated, etc.

For an approximate calculation of the absolute error R of Simpson's


quadrature formula (3), use can also be made of the Range principle, according to

which

where 2 and S are the results


h and // = 2/i, respectively.

of calculations

from formula

(3)

with interval

3160. Under the action of a variable force F directed along


the x-axis, a material point is made to move along the x-axis
4. Approximate the work A of a force F if a
to x
from x
table
is given of the values of its modulus F:

by the trapezoidal

Carry out

the calculations
the Simpson formula.

formula and by

3161. Approximate

J (3*

4x)dx by the trapezoidal

formula

putting rt=10. Evaluate this integral exactly and find the absolute and relative errors of the result. Establish the upper limit A
of absolute error in calculating for n=10, utilizing the error
formula given in the text.

384

\Ch. 10

Approximate Calculations
i

3162. Using the

* *

calculate

Simpson formula,

to four

J *

decimal places, taking n== 10. Establish the upper limit A of absolute error, using the error Formula given in the text.
Calculate the following definite integrals to two decimals:
3,63.

3,68.

Ife.

(-!<,.

Jl

-~.

3164. C

3169.

317

>

3166.

[x\ogxdx.

8187.

Q | 7i

dr.

fcos* dX.
FT
\+X
Jcos*
,

3172>

3173. Evaluate to two decimal


a PPlyi n S the substitution

the

places

-7--

improper

Verify the calculations

ft

by applying Simpson's formula to the integral

is

chosen

,so

QO

that

7^722

<

integral

Jy

rr
~r x

where b

10

plane figure bounded by a half-wave of the sine curve


*-axis is in rotation about the x-axis. Using the
Simpson formula, calculate the volume ot the solid of rotation
to two decimal places.
3175*. Using Simpson's formula, calculate to two decimal
3174,

f/=sin^ and the

places the length ot an arc of the ellipse


in the first

y+

situated

quadrant.

Sec. 5. Numerical Integration of Ordinary Differential

1.

A method

of successive approximation
be given a first-order differential equation

subject to the initial condition

(Picard's

y'^f(x.y)
= # when x = *

Equations
method).

Let

there

<1)
.

Numerical Integration

Sec. 5]

Ordinary Differential Equations

of

385

The solution y (x) of (1), which satisfies the given initial condition, can,
generally speaking, be represented in the form

y(x)=

lira yi (x)
-* 00

(2)

where the successive approximations


y

*//(*) are

from

determined

the

formulas

(*)=#<>+

If

the right side f(x, y)

R{
and

in this

satisfies,

is

|x

yt-i (*)) dx

defined and continuous

in the

neighbourhood

Vi)-f(x. i/JKJ-ltfi-tfil

(L is constant), then the process


converges in the interval

of

I*
ft

(x.

neighbourhood, the Lipschitz condition


\f(x,

where

= min(a, ^
M/
\

and

successive

definitely

(2)

Af = max|/U,

/?

approximation

y)\.

And

the error here

is

/?

- y n W \<ML* ~

The method of successive approximation (PicarcTs method) is also applicable, with slight modifications, to normal systems of differential equations.
Differential equations of higher orders may be written in the form of systems
of

differential equations.

2. The Runge-Kutta method. Let


,

To do

it
be required, on a given interval
to find the solution y (x) of (1) to a specified degree of accuracy e.

we choose

this,

the interval of calculations /i=

the interval [*
X] into n equal parts so that h*
Xf are determined from the formula
,

X|

=x +M

(i=0,

1,

2,

< e.

..., n).

By the Runge-Kutta method, the corresponding values


function are successively computed from the formulas

1900

by dividing

The partition points

/;

(/

(x/) of

the desired

[Ch. 10

Approximate Calculations

386

where
f

=0,

1,

2,

...,

n and

(3)

*i"

=/(*/ + A,,

correct choice of the interval h

To check the

it

is

advisable

to

verify

the quantity

e=
The fraction 6 should amount

to

few

hundredths, otherwise h has to be

reduced.

The Runge-Kutta method is accurate to the order of h 1 A rough estimate


Runge-Kutta method on the given interval [x X] may
be obtained by proceeding from the Runge principle:
.

of the error of the

Rn

y*m

Urn

'

15

where /i = 2m, y 2m and y m are the results of calculations using the scheme (3)
with interval h and interval 2/i.
The Runge-Kutta method is also applicable for solving systems of differential equations
y'

with given

= f(x,

y> z).

*'

= <p(x,

y,

z)

(4)

= z = 2 when x x
by the Milne method,
=(1)
X
we in some way find

conditions /
3. Milne's method. To solve
initial conditions y=^y when X
values
initial

yi=y(*i),

t/

Q,

y=0(*i).

subject to the
the successive

=*/(*)

of the desired function y (x) [for instance, one can expand the solution y (x)
in a series (Ch. IX, Sec. 17) or find these values by the method of successive
approximation^or by using the Runge-Kutta method, and so forth]. The ap-

proximations y t and y] for the following values of


successively found from the formulas

*=

\vhere

fi

= f(x

it

r/ t

(i=4,

5,

..., n)

are

+~(7"

y^ and7i = /(*i,

Hi)-

To check we

calculate the quantity


(6)

Numerical Integration of Ordinary Differential Equations

Sec. 5]

387

does not exceed the unit of the last decimal \(y m retained in the
y (x), then for f/ we take I// and calculate the next value y/ + 1
10~ w then one has to start from the berepeating the process. But if e/
ginning and reduce the interval of calculations. The magnitude of the initial
interval is determined approximately from the inequality h 4
10~ m
For the case of a solution of the system (4), the Milne formulas are
written separately for the functions y (x) and z (x). The order of calculations
remains the same.
Example 1. Given a differential equation */'=*/ x with the initial condition y(0)=1.5. Calculate to two decimal places the value of the solution
of this equation when the argument is x
1.5.
Carry out the calculations
If

e/

answer

for

>

<

combined Runge-Kutta and Milne method.

by

We

Solution.

choose the initial interval h from the condition /r*<0.01.


let us take h
0.25. Then the entire interval of
x
to jc=1.5 is divided into six equal parts of length

To avoid involved writing,

integration from
0.25 by means of points x f (i
0, 1, 2, 3, 4, 5, 6); we denote by y f and y^
the corresponding values of the solution y and the derivative y'
We calculate the first three values of y (not counting the initial one) by
the Runge-Kutta method [from formulas (3)]; the remaining three values
we calculate by the Milne method [from formulas (5)]
1/4. */s
y&
The value of // will obviously be the answer to the problem.
We carry out the calculations with two reserve decimals according to a
definite scheme consisting of two sequential Tables 1 and 2. At the end of
Table 2 we obtain the answer.
#
#, x =^0, // =1.5
Calculating the value y r Here, / (x, */)

fl

/i

= 0.25.

A^

=i

(*<

= 4-D (0.3750 + 2-0. 3906 + 2.0.3926 + 0.4106) ==0. 3920;


(

0)

*(>

= / (*o.
=

/
/

0o) h

+~

)=s

0+

.5000)0.25-0.3730;

(0)

\
+ -y- j = (- 0.125+1.5000 + 0.1875) 0.25= 0.3906;
fc

Xo

/i

& (0)

*i

=n *o+y.

0o

\
+ -|-) ^ = (0 125+1.5000 + 0.1953)0.25 = 0.3926;
^=
0.25+1.5000 + 0.3926)0.25 = 0.4106;
(

= 1.5000 + 0.3920 =1.8920

(the

first

three

decimals

in

tins

approximate number are guaranteed).


Let us check:

20
|0.3906Q.3926|_ .1
= 0. 13.

10.37500.39061""

By
in

13*

we chose was rather rough.


values y t and y 9 The results are tabulated

this criterion, the interval h that

Similarly

Table

1.

we

calculate

the

156

388

Approximate Calculations
Table

1.

Calculating^,,
f(x, y)

Calculating the value of


j/

=1. 5000,

^=1.5000,
Applying formulas

jr 2 ,

y9

We

have: f(x, y)

jf4 .

= 1.8920,
=
1.6420,
^'

(5),

we

by the Runge-Kutta Method.

y t = 2.3243,

r/,

[Ch. 10

^=1.8243,

/z

y,

= 0.25,

,== 2.8084;

= 2.0584.

find

-^

= 1.5000 + 4

A OK

(2-

1.6420-1.8243 + 2. 2.0584)

= 3.3588;

1+3.3588 = 2.3588;
=

02

+ ^(01

h y'
t)

.
hence, there

is

'

= 2.3243 +

3.3588-3.3590

no need

(2.3588

+ 4- 2.0584 +

.8243)

to reconsider the interval of calculations.

= 3.3590;

Numerical Integration of Ordinary Differential Equations

Sec. 5]

389

= =

We obtain i/ 4 y4 3.3590 (in this approximate number the first three


decimals are guaranteed).
Similarly we calculate the values of y^ and y^. The results are given in
Table 2.
Thus, we finally have
y

(1.5)

= 4.74.

4. Adams' method. To solve (1) by the Adams method on the basis of


the initial data */(x )
we in some way find the following three values
/o
g
of the desired function y (x):

[these three values may be obtained, for instance, by expanding y (x) in a


power series (Ch IX, Sec. 16), or they may be found by the method of successive approximation (1), or by applying the Runge-Kutta method (2)
and so forth].
With the help of the numbers X Q x it * 2 x t and */ y lt y zt ys we calcu,

late

</

q lt q 2

We

?, where

= h y' = hf (x
2

2,

// 2 ),

qs

= hy' = hf (x,,
3

then form a diagonal table of the finite differences of

V'=f(x,

I/O

f/2

i/5

y)

q:

_ _
Approximate Calculations

390

The Adams method

Thus,

Adams formula

numbers

the

utilizing

we

= + --

A#,

it,

<7,

+ A*/

A? 2

</ 8

A<7 2 ,

calculate, by

in

table of differen-

consists in continuing the diagonal

ces with the aid of the

Hhe difference table,

[Ch. 10\

A 2 ?,, A 8

means

of

</

situated

formula

+ IZ A <7,+
2

-r^

A*<7

After

And when we know * 4 and #4

(7)

finding

diagonally in

and putting n-- 3

A//,,

we

calculate

we calculate q^~hf(x^ f/ 4 ),
difference table and then fill into it the
2
A 3<7,, which are situated (together with 4 ) along
finite differences A</,, A ^ 2
a new diagonal parallel to the first one.
Then, utilizing the numbers of the new diagonal, we use formula (8)
4 in it) to calculate A// 4 */ R and q t and obtain the next diagonal:
(putting n
A 2 <7,, A*<7 2 Using this diagonal we calculate the value of */6 of the
ft, A^ 4
desired solution y(x), and so forth.
The Adams formula (7) for calculating by proceeds from the assumption
that the third finite differences A 3 </ are constant. Accordingly, the quantity h
of the initial
interval
of calculations is determined from the inequality
4
/t
<10~ m [if we wish to obtain the value of y (x) to an accuracy of

#4

*/3

introduce

8-

*/ 4

A# 8 and

</

into the

].

In this

sense

the

Adams formula

Milne (5) and Runge-Kutta


Evaluation of the error

is

(7)

equivalent

to the

formulas of

(3).

for the Adams method is complicated and for


practical purposes is useless, since in the general case it yields results with
considerable excess. In actual practice, we follow the course of the third
finite differences, choosing the interval h so small that the adjacent differences A 8 <7f and A*</; +1 differ by not more than one or two units of the given
decimal place (not counting reserve desimals).
To increase the accuracy of the result, Adams' formula maybe extended
by terms containing fourth and higher differences of q, in which case there
is an increase in the number of first values of the function y that are needed
when we first fill in the table. We shall not here give the Adams formula
for higher accuracy.

Example

2.

Using the combined Runge-Kutta and Adams method, calcu(when #1.5) the value of the solution of the
x with the initial condition f/(0)
1.5 (see

late to two decimal places


differential equation

y'y

Example

1).

We

use the values y lt y z //, that we obtained in the solution


Their calculation is given in Table 1.
We calculate the subsequent values //4 f/5 #6 by the Adams method (see
Tables 3 and 4).

Solution.

of

Example

1.

The answer to the problem is #4 = 4.74.


For solving system (4), the Adams formula (7) and the calculation scheme
shown in Table 3 are applied separately for both functions y(x) and z(x).

Find three successive approximations to the solutions


and systems indicated below.

differential equations

=
=

3176. y'=jf+y*\ y(Q)


Q.
3177. y'
x y
2, z'
y-z\ f/(0)=l, z(0)
3178. */"
-#; y(0) 0, y'(0)=l.

= + +
=

= -2.

of

the

Table

3.

Basic Table for Calculating


/(*,

</)

y4 y

= -* +

*/;

fc

by the

=y^0.25

Adams Method.

(Italicised figures are input data)

Answer: 4.74

Table

-1

153

Auxiliary Table for Calculating by the

Adams Method

Approximating Fourier

[Sec. 6]

Coefficients

393

/i = 0.2,
use the Runge-Kutta method to
approximately the solutions of the given differential
equations and systems for the indicated intervals:
3179. 0' =
x; 0(0) = 1.5 (

Putting the interval

calculate

3180.
3181.

=|

0'

0(1)

0';

y'=*z+

z'=0

1.

x,

(Kx<2).
0(0) = 1, z(0) =

(0<x<l).

combined Runge-Kutta and Milne method or


Applying
Runge-Kutta and Adams method, calculate to two decimal places
a

the solutions to the differential equations and systems indicated


below for the indicated values of the argument;
0.5.
3182. 0'=x+0; 0=1 when x=--0. Compute y when x
0. Compute y when x=l.
x*
3183. 0'
0; 0=1 when x
0. Compute
when x 0.5.
3184. 0'
3; 0=1 when x
20
x
3185. J0'
20+z,

=
=

=
=

=
+
z' = x + 20 + 3z;

Compute y and
3186.

f0'

3187. 0*

30

=2

3188.

y=

when

0V +1=0;
and

when x = 0.

when x = 0.

when x = 0.

0'

when jc=l.

=1.5.

A:(JI)

A:

0=1,
A:

^ + |-cos2/ =

3189.

=
= 0.5.
=
0'
z

2,

= 2,

0;

Compute y when

Find

z,

z;

when x=l.

Compute

when x = 0.5.

)2r'=y
and
Compute

= 2,

0;

x = Q,

Jc'

when

f=-0.

x' (n).

Sec. 6. Approximating Fourier Coefficients

Twelve-ordinate scheme. Let y n


of

the function y

and y Q

-f/ 12

We

~f(x n

(n

points
f(x) at equidistant
equidist
set

= 0,

1,

xn = -

...,
of

up the tables:
y* y\

yu

Sums

yz

y* y*

y\9 y> y*

(2j)

Differences (A)
Q u l uz
Iu
U* U 6 M 4

Sums

s l s 2 s,

Diilerences

tv

Sums
Differences

12)

be the values

the interval lU,2ji],

Approximate Calculations

394

The Fourier

may

\Ch

coefficients a n b n (n
1, 2, 3) of the function
0,
y
be determined approximately from the formulas:
,

6t

a2

=s

&,
s,

+ 0.5(s,

&8

s 2 ),

= 0.50! + 0.866a + a
= 0.866 (1, + *,),
= a, a
2

10

= f(x)

8,

8,

(1)

where 0.866

We

have

f(x) zz

Other

'

30

10

schemes

are

also

used.

(an cos

nx +

Calculations

b n sin nx).

are

simplified

patterns.

Example. Find the Fourier polynomial for the function y


represented by the table

From formulas

(1)

by the use

of

= f(x)

we have

= 9.7; a, = 24.9;
=
13.9; 6 =
8.4;
6,
2

=i0.3; a a = 3.8;
6,

= 0.8.

Consequently,
/ (x)

^ 4.8 + (24.9 cos x + 13.9

sin x)

+ (10.3 cos2x

8.4 sin 2x)

+
Using the
for

(3. 8

+
cos 3*

+ 0.8 sin 3x).

12-ordinate scheme, find the Fourier polynomials


the following functions defined in the interval (0,2:rc) by the

Approximating Fourier Coefficients

Sec. 6]

tables of their values that correspond to the

395

values

equidistant

of the
.

argument.
3190. </ =
7200

= 4300
=
5200
y =
=
9.72
y,
= 8.97
=
8.18
y,
= 1.273
y = 0.788
y = 0.495
#,
4

=
=
6.68
y,
= 9.68
- 2.714
=
3.042
y,
= 2.134

3191. y

t/,

3192.

J/

/2

f/ 4

t/ s

4
s

= 7400
- 2250
=
y,
/,

= 3850
= 7.42
y,
=
6.81
y,
= 6.22

yg

</ 8

= 0.370
=
0.540
y,
=
0.191
y,

y,

= 7600
= 4500
^, = 250
0,

</,

=5.60
/ 10 =4.88
/ n =3.67

y.

y,

=-0.357

= 0.437
n = 0.767

#,
t/

3193. Using the 12-ordinate scheme, evaluate the


Fourier coefficients for the following functions:
a)

f(x)

(x>

first

several

ANSWERS

Chapter
1.

then a
b) + b
+ \b\. Whence (a
= & a (^ 6 \<\ab\
& ^ a
Hence, |a
|a
\ab
= |a/ + |6 3. a) -2<x<4;
\a-b = |a +
b) |<|a| + -6|
< x < 0; d) x> 0. 4. 24; 6; 0; 0; 0; 6. 5.
3, x >
c)
.; ~; 0.
(*)= --x +

a=(a

Solution. Since

>|a|

and

\b\

Besides,

|.

<

b) x

1;

|.

|.

1;

7.

b)~oo

<*< + oo.

/2<*< +

be 2

+*

*
but

16.
19.

oo

2<0,
this

22. q>(*)

= 2^

i.

5* 2

10. i|>(x)

a)

T = JI,

T=

Periodic,

e)

y=

*-|-KO,

or

e.,

should
either

x<l,

15.

18.

a)

Even, b)odd,

= i[f (x) +
/ (x)

(-x)}

7 =ji,

c) periodic,

0<x<c; = 6
t/

if

even, d)odd,

+ ![/ (x)-f (-x)}.

2n

r = -r-,

c)

2,...).

1,

3^ + 6x23.

if

i.

Arji<A:</ Jt + ~(/2 = 0,

21.

x,

<*<

2^0,

K*<2.
\<x<

2<x<2.

2
n, b) periodic,
-^
u

nonperiodic. 27.

is,

oo

13. a)

l<;t<2. Solution. It
2)<0. Whence
(A:+ 1) (*

Thus,

impossible.

0<*<1. 17.
20 K*<100.

e) odd.24. Hint. Utilize the identity

26.

that

14.

l<x<2-

e.,

+00).

2), (~-2, 2), (2,

2<0;

is
1,

-4-< <

00,

*=0,
|jc|^ K2.
2

or ^

<^<
Jt

12.

b)

x'^0,

A'+l^O,
Jt^*2,

oo;

d) periodic

C<

m q^ when
=
m
+q
qil +q
/^A;^/,-!-^;
28.

= ^i^i + ^2(^
when
+ /2<^</, + + =
-'i)

/1

37.

-^-;0;

7-. 38.

a)

/.

=2
x=

29. (p[x|)(jc)]

=Q

when

2l 2

2Jf

;^[(p(x)]

1,

>

= 2^

(x

2)

30. x. 31. (x

when #

>

1,

when

+ 2)
y <Q
2

= when x =
when jc<
and * = 2, / >
when
1 <x< 2,
1; b)
< when oo < x < 1 and 2 < x < -f oo; c) z/>0 when oo<x< -|- oo;
when x = 0, x =
]/~3~and A;= /" "3, r/>0 when
V%< x<0 and
d) j/ =
1

i/

i/

oo);

b)

and

l<x<-f

x=V y +

y<0 whenO<^<!

oo,

and

39. a)

= -- (y

3)

Answers

c)

if

y lanyf

= arc tan
43. a)
c)

*/

#=

y=u
= arc
l

a)

^=--;

d)

VlogT;

when

d)

= 2-10>

oo<#< +

oo);

e)

* = # when
oo<0<0; x=/l/ when
= 2x 5; b) # = 2a = cos.r, c) =
u
u=
x
42. a) # = sin x; b)
sinw, a = 3
</

y = 2(x*

!/""<

t/

c)

= -cosjc

oo);

40.

<y<^\

41.

oo<#< +

397

JC
|

oo<x<0 and

1)

and

|x|<l,

if

b) y = log (10
=
y x when 0<x< + oo.

< 1/2S;

10*),
46.

|*|>1.

if

-oo<x<l;

Hint. See Appen-

\j

dix VI, Fig. 1. 51. Hint. Completing the square in the quadratic trinomial
2
we will have y y
and yQ (4ac b 2 )j4a.
a(x * ) where *
b',2a
Whence the desired graph is a parabola y ax* displaced along the *-axisby
See .Appendix VI,
53. Hint.
XQ
and
the
t/-axis
along
yQ
by

= +

The graph

is

See

Hint.

58.

2.

Fig.

a hyperbola

Appendix

y=

shifted

VI,

along

Fig.

the

3.

*-axis by

Hint.

61.

X Q and along
2

the t/-axis by
(

x+

yQ

Taking the integral

Hint.

62.

\ (Cf. 61*). 65. Hint. See

Appendix VI, Fig.

part,

4.

we have y=-~
o

--13
9

/
'

67.Hint. See Appendix VI,

Appendix VI, Fig. 6. 72. Hint. See Appendix VI,


Appendix VI, Fig. 8. 75. Hint. See Appendix VI,
Fig. 19 78. Hint. See Appendix VI, Fig. 23. 80. Hint. See Appendix VI,
Fig. 9. 81. Hint. See Appendix VI, Fig. 9. 82. Hint. See Appendix VI,
Fig. 10 83. Hint. See Appendix VI, Fig. 10. 84. Hint. See Appendix VI,
Fig 11. 85. Hint. See Appendix VI, Fig. 11. 87. Hint. The period of the function
5 sin 2x with amis T
2njn. 89. Hint. The desired graph is the sine curve y
plitude 5 and period n displaced rightwards along the x-axis by the quantity
5.

Fig.

Hint.

71.

Hint.

Fig. 7. 73.

See

See

90. Hint. Putting

a=A cos

cp

and

b= A

sin

--- V

In our case,

cp,

we

will have

y=A

sin (x

cp)

where

=-

Hint. cos x

V& + b

and

(p

arctan(

4 = 10,

cp=0.927. 92.

= -jr-(l+cos2jc). 93. Hint. The desired graph the sum of the graphs
= sinjc. 94. Hint. The desired graph the product of the graphs
= sinx. 99. Hint. The function even For x>0 we determine
and 3) y =\. When x
at which
>+<,
# = 0; 2) y =
is

yl
yl

=x
=x

the

and
and

points

t/

is

is

i/ 2

y+\.

1)

\\

101. Hint. See Appendix VI, Fig. 14. 102. Hint.


103. Hint. See Appendix VI, Fig. 17. 104. Hint.
17. 105. Hint. See Appendix VI, Fig. 18. 107. Hint.
Hint.
18. 118. Hint. See Appendix VI, Fig. 12. 119.

Fig.
Fig.
Fig.

15.

Fig.

12.

120.

See
See
See
See

Appendix VI,
Appendix VI,
Appendix VI,
Appendix VI,
Hint. See Appendix VI, Fig. 13. 121. Hint. See Appendix
132. Hint. See Appendix VI, Fig. 30. 133. Hint See Appendix VI,
Hint. See Appendix VI, Fig. 31. 138. Hint. See Appendix VI,
Hint. See Appendix VI, Fig. 28. 140. Hint. See Appendix VI,

VI, Fig.

13.

Fig. 32.
Fig. 33.

134.
139.

Fig. 25.

141. Hint.

Answers

398

Form

a table of values:

Constructing the points (x, y) obtained, we get the desired curve (see Appendix VI, Fig. 7). (Here, the parameter t cannot be laid off geometrically!)
142. See Appendix VI, Fig. 19. 143. See Appendix VI, Fig. 27. 144. See
Appendix VI, Fig. 29. 145. See Appendix VI, Fig. 22 150. See Appendix VI,
x 2 It is
1^25
Fig. 28. 151. Hint. Solving the equation for y, we get
now easy to construct the desired curve from the points. 153. See Appendix VI, Fig. 21. 156. See Appendix VI, Fig. 27. It is sufficient to construct

y=

the points

(x, y)

to the abscissas x

corresponding

= 0,

a.

157. Hint.

-^

*]

the equation for x, we have *


10 \ogy
Whence we get the
y
points (x, y) of the sought-for curve, assigning to the ordinate y arbitrary
values (*/>0) and calculating the abscissa x from the formula *
Bear in
oo
as y -+ 0. 159. Hint. Passing to polar coordinates
mind that log y -*
(

Solving

haver=

cos 8

>

q>

+ sin

r="
x = 0.4

:2.9,

0002;

f)

= -2,

*2

2.5;

=i=

166.

= 0.86.
=2

= 2, ^ = 5; * = 5,
a) *!
^ = 2,^ = 3;^ = 3, y = 2; c)
2

A^=9;
0.0002.

a)

3.6,

4; b)

I.

172. 1.

169.

A^

c)

Xl

/i

>

logAr<

a)

Use the formula


182.

0.

192.

oo.

183.

193.

oo.

~~.

173.

+2

184.
2.

194.

174.

0.

3.1;

y,25s

A-

=.,

yi

10; c)

^=2.7, y z

= 2;

x,=--2,

^2

9;

Sit

**=
167.

32.

/i

>

W = 99;

b)

f/

168. 6

when

x>X(N); c)\f(x)\>N when \x\>X(N).


171.

8C

l,

163.

will

//

165.
;

^^=

d)

> -~
V

a)
c)

32)

= rsincp, we
161.
F = 32 +
^=

32)/

Fig.
ft

e)

'

Appendix VI, Fig


and

.),,-'

/2

when

y,

_1^
2
1=JV.

=15

*);

^=-2;

*,=_;

VI,

Appendix
KK

r = e? (see
= rcosq,

have

will

coordinates

polar

x=1.50;

e)

we

(p

1M.

.1

b)

= 0.6* (10

162.

b)

tancp=~

Hint. Passing to
3sin cp cos cp

160.

d)

and

YX* -f- y*

1.

175. 3.

+i = -g
2

-f

185. 72.
oo.

195.

186. 2.

196.

170.

1.

+ l) (2/i+
3a a

188.
197.

a)

a)

002;

>N

when

(e<l).

0<*<6(AO;

176.

187. 2.

0;

177.

b) 2*

b) 1; c) 2; d)

178.

j.

oU

Hint.

180.0. 181.

1).

179. 0.

oo.

189. 0. 190. 1. 191. 0.

3* 2

198.

-1.

199.

^-

1.

Answers

200. 3. 201.

i-

202.

-1

203.

204.

399

12.

205.

-- i

206.

207.

1.

-|
208.

=:. 209.

x
215. 0. 216.

a)

228.

A.

1.

4o

236.

217. 3.

b) 0.

224.

218.

225. cos

ji.

A:.

244.

245.

251.6.252.

e*.

ji.

i-

239.

a)

e" 1

Solution,

1.

-m

233.

).

a)

241.

1.

247.

lim (cos*)

221.

0;

lim [1

1.

1.

242.

-i
4

249.

"*.

cos x)]

(1

X-0

X-+0

1.

b)

234.

e" 1

248.

227.

240.

Jt.

214.

246.

0.

I(rt

-|.
250.

238.

--JL

226.

232.

I.

237.

2 19. 4". 220.


o

230. 0. 231.

Jt

243.

22'

---.

213.

~:j4=.
235.

212.

j/jt

sin a.

229.

211.

-^s\n2\
z

222. cos a. 223.

--

210.

2sin a

"m

Si nee lim V

2 lim

2-1- lim

=e

that lim (cos*)

1.

b)

*->o

(see

follows

it

the

in

preceding

lim

lim (cos x)*

a),

As

Solution.

"/==-.
e

7=0,

V
i

case

^n

X-+Q

=e

Since lim

X-+0

= _J_,
2

2 lim

=
Put

lim (cos*)*

1.

256.

1.

--

257.

6*^1=0,

1.

where a-^0.

Hint. Put

263. a)

267. a) 0; b)

1.

1;

b)

268.

= a,

-i
a)

259.

264. a)
1;

b)

-1;

b)

1.

258

'

Utilize the identity a

Ina. Hint.
-*

where

=<?

Hint

260. In a

262.

that

*-).o

lOloge. 255.

253. In 2. 254.

follows

it

269.

(see

1.

a)

Example

265. a)
1;

b)

-1;
1.

b)

259)
1.

270. a)

= e na
a

261.

266. a)

6.

1; b) 0.

oo; b)

+00.

Answer*

400
271.

Solution.

but

if

when x=l;

whenx = 0;i/ =

l<x< + oo.
280

x=l

x>

when

when

276.

281.

kn (fe=0,

then cos 2

&

If

/Swi,

x>0.
277.

^g.

6=0;

1
1

compound
c)

>

x\

1000. 289. |x

1|<0.005;

b) |x

= 0.01;

b) 6

c) 3.

293

299.

3.

6)

a)

300.

x
|

kn, where k

316.

a)

/(0)

x+ A*

= n;

b)

when

^* 2^

b)

A:)

is

<,\

we

/(0)1;

310. a)

an integer
I

c) /(0)

313.

= 2;

7=

301.

292.

a)

1.

= 0.1;

1;

2;

b)

298.

1.

Hint.

0.98(09804);

1)

d)

Take

3.167(3.1623)

c)

have

Hint.

< 0.05;

a)

15. 297.

296.

b) 4; c)

2;

309.
|

3.875(3.8730);

4)

jx

1 ~

0.985(0.9849);

Ax

<|
Ax

No

295.

a)

U|>100;

b)

-^-=6;

10.954(10.954).

d)

y^zT

curve

a)

2|<

|x

Second, b) third.

a)

the

|*|>10;

a)

\&x\<x

cos

285

=x

279.

si.

"

0<e<l;

d) 2; e) 3.

when

(x-f- AJC)

j/

proportionality factor (law of

290.

0.0005

l^"x)<| Ajc|/V^x.

integer; b) x
inequality

-5-

U|>

0.0095(0.00952);

inequality |cos

<

+1;

3)

Ax+

the

is

when

291.

c)

when x<0; # =

= 4"
of

asymptote

0.043(0.04139). 303,

then

Ax|/(V A:+

288.

<~

7)

the

is

1.03(1 0296);

=3

> 0,r

-i

a)

072(0.7480);

= 0.001.

b)

1;

c)

1.03(1.0309);

If

c)

)/"lO= l/"9Tl
2)

Q t = Q<fkt

interest);

1;

-!

where k

0<x <

278.

lim <4C n
n-Mo

284.

nj

*
.

y=x

line

j/

x 2 -*oo.

fL

straight

Q/^^Qo

287.

274.

y=x

when

0=1 when 0<x<l;

275.

the

= |x|.

*,-*---;

282.

4-

J/

cos*x<l and y=0;

then

...),

272.

/=!.

273.

2,

1,

and

1.12(1.125);

5)

Hint.

307.

J/x-fAx-- /"il
advantage

+kn, where

the

of

is

an

311. Hint. Take advantage of the


l.
315. No
>l=4. 314. f(0)

= 2;
kind. 318. x =
d) f (0)

e)

=
= 0;
/ (0)

f)

/(0)

1.

317. x
2 is a discontinuity of the second
1
is a removable dis2 is a discontinuity of the second kind; x=2 is a removable
continuity. 319. x=
320. x
is
is a discontinuity of the first kind. 321. a) x
discontinuity
a discontinuity of the second kind; b) x=0 is'a removable discontinuity. 322. x=0
is a remo vable discontinuity, x
kn (k=\, 2, ..) are infinite discontinuities

323.

x=2jifcy

(fc

= 0,

1,

2,...)

are

infinite

discontinuities.

x^kn (=0,
is a
1,
2, ...) are infinite discontinuities. 325. x
1
is a removable discontinuity;
discontinuity of the first kind. 326. x
1
is a point
of discontiauity of the first kind. 327. x
1 is a discon-

324.

x=

Answers

401

x=0

is a removable discontinuity. 329. x=l


tinuity of the second kind. 328.
of the first kind. 330. x
3 is a discontinuity of the first
is a discontinuity
is a
1
kind. 332.
discontinuity of the first kind. 333. The function is
continuous. 334. a)
is a
discontinuity of the first kind; b) the function
kn (k is integral) are discontinuities of the first kind.
is continuous;
c) x
335. a)
k
(k is integral) are discontinuities of the first kind; b) x
is integral) are points of discontinuity of the first kind. 337. No, since
(k
the function y
E(x) is discontinuous at x=l. 338. 1.53. 339. Hint. Show
that when * is sufficiently large, we have P ( x ) P (x ) <0.

x=

x=0

x~k

II

Chapter
341.
344.

3;

a)
a)

0.21;

b)

1560;

624;

2ft

c)

+A

0.01;

b)

342.

0.1;

a)

100; c)

3;

b)

c)

0.000011. 345. a) abx; b) 3x*&x

1;

-.;

2*(2-_l)

e)

g^'-i);
A*

jt+Ax-f-J^x

2;

354.

a)

~-

lim

A!?

A/-H)

At

lim

"^
d)

^ =E =

AJ^-^O

sin 2 x

a)

12
"~

c)

]im f(x
A*-*O

the

L^

0.16;

/ (x)

time

at

time

t.

at

time

t.

A =^

b)

t.

at

substance

21 -

Ax

temperature
of

347>

+ Ax)

turn

of

angle

quantity

0238;

21
2

357.

Ax cos x cos

"

is

"" h;

(x)

the

is

l;

_ _
2

"

35l

0.25.

= J^- = sec x.
cos x
(x+ Ax)
1
Solution.
359.

-.

the

is

356.

Ax

lim
Ax->o cos x cos

cp

where

Ax

0.249;

lim

lim
AJC-M)

201

'=

where

Ar

>

Ax

where

b)

l;

/(*+Ax)~ /(x)
2,

lim
Af-M>

b)

Ax

c)

b)

rX =
dt

355.

dt

353. a)

JP=

b)

Ar

~~

a)

IT)

348. 15 cm/sec. 349. 7.5. 350.


352. a)

346>

AX

c)

(x-f-

Ax)

a)

3x 2

358.

n8)=

Solution.

sec x.

AX
b)

lim

Ax

*o

-4
x*
^ (8

c>

7=

2 V^

A ^~^

(8)

Ax

A*-M>

+ Ax 8
= hm
Ax
AA:^O Ax
8
8
[ J/ (8 + Ax) + ^/ (8 + Ax) + j/
]
= Hm
= 1. 360. r(0) = -8, r(l) = 0,
=
12
Ax-^o 3/(8+ Ax) +
2^/8 + Ax+4
=
=
=
For
the given function the equation
x
Hint.
3.
0.
361.
0,
x,
/'(2)
= x 362. 30m/sec.
2.
364.
363.
/'(*)=:/ (x) has the form 3x
=
365. nx
^. 366 L 2 tanq>=3. Hint. Use the results of Example 3
8

..

_-

xo

and Problem

1.

1,

365.

367. Solution, a)

f'(Q)^ lim

Ll

!_=

lim

Ax
AJC+OJ^/

AX

4- oo;

Answers

402

1^15^=
Ax

/'(!)= lim

b)

lim

2fe+l

lim

sinAxl

=1.

Ax

A*->+o
5x

*+A

371.

'

3x

-l

+n-1

6ax

= x x* = x

9y2
^
A

sin/.

y'

= 0.

390.

sin 2

xe x

391.

xV(x-f-7).
x 2 e x 396. e x

sin x). 395.

^-x

cot x

387.

/^arc sin

3x 2 lnx.

398.

402

_ tanh2x

403

toco.h^dnh*
cosh x
2

2x 2
405.

x
407.

x
411.

Vx

12fl6+18^

-|-2x arc

(1x
412.

//.

fex2
.

>X

16x(3

+ 2x

2 8
)

"~

420. 2

419.
2

2 sin x

15 cos

xln 2

sin

x sinx.

_2cos^

sin>

3cosx)

arc sin

....

414.

2(l+Jt )V^arctanjc

_3jari!*)l
/"l

x*

x2

/I
1-tan

+ tan

COS 2 X

.Hint. x
3 cos x

cos*x

Ssin^^

I)

418.

y.

2
3a fax-}-b\
fax
b \

424>

cos 4 x

L
!

-f-

tanhx

421.^^^---

423>
(1

427.

xsmh 2 x

sin 8 2/

422.

425.

cot x

+ x coshx.

sinh x

-3(xln* + sinh*cosM

413.

In

401.

*~ 1)
.

x /

* (21n

397.

2 2
)

-.1/j/l;-!.
X
f

416.

8 2

v5

*>y*

393.

(2x

41B

^f-.

"

X2

j/~i

arcsinx-f-

*\ arc coshx

cos x)

(sinx

arc sinh x

406.

I)

384.

2x

388.

.
'

4()4
1

1
^1
x In 10
x

1+1H/-A.400.
x
x
x

399.

on

4***

x 2 (2x

392.

x+

wy
1*

383.
sin

386.

394. e* (cos x

3 sinx.

2)

380.
OOU.

+ 5)

5x

377.

5 cos x

fW-4-9ft
VJA
-7- z-o

(x

ji

374.

JL

Hint. y

373.

2x 8 370.

2. 382.

7=

x arc tan x.

389.

rn

/2*-fl

l + 2x

369.

2 (1
2

+ 2.

o*7A
OIS7.
379.

r,

1
x

376. -*

+ dx)

(c

12x 2

8
.

+ b(m-\-n)t

U*U.

Y
385.

3x- 4

l/x
r^

1.

5x

/'-

c)

sin Ax
~ L^iJLJ=-l;
,.

lim

5x 4

368.

-JL
2x

375.

==00;

\\

*JL
'=

mat m

372.

^_

lim

= sin- +
2

+ 2 sinx

2 V^lSsinx

lOcosx

1
'

2 1^1
>
'

x 2 >^1-|- arc sinx

^
'

(1

+ x ) (arc tan x)

Answers

403

429.

430.

cos(x

-.

5x+l)

2*

1)

-}-

asin(ax +

433.

(2x-5) X

4 32.

434.

p).

sin(2/+q>).

X 2 COS 2
X

""*

?21i

435.

436.

sin

n2

x cos 2x 2 sin 3x 2

437.

438.

Solution.

a
l

(2x)'

,-

V^l

"~ 2

lA

(2x)

439.

4x

x/V

442.

10xe~*

443.

-p^jf

440.

446.

sin2

+2

450.

t
/

cos 2 In

453.
(1

-j-

In x) x

455. Solution, y'


3
H- sin

5x2 cos
4x

456.

(1 -j-x

(sin

5x)' cos

2 -

sin --

tanx
x
6

10).

cot x log

4 arc sin x)

-}-5sinx

e.

VI

454.

x\

x f
-^

449.

arc

+ x In

(1

x
(e

+x

452.

xlnx

1
.

2xi0 2 *

448.

^^

451.

x2

441.

447.

2.

445.

In 5.

pX
t

"~

2 J^jc

2*5~* f

444.

2x \ In x-jx \
2

'

+ sin

5x

cos 2

= 15 sin

= 3 sin

-=-

6 J)

x2~sin

5xcos5xcos 2

5x cos 5x5 cos 2

xx
3

5xcos

-r-

sin --

x 2 -f4x

3
457.

x'
.

458.

459.

463.

4x 3 (a

465.

2x 3 ) (a

5x 3 ).

474. sin 3 x cos 2 x.

sin

479.
.

476.

x cos 4 x

10tan5xsec 2

3cos*cos2*. 480. tan'*. 481.

-- 1

484.

sin

arc sin x (2 arc cos x


-

-f

]/"l

x2

.
'

478.

3/

sin

2/.

482 .

arc sin x)

xcosx 2

477.

5x.

2
.
OB
485.

K a

sin 2

+p

AQC

V^x

cos 2 x

486.

'

1+x 2

487.

491.

1^2x

xa

492.

arc sin

...

25x 2 arc

sin

5x

Answers

404

494.

- -

495.

1+cos 2 *
a w *lna. 502.
f

e-*cos3x. 505.

504.

(1

Psinhpjf)'. 539.

542.

/?*

499..

506.

548.

l(/tanx

-^-

544.

545.

sinx

y+

I1^3
-.
5

V (0)= =^;

/'+

(0)

tion.

or

567

do not

We

sin ' x
.

+ l/Tosxln a).

1
,' =

549.

when x

have

c)

6n. 554.

553.

/I (0)=

exist. 555.

y'

xy'=y(\x)

= e" x (1
566.
*

/_

/+

d) /'_ (0)

=/'+

(0)

+ (0)-0;

1,

a)

/'

x.

556. 2

+ ^-.

=^

557.

(0)=1;
(0)

< 0;

y' (0)

does

f'W-

= -1,

xarctanhx

546.

y'

550.

^L

x2

{
552.

(1

2coth2x. 541.

540.

y'=l when x>0;

a)

'-,*,.

b)

exist;

tanh 2 2x)

(1

cos2x

547. xarcsinhx.

sin 2xe

500.

x
p sin pO- 503. e" sin p*.

e*(acos p/

2xMna).

+ 4 sin*

_)/.

6tanh 2 2x
543.

x y In 2 x

not

"""

1+*-

^-'a-^fn

496.

2xcosa + xt

= 0,
1.

b)

/.

(0)

e) /'. (0)

558.

2
,

and

561. Solu-

= -(lx)
(1 +2x)(l +3x) + 2(l+x) (1 +3x) + 3(x + l) (1 +2x).
~
x).

Since e' x

it

follows that y'


2

Answers

40S

-24
573 '

>-

574.

'n*.

575.
*

2!li

+ cos x Inx

SIn

578.

576.

'

- sin * tan x)

(cos * In cos *

(cos x)

58

'

581.

"

.-- 4'<(2 "" <t)

586.

590.

594. tan^. 596.

601.

tity.

,;

603.

-.
2

Yes, since the equality


604.

2j/)

arc tan 2

=^63

(-2, -12).

45.

623.

26'.

626. (1,

-3).

620

624.

627.

</

arc tan

b)
d)

y-5 = 0;

^2y
=
x

+ 2 = 0.

l=0;

34 = 0.
13

= 0,

GA;

(3,0):

t/

2x

y=l

1;

x\

616.

xy

a) 0; b)

~^r 36

1=0; y = 0.

the

for

625.

21'.

633.

(0,20);

= 0;

6x
Jt
2//
x
2f/-fl=0

f-i
o

point

1,

1).

6;

t/

= 0.

at the point (2, 0): (/=

639.

14;c

638.

634.

At

x-f-2; y

(1,

15);

-1^
lo

2^

for the point (1, 1);

7x

= 0.
637.

45-.

y=2

a)

635.

5y-f 21=0.

622.

0.

c)

+y

= **- *+ 1.628. k = ^.629.

+ y2 = 0; c)
3 = 0;
e) 2x + y

1=0

= -Ilf

= 2x

632.

3 cosy

10

11

631.

r
10

615.

y,

6 08

an iden

is

605.

_=_

\nx-xx

-*-.

593.

-.

l-f3ju/ -f4(/

yVx*+y*

589.

f.

={

599. No. 600.

oo.

-.

602.

ex

tan

-tan^.592.

597.

1.

fir-*- ra.

588.

W.

a);

+x*)arctanx

585.

(arc tan x)X

=x

0;

636.

the
2\

point

(1,0):

at the point

41

=0.

Answers

-406

640.

The equation

Hint.

of the

tangent

Hence, the tangent

^-{-^-=1.
2y Q

is

ZXQ

crosses the x-axis at the point A (2x 0) and the y-axis at B (0, 2f/
the midpoint of AB, we get the point (* y Q ). 643. 40 36'. 644.
at
the
at
bolas are
and
intersect
point
(0, 0)
tangent

The para-

arctan-i=^88'
648

652

'

'

the

at

= 2jia

/1+4JT 2
t

--

~.

arc tan

=a
fl

-^-in/sec.

The

2u g/sina

+g

To determine
system. The range
on

velocity

the

Sf =

'

S,

= a;

cm /sec;

0;

656.

2jt.

q>

is

the

of

slope

657.

The equation

660.

range

2 8

5,,= -^;

of the trajectory
*

sin

\sy=x tan a

2a

The

vector

velocity

cm /sec

is

velocity,

cosa

axes

the

~
'

are

and

The magnitude

j-

the velocit y vector

of

the velocity

CO8

cm

40

of

he
2

360

at

the

g,

density

x2

681.

5* g/crn,

is

+~~210,Y

4
.

u = 5;

x*)
.

671

+ 2).

i*

cosh

682.

a
vl
//

6 79. y"'

= 6.

680.

~64sin2x

684. 0;

1; 2;

2.

of the rod

the density
2 cos 2x
669.

2 arc tan x

672.

is 0,

674
(1

The mass

666.

the density at
x*
668.
e
(4^

3(1

y cm/sec

m'/'sec. 665.

56x 6

667.

60g/cm.
2(1 ~"* 2)
.

673.

JT

at

is

670.

Q
,

- '6. 8 cm/sec, the area, at a rate


diagonal increases at a rale ol
2
664. The surface area increases at a rate of
2ji m /sec,
/sec

the volume, at a rate of 0.05


is

is

directed along the tangent to the

is

~
663.

the trajectory, eliminate the parameter t from the given


is the abscissa of the point A (Fig. 17). The
projections

Hint.

of

655

'

tan|i=

2_|_p2.

uj

tan^i

angle

S,

-y +2(p

}/~l-|-4ji

658. 15 cm/sec. 659.

654.

--

rt

~
n -~

>

647.

(1,1).

an

'

653.

point

Finding

).

685.

/'"(3)

The velocity

The acceleration, a 0;
0.6. 686. The law
0.006;
the
is
x
acoscof; the velocity at time / is
point M,
2
acD cos(of. Initial velocity, 0;
awsincof; the acceleration at time / is
aw 2 velocity when *
initial acceleration:
is
-aco; acceleration when
The maximum absolute value of velocity is aa>; the maximum
*
is 0.
2
n
(n)
(n+l
value
of
acceleration
absolute
is aw
687. y
n\a
688. a) n\ (1
\
x)~
is

of

4997;
motion of

4.7.

b)

_ ir +

i
.

689

=
=

Answers

407

d)

2 n jc

691

^(OJ^-tn

692.

1)1

a) 9/

8
;

b) 2/

+ 2;

XU+*-);

b)

sin

693. a)

---.
$

<

=.

696.

708.

-o

711.

x=\
718.

721.

725.

^b)
3

a)

and

AJC

No.

719.

dy= 45

^~^-

=l-

697.

f- .712. Ar/

714

d//

cf(/

= 0.009001;

=.
A:

M+

(AA')

720.

7,0.

d(/

when

717. For *

16

713. cf(l

^5 = 2^Ax, A5 = 2^ Ax +

722.

"J
256

//

d/-0.009.

^=^-0.0436.

0.0698.

'

709.

2
2
ft-4-.

'

726.

-j;

4o

"
707.

694 a > 0; b) 2e>al 695 - a)

777
4a

/"T^?.

c)

= 0.

= ;=i 0.00037.

723.

2xe~**dx. 727. \nxdx. 728.


x

y*xe
735.

738.
740.

~dx.

IJ

737. a) 0.485; b) 0.965; c) 1.2; d)

0.045; e)

5"=^ 2. 25;

739.
565_cm
^^4.13; ^70^=8.38;
200=5=5. 85. 741. a) 5;
/70=^4.13;
J/
^/10=^2.16;
.

"""

d) 0.9.

742.

1.0019.

743. 0.57.

744. 2.03.

748.

~(^)

sin,ln, +
fc).

753.

2
.

-.754.

(d **'
iy

(1

750.

~ + 0. 025^0

^)*/i

^640=^ 25
b)

749.

3.2"

sin

3.

l.l;c)0.93;
*( dx ?
.

(1

(dx)*. 752.

751.

81.

X2 )

-e

/a

__
Answers

408

755.

xcosa

sin(*sina + na)-(d*)

758.

No. The point

763.

(2,

765.

4).

2(

3U'

C<8,<1.

0<B<1.

0<6<

c*

770.

sin

error

(
_ = H

x \
(1 --

2!L_
8

= 77:.

get:

x \

-f

where

|2

where

J,,

= 8 *,
2

?-6*,

775.

both cases g

We

Solution.

6jc;

have

factors in

powers

of

#,

in

__L

^ + -_x -- x.

^1-f-)

40

Expanding both

= 0.

~ sin

x-^- +

sin g 2 ,

-L
.

&

_i_
*

we

*=

than =7
5!

762.

x=(x- l)-i (*-l)

b)'

less

is

exist.

j_
a

In

not

function.

769. sin

1.

~ + ^-

The

773.

768.

0<0<

x = x-

the

of

does

(2)

/'

=i+x + J+|J. + ...+ ^lL.+5jeS

772. Error: a)

1.

b)g = -~.

since

No,

discontinuity

|= +6 (x- 1),

^ = 8,*, 0<9 <1;

where

757.

is

=^;

a)

where

1)8>

x = ~x

/l

-^~^1H
//T"

y
---

-x \

8
1

J
y*

^~9~2-

Tne n

expanding

powers
778

oo

788.

1.

we

779.

~.

788.

same polynomial

get the

780. 3.

n<l.'

for

802

of

789.

793. 0.

781.
790*.

1.

795. -1

804.

Find

lini

-^

u ~>0

791. a.

0.

796.

805.

where S

hh

783.
792.

797.

806.

oo

1.

IT

sin a)

(a

807.

>

799.

1.

1;

for

800.

808.

1.

the exact

is

784. 0. 785.

oo.

for

^77.

810.

1.

expression

for

801.

e*.

R2
= -~-

x
x
=^rlH---r-n~-2

\2

782. 5.

803.

the

l;

1.

Hint.
area

3"^
of

the segment (R

is

the radius of the corresponding circle).

Chapter

(00,

III

812. (
increases; (2, oo), decreases.
oo, 2), decreases;
813.
814.
increases.
(2, oo), increases.
oo),
0) and (2, oo),
increases; (0, 2), decreases 815.
2) and (2, oo), decreases. 816.
1),
817.
increases; (1, oo), decreases.
2),
8) and (8, oo), decreases.
818. (0, 1), decreases; (1, oo), increases.
819. (
and (1, oo), in00,
1)

811.

2),

(00,
(00,
(00,

creases;

creases;

1,

1),

decreases

oo

820.

increases. 822.
j,

(00,

(00,

(2,

00,

(2,

oo),

0),

increases

increases. 823.

821.

(0,
oo,2),

],

de-

decreases;

409

Answers

(0,

824.

increases.

oo),

(2,

oo), increases

decreases; (1,

1),

and

00, a)

(a,

*=-

when

0)

and

828.

No

(00,

825.

decreases.'

oo),

827. ymax

1296 when x = 6.
extremum. 830. t/m in = when x=Q; t/ mln =0 when x= 12; t/max
when x= 1; t/ mfn =^ 0.05 when
0.76 when x=5=0.23; i/ max =
831. t/min^
2
833. #max =
832. No extremum.
x=^1.43. No extremum when x = 2.
-

when *=0;

|/

= 3/3
when x =
i/

840.

[/

=2

x=

when
837.

j/

2,

845.

y mln

when x = 2
value

is

m=

when x =
Af

75-

^= 12

for x

fjkJ:^

c
f/

= ~;// min ==0

min

for

when x =

1;

j- (fc^O,

for x

2,

1,

is

is

(2fe

r
".

(fe-^0,

1) JT

when x =

844.

f/

m?n

-.

-l when
4

when x = 0;

No extremum.

M = 7rwhen

value,

= 5.

//max--2

m^=0 when

852.

1; M = 27 when x = 3.
m = 1579 when x =

Each

side

whose

863>

-f-

1)

= l; M=JI
854.

-j-

when

m-

a)

M = 3745

when

be equal to

--

10;

864. The

Isosceles.

865.

850.

\.

The

side

The altitude must be

866.

terms must

of the

Smallest

849.

when x = (2fc

m=~

851.

must be twice the other

of

tlit

half

the

-g-

altitude

-^L
y3

the radius of the given sphere.

mln

-7--

Altitude of the cylinder,

where

f/

=6

= --

x=\

square with side

cut-out square must be equal to


867. That

848.

...).

b)

861.

4.

<7

side adjoining the wall

868.

846.

= g when x=l.
x~
greatest

= 10; M = 5

2,

when

842. y mln

= /2

s/"3 when

y mta =

839.

max

when x = 2K

K^3"

x=1

when

1.

y min

r/

max =

*,

The rectangle must be

base.

= 0.

t/

when

y min=l

Ji;

835.

836.

^==^

9rr

^ n; ^/max^^cos

l
when
853. /n=s
when x=l; M=-^2o6 when x = 5;

862.

when

fcjT

p=

when x = 3.2.

yg

x== _l.

856.

=9

x=

x=12.

#max

= 3/3"

min

ymln ==0when

and x

f/

(/

847.

834.

= V"3 when x = 2^3;


*=1; ma x = when x =

w ^n

t/max^^-

=2

max

841.

...).

^L;
yO

*=12

Scos^when

843.

when

m ax-= 5 when

1,

mm -0 when

838.

is

to

equal

radius
869.

the radius of the given sphere.

of

the

its

Altitude

diameter

base
of

R ]/

the

of the base

ro

where

cylinder,

870. Altitude of the cone,

i<

__

RV'2
--

410

__
Answers

where

is

the radius of the given sphere.

where

is

the radius of the given

cone

-jr-r,

where

871. Altitude of the cone,

Radius

872.

sphere.

876.

The altitude

of the cylindrical

+ *yo = 1.
^* XQ
-^-

878.

879.

The

the vertices of the rectangle which

3 \
-j J.

881.

~Y=.,

arc cos

4-

885.

a)

and

t-y-.

P min = y%aqQ.
spheres,

the

impact

with

m*V
+w

b),=

=;

If

n=

"I/

(if

this

is

battery

number

the physical

points
^"3")

points

3),

oo,

6)

2,

J,

*.

^_

884.

886.

not an integer or

is

meaning

M,

M lj2

concave up;

...);

oo,

-4r

,=

obtained

is

as

as close as possible to the

concave

2),

0),

(4/e

+3)

0)

1/~3,

(|/*

0)

and

-^, (4

Af 2 ( 6,

down;

0(0,

+ 5)~

l)it,

2&Ji),

f(2fe+l)y,
concave down(fe=0,

sas of the points of inflection are equal to

xkn.

0).

895.

|-V

and (1^3,

points of inflection,

concave up; ((2k

not a divisor of

(2,

oo),

892.

(6, --|Vo(0,

^3), concave up;

(0,

is

of the solution

must be

of

inflection

of

^L

inflection.
(00, oo), concave up.
12),
point
concave down, ( 3, oo), concave up; no points of inflection.
and (0, 6), concave up; ( 6, 0) and (6, oo), concave down;

inflection

and

-jr-

1,

M (2,

oo,

of

I/ ^M-

a divisor of TV). Since the internal resistance

is

(-~-a;

of

follows: the internal resistance of the battery


2
external resistance.
889. y
h.
891. (
o
893.
894.

coordinates

\f~Mm. Hint. For a completely elastic impact of two


imparted to the stationary sphere of mass m after
sphere_of mass m 2 moving with velocity v is equal to
v

concave up;

where

887.

the

6J/~2,

equal to the greatest of the numbers

AM=a

883.

7V,we take the closest integer which


of

is

The

velocity

888.

880.

that

zero;

= \l*

a^Tand

on the parabola

lie

The angle

882.

arc tan

of the ellipse.

the crossthe sector

is,

must be

part

sides of the rectangle are

a and b are the respective semiaxes

/HI

jr, that
angle of

the vessel should be in the shape of a hemisphere. 877. h

is,

R,

of the base of the

the radius of the base of the given cylinder. 873. That

is

whose altitude is twice the diameter of the sphere. 874. q>


section of the channel is a semicircle. 875. The central

/~2

-^

oo),

896.

concave

down

oV

(00,

concave down;

(&

897. (2/m,

1,

898.

i2,
[0,
\

...);

the abscis),

V&J

concave

Answers

[7^,

down;
;

899.

oo

V/>

concave

up;

0),

concave

up;

900.

(00,

3)

_JL

^=

(0,

oo),

and

(1,

W)

(Y*

oo,

inflection.

411

a point of inflection.

is

concave down; 0(0, 0)


concave up; (3,

a point

is

oo),

of

concave

1),

=
901.
of inflection are M (
3,
T and M (
=
=
= 0. 902. *=1, x^=3, =
x=
903.
904.
x.
2, y=
*,
905.0
= right 907. jc-=
= x, right. 906. =
left,
left,
y=-x, left,
= 2, left, y-=2x 2, right. 909. 0^2 910. jt-=0.
= *, right
908.
911. * = 0, 0=1.
912. i/^O.
913. *-=!.
0=1, left, = 0, right.
= x n, left; y = + right. 915. y a. 916. ma x = when x 0;
914.
4 when x = 2\ point of inflection, Af,(l, ~~ 2
917 ^/max^
when
min =
r
= 0; points of inflection Ai l|f
when
jf=V 3; m
~)
== 4 when x =
918.
y min = Q when je=l, point of inflection, M, (0, 2).
m ax
919- f/max^ 8 wnen ^ =
2,
m n = when = 2; point of inflection^ M (0, 4).
when A'^0;
of inflection
920.
M lt2 (Y5, 0) and
points
//min^
down; points

A'

1,

J.

1.

iy

1,

1,

l,

JC

JT,

)-

r=:

in

jc

1,

1;

//

Jt

\,

totes, x

923.

924. (farm

0=-xl.
~

1,

= 0.

= 0.

when

Ai

J2/"3,

lf

of

point

= --J;

Ai(5,
\

of

0(0,

inflection,
jc^=

*---

1;

0min

and /^(4,

(8,

and

4)
3,

940.

938.

= 0;
m n=
x
s

asymptotes,
x

0,

=4

when x =

V%

0max=

935.

2.

when jg=

^nax^

936f

~YH
max

when x =

of

points

= 0.

when

Asymptotes,

941.

= 3.
jc=

1;

1;

4;

max
j

ym n

=y

942.

and

max

\\

4r J
0.
y

=l when

x-^4;

929.

Point

0.

--97
930.
nnx =
4 when
max =

'16
,

931.

= Q and
=2

f/

= 3v
933.

;c

0(0,

/I

0).

932.

(8,

934.

(0,

End-point,

A(Y$* 0), 0(0. 0)


inflection, M (1^3 -f 2 f^J,

point of

(Q

x
t

m =
M 1>2 (1,

= 0,

points

of

and M f (1,
(when x = 0)

0);

1)

in

~~

=2

= 0,

2)

and

4)

End-points,

when

inflection,

asymptote,
939.
max -=2

K 2); asymptote,
4 wh'en x==4; point of inflection, 0(0,
~~
^
3 /
4
4 when x
when x
2, 0mi n

inflection,

when x =

'

and

=
x

asymptotes,
r

asymptote,

0);

M lt (\
\
x--2 and

= 0.

and

and

937. Points of inflection,

0).

asymptote,

943.

0);

V
M, t(l,
when

=2

x~\\

vvnen

m in^=

0);

x.

=4

928.

2 V 2 when
2) are end-points;
ma x
are end-points. Point of inflection,

B(Y$*

0=^

0);

asymptotes,

-;

=-

points of inflection, 0(0, 0)

asymptotes, x

77-];
y

T2); asymptote
O.
asymptote, AT

1/2,

when x--0; asymptotes,

asymptote,

when

when x-=2; asymp-

inflection,

m jx ==l whenx=l;

-^)'.

inflection,

M(

of

points

926. # ma x-~

^2

mm

inflection,

= 0,

min

(l,

f/

of

/y

inflection M,
lt
4 when i=l;

of
1;

point

wnen
0max~'Qo

925.

asymptote,

and

Points

922.

when jc=l;

when x^O;

^max^

when x

max

=3

927. y m n

921

when
944.

= 0;

0min^

= 4;

=K

asymptote,

T7=
1/2

0.

0);

when

x=2.

Answers

412

J/max

1/~JT

TT= when
V 2
-o-

of inflection,

of

M,
x

3a,

= 4;

949.

=2

rnax

of

point

/min=

953.

=g

min

f/

2
2,

of

= 0;

when x = g;

*-

as

1+0

-- ,0

jc

points

j n+

2Jfeji;

t/

~+

fare cos

On

the

*=n;
M, 4
nal

2fen

[0,

(fe

= 0,

= 0, # =

1;

n + kn,

-j-

~^-J+2fejx,

m in

2 20,
2it],

ji,

...);

i/

max

when

2A5Ji

(fc

t/

max

Odd

of

when

of

x=

Asymptote,
x oo).

2;

points

defined on

max

the

=
2,

...);

function

with

1,

Periodic

inflection,

~;

V$

M k (kn,

ml n=0.71

when * =

86;

^=0.37);
J

when
and

0)

961. Periodic function with period

1;

956.

m in =

(0.57,

periodic function with* period

when x = 0;

^=

2n. y m n

inflection,

x=

= 0,

39

# = 0.

"""472)-

x (as

period

j Ji-f2/5Ji;

-y

not

is

asymptote, A:-

1^0.63;

with

960.

points

yg )^l5j.

962.

jc

the function

*=

when

when ^ = 0; points
0.95).

when

max

and

(-rr==

~V,

(e

t/

^e ^ 7

= 0.

when

74

2,

ji],

J.

Oj.

1,

950.

1,

y m{n =

j+

wn en
y

asymptote,

);

f\ e

function

interval
i/

inflection,

^3

y mln ==

2ji.

Ai

inflection,

of

948. f/max 1111162

f
lf 2

= -F ^0-54

max = ^2" when A:=

Mk

of inflection,

= 0.

asymptotes, x=*l.
(when x-+
oo) and y

period

max

Points

'

~\

inflection,

959. Periodic

point

1.33);

Asymptotes,

when

Nk

of

when x=l;

inflection,

(limiting end-point). 955.

interval

x=

f/

inflection,
1>2 (1.89,
957. Asymptotes, r/
Q.

958.

of

point

point

point

asymptotes,

point of

0)

when x = 6;

/2
max =

t/max^

0.70);

0(0,

3
= -0-7=

947.

-n-l,

951.

^14.39,

954.

*/

M^J

^ = -4=.,

wnen

x-*0.

946.

points of inflection,

(e"'*

3,

/ 8 ^ 2 i/"~
^2

when ^ = 0.

asymptote, y

inflection,

>:

j-

a,

2 (

Af,

= Q.

asymptote, y

and

-j-

when * = 0;

-^

of

w min

when

y-+Q

/iooy

inflection,

952.

945.

j^r=V, asymptote, x = 2

when

jc=l;

when

x=l

/12,

points

//

/min

5-

points of inflection,

inflection,

asymptotes,

J ')

(3
3,

*.

2;i.

2xc.

when

0.13) and

On

inter-

i/max=l when

__
Answers

Ji

^nin

'

When * =

=l

(1.21,

when

= 2n;

M,(2.36,

0.86);

A4 e (5.50, 0).

of

points

with

function

when * =
3

asymptotes, x=-r-Ji

Even

965.

=
c

^+

Mk

inflection,

+ ^Jt
kn,

^min

skill:?,
function

A:

= arccos

^zr

(/

lO^
with
m

J^

(jt^arcsin

On

2it.

the

when x^Ji;

0(0,

0);

M k (kn,
function,

//

and

Odd

m in =

n + 1 +2^n when

*-*+oo).

inflection (centre of
974. Odd

Oj

Even

966.

when

^min=--F-

n]

t/

(/j

ma x

M!

t/max^

-y

Oj;

-i/g)-

0,

2,

1,

57

Odd

969.

when

...).
A:

function.

t/

JI

A;

(/

+-fr

+ oo). Point of inflection,


+ ?JT when
max = -^-

when

972.

(/

JC==1;

symmetry)

of

points

/JJT;

2,

1,

(as

inflection,

2fcrc);

when x =

^max=

(0, Ji);

"

-^-^

--

asymptotes,

...).

(as

971.

x-*

Even
and

oo)

(node); asymptote, y

Whe "

X=S ~" I;

y = x + 2n

(left)

P oint

and

l.

y=x

= 1.856 when
-=
M
of
0,
inflection,
+ (when
asymptotes,
point
-^y
and # = * In2.
(as ^-^+00). 975. Asymptotes, # =
oo) and y = -^

(right).

l;

function.

t/

m in=1.285 when
f

^-^

(1,

function.

[0,

^= when

2k 4-

=^.^1

#==

oo)

970.

j JI+/JJT.

vvhen

0.34).

= -J +
1
asymptotes, x^-y -^ (6 = 0,
= 0; asymptotes, =
t/mln^ when x

+ /en;

-j-

1,

=
-

of

points

interval

inflection,

57)

n;

...);

[0,

fejt)

(fcjt,

'

asymptotes, x\.

2,

1,

the

of

points

(2

graph

= 0,

^-)

when

inflection,

interval

function. Points of inflection, M*


Even function. End-points, 4, 2
83,
Af J(1.54
(cusp); points of inflection,
lf
of

0.86);

(- (-/I)-=

-.(/S-i/S)'
Odd
points

of

-=.

= arccos

967.
968.

Limiting

(4.35,

= 0whenx-n;t/ mln =

when

max

0.86);

period

On

when

Ai^O.36,

asymptotes, *

...);

2n

wnen ^==0; points

period

= -F=

(fc

#min

V~2
~
ym n = L

2n.

+ 2foi

2,

1,

period

t/

arccos-^;

= arccos --7=)'

(arc
periodic

with

when ^ =

rr^s

Af t

= 0,

(fc

-y- J

0.86);

period

ji

inflection,

964. Periodic function with

function

periodic

*=

wnen

M 4 (3.51,

0);

Periodic

963.

= ^- 7

ymax

413

JC=1;

t/

t/

max

ji

Answers

414
976.

j/

*=1;

m n =1.32 when
i

2jt.

period

t/

min

asymptote, x

when

x=^-ji

-g-

= 0.

+ 2&JT;

t/

...); points of inflection,

2,

1,

arc sin

and

^- +

2;ri

i/^ + 2/m, UlA

M^ Urcsin

wnen * =

max ^=e

= 0,

(fc

with

977. Periodic function

978.
(2fe+l)n,e
4(0, 1)
End-points,
^
B(\, 4.81). Point of inflection, M(0.28, 1.74). 979. Points of inflection,
4.81 (as *-*-|-oo).
0.21 (as x -+
(0.5, 1.59); asymptotes, #
oo) and r/
980. The domain of definition of the function is the set of intervals (2kn t
where fc
Periodic function with period 2ji.
...
0,
1,
2,
2/jJt-f-ji),

A^
and

t/

when * =

max

The

981.

domain

of

Points

-^-

(/e

r/=1.57;

lf

2,

1,

2/jJi

i/-^
1

y mln

as

-*

Domain

of

(limiting

period

+ kn.

i/

min

Limiting

asymptotes,

...);

definition,

increasing

<~

2/jn

]x

y mm =l when

2jr

984.

Asymptote,

985.

End-points,

^ 0.69

2ji.

period

monotonic

0;

with

986.

987.

>

end-point).

when ^^0.

^-^ +

2,

1,

definition,

with

function

Periodic

kn.

of intervals

= 0,

asymptotes,

...);

asymptotes,

...);

the set

function

Periodic

1.57

(fe

983.

as

-+

0)

(2fcji,

0.

2,

p0.37; y

integer.

1,

57);

an

is

of

...).

= 0,

,(1.31,

*=

Mk

asymptote,

(fc~0,

is

2,

1,

Domain

982.

2fejt.

0,

(&

definition

inflection,

function;

2&Ji

of

where

(2fc+-jrjJi

x=

when

4 (+0,

end-point,

Q);

t/max^ 6

^ 1-44

when # = e=^2.72; asymptote,

(/

of

point

!;

inflection,

when / = (0 = 3); y mm = 1
M, (0.58, 0.12) and M 2 (4 35,1 40). 988. Jc inln =
when / =
(x^3) 989. To obtain the graph it is sufficient to vary / from to 2jt.
a (cusp) when
^min^ awhen^-ji(// = 0); ^max = awhen / = (#-0); min 1

t/

+-2-

3T

inflection

when

fl

(cusp)

when

<

7--

V'Se
\

when

8
,

-y

U = 0);

points of inflection

7jT

-j-

*=

I/" 2,

i.e.,

to=

^)l

^max =

XJ.

i.e.,

44

r wnen

m in=

Jf

ma x
5jl

r4

990.

3jT

= -j-

when

(x;

= 0);

-TT=;

i/

= 2x

when

_ /"^"t

asymptotes, jc=0 and y

(cusp); asymptote,

wn ^n

-*

= 0.991.

+ oo.

992.

l(jK:

= e);

and when

points of

t=

xmin =l
i/

m in =

andymln =i
when

= 0.

Answers

$93. ds

cosa =

dx,

415

<*>

where

=
x
sin

tanh

998. ds

-.

997.

= 2asin~rff;

cos

ds

sin--;

cosa=

cos/; sin

sin

=a

sin/. 1000. ds

"(p;cosp=

cos

dx:

d^;

= cosh

cos

1002.

a=

-~-

999. ds

cosp

V^l +(p

ds^

995.

tit

I/

(X

~3asin ^ cos/

62

Ka 2

c=-

d<p;

ds==

dcp;

tp*

cos*

1003.

=
an2z

zzz

K^36.

1006.

4^

/(-=

1010.

^"3^2"

1007.

1012.

both

at

1005.

"

vertices.

&2

;
'

ds==

KB =

1011.
(

^).,0,3.

(-!"*..

,=

a sin 2t

1016.

!023.

.
|

^a

-ilfl,

3)

1019.

-8.

-^-

a cos

1017.

-pK

= ~(X

p)

and

sin

= cos 29.

K^
[

-f-

R^-

= |a/|.

1018.

1020.

1026.

1009.

1014.

-^-

f k2

d(p;

3
1015.

1004.

cos-|-.

/?

(2,2).

l025 -

(semicubical parabola). 1027.

_
8

^c

8
,

where

=-a 2

2
.

Chapter IV
In the answers of this section the arbitrary additive constant

ted

for the

sake of brevity. 1031.

aV.

1032.

2x 3

+ 4* + 3*.
2

is

1033.

omit-

416

Answers

1037.

'-+--

--'
a 2 x---a'x

1038.

J.

j/

<*

''---~.
'*'---

JC .

3x 2

3x*

fi

1040.

'

13

4//2

1042.

+
1043.3=

In
-4=
2/10

1044.

= sec

-x;

b)

1045.

ln(x+

/4 + x

1048*.

tanhx. Hint. Put tanh z

b) x

1;

/"lO

-^~ln (x+ /x ^).

1047. arc sin

x-cothx.

^j~

1050.

'

_i

J
Solution.

numerator

the

|.

^
1063.

1065.

--^-.

cotx

1052.

we

1057.

c--l|.

Inlx

1059.

|.

~~'

3*- 2

]/

_Jiy(a_-

1062.

y.

1066.

^=

2
).

In

/7 + 2

+ b + x V ab

+ V7 + 8x

1064. 2

~.
.

fll

,,

-L arc tanx

get

1054.

I06li

:ln

In

1049. a)

Solution, f

In (2

1073.

2/2

*
1

1067.

Put

denominator,

~+

(x+W

/x +l.

Hint.

Solution.

1056.

1060.

"

1058.

the

by

-I' + T

sin-

r-j

Dividing

arc

1053.

1055.

x.

tan

arc

1046.

).

x=l

1051. a in

tanx

a)

1009.

1068.

1072.

/"2

arc

-~=

tan

arc sin x

Answers

1075.

^ In (a * +
2

1079.

V^=

31n|x +

1077.

2
ft

=l n
5

1078.

1080. -i arc sin

^.

^*

1). 1076-.

-ln|jt

arc tan

)-}-

417

^-

1081.

are tan
1082.

1083.

1|.

^(arcsmx)

1086.

\-e~

1090.

2
1092.

1084.

,/

-H-

r?

3 Ina
i

1095.

In

1097.

e*

1|

+ 3)

In (2-

3 In 2
1102.

1098.

1109.

Xcos(logA:)

Hint. See

arc sin

HOI.

__

!. Hint. Put
in

1109

lilt.

sin

jc=

~ tan

sin

(fljc

cos 2).

tan(x

).

1117.

-^-cos(l

^'.1119.

Xln

sin

1122. 5 In

sin

4-

1125.

In

1120.

.
|

-^ cot x

ln|sinx|.

1121.

21n|cos/"x|.

1123.

1.

tan*

1118.

r).

ln|cosx|.

ft

--

1110.

tan
1

lOx

1112.

ft).

a In

1116.

+ ^)-

In

1108.

/"x.

~(1

008(0

ax
1113.

arc tan (a*).

jJ

1104.

e*

^-cos2ax. 1107. 2

1106.

hint

'

!n~5"
1100.

_^L_J.
1103.

2sni -^r.

1096.

Hint.

-Jr
V

1099.

(a

1105.

1126.

^.

|sin

1124.

(a

ft)X

1127.

1129.

1128.

4a

1130.

sin

1132.

1131.

-^-tan -^.

U33.

-|
5

1134 .

_3^_lJL.

14-1900

M55.1

U 36.

tan

j).

+
2au.)
l(ln tanf
|

23
Answers

418

4- In
3a

ocot3x|

tanh

1140. In

1144. In

-r-

sinhxl

ln(x

1152.

lr

1155.

1153.

In

tan x

fls n

n49

1147.

arc tan

J/

1150.

In

ln|x*-4x+

Incoshx.

1143.

ln|tanhx|.

1146.

"-

1148.

Xarc tan

1142.

*/(5=x^.

1145.

1139.- +-

2arctane*.

1141.

1138.

_. -------

1137.

| )-

]/
1151.

sec

1156.

1)

'

1157

*. 1158.

arc sin (x 2 ).

1159.

&

in

1160.

taniw-x.

/(^
Vx

2 In cos

1165.

1172.

sln

1168.

1169.

1170.

-l^arcsin^A

1173.

1175.

\n(e*

^21n

1171.

tan

1167.

tan^-l.

K.

=.

-i-ln

1166.

a
f

1181.

/"l
<40 -

1187.

2
jc

- 1=r:
7

1182.

1185.

arc sin

ln(secx+

arc tan

/tanx\
I
== }

1183.

-2cot2x.
1

l/~sec x-i- 1).

1186.

.-

2).

= In

-p=.

+ 2arctanx.

ln|x|

1174.

Ve*x

tan

jr

ln|tanox|.

arc cos -^

1184.

V 5 -f

sin

2x

sin2x
.

Hint.

=
x

1189.

s inh(x

1190.

-Latanh*.

1191. a)

-~=arccos^

when x

> ^2;

b)

In (1

+*-*);

Answers

(5*'-3)';

c)

d)

2arctan \e*

1195.

V 2x4-1

aarccos

arccos

1204.

if

> 0,

and arccos

In*

1201.

-.

-~ V2~x -^ V2-x

1202.

1196.

1.

Putx =

Hint.

TT arc sin x.

ln(sinx+- J/"l+ sin**)-

e)

1194. In

419

In

21n Inx-f
|

--

2
.

~J

x<0*)

if

Hint.

'

Put

= ~-

substitution x

be used in place

may

~
x arc

1216.

sin x-}-

x2

1217.

^r~-

1214.

^T^Y-

the

of

(9x

In (1 -|-x

x--^
*

xcosx.

sinx
12 18.

x arc tan

3*

st

1215.

Note. The

substitution.

trigonometric

1212.

x.

^
o

igf
y

of undeter-

xV*dx = (4x 2 + Bx + C) e*x

= (Ax + Bx -f C) 3e + (2Ax -f B)e9X


2

3Jf

* and
Cancelling oute
equating the coefficients of identical powers of
3

122
B=

whence ^

-^-;

=Q

(x)e

ax

where P

polynomial

Hint.

*)
is

).

after differentiation,

X 2e* x

6x-f- 2). Solution. In place

by parts we can use the following method

of repeated integration
mined coefficients:

or,

xlnx

1211.

a|.

1213.

1206.

good

See

rt

of degree

Problem

c=

2f

= 3C +
In

the

x,

we

get:

6eneral

^ orrn

p n to e ax dx =

is the given polynomial


of degree n and Q n (x) is
n with undetermined coefficients 1219.
e~* (x 2 -f-5}

(x)

1218*.

1220.

-3e

"(x-h9x

+ 54x+ 162).

Hint.

See

Henceforward, in similar cases we shall sometimes give an answer thai


for only a part of the domain of the integrand.

Answers

420

Problem

cos 2x

mined

1221.

1218*.

Hint. It

where P n (x)
polynomials

px dx

(x) cos

= Q n (x) cos PX + R n (x) sin

arc sinx

+ In

sin

-|-

1236.

x8

Inx

O^2

-^

(x+ V^l+x

Ay2
^Ar

tan ~^
2

arc sin x

-j
4

2,

e* (sin x

1232.

xcotx-f

1230.

'

1228.

sinx

cos x)

x2

ft

+2 V
,

get

+ -^ In (1 + x

rt

1246.

1+

- -- -- ,

In

cos 2x

]/T=Ixx

""

xcos

2
<rt

'

x2 x

and

+2 V

1248

'

(2 In x)

'

2xsin (2 In x)

10

a=x

du = dx

x (arc sinx) 2

1244.

).

xtan2x

._

1247.

X
2

In x
2 In x
~^-^--

^ arc tan 3x-^ + ^o In(9x +l). 1243. ~-^X

1242.

/ cos2x -2sin2x

1240.

x.

fy-*

1238.

-1).

Inf-^-x.
+X

1245.

2x.

1237.
1,

1239.

x arc tan x

x2

-/m -_

+ -3x.

1).

arCMn *

Xarcsinx
Xarcsin

1234.

(x*+

[ln(lnx)-l].lnx.

X (arc tan x) 2

we

1225.

1250.

1218*).

"

cos (In x)J.

x In

1229.

x*.

1238.

1241.

(x) are

ln *

arc tan x

1227.

1231.

and R n
Problem

(x)

(see

ij(,

l/T

-|-

undeter-

of

Qn

2xlnx + 2x.

xln x

1224.

/"xlnx

method

the

apply

the given polynomial of degree rc, and


degree n with undetermined coefficients

is

of

^-Inx--^.
y
o

1226.

1222.

form

1223.

2x

also advisable to

is

coefficients in the

Pn

sin

4^8

Whence

^ ( x ~r

x dx

and

x*dx

2(x +l)

1)

125K
Hint.

xy a

^x* + -s-

a=-

Utilize

arc sin

the

identity

Solution. Put

j..

x;

we have f }/aa

a ~2

1252.

= Ka x and
-x'dx = x ^a -^-

xa

f -/-^2
\

Ka

=.
,f
x 2 dx+a2 l

dx

whence

-;

a*

x 2 ].

^a2

Answers

Consequently, 2

y~a*x

l257

Fn

arctan

x*

2
-fa arc

Problem

1258

TTT

-ln

1259.

~.

sin

1252*.

Problem 1252*. 1255.

Hint. See

Va

See

Hint.

arc sin
-|
o
2

dx = x

421

4A

^~-

/
f ~<T=

ln( *

1260. x

2).

arc tan

7=-.

Larcsin^P^.
^

,262.

x-f31n(x

1261.

arc tan

1263. arcsin(2jc

1).

(jc

--T

1256.

2t

arc tan (x

(x

-~

1254.

arc tan

'

~ V& +**

1253.

(A:3).

1264. In

1/2

1265.

-2 V -x-x* -9 arc sin

1266.

4jc-f5.

=^

In

1267.

'71

5/"1
1268. In

?=. 1270. arc

arc sin

1269.

x
arc sin

1271.

"*

,273.

9
arc sin

+ 2*

2 In

1272.

sin -

y$

+5).

1274.

2x4-1
-

arc tan

1276.

1278.-ln|cosx + 2+
~'

y\

1279.

1281.

jc

In 2 x

4 In x

+ 31n|x

1283. In

2 arc sin

l!L~.

1280.

o
3|

31n|A;

2|

5*

+ In

1284.

1282.

(x-4)'

- -b

x
\

+a

.1285.

1+*

(*-!)'
1286.

30

27
1288.

1290.

11

4-j
1289.

2U-3)

-^-r
.

49

5)

49 (x

+ 2)^4*

x~5
x-f-2

1292.

1293.

ln|

x-3|-

In

U~l +
[

In

Answers

422

X arctan(* + 2).
x*-x V

arc tan

In

1300.

+ji-i*-

_|arctanx- TT^-jT -^

1305.

1304.

5=41.

arc tanx.

In (x

1310.

y ln|* +H
7

ln|x|

FT

^/(a* +

6)']

1315.

56

X -6

-3

In

In

1324.

(A:

1
|

2 arc tan

-J-

x7

1)

tan x

_ J_ J

'

Sx^Sx*

2 arc tan

1317.

-^=.

1322.

I)

arc
+1)-^o

^3

Xarctan]/|-.

=il.

3 In

7(jc

(l+/l.

3^/1-6 ^/"x
1320.

l)"

(A:

._-

4(x

9(jc_l)>

= (*

arc tan

1312.

1313.

1
.
i

Put

Hint.

*+l

tan (x-^1).

1311.

1306.

X[2

1303.

4r (8

+ 21n

'+4^

arc tan

1298.

1299.

1302.

arc tan x

,,.,
1297.

In

1295.

K"-\

+ arctan(*+l).
-

= arc tan

In

1294.

1316.
1318.

1319.

+ J/l + 6 arc tan


|

*/l.

arc tan

^1-x.

1323.

(jf_2)+-I

In

where

423

Answers

=7+l).

1327.

*-li

-+i

1329.

-arc sin -^

1330.

1331.

4/7=1

-* + l).

-retan

-2

-cos.,.

(*

1340.

1334.

where

1337.

1332.

+1)

,336.

1
.

1338.

sin

1339.

g-sin'x.

-cosx+-jtos>

1342.

*J^-**. |841.4eo.|-^|.

>-.-.+
--^=

cot*

_
7 i_sin6x.
144

1347.

__-.
35

,849.

-1

21

._.
._

, 344 .

3 In

tan x

cot

jc

Jt

1348.

tan x

tan
+ -^
o

+2lnt.n.

+ To tan

1351.

^.

1350.

1353.

COS 2

"[

ln

,357.

3 cos*

*+i

t,n|

_S2*lf_in|sinx|.

1354.

1358.

cot'

sin 4 A:

In

8siu*x

+ cot x + x.

1361.

1362.

xlB

-1 J/55?x + -| J/cSS*-^ J/cw

SrRj{"^
.

-cosx.

,366.

.369.

arc tan

^'
.

1367.

,370.

11

-tan! --f

1359.

!. 1363. 2 J^twix. 1364.

where I

" iain

-+3-.

1365 '

.368.

lE_^M+<P).

J-J^

cos-

137 ,.

!!H +

Answers

424

5*

sin

sin

7*

^
Jo

tan

-!-

1374.

tan

In
-f
*

1373.

1376

--

tan|

cos 4* --5- cos 2*.


^ cos 6* -^
z4
o
lo

1372.

'

*
In

1377.

1378. arc tan (l


\

tan

We

3 cos x
Solution.
Whence
4- P (2 sin* -{-3 cos*)'
12
5
.,,
Q

a= !3'
r

iiC

-f-

arc tan

1381.

5>

have

lO

-^^

fraction by ccs 2 x

= j2^_

<.

In
||*--^
lo
id

2 cos x

===

+ 2p = 2
p3sinx + 2crs*

= 3,

3^

(2 sin x

3a

and,
,

2^77+3^7^

J
rfx

Si

1379.

3 sin x

put
2u

We

P== -!3-

+ tan 4V
^

2sin *

3 cos x)
consequently,

12

==

13

5
2

ln

1380.

In |cosx

situc(.

1*3

Hint. Divide the numerator and denominator of the

-=- arctan

1382.

See

Hint.

V*

Problem

See

Hint.

Problem
1384.

1381.

1381.

4 In x

1387.

V2

/" 2 -f

sin

Vi

sni2*'

~~"

/\

r~~~

/"

t388.

4-ln^^^.

arc ian

1
.

Use the identity

sin*

sin*)

sin

In

cos*

-.

r
cosh *
!

1396.

2coth2*.

12
or

-j=.
O

arc tan

1394.

5)

+m+
1401.

-JT-

In (cosh

1397.

If

sinh*

*)-^^
2

1399. arc tan (tanh*). 1400.-~=.arctan

* iA-?\l

(f V

Hint.

'

*
,

cos*

*
*

-j-sin

cnth 8 *

coth*

1398. *

identity

+ 21n
'

tan

1393.

tanh

the

arc tan

x+cos*

1-j-smx

^>

1395.

1390.

sin*

Sm
1392.

Use

Hint.

sit

(2

1389.

sinh *
^
*

sinh 2x
r
*

^^

5
,

.,

..

..

...

Hint. Use the identity

cosh*

-^ (sinh*
%

-J*

cosh*).
'

1402.

-7=1 In (/" 2 cosh* 4-

^2

^ cosh 2*).

425

Answers

1403.

~ ^2 + x + In (x +
2

1404.

H06.

1405.
-

J/"^IT~2

1407.

|~

1408.

3+ Kx

8ln| x

In

6x

2x

+ +2
1

1410.

7|.

+ yV

^ VV

1409.

In

17)

4|*

6x

JT

97

**

1413.

--Uarctan.
2

-54-~']

1416.

1419

1423.

4-

4 sin 4x4-c<
c s4x:\

sin

_~Z + o

._.

142

*!
a1)

V>

2*

+2

|arctan(2x+l).
1436.

,.

V\+x*)2 V\

arCCOS (5x "~ 2)

/.^-x'.-

2x+10-9...

4 arc tan (x
.434.

n
cosxsin -'x

l428 '

1430.

...

1432. In

arcta "
+,3
2?

tanx.

e*

1425.

sin2x

xln'(x+

1424.

1426

-TT (2

1422. x

lnk*-l|4--O

-25.v'-3.

Y*

1415.

1418.

'

1417.

1421.

1412

L=ln

1414.

1-x

1411.

1).

1431.

1).

_+

1433.

j In

1437.

1435. 2 ,

arc tan

In

x'

Answers

426

l-x 2
2

'"

'

2x

arc tan

1439
l439

-^r

1440.

3
1442.

l"\

In

/2X-45VJ/(2jc)

1443.

J/5^x

x 2 4T,'

4 In

I)

^lVr>_ 9 _
arc sin (8*

(8x--l)

1458.

----

arc tan

,r-

where

cott-

1462.
*

cos5<

1463.

3cos5x
tan

T^ In

~sin2x.

1467.

X arc tan

+2

In

+ secjc|-i-cosecjc.
2

A.n|\
-^ ).
>

ax

y 2

1473.

In

tan *

'

r-^-^.

ax).

1477.

1475.

^uu-1).

~ (cos

|~cot

*-

cos 2

6)

t^ +

tanS *

1470. arc tan (2 tan

an

1472.

^"tan

x tan
1478

x+\).

T -r=X
1

-^= xarc
j/

+ 2+
-

tan x

,465.

-= arc tan

1469.

hi

In

ln| tanjc

Xarctanl

X In (sin

tan 2

1)

=
1461.

J/JT^t).

1453.

1471.

(1

1450.

1452.

1466.

J/x+1

-1

1448.

X arc sin

--5-^

1444.

1446.

tan

3*+--

?=?

|A3

tan

1479.

^74. J_ x
a

In |cos 3x

le-.

1476.

fin |O=5-

427

Answers

1480.
1483. .n

J481.

^-?

+ ln

sinhxl

1485.

4- 1" cosh 2x. 1487.


5

1486.

x.

gL-^ + lln |e*

1488.

|l+cot*|-co tje .

1489.

2|.

*coth*4

i-arctan^^-

-1 */(?

1490.

x^-'+fir
1494.

In

1496.

(coslnx

lcos5*~

t499

'

+ sinlnx).

|sin5*V

1497

1498.

x2

2) arc

tan

1 [(^

y x sin 5x + 3x cos 5x +
-|

^^r

'

1500.

In (2
-'
,

Chapter V
1501.

vjg~.

1502.

a.

fc

x=\

Hint. Divide the interval from


tervals so that the abscissas of

metric

progression:

sina

Problem

See

Hint.

xl

x =^l,

the

=x q

in

2)

at?

1512.

-f- cos
x

1,2,
sum

-.
x

5-^ = 2 sinh*.

**

1516.

gral

in o

/i

cos

1513. *

1517.

.,

xn

= x$ n

Hint.

'

'

Solution.

1\

+ ~~T" J
,

'

formula
3da

1)

1511.

1518.

the

1508.

1,2,3, ...). 1514. In

sin x.

In.

1506.

Utilize

156.

the x-axis into subindivision should form a geo-

cos n-}--^ a
^~
2
V
2; J

1510.

1/1,2,
^r-^V^=+ 7T +

X on the interval

of the function /(A:)

1505.

on

cos*.

In*.

1509.

x b q*,

_l

2sin

=5

points of

x2

1504.

3.

to x

1507.

1505.

+ sin2o+...+sin/io =

- r -;

1503.

2.

The

be

sum
as

mtc -

lim

Therefore,

[0,1].

may

~~

1515.

sn

-*oo

-^
2

1519. In 2. Solution.

--o-+."H-- \ may

M\ 1+1arH
[

The sum

i+Ji
n

n /

s"
n

= -i- H
n+1

/i

\-^ +
+2

. . .

H
/I

be regarded as the integral

+M
sum

of

428

__
Answers

the function /(x)

form

the

t520.

p
1526.

xk

=l+ n

= arc tan

tanh

;
1

1552.

1555.

-=.

1556.

1524

"T
4

^2- =

f
J

1525

^r
o

-1=

-4o

arc tan 2

~.

1533.

|o

15 38.

~.

1543. sinh

^-r

if

1557.

Diverges.

= -i-

p^l.
p>l;

2ji.

(*

1549.

if

1547. Di-

p<l.

f(x)dx+{ f(x)dx where

(jc)

= x^-

1564.

l + ~ln3
3

x)?"

(1

n.

Diverges.

1565. 3
1569. Converges. 1570.
1566. Diverges 1567. Converges 1568. Diverges
verges. 1571. Converges. 1572. Diverges 1573. Converges. 1574. Hint. B(p

1563.

~.
2

1554.

1559.

e J

1550.

Diverges.

diverges,
1558.

1546. 2.

cosh

2 \

+ -|-sinh

Diverges.

t560. ,-!-. 1561. Diverges. 1562. -i


k
Ina

~.

1534.

1539.

In 2.

^-

if

diverges,

1553.

1.

tang

1545.

In 2.

H-*

1529. arc tan 3

-^-

p<l;

if

1537.

1542. arc

(In 2)

1551. Diverges.

1532.

^.

%-+\-.
4
o

^7= + ir.
9^36

1541.

1548.

1531.

1523

35~-321n3.

ln-|

oo

1528.

1536.

tanh(ln3)

verges.

sn
oo

lim

Therefore,

= 334-

152

1522.

In

1530.

~ In

1544.

2 ..... n).

!,

where the division points have

[0,1]

n ->

1527.

1540. 0.

-|- 1

-jln-|

1535.

on the interval

(&

1521.

j-7

--

--

1
;

since

lim

/(x)x

Con%

q)=

-^=l

and lim

^)

(1

"^/(x)=

1,

both integrals converge when

p<\ and

q<l,

-> i

that

is,

when p>0 and

-x p

~l

Hint. ?

1575.

(/>0.

(p)=\f (x) dx+(

where

(x) dx,

e~*. The

first

integral converges

when p>0,

when p

the second

is

IL
2

1577. 2 /~~2

arbitrary. 1576. No.

("

In

/7rff.

1578

00

1580.

1584.

C^f1+'
f !?"**#.
1
J

2~

1585.

1589. 4

n.

1581. x

-^

1586.

1/5
1590.

~ 1"
o

= (6

a)/+a. 1582.4
-^

2 1^1

H2.

1591. In

4-^

^-|y

1587.

21n3. 1583.8

1~~.
4
1592.

1588.

~ + T^

^JT.
21^3
/""S-

1593

nr
o

Answer^

1594.

--

-Z

1599.

600

^rr^
o

1601.

1-

429

+ l^.
4-(^
2

1 6(>2.

1603.

1.

00

1604.

1605.

z\

b z-

606

r(p+l)=C

Solution.

we put *P

the formula of integration by parts,

~
du = px. p l

e~ x dx

U,

v=

dx,

xPe~*

dx.

= dv.

Applying

Whence

e~*

and

If p is a natural number, then, applying formula


account that

we

p times and taking into

(*)

get:

"

1>35

(2fe

2-4-6

2
,

n=

ii

2k-\-

number;

an odd number

is

even

an

is

246

2k

1-3-5 ... (2k


/

t608

^7 V

...

128
_
"

1609.

'

3l5

_ 63jt
10

~"

^,

^B

'

512
Hint.

'-Hr^

Put

1610. a) Plus; b) minus; c) plus Hint. Sketch the graph of the integrand for
values of the argument on the interval of integration 1611. a) First; b) second;
c)

1612.

first.

1617.

Hint.
1624.

2</</"5.

1618.

The integrand
1.

1613.a.

1625. --

|-</<l

10

4O

1639. ab[2
1641.

2a2<r 1

1636.

In 3.

~.

In

1642.

(2+

2n + -

and

1631.

na8

1640.

-g-

1643.

6n--|

1620.

t638

1632. -~-p

2
.

= 32

1626.

4~

1633.

4-i

1623

%< l <-^-12.

0</<~
-

= 2(coshl -1).
^+~~2
6

/~~3)).

-^a

naa

~~4-O

1637.

15n.

1644.

See Appendix VI, Fig. 23. 1647. a2 f 2


1648.

^jt</<yJi.

of the sign of the function.

1630.

2 arc sin -i.

1616.

-|

1619.

Take account

Hint.

1635. 4.

V"l

1615.

increases monotonically. 1621.

1627. 2. 1628. In 2. 1629.


1634.

~.

1614.

1049.

Hint. See Appendix VI, Fig. 27.

yln3.

+ ~-V

^ji-i|-?

1645.

Hint. See

and

1.

1646. 3jra2 .

Hint.

Appendix VI, Fig. 24.

~ n+ p

1650.

Answers

430

1651. 3na2

1655.

-|na

Hint. See

Pass

Hint.

1664.11)^2.

VhT^tf.

1667.

l/T+ln(l+/n

1673.

1674. 2a

1687.

1691.
1696.

b)

(5

t;^

= ~;
&

^(15
2

6jia;

,705.

+4

c- 2 ).

av

16 In 2).

^. 1710.

~a
o

s
.

na 2 /"p^.

V5

1)

2n6 2 -f

(eccentricity

of

Ao "P

ellipse).

8
.

1695.

1U
1701. a

10

1703.

-na.

1707.

}.

nabh (

1704.

4U

1708.

+ -^oC /)

nabc.

1713.
5

1715.

1717.

Do

1719.~rta 2 1720.
.

^tfx*.
8

~nh*a.

1699.

no*.

1690.

1694.

1688. -1

lUi)

1712.

nln.
+

+ 4) = ^(2
+ sinh2).
z
1722.

*.

i[5^T-8].

torus.

^^.
z

na'.

1721. 4n*ab Hint. Here. y


b
external surface of a torus; taking the

face of a

Jia

n(5-^T) +

17l8.^(e*-ff
4

1711.

je

0]. 1682.

1685.

In 3).

10

1706.

+l).

1676.

V2" + ln ( 12"+

1693.
1698.

1678. I6a. 1679.

1689.

1716.

1702.

+$ +fl 6).

DOT -!;

1714.

2n a.

1697.

+
*(x*
^
4-JW
&

(9n -16).

c)

-Ji

= 2n. 1692.^^.
o

1709.

1688.

1672. ~-(e

fc=ln

4- [4 +

1684 .

ln(2+VT)

^^

1680. 8a. 1681. 2a

JQ!.

, 68 3.

^(10/161).
a= 1.

sinh 2

cosh q

ln~

1675.

Fig. 29. 1677.

1665.

V'"?11!). 1671.

/T".

Appendix VI,

See Appen-

See Appendix VI,

Hint.

coordinates.

polar

Hint. See

~.

1659.

the

the lo P

1668.

1670. In (e

l-f-lln-|.

aln.

to

For

See Appendix VI, Fig. 22.

1656. 8jtV. Hint.

Fig. 28.
.

Hint

fl2 -

0<+oo

1658. a2

1654.

Hint. Utilize the formula

1666.

1669.

6na2

1653.

Appendix VI,

1657.

Fig. 30.

VI,

+ 2ab).

varies within the limits

parameter

dix

1652. n(b*

-^(e1

Taking the plus sign, we get the


minus sign, we get the internal sur-

arc sine; 2) 2na


1723.

a)

-\

81|-^
In

where

~~~8

b) 16n

a 2 ; c)

~
a

Answers

~jia 2

1724.

1725.

2ita

(2-

J=y = y.

M x =M K =-~a

1730.

a 24-sinh2

1729.

= y = ~a.

1737< *~=Jia

^^SB-

I736t

^ss-

<

2na 2

1731.

1732. x

^=|-

M x =- V
M X =M K = ^-

1727.

M & = ^.

asina

._00

na 2

1726.

/2~).

M = ~-

1728.

43 1

=
4a

,__

1738>

Divide the hemisphere into elementary spherical slices of area da by


horizontal planes. We have da
2jiadz, where dz is the altitude of a slice.

tion.

az dz

2ft f

Whence

z=

2nfl

tance of

-j-

=
z

Ct

Due

-?r.

from

altitude

symmetry, x = y = Q.

to

vertex

the

the

of

cone.

At

1739.

dis-

Partition the

Solution.

cone into elements by planes parallel to the base. The mass of an elemen2
tary layer (slice) is dm,YftQ dz, wnere Y * s * ne density, z is the distance

the

of

from

plane

cutting

vertex

the

of

the

cone,

= -J-Z.

Whence

h
Jt

1740.

h.

0;

0;

+-g-

_^

a )-

Due

Solution.

to

symmetry.

we partition the hemisphere into elementary


#~=0. To determine
layers (slices) by planes parallel to the horizontal plane. The mass of such
an elementary layer dm
^nr^dz, where Y s tne density, a is the distance
2
2
is
2
the
of the cutting plane from the base of the hemisphere,

5T-=

r= ^a

[a

radius of a cross-section.

11
"33

1742. I a

1745.

= -z-ab*\

/=yjt

concentric

the

moment

/?J).

The

circles.

of inertia 7

have:

zdz
o

3
=-Q-a. 1741. /==jta

11

4""

I b =z-^a

(#J

We

b.

1743.

7=-/?& 8

1744.

15

Solution.

We

mass

each

of

= 2nCrdr =

partition

such

y n(/?*

= JW&
"4
--

the

ring

element

IW
b

into

= -T na*b.
4

elementary

dm = y2n.rdr

#J);( Y ==1). 1746.

and

^nR*Hy.

Solution. We partition the cone into elementary cylindrical tubes parallel


the axis of the cone. The volume of each such elementary tube is
dV 2nrhdr, whe r e r is the radius of the tube (the distance to the axis of

to

the

cone),

=H

s-

is

the altitude of the tube; then the

moment

of

Answers

432

/=Y

inertia
ia

cone. 1747.

2jttf (

Ma 2

cylindrical

volume dV

dr

^lr^

the

Then the moment

where y

is

radius of

= |- Ma

lows that y

of inertia I

V--=2n 2 a 2 b\

1748.

= 4nay

I/

S--=4n 2 ab.

4
f/^-rr

b) x==(/

= T7-n.

sen

that

1U

so

centre of

1750. a) x^=0,

the

the

circle; b)

^=4o

cone obtained from rotating

where b

is

the base, h

rem, the same volume


of

trom

gravity

is

1749.

= ~^/?

//

a2

=A

na'v,

a)

fol-

ita'y, it

7-=//~=4
o

Solution.

The volume

about
of

its

the

base,

is

\\hence

base.

of the solid

is

triangle.

where x

-z-b'i.

and the origin

diameter

the

1753.

x=<Mua>t:

The elementary

Hint.

a;

the

double

--

equal to V

rft/i

By the Guldm

distance

of

the

is

force

of

(force

centre

tlu>

H
-

D flt,=

theo-

1751.

vt'
^

-lnl+--.

1752.

The coordinate axes are cho-

Hint.

the altitude

the

"

71

with

a triangle

2Jtx

I/
f

coincides

jc-axis

M=

---

elementary

L./-*dr

of the

An elementary

h = 2a

a tube,

and since the mass

e density of the sphere,

densit y

into

sphere

a
is its altitude.

the

is

the given diameter.

is

the

is

where

'

partition

which

axis of

2nrhdr, where

We

Solution.

the

tubes,

^)

-f

1754.

is

gravity)

the

equal to

the volume of a layer cf thickness dx, that is, dF =


where y is the weight of unit volume of water. Hence, the elex} dx, where x is the water level.
mentary work of a force dA ynR'* (H

weight

= ynR

1757.

of

water in

dx,

A=ryR

2
.

l^&

1759.

A = ynR*H.

on

mass

is

1760.

equal to

of the earth. Since for r

1758.

^=

F= k

=R

A ---^ R*TM

tn0h

j-

where

A=

When/i = oo we have A

The

kgm.

force

acting

the distance from the centre

r is

F=mg
+
R

m^-

Solution.

/lo.-mg/?.

we have

sought-for work will have the form

^Q 79-10* -0 79- 10

it

follows that

kM=gR

The

h.

kr-dr
*

= mgR.

1761.

kmM
1.8-10

{ -^-

\KK-\-nJ

ergs.

Solution.

__
Answers

The

force of interaction of charges

is /

433

= ^i

work

dynes. Consequently, the

-J-=S
Jd.x

= *o*i(/ --- \ = 1.8-10


X
X /
isothermal process, pw = p
1

*'
4

4=800 nln2

1762.

ergs.

kgm. Solution. For an

tV The work performed

in the

expansion of a gas

v\

from volume v

volume v

to

A=

is

p di>=p

In

i>

1763.

UQ

an

For

Solution.

adiabatic

the Poisson

process,

i>i

*^1.4,

holds

A=

Hence

true.

=5=

15,000

kgm.

^0

(
J

law

pv

= pj>^

where

-Ms
-^dvk
k
v

4=~jiu.Pa.

1764.

the pressure

Solution.

on unit area

If

of the

the base

the radius of

is

support p

p
z

The

3ta a

by

frictional

forces

Therefore,

on

of

revolution

complete

A=^x

work

the complete

force

-^~rdr. The work per-

is

one

in

ring

frictional

2uP

ring of width dr, at a distance r from the centre,

formed

them

of a shaft,

r*

dr

is

JifiPa,

Solution.
j M#
= ^~da where
dK=^L2

1765.

<D

from

the

axis

of

The
da

rotation,

kinetic

an element

is

the

is

of

of

energy

particle

area, r

surface

of

the disk

the distance of

is

= -^j.

density,

it

Thus,.

Whence
1767.

K = ^/?

co

= 2.3-10

K-^J,-*-^
8

kgm. Hint. The amount

to the reserve of kinetic energy. 1768.

1770.

P = abyxh.

1772. 633

Igm

1771.

P = ^-_

1773. 99.8 cal.

p = -^-.
o

(the vertical

1774.

M^~

1769.

of

766

work required

p^fo +

component
gf

* = | X M*W.

fe

^fr)

1776.^-

__
1777
15-1900

Solution.

Q==

cm. 1775.

^^-^
2

10>r

(k

is

the

Q= (v2nrdr =
.

^n

equat
,

directed upwards).

is

gravitational constant).

is

a6

2
f( a

UJ A
Hlnt

r*)rdr

Answers

434

along the large lower side of the rectangle, and the y-axis,

perpendicular to

ua
it

in

middle.

the

S=\J

Solution.

1778.

on the other hand,

dv,

7^=a,
at

vt
t'a

whence

1780.

dt

dv,

M*=:

and

acceleration time

the

consequently,

(xt)ktdt + Ax=j(l'x*).

/=l- = S.

is

1781.

Q =0.12 TRI*9

S=

Hint.

cal.

Use the Joule-Lenz law.


Chapter VI

V=

1782.

2
(i/

x 2 )*.

1783.

(x

/"4z

t/)

Oje//

1786.

/<*,

x2 )

= \+x-x

replace

by

x.

_,

Then
.

It

1=^;

1789.

f (x,

y)

in the

form

^ Ri

Solution.

o~~^

= x~\+V"y.
* === (a

In

Hint.
2

~|-

and,

Solution.

the

Then

and

y.

+1

and

+ y~u,

1790. / (a)

*=1+/(VT

/(w)=:w

When *=1

a)

= l/V

Designate

v, x

identity

hence,

f (x)

/(-=M= I/

remains to name the arguments u and

then

1788.

(xy) 2

UV

^=

Hint. Represent the given function

/?

1787.

-}-3

_...

-f.
j^S.

we

1791.

-}-2a.

have

the
"

1)

put

/(</)

identity

Single circle

with centre ai origin,

*; c) halfincluding m the circle (x


y
1); b) bisector of quadrantal angle #
#==0 (x y>Q)\ d) strip contained
plane located above the straight line *
lines (
between the straight lines (/=
1, including these
e) a
1 and
/= 1, includsqua're formed by the segments of the straight lines
l<t/<l); f) part of the plane adjoining the
ing its sides
x including these lines and
*-axis and contained between the straight lines
when x
x when
0,
excluding the coordinate origin
and
2,
2<i/<2;
2<i/<2
h) the
*<0); g) two strips x^2,
2
2
2
2
2
2
a and x -ff/
2a including the
ring contained between the circles * -r-i/
boundaries; i) strips 2/uK jc
1) n, r/^0 and (2n+ 1) Ji<x<(2n
(2/i
2) it,
where n is an integer; j) that part of the plane located above the

K#<1);

#=

(Kx^l,

(x^y^x

<

y=

*<

>

x<t/<

_ __
Answers

435

~y > 0); ,k) the entire j/-plane; 1) the entire *t/-plane,


x*(x*
parabola y
with the exception of the coordinate origin; m) that part of the plane located
above the parabola y* x and to the right of the (/-axis, including the points
of the t/-axis and excluding the points of the parabola (*:^0, y > V x)\
0; o) the
n) the entire place except points of the straight lines *=1 and t/
x2 + y 2
n (2k + 1 ) ( = 0, 1, 2, ...).
family of concentric circles 2n k
1793. a) First octant (including boundary); b) First, Third, Sixth and Eighth
octants (excluding the boundary); c) a cube bounded by the planes x=
1,
1, including its faces; d) a sphere of radius 1 with centre
y~ 1 and z

<

<

1794. a) a plane; the level lines are


at the origin, including its surface
0; b) a paraboloid of revostraight lines parallel to the straight line *-f */
lution; the level lines are concentric circles with centre at the origin;
a hyperbolic
hyperbolas;
paraboloid; the level lines are equilateral
c)
d) second-order cone; the level lines are equilateral hyperbolas; e) a parabolic
0;
t/rf- 1
cylinder, the generatrices of which are parallel to the straight line x
the level lines are parallel lines; f) the lateral surface of a quadrangular
pyramid; the level lines are the outlines of squares; g)_level lines are parabz
C ]fx i) the level Ifnes
olas y^-Cx
h) the level lines are parabolas y
2
2
2*. 1795. a) Parabolas
are the circles C (*
(C
y )
0); b) hyper2
2
C 2 d) straight lines y ax-{-C;
bolas xy^C(\ C
1);
c) circles jt -f*/
lines
y-=Cx(x^Q). 1796. a) Planes parallel to the plane
c) straight
0,
x-\-y-\-z^=Q\ b) concentric spheres with centre at origin; c) for u
one-sheet hyperboloids of revolution about the z-axis; for u
0, two-sheet
hyperboloids of revolution about 2 the same axis; both families of surfaces
2
za
are divided by the cone * 4-r/
1797.
(u
0).
b) 0;c) 2;
a) 0;
k
not
limit
does
limit
e
does not exist. Hint. In Item(b)
exist; f)
d)
e)
coordinates
In
Items
and
to
consider
the
variation
of x
(f),
(e)
pass
polar
kx and show that the given expression
and y along the straight lines y
may tend to different limits, depending an the choice of k. 1798. Continuous.
1799. a) Discontinuity at je
0;
0,
y
b) all points of the straight line
of
line
of
is
the circle
x
c)
(line
y
discontinuity);
discontinuity
2
the
tines
of
the coordinate axes.
l;
d)
discontinuity are

|<

>

y^Cx*

>

<

1800
is

Hint. Putting y

=y

continuous everywhere,

when
<p2

f/ 1

q^M^O.

^0,

2v u
(y) =
is

since

we

for

get the function (?,(*)

yl

discontinuous at the point

the denominator * 2

when

Similarly,

jt

From

everywhere continuous.

is

function z

=^ const,

(0, 0)

= *, = const,

-|-f/

the

^0, and
function

the set of variables x, y, the

since there

is

no limz. Indeed,
X

which

-*

r cos cp,f/
r sin <p), we get z~ sin 2<p, whence it is
passing to polar coordinates (x
>
if x
and y -+ in such manner that (p const (Os^qx; 2rc),
2(p. Since these limiting values of the function z depend on the
direction of cp, it follows that z does not have a limit as x and y -+ 0.

evident that
then z -* sin
1801.

~^3(jc 2

1803.

^=

1805

dx

ay),

^ = 3(#

dy

=
(

x*

l*'

dy

Answers

436

.-s,^i/
ox

xy*

x*

dz

V2x'-2y*

yx*

cos^.

dy

ln

,810.

ox

dz

V2x*-2y*

oz
4813.

1815.

-- -

1820.

/;(1,2,

= 4,

0)

1821. r.

5^-.

1828.

-.

tan p

x-=zxtf2*"

-5-=x2*"ln2,

-r-=i/2*'ln2,

tan Y =-~- 1829

1827.

cp (x).

2-^

tana = 4, tanp=co,

1)

^=y *

= arc tan ^ +

1826. z

^(1,2,0) = !

= ~,

fy (\, 2, 0)

1,

1814. r

1'

If^T'

to see that the function is equal to zero over the entire x-axis and
the entire i/-axs, and take advantage of the definition of partial derivatives.
1831. A/
Be convinced that f'x (0, 0) ^(0, 0) 0.
4Ax+ A(/ 2Ax 2
2
4dx
&\
-f 2AxAi/-t-Ax Ar/;
a) A/
b) A/
df
dy\
d/=- 0.062.
df
2
1833. dz'^3{x*y)dx
3(y*x)dy. 1834. dz 2xy*dx 3x*y dy. 1835. de

Check

2
\**

i. 2

(xy

dxx

y dy). 1836. dz

= sin 2xdx

sin 2(/di/.

1837. dz

y.

18401 dz

=
+

= 0.

1838. dz

1841. d0

=-

(xdx

+ ydy).
dx

1839.

^=

1842.

d/(l,

1)

dx

sin

x
1843.

du =

1845.

du=xy +

t/z

dx

+/x dy + xy dz.

y^.
(5dz

3dx

1846.

1844.

4di/).

1848. d/

live to inner dimensions).

r/dx

:74

= 0.062

1850.

(x

dx

y dy

+ z dz).

zdx+\-xzdy +

dw-

du

-r-

+ xdt/--dz.

1847. d/ (3, 4,

cm; A/==0.065 cm. 1849. 75

cm. Hint. Put the differential

cm 3

5)-

(rela-

of the area

the sector equal to zero and find the differential of the radius from that.
1851. a) 1.00; b) 4.998, c) 0.273. 1853. Accurate to 4 metres (more exactly,

of

4.25 m).

1854.

n ag ~Ef.

1855.

da==-^

77

(dy cos

dxsia

a).

1856.

^=

Answers

-sin*

In sin*). 1861.

d*

-.

=_.;
x'-l-y

= 0.

<!/+
(

z)[^(x,

i/.

y)

1865.

l867 -

S=^

-j)

dx

=W:
*

dx

ty'v (x,

^l-

.l.

_
221
66

+ /^(*,

1862.

dx -,--,.
1+x 1

= 2x/,

1863.

1864

437

( *-

The

1873.

y)<f' (x)].

>+<

"

increases

perimeter

at

a rate of 2 m/sec, the area increases at a rate of70m*/sec. 1874.

20/52 V5 km/hr.

1875.

- 9 ^3

1876.

1877.

1878.

1.

J^..

1879.

<fc

68

c) (7,2,1).

1884.

cosa=s-r-,
o

cos p

cp

-^QQOQ-r
^=83 37.

9/-3/

j (5/-3/). 1886. 6/-f 3/ +

1885.

COSY =

1891.

I7
1/1

9 = -7=.
y 10

=
1894.^=0.
1895.^
^ xd^
^x

dy dz
A

= mn;

0)

v (0,

and

tiation

the

,,

irp-yi+^i
[~2
that For x
ticular,

fj,,(0,

2
f/

cos

(0.

0)

/y

= /i(n

and

for

0)=

of

+ yvj

(when

any
1.

1899.

2jc sin (xy).

(jry)

definition

4\

grad a|=6;

1889. tan (p^= 8. 944;

,897.

^yi^

^-;=
^'

'

1898.

1887.

1888. cos

-O-.

^
^=

lfiol

2A?.

(/,

1902. Hint.

1).

partial

A'

/^ (0,

Similarly,

1
J/

y.

MO.

Whence

find that

0)

verify that

f",IJC

and

= m (m

Using the rules

derivative,

?4 0)'

y)^
we

(0,

of

differen-

f'
x (x,

1;

in par-

0) = 1.

1903.

"i-i-

-g^r--/ii\".
1904.

-if

/("'

0)

+ 4*0/

Jll ,

(M,

y)4-*

w (,

y) =3

consequently,

fxlt (Q, y)=

\Q,

1);

u).

o);

438

Answers

1905.

= q>W +
t915.
+ * (y). 1916. d*z=e*y x
(*, /)=*cp
d*u = 2(xdy dz + ydzdx + zdx dy).
1917.
+ x dy)* + 2dx dy].
Y* X
1919. dz =
d*z = 4<t"(t)(xdx + ydy)* + 2<f'(t)(dx* + dy*).

1914. u(*.

I(y

1>(</).

!/)

((/)

dx

1918.

(f

= a M (a, o)
x
x
1921 dz = (ye*f'v + eVf"uu + <2ye +yf'nv + y*e* w dx* +
+ 2 (&f'u + e*f'v + xe*yfuu +e x + y +xy f + ye* x fo dx dy +
1922.
d'z = e* (cos y dx* + (xeyfu + x*e*yfuu + 2xe*+yfuv + e**Q dy\
1923.
d'z =
3 sin y dx dy
3 cos y dx dy + sin y dy
# cos x d*
1924. d/(l, 2) = 0; d f(l, 2) =
3sinxdx*dy 3cosy dxdy + xsiny dy*.
2

1920. d z
.

f'

(\

tlv

).

d 2/

1925.

+ C.

1926. xy

^?+p + C.

1931.

1927.

y ln(^ +

1929.

a=

1932.

1,

|/

0)=

0,

(0,

+ 2arc

6=

1,

^,=

1938.

1.

Hint.

Xdx+Ydy.

expression

condition

1939.

f^f'y

of

1940.

+C

'

l933

total
xy

C.

JC

2
//2

+ y + 2 +C
f

1937.

the

~+

1930.

2z-j-C. 1935.

t/z

the

+ C.

tan

+ 2^ + 3x2 + ^
1936.
+ + + c.

Write

1928.

8
1934. x

~ + sin x + C.

x't/

for the

differential

a=f(z)d2 + C.

1941.

-^y

;ri=
dx2

a2

^I=-T-I.
dx*
a*y*

2
a5^1;
y*

of straight
equation of a pair
r

dx 2

1946
1943

1945

=3

The equation
M
defining//
&

^= ^

ln

-1;

or

x^ *

1944

*%2

=8

is

the

=
^
dx y
_

or

-8.

axy'

dz_x
<9jc

1943.

lines.

(\-y)*
dy

^42.

xf/

'

%~~

dx

(axy)*

dz_Gy

yz
z

'

dx 2
2

3xz2
(xt/

'

a
)

^^

dz_zsinx

'

dx 2

cosy

x2

dz

'

dx

cosx

t/sinz*

d/y

'

Answers
d*z

d*z
:

dx*~

a*b*z>

dxdy

*;*;

1954. d2

439

1953.

dy*''

dx
2

dw;
*

i/

-^^dr/
2*

f
.

1955. d2

= 0;

d 2 2=

).

Tc(^
IO

1961.

-r-

2
rf
l/

l956

)-

T"

r-^

1;

dy);

=00;

dxfy

dz

11111

"^"*

= 0.

T~9

1964.

dw

:^;
1965.

=-

,966. a)7

dz

c cos y

1967.
,-

sin

|).

1971.

a)

-20;
pp^-.

sina

1974.

q,^.
1969.

=F;(r,

cos

q>

9)^.

- +

+ ^O.

b)

g-0.

~=0.

1975.

tt

cp)

cos

<p

dz
1968.

dx
1970.

1972.

~-. z== 0.

1976.

Answers

440
3*

1983.

+ 4j/-H2z

/ a 2 -f &2

169

= 0.

+ 4y + 6z =

1985. x

21

1986.

the tangent planes are


the points (1,
1. 0),
-f
and (2, 0, 0), to the j/z-plane.
parallel to the xz-plane; at the points (0, 0, 0)
There are no points on the surface at which the tangent plane is parallel to
c

Ai

1987

the x#-plane. 1991.

-^

*
Projection on the

on the xi/-plane:

1994. Projection

._w_ )=0

<

/z-plane:

_
~n

1.

3y*

on

Projection

I^Q

the

xz-plane:

=
|

3x 2

4-z

A
1=0.
,

Hint. The line of tangency of the surface

with the cylin-

projecting this surface on some plane is a locus at which the tangent


plane to the given surface zis perpendicular to the plane of the projection
2
ax + 2bxy
1996. /(*
k)
/t,
y
cy*-{- 2(ax + by)h + 2(b* + cy) k i-ah
2
2
1997.
(xH-2)
/)=!
/(*,
2(jc-f2)(t/~l)-f3((/
-f 26/ifc-K/z
1)2,

der

i/)

= 2/i + fc-M + 2/i/e-f/i

-_1)H2(*-1)

(2

/(^

2001.

z)-f

(/,

A/(x,

1998.

+
=

2[/i(x~t/

*.

1999.

2005

>

[(jt

+
=

I)

4-

+ k,

I)

(//

z+/)

z)
.

1)

f(x

+
(y

2003.

Dl

Un?^

a)

= (x
+ H,

/ (x, y, z)

2000.

-z) + k(y-x

2004.

1).

(*/-!)--(</-- l)(z-l).

+ x,+.2002.

-!)(!/-

b)

Hint. Apply Taylor's formula for


2006. a) 1.0081;
b)J).902.
r
in the neighbourhood of the point (1,1);
the functions: a) f(x,y)=\ x
y* in the neighbourhood of the point (2,1). 2007. z= 1 -}-2(x~-l)
b) f (x, y)
2
when x=l,
10 ( X -l)(y-l)-3 (y- !)'+.. 2008. z mm
-(t/-l)~-8(x~l)
2010.
z min -=
2011. z max =108
1 whenx=l, 0=0.
i/=0 2009. NocxVcmum.
(3 n z_4M)p2j

yy

= 3,t/ = 2.20l2. z min =


There

is

when x =

no extremum

Y^ y =

for

1^2

x=j/ = 0.

when

and

2013.

2 max

at the

jc

= = 0.
i/

1"\ at points

2017.

min

T^
2015.

y==

a"d

""Ff

zm i n

of the circle

4
3-

and*=

X^-TTL, t/---^

points

when

*2

when x =

y==

'~'Vf'

x=0 = 0;

wne "

21

+y =
2

-j

1.

2016. zmax

2 14 '

F?'

nonrigorous

= 1^3 whenx=

z=l.

ab

* ==

at

3
the points

2018.

w min

;?Tnax

"=1

maximum
1,

=4

1.

when

=Y

J/1' z== ^ 2019. The equation defines two functions, of which one
has a maximum (z m ax
8) when *=1, f/=2; the other has a minimum
2jwhenx 1,0 = 2, at points of the circle (x 1) 2 + (t/ -f 2 2 -^ 25,
(Zimn =

JC

functions has a boundary ext-emum (z


3). Hint.
tions mentioned in the answer are
explicitly defined by the

eacn

of

these

The

func-

equalities

__
Answers

e=3

441

2
1^25
(x\)* (f/-f2) and
consequently exist only inside and on
2
2
the boundary of thecircle (x
25, at the points of which both
I) -|- (l/
2>
functions assume the value 2
3. This value is the least for the first function
and is the greatest for the second. 2020. One of the functions defined by the
the other has a
2,
1,
r/
equation has a maximum (*max"-~~~ 2) for x
minimum (2 min 1) for x
2, both functions have a boundary extremum
1, y

at the points of the curve

*=#=-.
2023.

=5

2022. 2 max

12*

4i/

x=l,

for

for

imta

4jt

= 2;

2 min

2024.

max

f/

~.
^~.

+ 16r/

33=0.
5

----

for

(f
2,

1)

j/

fl

2)

=2

x==

for

/"T

^^ T

I/

-IT;

b) greatest

= 0,

2030.

2).

1,

= 0.

2=

j/

value

1.

ll
m,

m 2 -f m,

i>

-rp
3

of

~T

the

ellipsoid.

2046

Major

for

/""2"

value

= =

for

for

smallest

y-^0;

1,

t/

(in-

j/2V,

44
a

2b

^2V*

s=s3i

j/

= 6,

-|- J

The dimensions

2 Q43.

of

the

= = 26+2V,

2a

Isosceles

2041. Jgaa

2c

2044. *

2036.

"T^' where
V 3

axis,

--

>/a. 2039. Ai

\/a

>

~p,~/>, and

7^-*
3

J/2V,

\/a

\/a

2Q42

axes

*=

for

x==

for

y=l;
,

*2a

parallelepiped are

TFT

= 0,

y-~Q (boundary minimum).


= 2, y = 1 (boundary maximum); smallest
(boun(internal minimum) and for x = 0, y =

Cube. 2038. a

1a
-i-

z== "~

2032. Greatest value 2

are
2040. Sides of the triangle
fe

2031. a) Greatest value

value

2034. Cube. 2035.

dary minimum).

27

for

smallest

t/=

2 for x

triangle. 2037.

2.

f T)'--(f
=3
Greatest value

2025.
r/

a)

ternal maximum); smallest value


13 for x
2033. Greatest value 2

value

I/

f T)=

2,

(1,

"3

for x

= 5 + *:i,
/

/^
\

#--2

1,

b) smallest value 2

I)' (41,

(2,

for

=-j

= 2, 2 =
= 2, 2= 2, "max^ 9
*=1,
= 2 = 0; Wmin^C for x = t/=0 2^C.
*=
at the points
.6Mor x = 2, y=4, 2=6. 2028. u max -4
1,

2026.
MMIX -=fl 2
2.4
2027. w max

for

2 max

-j/

2021.

minor

b,

a,

and

= --

axis, 26

= 2.

are

2045.

Hint.

the

x=

semi-

The square

of

the distance of the point (x,y) of the ellipse from its centre (coordinate origin)
2
2
The problem reduces to finding the extremum of the function
is equal to x -f-r/
2
2
9. 2047. The radius of the base of the cylinder
5t/
x*-\-y* provided 5*
Bxy
.

Answers

442

is

|/2 + -~=,

the altitude

must connect the point

sphere. 2048. The channel

with

the

^
14

2049.

1^2730.

the ray passes


a

cos

the ray
J

of

the

of

point

sin

BM=^

of the function /(a,

a=p.

function

/^/ji/^

2052.

7 2 , / 8)

f (/

must

lie

ft

which

at

between A, and B t ;

to finding
* the

01

t/2

-5t\2

AI

~ provided that a tan


COS p

-5-

Find

Hint.

the

minimum

a + b tanp = c.

minimum

the

of

Kl

= /J/?; + /*# +

that
provided
second kind

/J/? t

The isolated point

M,

is

B^M = b tan p. The duration of motion

-- --

UjCOS
2051.

length

The problem
reduces
r

-5

y 2 cosp

|J)

its

into the other,

A,M =a tan a,

a, cos

Uj

-- --

is

"T

7 V^L
-

5 \

1 1

Hint. Obviously, the point

medium

one

the radius of the

is

the parabola
(-o-

of

-^-

EE^^!

2050.

frm

line

straight

--^=, where

|/2

+/ +/

/!

-/.

2055. Tacnode (0, 0). 2056. Isolated point (0, 0). 2057. Node (0, 0). 2058. Cusp of first
kind (0, 0). 2059. Node (0,0). 2060. Node (0, 0). 2061. Origin is isolated point
it
if a>6;
is a cusp
of the first kind if a
and a node if a
b.
6,
2062. If among the quantities a, b, and c, none are equal, then the curve
does not have any singular points. If a
b<c, then A (a, 0) is an isolated
c, then B (b, 0) is a node; ifa^fc^c, then A (a, 0) is a cusp
point; if

2053.

2054.

(0, 0).

of

Cusp

(0, 0).

<

a<b =

of the
2

-f-

/3

kind. 2063.

first

y=x.

/ ''.

2067.

whose equations,

2068.

if

2064.

2066. **/

pair of conjugate equilateral hyperbolas,

x#^=-^-S.
the axes of symmetry

coordinate axes, have the form

y^R.

2065.

y ^2px.

of

the

ellipses

xy~

2069. a)

are

taken

as

the

The discriminant curve

the locus of points of inflection and of the envelope of the given


y
of the envelope
is the locus of cusps and
family; b) the discriminant curve y
Q is the locus of cusps and is not an enof the family ;c) the discriminant curve y
velope; d) the discriminant curve decomposes into the straight lines: *
(locus
is

of nodes)

2073.

x^ a (envelope).

and

1/~3V

1).

2074.42.

2079. a) Straight line; b)

2082. 4/

(*

for

f:=0,

A;

= 2cos_^,

0=1/^13

z?=-2y+3ft,

f/

for

y=~
*8

2075.

parabola;

+l). 2083. ^ = 3 cos

= 4/ 2084.
+ 2ycos/-l-3Ar;

2070.

#=4

c) ellipse;

= 2sin/,
any

w=

/;

2i;

for

XQ

+Z

Q.

= 3/

^^-

11

2080.

= Q, v--=4j, w=.

(screw-line);

-2/cos

2072.

2077.

d) hyperbola.

(ellipse); for

w=

i
^

2076.

5.

sin

2071. 7

2v 2

2ysin

t\

u=

3/; fo

2/sinH

w2tor

any

Answers

x=cosacoso);

2085.

co/ sin

of

sin

uy

a>

2
2088. 0"|/ a

screw

(*

-y-

= --

[(sin

a cost

are

2090. T

sin

cos

(*

x + 2z

COS(T,

0./J*.

acos/
,

sin

cos p

==

+ 4r/-f-12z

114

2096.

(normal

|,

(4,
.

ing

(tangent);
j

-_
cipal normal);

2V

=y+ =

plane);

(tangent); x

(tangent);

(principal normal);

-,
1

z=

/2

of

= sin/;
plane);

(tangent);

plane).

(pri "~
t

cosv,

= 0.

^ilx*

(binormal);

cosp 2 =

=-=;

cos|J 1

=_ 2 ^

/4

(oscu^ating

TT~

^-~j-

cosines

(osculating

~T?T

= _

^.=^l = i^l

2097.

= cos/;

6r/-f-z

cos Y

=
__t>r

12*

plane);

=
j

=^T

*"T

...

(rectifying plane). 2095.

(binormal)

directi on

The
a cos

2093.

asint

normal)'

a
=--===

k]\

/\

- zbt

VI

/;

cos(v,z)=0.

^r&cos/r =
y

^ sin

-;

"; P^=.
/5

/105
..

1^3^

2)

a>/

cos t)J

The direction cosines of the principal normal are cosa,


1
z
2094. 2*
0.
cos YI
(normal plane); y

art

cof

+ *);

(sin

^
cos

cos a sin

cousin a cos

the angular speed of rotation of the

is

(o*

[(cos

^--

smt

tangent

2aa>u sin

.,

y-

cos

\
*>
(tangent);
/

x -acost

v=
a cos CD/

(circle);
2

co /

|;

^"

co

where w = -

+ cosO /+(sin

the

/i

T= --

2091.

fc).

2092. t

cos

V aV + u

2089.

=
u=K

CD*

2086.

= sincof
w
+

sin acoscof; z

y
cos

cofc

443

jc

"""3"'

+ = (osculat= + 2- = z--2
i/

i/

-g

2098. a)

(normal plane); b)
__

(tangent); x
(tangent);

+ # + 4z-10 =

2_>^3
2|/*2^=0.

r/

a)

6 2 )ejz

-=

c)

^|

2/*3z =

2101.

c) b*x\x
a*yly + (a
~ =
ing plane);

(normal plane);

(a

6 2 ).

==

~~A""J:
f 3

+ y = 0. 2100. x
=
18 = 0;
6y + 2z
2102. 6.r8/z + 3 =
(osculat-

(normal plane); 2099. x


9x
0;
# z 9
b)

4*

=a6

=^"' 2

(P rinci P al normal);

Answers

444

* +6
J
3y

=o

binormal )

(osculating

TSBS

= 0.

\i

fl>

2107.

when

M
.

/=!,

=
=u

'

>

(principal normal);

/+ *

v== /

>

J7*

K=
99

/C=4-1/T7
/
r
14

w,

2jc

=*

= 0,

/"To

2106

2108. a)

When
i

w w =2;

V~i\ b)

a)

2112.

plane);

|7^=i

19z~27

2111.

*=0

bx

(binormal). 2103.

= -^=.,

oy x

2,

=2

wn

= 0,

/~TQ

I/

'

"^ j^

Chapter VII
9
2113.

2120.

x=l;

9^

44.
O
-.
O

2114.

= ^r

*=-!;

2125.

= 0; t/=y25

= 2.

J*Jf(*.
f/

2117. 50.4. 2118.

6;

x = 0;

It
00
y)d(/.

= 2.

= 3.

2122.

2126.

2129.

=x

r/

2JC

JC

+3

2Af

72

f (x,

12
'

2I32

'

/ (x,

2130.
1

2119.2.4.

21

00

f(x,y)dy.

t/

^^

J dj/ J

12

J/^

dy

^2

ix
=
JdxJ/(*.
00

10

00
42

2131.

i/)rfx

y\

2127.

11
2128.

x=2

1;

TT/7*

44

2116.

\JL

= 3.

~.

2115.

L**

2121. ^

7t

ln~

,/T
y T

t/

= + 9;
A:

Answers

2133.

dx

Vl -

f (x,

""

i/)d</

d*

y)dy =

f(x,

-K 4 -*

+J

d</

"

dx

*,

dy

J
~

dx

r
dy

-V*--tp

c/

~yr^p

vr~^y~*

V* -X*

y)dx +

f(x,

*
-

2134.

/(*,*/)

x*

+j

445

Kl+

JC

/(Jt,

f(x,y)dy=

J
V* -n*

!/)

^+

U. y)d^+

dy

-vihT

I-A:

2135.

1+ Kl

(x,

r
e) \

\)

+ 2aa
r

dy

48

2136.

dx. 2137.

'

l dlj

jc

f (x,

y)

dy=

*j

i/

f(*>y)** +

f (x t

%)

81

dy

f (x,

y)

dx

_
1

y*

dy

JL

12

2138

o^

ao

Ji

Vu

Vx

22
f (x, y)

dt/

20

ax
oo

w
"

y) dx\ d)

dx

-V
i^Kr~7^

/a

/(AT,

rfy

Jf

-4f/ a

y) dy

Vci* - v*

i-#

/ (*,

_o^

r~;
- x?

dx

a)

f(x,y)dy-

-^rn?

]*y

Va?

y*

f(x.y)dx.

d[/

(x t y) dx*

t/)

444

__
Answers

*=0

bx

(binormal). 2103.

(osculating

plane);

'

>

__ Q

(principal normal);

v,y.

30

= 0.

192-27

2107.

9100
2109.

21

2108. a)

- n -K-e

.1
a)

fl

flfj*

when

OC

2113.

4~.

2120.

~,

1*

2114.

2121.

= ^-

#=3

2123.

2125

t/

2115.

= x\

= 0'

y\

/=10

r,

1^*25

x*'

11

t/

=
</

= 0;

y = 4.

= 0*

=3

2122.

TT/7.^

2124.

y)d</.

2119.2.4.

2
;c

t/

= * + 9;

= ~\

=x

z'

= 2x;

21
=
/U.
00
dr

y) dy.

2-0

oo

2130.

y)dy.

2126

V)<te

2129.

^y
/

ZX + 9

dx

/ (x.

2X

72

12

y_

y = 2.

6;

10

00
42

= 2,

oo

= 0,

12
*/(*'
00
jc

*/(*. </)dx = d*/(x,


o

7=

-nM
b) P
R-Q-

2127.

2128.

Chapter VII
Q
2116. -~
2117. 50.4. 2118.

x=2

1;

= 3.

jc

~
jf

Ing.

K=

^=

/=!,

jc==l;

When

2112.

i^i

^=2;

K~=-

>^2; b)

a)

f(x,y)dx.

r
2131.

dy

/(^,

i/)^+

f(x,y)dx=

dx

-i

f(x

y)dy+

-*

12
/.

2132.

dx
Ja

2JC

f (x,

y)dy=\dy

f (x t

y)dx.

Answers

445

2133

4-

J<*x
i

J
"
~~
*

dx

(x.

dx

f (dx

c/

y) dy

dy

+J
"

y) dy

/ (x,

y) dy

/(*,

y)dy =

dx

/(x, y)

dx+

dy

dy

f(x,

rr

f (x,

f(x,

y)

y)dx +

t/

i/

+J

dx

Vl + *

f (x,

dx

y)

/ (x,

V9 -X*

dx

V&~^

/
j
-vr^*>

2134.

+J

dy

/ (x, y)

dy

/(x,

\y
f (x,

2135.

y)dx.

a)

- x*

dx

b)

y)

dy

dy

/ (x,

[/

48

2136.

2138.

f dy

dy

/ (x, y)

'

!L

- x*

f(x>

dx. 2137.

+j

ta

dy

2(j

dy

2a

SI

<

dx

*. ffMir.

Ji

y)

12

f (x,

Vx
d*

y) dx c)

JL

2a

y) dxt

y) dx; d)

ax
oo

+ 2u

f (x.

Jdy J

f (Xt

-1

Ki~^

dy

y)

f (x,

-t/'

*
~a

'a

Va*

a
f (x>

_L.

EHi
2

dx

f (x,

y) dx

\
2

dy

JL
3

/ (x,

y) dx.

/ (x,

y) dXr

Answers

446

+
dy(f(x,y)dx
J

(
J

2139.

2
fl
a

20
za

/(*, */)dx

+ jd*/
o

2141.

yf1

10J

f(x,y)dx +

2d

20,
2a

di/

j
o

a+Vtf^H*

11-*

f(x,y)* + jdxj

dx

VT

2
i/

2~~

[dy

f(x,y)dx.

c/

Va*

_ ]/ aG a2 _ y*
yl

2140

dy

V8

a_

00

VT

f(x,

dx

2142.

y)*.

VT^lfi

^I*

VR*-y*

/(A:,

2143.

t/)efy.

/ (x,

J
j
00

f(*,y)dx.

dy

arc sin

jt

2144.

fdf/

arc ^in

2149.

2153.

-p

/(A:,

f/)dx. 2145.

2146.

~ a.

2147.

2148.^-

-^

-g-

y
15jt

1.

2150.

6.

In 2

2151.

a)

dx

2154.

2152.

-|

xr/dt/

b)

16
;

c)

1"

2155.

2JT/?

//

Hint,

2156.

|-Ji/?'.

//

dx

==
dr/

/?(1-COS/)

*Jl

(S)

C dx

cosO^

/?(!

ydy> where

the

last integral

is

obtained from

D4

the preceding one

2159.

by

flf

the

substitution

+i--

216

x=R

(*

sin t).

2157.

2158. --

Jt

T"

sin q>

^T

no
T
[(p

r/(rcos9, rsin(p)dr.

2161.

J
o

cos

(p

Answers
jn

sin

sin

*Ji

sin (p

a /COS

2164.

(tan

r/ (r cos

r sin q>)

cp,

r dr.

dr

5Jt

a /COS

dq>\

a cos

2165.

f
J

f
J

2171.

~nab.

r/ (r cos

dcp

r2

Hint.

-sin(pdr=-^.

2166.

The Jacobian

= abr.

is

1.

= u(\
u

as

2172.

v)

and

cfu

The

0);

the

when

c,

/(w

of u:

when x

limits of integration are

wy,

uv}udu.

Jacobian
x=r-0,

Limits

is

uv^au

(1

t;),

2173.

2~y
dv

Of

whence

fr
\

-TT

variation

of

the

define

whence
of

since

v:

1;

= ^-^

+a

for

= ftx

we

find

dw

"1

(2

'

"~9~)^

Hint '

ablf? 2
L\/i

-r]arctan^-r-

k2 J

ter

chan S e

variables, the equa-

tions of the sides of the square will be u

2174.

= u. We

4-

We

Solution.

u(l~u)=0,

y--^uv\

functions

follows that

wr^axjt

u=r-r-5

dr.

i-

+p
q

(since

q>)

P
i

2167

4-*-

r sin

<p,

<p

limits

2(J)

""T"

have

<p

<p

dq>

<p)

2<p

d<p

/(tanp)dcp

<p

cs a

n
r dr

/ (tan q>) dcp

n_
4

4-

oo

2163.

r/(rcosq>, rsinq>)dr.

(f

cos 2

<p

d<p

sin

t_

2162.

447

bli

+ -Ahk

= v;

Solution.

+ u = 2;

The equation

= 2;
of

u
the

w.

curve

448

__
Answers

r4 **/*

rj-cos*(p-- rrsin

4ht upper

follows that

we

r= y

limit,

cos 2

p-

to the

sin

pDue

to

sin 2

T^

cp

^2

cp

limit

whence

0;

the

entire

of

first

quadrant:

dxdy = 4

a)

4-g-;

-;

a)

^dy
2

b)

dx
~

jJia

2184.

Vy

+ ^dy

a *-

2177

T5.

2180.

2183.

6.

2188.

1(6
o

1188.

,.96.

2202.

2205.

2209.
Hint.

2214.

*.

^dx
2179

flZ -

a-- + -.

dy.

Hlnt

2182.

Change the variables

x2y = u,

I(R
o

a )ln-.

2187.

a).

2193.

A.

abrdr.

~.

2194.

^.

2195. ~-

2197.

iia'(a

J.

2,98.

2206.

.19..^. 2200.^. 220,.^.

^"2"~ 1)-

2207.

nafrc.

2^'

|-

IB (!--). 2210.
Change

~^

2203.

P).

^.

4(m

2l78

u=di/(l-^^=:C^r(l-x)dy.

the
1

ft)/?

2217. 8aarcstn

3
.

2218.

-~ a

-i
O

2204.

(6

J~

2211.

yT

xt/

= u, ~=f.

-1^(3
/"T
o

to polar coordinates. 2221.

1).

2212.

|-jw

\(

(2

~-

2213.

2219.

"2

1).

^a

Hint. Integrate in

'

2208.

5).

ft

variables

2215.

Paas

a)(p

IX

^-^-

dx\ b)

Hint.

lOjt.

sin'cp

aVa*~x*

2181.

2185.

ourselves

_j>

Vy

1175.

- b*
-

<p

dy

(S)

cos'

it

real,

quadrantal angle

confining

fa*

be

integration relative

of

integral,

ak

the

first

region

tan

must

for the

of the

arc

fo

and

be

r will

for

Since

q>

symmetry

we can compute

axes,

lower

the

J,

2
rr cos

q>

ak
have fan<p^7-r-

whence

cp

2 2
/"a 6

KT-1).

+6 c +c a
2 2

the j/z-plane. 2216.

8a 2

1+5LV

2220.

3na2

2
.

Hint.

Hint. Pass to

__
2222.

coordinates.

polar

-5-

2223. 8a arc tan

8aa

Hint. Pass to polar coordinates.

and

fl

T T

V2
L

Integrate by parts, and then

ya

9
2

x'

=?

9
2

+c

8fl

arc sin

\r

^^

variable

the

change

2
Vb* + c*-a }/a + c

^-

ady

Hint.a=\J dx\J

-^5

the answer. 2224

449

a1

/.__

Answers

sin

In

Iransform

t\

'-=)

Hint.

Pass to polar coordinates


-

H=0.

2228

/x

2231.

2226.

^ ^=

=4

2232.

2229
/t

a)

"'

^J;
lz

^=

'

L.

2225.

2227. *==*?,""
o (4

J*

*-;lr-o.

(D*-d4 );

b)

a4

2234.

|-

-3-

/=- f

Hint.

a*.

2230.

/^
Voj?

2233.

JT)

-K^I
16 In

2235.
x

is

(/

29 ~

d=

to

equal

V
of the straight line.

The distance

Hint.

-JL and
2

2236. 7

is

by means

found

^fca

of the point (x, y)

[7

/~T+3

In

from the straight lint

of the

normal equation

(}^"2~+l)],

where k

is

the

proportionality factor. Hint. Placing the coordinate origin at the vertex, the
distance from which is proportional to the density of the lamina, we direct
the coordinate axes alonjj the sides of the square. The moment of inertia is
determined relative to the x-axis Passing to polar coordinates, we have
jt_

_jt_

a sec

/x =

oo
I

d<p

2238. /o

^--

a cosec

(jp

kr

(r sin q>)

dr

dq>

n_

jia

4
.

VR*~^7*

2242.
a

-1

Hint. For the variables of integration take

( dx
)

-*

dr 2237. /

Jg

Jifl

4
.

T
2239.

y (see Problem 2156). 2240.

2241.

(r sin q>)V

<p

kr

dy

x
\

/ (*,

0, x)di

and

Answers

450

VT=T*

2243.

^1 -*'-/,
dy

/(*,</,

-y

-i

2244.

2248 .lln21

4n#
15

_^

2254.

2258.

=f-

2253..

2252. -jutfc.

2257.

2249.

|.

-~a2

2255.

/?.

??fl%.
9

2259.
10

2256.

~ Jia

2260.

?c

|-r
3

Selut ion.

2acos(p

C dx

40

dz

= 2fdq>

L
20 COS

r'dr

"25"

00

-T^^T-

<p

f(2acoscp)*

2na 8

nnot

226h

Hint.Pass

19

2263.

4).

=0;

t/

the cylinder
the jq/-plane

_
=0;

^(a + b +

2265.

2266.

c).

=0, 2=0.

Introduce

Hint.

a.

2269.

~
iz

coordinates.

a2

^(6c

6 2 ).

(3a

coordinates.

spherical

4/i ).

Hint.

For

the axis of

we t^ke the z-axis, for the plane of the base of the cylinder,
The moment of inertia is computed about the x-axis. After
the

passing to cylindrical coordinates,

rdydrdz

to cylindrical

2=-^D
-

*=~,
o

2268.

nabc.

2264.

_
x

Pass

-^-jt.

~(3jt

2267.

Hint.

2262.

coordinates.

to spherical

from

the

x-axis

is

equal

to

square of the distance of an element


r z sin 2

+z

q>

2
.

2270.

(2/z

+ 3a

2
).

Hint. The base of the cone is taken for the xr/-plane, the axis of the cone,
for the 2-axis. The moment of inertia is computed about the x-axis. Passing
to cylindrical coordinates, we have for points of the surface of the cone:
r

= j-

2);

(/

and the square

of

the distance of

the element

2
2271. 2jtfcQ/i (1
the x-axis is equal to r 2 sin cp-f z 2
proportionality factor and Q is the density. Solution.
.

cos a),

rdydrdz from
where

The vertex

is

the

cone
is taken for the coordinate origin and its axis is the z-axis. If we
introduce
spherical coordinates, the equation of the lateral surface of the cone will be
a,

\|)r=--

and

the

equation

of

the

plane

of the

base will be

2t

From

the

follows that the resulting

r=-sin

-.
ip

stress is directed along the

symmetry
The mass of an element of volume dm = p/ 2 cos dcp dtydr, where Q
the density. The component of attraction, along the z-axis, by this element
it

z-axis.
is

of the

of unit

tj)

mass lying at the point

is

equal to

sin

ip

sin

i|?

cos

ty dtp dq>

dr.

Answers

451

L-a
The resulting attraction

oo

equal to

is

h cosec

27i

dq>

\f

Q sin

d\|?

\f

cos

dr.

\|>

2272. Solution. We introduce cylindrical coordinates (Q, cp, z) with origin


at the centre of the sphere and with the z-axis passing through a material
point whose mass we assume equal to m. We denote by % the distance of
2
be the disfrom the centre of the sphere. Let r= >^Q 2 -h(|
z)
tance from the element of volume dv to the mass m. The attractive force of
the element of volume dv of the sphere and the material point m is directed

this point

along

and

equal to

numerically

is

kym

density of the sphere and dv


qd^dQdz
jection of this force on the z-axis is
._,

dF

/\

kmydv

y=

where

the

The

pro-

-^

...
dQ

kmy *^~

is

la

the element of volume.

is

c s (rz)

dz.

d<P

Whence

R
4

But since -x-ynR'

M,

follows that

it

F=

kMm

(p>

2275. a)

2273.

for

>

a; c)

(p

> Q);

Hint. Differentiate
L. 2277.1.
3

n2

dye~ x

y*e~*y

-^-^
A
!'?

d)

P~1~P

> 0)

(p

00

2276.

---

b)

0);

?.

*5

C-^f/=
J

2278.

twice.

ln~.
a

2279. arc tan

--

arc

tan

2282. arc cot

2280.

n (l+a).
-^l
2,

2284. -1

2285. -5.-

24

AW

AW

2283.

4--

1.

polar coordinates.

from S

In

-^

2288.

~
o

coordinate

the

consider 7

2287.

x*

-^
4a-

2286.

'

a2

its

that

e-neighbourhood,

+ y*dxdy, where the eliminated region

is

at

JJ
o6o
\d(p

Whence

\ r

the

Passing to polar

origin.

271

Inr

lim 7 8
e->o

dr=

--=.

round the straight

we have

coordinates,

JL^

-5-

In r

le^JJ rfcp~2jt(-j~-Q-lnen

rrfr

\ *

-T-).
* /

-5L. 2290. Converges for


2
line

is,

a circle of

(S 6 )

radius e with centre

Pass to

Hint.

1).

2289. Converges. Solution. Eliminate

with

together

origin

2281.

(/

=*

with

a>

narrow

1.

strip

2291. Converges. Hint. Sur-

and

put

- =a

J.'
(S)

K(*
K
v

2
I/)
y/

Answers

452

i-

2292.

2297.

l-2-^! arc

2301.

1).

~ll

|-[(I+4Ji

2298.

tan

M*

2302. 2jra

^(lOVTO

2303.

/7~

a*V 2. 2300.^(56

2299.

Hint.

1).

ftl

be interpreted geometrically as the area of a cylindrical sur-

may

y)ds

flu
\

5m

fot

Converges

c
face with generatrix parallel to the z-axis, with base, the contour of integra
tion, and with altitudes equal to the values "of the integrand. Therefore,
o

S=

xds, where

the

is

OA

arc

parabola t/=--x
8

of the

e
(0, 0)

and

a V~3. 2305. 2

(4,6). 2304.

V^ + u

2306.

2308.

2ita

f ji

fl

a)

+ 4jTu

^aTT^

"

-5-;

b) 0;

c)

2309.

Use the parametric

To

,/

(a

+&

equations of a

fl

(-q

.o a

2311.

cases 4. 2314.

all

2315.

circle.

2jta

2
.

30

4;e)4. 2313. In

d)

2307.

2310.40-

h*\

fl

"*"

4-|rln

10

4
2312.

l/~a 2

arc sin

b*-\

that connects the

a *b

/
points

-r-a&
o

2316.

Hint.

2rc.

sin 2.

xa

2317.

2318.

0.

a) 8;

12;

b)

c)

2;

d)

e)

\n(x

+ y);

//a

+ J{

ty (y)

dy-

2319.

2322.

a) 62;

b)

+ ln2;

c) -L

1;

d)

f 9(jc)rfjc-f
J

f)

2.

2320.

y\

Vl-fft
e*~ y (x +
.

c)

2325.

fi-

y)

+ C\

d) ln|Jt

+ j^"M
//,

2328.

JJ

x2

a)

8
-

+ |/| +

2326

4o

+ 3xy

a>

2329

2f/

C.

+ C;

b)

2na(a +

2323.

~ 20

b>

0^
233

^r-^

1;

c)5
4--

x8

x y-{-x
2
2
n/? cos
b). 2324.

d) 0.

2;

2331

2327. /

2332

a ) Ol

(S)
2/iJi.

Hint In Case (b), Green's formula is used in the region between the
b)
contour C and a circle of sufficiently small radius with centre at the coordinate origin 2333. Solution. If we consider that the direction of the tangent
coincides with that of positive circulation of the contour, then cos(X,n)==

= cos(y, 0=/.
as

is

hence,

<cos
J

(X, n)

ds= J

ds = <P dy = Q
4
J
as

the area bounded by the contour C. 2335.

not applicable. 2336. nab.

2337. -| Jia

2
.

Hint. Green's formula

4.

2338. 6jia

2334. 2S, where

2
.

2339.

-|a
*

2
.

Hint.

is

Put

__
Answers

45$

for
2341. n(R + r) (/? + 2r); 6n
y=tx where t is a parameter. 2340. ~.
R r Hint. The equation of an epicycloid is of the form x = (R -f r) cos f
r
~rcos^JL. *, y = (/?-f-r)sin/ rsin^J^/, where / is the angle of turn of
2

radius

the

2342. Ji(/?
is

cycloid

Problem
2345.

V
c)

of

r)(#

~ nR

2r),

drawn

circle

stationary
2

for

-^-(a

where k

),

work,

mgz,

potential,

mg(z

is

2 2 );

L(jc

of

the

+ + *)

work, 4r (#

2346.

= -ii-,

potential,

epicycloid
2344. mg(z,

FR.

factor,

proportionality

b)

tangency.
the hypo-

of

corresponding
2343.

of

point

The equation

Hint.

'=-7-

obtained from the equation


2341) by
replacing r by

the

to

(see
z 2 ).

Potential,

a)
-

work,
2

2347.

).

-|jta*.

2348.

2353.

2354.

a.

/J*"

/i*.

51)

10 (5

2355. a)

0;

b)

-{(

(cos

+ cos p

2356. 0. 2357.

na\

2358.

4;i.

2359.

2360.

a*.

(V)

2365. 3a4

2366.

-^
2

2378. grad(rr)

/'(/)

the vector

=---

COS

2385. a)

>r)

^=0

r ot

r) t

2387.

2388. div grad

2392.

2a)/i

U=
2

a)

5^

for

j~Ji/?

(/(r)<7)
,

where

57

= lgrad(y|

= ^-^cxr.
is

The

3y

3Jfe;

c)

b)

div

rot (/r)

a=^

2380.

/ (r)

~=
/'

(r).

-^-;

2386. div tf=-0;

c)

div

rotv = 2co,

where

unit vector parallel to the axis oi rotation.

2
);

when a = 6 = c.
2383.

div(r^)-=~,

b)

+ 2//

points except the origin.

= c 2 2376. grad (/ (>1)=9/


x = y = z.
2377. a) ~; b) 2r
,

2382.-?-.

IJ_r.

^+^+^

//(3/?

the level surfaces are planes perpendicular to

7^,

/i

2371. Spheres; cylinders.

2363.

= xy\

= c;

divr=3, rotr=0;

= LW( Ct
co=co/f

2379.

c.

+/=c
*

d)

Jia

2373. Circles, x 2

2372. Cones.

2367.

rot

~ n/?

//

grad
2
(

flux is equal to

U = 0.

+ 2//

2
).

2391.

2393.

4jtm. Hint.

3;i/? //.

divF=Oat

When

all

calculating

454

__
Answers

the

flux,

use

theorem.

the Ostrogradsky-Gauss

2394.

_ ~~TtD

2395.

2n*h*.

2396.

/=f

rf(r) dr. 2397.


.

2398.

-L.

2402.

24 8

potential;

U=xyz + C',

b)

2400. Yes.

Chapter
2401

No

a)

2403.

VIII

2404.

2405. -

24 06.

2416. Diverges. 2417. Converges. 2418. Diverges. 2419. Diverges. 2420. Diverges.
2421. Diverges. 2422. Diverges. 2423. Diverges. 2424. Diverges. 2425. Converges. 2426. Converges. 2427. Converges. 2428. Converges. 2429. Converges.
2430. Converges. 2431. Converges. 2432. Converges. 2433. Converges. 2434. Diverges. 2435. Diverges. 2436. Converges. 2437. Diverges. 2438. Converges.
2439. Converges. 2440. Converges. 2441. Diverges. 2442. Converges. 2443. Converges. 2444. Converges. 2445. Converges. 2446. Converges. 2447. Converges.
2448. Converges. 2449. Converges. 2450. Diverges. 2451. Converges. 2452/Di2455. Diverges.
2453. Converges.
2454. Diverges.
2456. Converges.
verges.
2457. Diverges. 2458. Converges. 2459. Diverges. 2460. Converges. 2461. Di2462. Converges.
2463. Diverges. 2464. Converges. 2465. Converges.
verges.
2466. Converges. 2467. Diverges.

k- >
!

2468. Diverges. Hint,

2470. Con-

verges conditionally. 2471. Converges conditionally. 2472. Converges absolutely 2473. Diverges. 2474. Converges conditionally. 2475. Converges absolutely.
2477. Converges absolutely. 2478. Converges
2476. Converges conditionally.
absolutely. 2479. Diverges. 2480. Converges absolutely. 2481. Converges conditionally. 2482. Converges absolutely. 2484. a) Diverges; b) converges absolutely; c) diverges; d) converges conditionally. Hint. In examples (a) and (d)
CO

consider the series

( a 2k-i

+ a zk)

anc

in

examples

and

(b)

(c)

investigate

2486

Converges

fe=i
00

separately the series

2a
k=i

00

2k-\ and

2 a^'
k=i

2485<

Diver ^ es

absolutely. 2487. Converges absolutely. 2488. Converges conditionally. 2489.


Diverges. 2490. Converges absolutely. 2491. Converges absolutely. 2492. ConCD

verges absolutely. 2493. Yes. 2494. No. 2495.

7.9*1

f9n_iv

^T

3/2

converges. 2496.

n=l
converges.

2497. Diverges.

2499.

Converges. 2500. Converges.

fi=i

Hint. The remainder of the series

geometric

progression

may

exceeding

be evaluated
this

by means

remainder:

Rn

'

of the

an

"9*

sum
,

of

__

Answers

455

--

-'' 2504.

m + "-=~

-f

2505

For tne

<.<.

iven series

it

is

Solution.

to find the

easy

exact value of the remainder:

Solution.

We

multiply

-:

by

*"+ 2

M+
"

Whence we obtain
is

nv nv
.

=n

\ zn

VTj

v n+

*
.

"**,

.
'

= / + 16
T5

4 /

+\

}_

16

From

this

the series

we

find the

above value

Rn

of

Putting /i=0, we find the sum of

2507. 2;

2506. 99; 999.

S-=(pY-

3;

5.

2508.5=1.

Hint.

when x < 0; S-=0 when


> 0, S=
2511. Converges
diverges for x<
Converges absolutely for x>
for 0<x<l,
diverges for
for
conditionally
x>l,
converges
absolutely
^"
for
conditionally
forx>*,
converges
2512
absolutely
Converges
<x < oo..2514. - oo <*<.
2513. or X <\.
*c, diverges
verges for^
2515. Converges absolutely for x > 0, diverges for x<0. Solution. 1)
<~; and when x > the series with general term ^.converges; 2)^5^1

^~
x = Q. 2510.

an

2509.5-1 when

1.

1,

x.

oo, since from cosn*->0


xs^O, and cos n.x does not tend to zero as n
thus, the necessary condition for converwould follow that cos 2n.v -*
when 2/m < x <
gence is violated when x<0. 2516. Converges absolutely

for

it

it diverges.
1,
2, ...); at the remaining points
<(2fc + l)ji(fc=sO
1
for x ^ 0. 2519. x
2518.
absolutely
Converges
Diverges everywhere.
f

2517.

> x<
2523.
x < 4,

2520.

x>3, x<l.

x>l,x<-l.

2521.
2524.

x^l,

x<l.

-Kx<

i-,

2522.

x^5j,

y<x<l.

Hint.

1.

For these values

00

of x,

both the series

x k and

the series

converge>

When '^l^

__

456

and

when

=-

the

Kx<0,

2525.

Answers

term of the series does not tend to zero

general

0<x<l.

l<x<K

2526.

\<x<\ 2529. -- <x<-~


2

2528.

1<X<1.
< x < 4.
3<x<3.

2532.

2536.
2540.

2530.
!<*<!. 2531. !<*<!
2
2535.
oo<*<oo.
2534. x^=0.

~ <*<

2538.

-5-

2542.

<x<2. 2539.
< x < Solution.
2

\<x<{

2541.

2<x<2.

2527.

K
V
oo<x<oo.

2533.
2537.

_
1

<x<e.

The diver-

x
1
of the series for
is obvious (it is interesting,
however, to note
that the divergence of the series at the end-points of the interval of converthe aid of the necessary condition
is detected not only with
1
gence

gence

x=

by means

of convergence, but also

d'Alembert

of the

<

When|x|

test).

we

have

lim

n\

nIn

lim

|(rt+l);t

|<lim(Ai+l)|jcr==
rc-- co

oo

lim
n-* oo

readily obtained by means of 1'Hospital's rule).


2543.
l<;x^l Hint. Using the d'Alembert test, it is possible not only
to find the interval of convergence, but also to investigate the convergence
of the given series at the extremities of the interval of convergence. 2544.
Hint. Using the Cauchy test, it is possible not only to find the
interval of convergence, but also to investigate the convergence of the given
series at the extremities of the interval of convergence. 2C45.
2
x
4. 2548.
2.
2547.
2549.
2546.
(this equality

is

l<x<l.

< <

2<x<8.

2550. x

2554.

3<x<e

3<x<

2558.

0<x<4. 2553. - ~ < x < ^


2<x<0. 2556. 2 < x < 4 2557. 1<*
< x < +-I6 Hint. For x=
the
G

7<x<

2551.

2555.

3.

2559.

2<x^8.

4<x<

l<x<3

2552.

-j^^
~
\

series

2561.
2566.

diverges,

lim
n -+

oo

(~-l< x <l)

-ln(l-A:)

ll n

lf(|xl)

2579.
.

arc

2577.

tan x

sum
2586.

^ farctanx--

In

x
of

3.

the series

2587.

ax

\^}

|<

(|xl<1)

(\x\<

1).

2585.

2580.
.

<
<i
|

(-

In(l-f-x)
1)..

2563.

2570.

2583

^---^

Hint. Consider the

x*

^-+-=

=l+

(|

<x<

2560.

2582>

2384.

ye
1<*<3 2562. l<jc<5. 2563. 2<x<4. 2564. |z| <
z
2/
< 3 2567. \z\< V^ 2568. 2 = 2569. z \< ao

2576.

2578.

since

...

(see

Problem

_ 00<x<00

2579)

for

2688. sin

jc=~^=:
x

'

Answers

xsm

cos (jc-|-a)=cos a

2589.

457
**

gating

the

)""

+ --^5S
A

-j-

~2<x<2.

Hint.

When

investi-

7*

use

remainder,

theorem

the

on

integrating

power

series

2592

_ l\"-

on-i

n=o
oo

< *<

e**=

2595.

oo

+V

oo< x <

oo

2596.

n=i

(_-oo

oo

<x<

<x<

2599.

(-!)"

2600.

2608.

oo)

oo

(-3<x<3).

rn-

T (-0" H

04/1-S y2n

2609.

(-OO<AC<CO).
1 -4-

*
2611. 2-f

22^11

+3

2610. 8

-|.._ +

2601.

(2rz) |

**

2s 32
-

...

2591

o.

+(-o^
3Z
^Z + TTi-"

x*

^ cos a + %r sma+'-rr cos a

2!

"*"

2W

-4-

3"

""

^ (=

W
JC

(~oo<x<oo).

I)""

vn

458

Answers

<-<

<X <oo).

2612.

+V

f|

(l+2- n )

l)

+ o7rFT

*"

'

~ 2 < * < 2)

'

2614.

).

2615. ln2

Ti

2616.

(-.i<jc<l).

V(-.l) x
=o

(-D n+I

2018.

(I*KD.

'"

2622.

e(

T+S-;2624.

219.

c+

1-^4-...).

262S

-(^+75+^+---)

!+

2623.

+...

+ x* + jx>+... 2626. Hint. Proceed= &sin<p, comthe ellipse x = aco$<f,

ing from the parametric equations of


i/
pute the length of the ellipse and expand the expression obtained in a series
2
of
2628.
*'
e.
2x*
5x
2= 78 59 (x-H) 14(x
of
powers
2629. / (je
5x 8
4* 2
4)
(
oo<At<oo).
(x
A)
z
*
oo
h
oo).

+
+ =

+ +

< <

2630.

Ar

n -1

V(

l)

^"^

(0<JC<2).

2631.

n=i

00

2632.

1)" (*

1)"

(Q

< x < 2).

AI=O
00

(n

6<x<

(x

1)

2).

+ \) n

< x < 0).

2633.

2634.
/l=0

(,<.
1.3(^-4)'
r 4-6
2

L3.5(*-4)
4.6-8

2"

2J(-D"

n=i

n=i

(0<x<oo).

2637.

4-i

"T---TI

JL

(2 ttll)!

2636.2+^-1^
1.3.5...(2n-3)(x-4)
4.6-8...2n

O^K

00 )-

""^

*""

2638.

~+

Answers

Make

Hint.

the substitution

45J>

and

expand

\RI<<1.

2641.

2642.

\nx

in

of

powers

R \<

t.

2643.

p r ve that

exceed 0.001, it is necessary to evaluate the remainder by means of a geometric progression that exceeds this remainder. 2644. Two terms, that is,
7

X2
1

TT.

x*

Two

2645.

terms,

2647. 99; 999. 2648.

*J<

0.7^68.

2659.

-"

/y

\2
.

0.005. 2650. 2.087. 2651.

<0

0621

2656.

+ V (-l)JfZ^L.

2660.

R \<

x |<0 39; \x\

0.608

2655.

2646. Eight terms, i.e.,

2649. 4.8

U|< 0.22. 2652.

0.39;

2fi54.

1.92

i.e.,

~-

~~ 03
Z

2657.

0.2505

V-L
jL*

2658.

0.026.

(2/0'

00

2661.

V
n=

JO

2662.

1-1-2

x)";

(//

l-^-y +

ry

i/1

<

Hint.

x^

2663.

geometric progression

Hinl.

lJC

= (l-.^)(l-y).

yW

|T^^=- +
!

(_

2664.

~~

K//<1).

Hint, arc tan


-j

2665.

/ (Jt

\-2bh-\-ck

/i,

#+

2666.

A;)

~^-=arc tan x + arc

tan (/(for

A^y

= ax* + 2bxy + cy* + 2 (a* + fy) + 2 (bx + cy)


/i

f(\+h,

2-\-k)

f(\

2)

=9/i

21/r -f3/i

-2^.2667.

2669.

l+x+-

-+...

n\

\x"< 0.69;
-^ 0.4931.

\ o
o

<x<oo; ~oo <t/<oo).

(-00

18 2653.

1-f

2670.

Answers

460

%-l
n=o

(n)=^=%.

^ + 4^(-l)";S(

2873.

n)

= n. 2674.

n=i

[^-

L2a

x ^4

V tr^!
+ <*a*
+

(fl

cos n*

sin

n*) 1

rt

l>fl

l)

|tlyl

m
nf

a*

S (+

Ji)

= cosh an.

2 Sln
2675.

is

nonintegral; sin ax

if

is

an integer;

gn

5(n) = 0.

n=i
2676.
Ji

2a

S(

integer;

2678.

Jl )

+ jL*
y _i )B
(

if

or

n=i

= cosan.

is

nonintegral; cosa*

t\r

J
2677.

=
:

CO

2680.

tin

sin

"-

if

is

an

Answers

2689

461

slnn/t
.

2691

i_

2694.

4n-l

Solution.

2 P
--

1)

2a

= -~

f /(*) cos 2/i* dx

(*

-jj-

cos 2/ix d

Ji

/ (x)

cos 2nx dx.

we make

If

the

substitution t**-=

in

the

first

Jl
integral and

identity
(n = 0,

=x

second, then, taking advantage of the assumed

in the

/(-^+<
1,^2,

= -/

will

<

(^

&,

(y +

/ (^) sin

=/

2nx dx =

2697.

~. T

sin

Sin

-|-

Case

as in

(\^

(1),

with

account taken

^ leads to the equalities 6 2II

of

the assumed

(/i=l,

2, ...).

'

/cos
sinh

o 2n =0

The same substitution


Identity

that

be seen

...);

n
2)

readily

[t
/IJTJC

> (-1)"

2698.

cos

sin-

_8_

where

1
^

nx

-16\\-1)"-'

-~,

(2'?

1^1

_-!?
,(2'H-l)Jix
o

-il

2702.

b)
1)2

n=i

_ A V ^ J 2 1+
1

n=o

!)

n*

2'mx

rs

mx

Answers

462

Chapter IX
2704. Yes. 2705. No. 2706. Yes. 2707. Yes. 2708. Yes. 2709.

no.

Yes;

a)
b)
= 0. 2716.
= 0. 2715.
2xy' = Q. 2717.
=
=
x
2xyy'. 2720. xyy (xy*+ 1)=
y' = y. 2719. 3y
= 0. 2724. /
2721. w=-xr/'ln~. 2722. 2xy" + y'=Q. 2723.
2725. #"
3f/V -^0.
^"'^O. 2728. (1 +y' )y"'
+ = 0. 2726. = 0. 2727.
2X
= cos*. 2732. r/==
2730.
2731.
x = 25.
2729.
y = xe
^^.r^Sg-^
+ g^ 4g 2X 2738. 2.593 (exact value y = 2739. 4.780 [exact
o
= 3(e 1)]. 2740. 0.946 (exact value y=\). 2741. 1.826 (exact value
value
2742. cot //-tan x + C. 2743. ^=
y = 0. 2744.

Yes.

2710.

2714. /
0. 2718.

xi/'

;u/'

2{/

r/

1.

(/"

2r/

i/'

I/

2//'

f/"

t/

//

e).

).

i/

..

= lnCx

2745.

= a + -^-.
2746. tan y^=C (1
-~

2748. 2e

=1^6 (l+e x
7|

= C.
ment

2760.

2762. if

8x
5x

2752.
2754.

*=2px. 2761.

t/

= 0. 2747. = C sin
i/

jc.

2751.

y=l.

2750.

or

hyperbola #

+^

2
J/

= ax

Hint.

to

respect

x,

2758

^2

2
-

I/

By

hypothesis

we

get

The

Hint.

seg-

= C. 2759. =
r/

differential

equation.

-*.

y=V 4

^2

+ 21n-

Family

of circles x

2764.

Pencil of lines y^=kx. 2765. Fa-

+ y = C 2766. Family of hyperbolas x y = C.


=~ ~
2769.
+ (y b) = b 2768. = xln~.
x
x
&
z

mily of similar ellipses 2x*


2767.

+ 2y+l=2tan(4x + C). 2753.


+ 10y + C = 31n lOx 5y + 6| 2755.

f/^Cx

t/

with

twice

equal to

is

i+^srrf-j.

2749.

).

2757. Straight line

Differentiating

2763.

= C.

of the tangent

= Ce a

+ if

f/

r/

X
.

= Ce y

2771.

+ ln|(/|=C.
2775. y

xl

C)

2773. y=5

x.
C,

(x

2776.

2778.

2
t/

=C

2
;

^-;
(x

+y

ln|4jc +

8t/

(x

2)

= 0.

2
//

= 4;

2774.

= (xy + 3).
+ 5| + 8j/-4;t = C.
!)

(;

i/

(x*

jc.

2772.

+ y*)* (x + y)*C.

2777.
2779.

x 2 =l-2[/.

__
Answers

463

2780. Paraboloid of revolution. Solution. By virtue of symmetry tUe soughtfor mirror is a surface of revolution. The coordinate origin is located in the
source of light; the x-axis is the direction of the pencil of rays. If a tangent
at any point
(x, y) of the curve, generated by the desired surface being cut
by the xi/-plane, forms with the x-axis an angle q>, and the segment connecttan 2q>
(x, y) forms an angle a, then tan a
ing the origin with the point

= y The desired differential equation is


2=
= 2Cx-\-C*. The plane section is a para2jq/' and its solution is y*
The desired surface is a paraboloid of revolution. 2781. (x y) 2 C*/ = 0.
Hint. Use the fact that the area
x 2 )-=Cx
x = C(2# + C). 2783.
But tana

tan 2 q>

tancp
^

j/'.

*/*/'

bola.
2782.

(2(/

X
is

to

equal

~--* 4 +

y dx.

2787. x

t>x and ~.

respect

2793.

r/

= Cx

2784. y

In

|x

+ + cos y = C.
2

= Q/

f/

= xln-.

2789.

x
(y

+C

+ C)

x(

2C|/)

=x

2808.

2799. x

= 01n

is

lnU|

with

2790.

singular

.v

1/

= C.

is

no

singular
2
/

2801.

2812.

= 0.

2810.

2813.

= 0.

2815.

integral.

General

integral

"o"""^~ r"^

General

integral

1/^3

2816.

C2 +

(A

HO singular integral. 2814. General integral


there

1.

x*' =

2806.

~ + ~==C.

2809.

integral

+-=

2803.

2
singular integral x

^2Cx;

t/

linear

2800.

(xsiii[/-j-f/cost/~ sin^)c =tC.

= 0;

there

~+C J=0;

ry2.|-C

=2

2811.

c.

is

y=^+

2 arc tan-

//

2786.

2
2794. x ^=

= C.

ab

+ arc sin x)

= 0.

?807.

= Cx + x

The equation

Hint,

!/

2788. x

2785.

|.

f/^-^cosxi

-~-

sin x. 2817.

- + C,

Singular
2821.

In

solution:

V p

t/

= 0.

2820.

+ + arctan~ = C,
2

i/

4y = x

x^\n

+p

1
,

In |p

p
.

Singular

x|

= C+

solution:

y=e

Answers

464

2822.

2823

g-C*+;

y=2*.

L=4

l* =

nlPL-zIl'sin P + C '2825.<

from which x

equation

=
*

y=xu\

j/

2829.

= Cx + C;
*/

= Cx +

The

Hint.

defined as a function of p

is

2827.

+ y*=\.

and the family of


geneous,
equation;

2824.

homogeneous. 2826.

is

no singular solution
2

-g

= 4*.

2830.

differential

2828.

JO/-C

2831.

circle

astroid x*"
a z i>. 2833. a) Homoy*i>
uv\ c) linear in y\ y
uv, d) Bernoulli's
with variables separable; f) Clairaut's equation; reduce

tangents. 2832.
b) linear in x\ x
its

The

y = uv\_e)
to y^xy'
V^y^l g) Lagrange's equation; differentiate with respect to x\ h)
Bernoulli's equation; y = uv\ i) leads to equation with variable? separable;
u--x-}-y\ j) Lagrange's equation; differentiate with respect to x\ k) Bernoulli's equation in r, x=uv\
1) exact differential equation; m) linear; y = uv\
n) Bernoulli's equation;

+ = Q/
4

2
2835. x

(/

2836.

= uv.

/=i~.

lnC; singular solution,


solution,

y = ~.

2840.

3//

2842. y

2834. a)

i/

2837.

In

sin-^

xi/(C-~

f- (x+l)

jc|

In

+ C;

x)=

x^y-

b)

2838.

1.

y=-Cx+ VaC\ singular


2841.
***-^ -arc tan y~

2839.

+ L~1J = C.
ln

= x(l+Ce*).

2843.

y=

Ar=</*(C-e-> ).

2844.

X -f2847.

x = Ce

sIn

^-2a(l+sini/).

2848.

i + Sx + y + ln [(x-3)

10
1

y-

8
1
|

= C.

-= In Cx.

2849. 2 arc tan

2852.

l- +
r

cos*.
+ 4o

-l.

2860.

2863. y

ln|x|

2855.

xy

2858. x

= C.

= C(y

2850.

2853.

1).

(/

**=! ---\-Ce

= * arc sin (Cx).

2856. x

y
.

2851. x 3

2854. y i ^Ce" ix

= C^-- (sin y + cos

= CeV-</-f,'-,/-

=-Ce^

2859.

/).

y
sin

2857.

jc+

py-

-0.
2

y x

ij

2879. f/-0.

y^Cx + -~,

2873.

i/

= ~^/2?.

+ 4# ^Q/ 2876. 0=-=*


2880.
2881. y =
(sin x + cos x).
i/^-^=
=
2. 2883. a) y
x; b)
Cx, where C

^C.

2875.

1.

-i-

2874.

2877. r/-=x. 2878.

(2x

x'-f-x

+ 2x + l).

*/

= 2.

2882.

2x
is arbitrary; the point (0,0)
t/
2
singular point of the differential equation. 2884. a) y 2
x\ h) #
2px;
2
2
2
2
2
(0,0) is a singular point. 2885. a) (x
C) 4-(/ -=C b) no solution; c) x
-=x;
t/
---=e-*

is

2891.

2-=0

8
i/

e~ x

--1

-C

f
.v

A:

that

the

area

is

to

equal

:c

y dx

and

arc

the

length,

to

x=^^--f

Cf/.

2897.

//

H^l/ 2

-^^

<>

2896.

a singular point. 2886. // --e". 2887. y^(V~2a


Kx) 2 2888. f/ 2
2889. r-=Ce^. Hint. Pass to polar coordinates. 2890. 3y 2
2
2
2892. x 2 -|- (//
2893. // 2 [- 16x^0. 2894.
6) -_6
Hyperbola
r-^/fq)
2
2
2
--C or circle x i-j/
2895. t/--i- (e-v
e- x ). Hint. Use tha fact

is

(0,0)

- 4C (C

-j-

x).

2898. Hint. Use the fact that tha

resultant of the force of s>ravit\ and the centrifugal force is normal to the surface.
'lakmg the r/-axih as the nxis of rotation and denoting by co the angular velocity "of rotation, \\ e get for the plane axial cross-section of the desired surface

the differential equation g-'-'==o) 2 x.

2899. p_^ e --*<wi7*.

Hint,

The

prts-

sure at each level of a vertical column of air may be considered as due solely
to the pressure of the upper-lying layers Use the law of Boyle-Mar otte, according to \\hich the density is proportional to the pressure. The sought-for
differential

equation

.-.kw-dx.

2901.

is

s=/>-j--a;
~
/

one hour.

2904.

kpdh. 2900.

dp

will decay in 100 years. Hint.

=^35.2

dQ==

sec.

kQ

Equation

Hint. Equation n(/i f

dfi.

Qo

2905.

m^ = mgkv

~=

*f~V

a)e~

Q-=Q (4-)

as

;v=

2910.

ds

Equation
ki

In

2903.

the initial quantity

of

~V udf.

=n

2908< u "~^

2909. 18.1 kg. Hint. Equation

4 2?6

^ =kQ.

2fc)d/i

(y)"*

tionality factor). Hint. Equation

16-1900

rpm.

Hint.

T^a + (T Q

2902.

(o^lOO(-g-J

s^klw.

2906.

2907.

"^

'

n^-

is

__

=^=

Hint,

g~tanh ft
I/

~)

[(/? sin

wf

Answers

466

+ L~ = E sinco*.

Hint. Equation Ri

].

2912.

+C

z.

2914.

#=<:,

+ {;, In

2915.

= C ed?

2916.

2918.

(x

C,)

i/

/=

2913.

2911.

r/

= aln
: 2 ;y

= C.

+C

in~|

2021.

2923.

</-

(singular solu-

2925. y

tion).

= C x(x

2929.

=C
x=C

2935.

2930.

1).

sin(C,

=x+

x^^

2933.

2y

4x z

2934.

=\.

y=^x

2941.

2944.

-'e-r

2947.

x=

2950.

2954. w

--e"^ 2

= + Cj + D

x2

n-

(^i

2951.

No

solution.

2932.

~
2939

^8

-2)

(i/-|

2948.

x.

//-C,
//

In

2938.

"--*

2943.

//

2028. y

//=-*-]-!.

2945.

1-e'

t/^sec
.

x=

2942.

t/-

= Cx

Jt^C,

2937.

ln|

2940.

2931. y

l.

2936.

2926.

(singular solution).

y=

2927.

r/-^=

= ~- + C

2946

1
-

e*

/y

-"

2949.
2952. y

= ex

2953.

//-2

In

--

A'
|

+C

o-Cj^+l)*

C 2 )A: + C 2

2.

Singular solution,

y^

4-

2959.

(x

(x.

Singular solution,

^
;

r/

= C.

2955.

+C.

2956.

2957.
-singular solution, t/=-

2960. Catenary,

2
)

r/

= 2at/~ a

=a

2
.

2958.

cosh

Cycloid, x

Circles.

Circle,

=a

(t

(x
sin /).

y = a (1

C^-\-kC\

C,)

= a2

cos

2961.
/).

<7

O. 2963. Parabola. 2964.

^-e""* +

2962.
</

"^

-0.

Parabola,

__
Answers

4- C*

x.

Xcosh
The

differential

=g(sina

-fC 2 lnx.

2973. y
the

= A + Bx

the substitution

2967. In

No, b) yes>

2972.

= C,x-f

f/

Hint. Particular so-

AT

/2

---

method

the

By

x3

C ^~-\-A, C z

find:

\"*/

= */,.

//

we

-}

y< 2968. a)

= --!- 4 * + ^L

2974.

-{-x\

homogeneous equation r/j.v,

variation of parameters

Use

Hint.

3y"+4i/~ 2//-0

y'"

(C,sm x-}-C 2 cosx).

2966. s

2969. a) t/'-{-y
Q\ b) y"
2y' -f f/-=0;
3jt
5x f
2x 8 2971. //-=
2970. j/

no, g) no, h) yes

f)

mg

m-r^
at*

Hint*

E(l uation of

'

cos a)

u,

300 d~x
~A~Z~

Hint. Equation of motion,

2x//'4-2f/-0, d)

2965

s^^- (sin a

motion,

of

^'
^\T)'

Hint. Equation of motion,

).

d) yes, e) no,

yes,
A-

tn/

6.45 seconds.

c)

= 77

^22

= a.

constant horizontal tension, and

is

Law

jiicosa).

c)

equation

I/

Xlncoshf/

where

|-C 2t

467

2975.

-\-B

the

of

= A-\-

=
+

2X

Ccos x-Hn|sec x-f-tan ^I-f-sinx In cos v| xcosx. 2976. f/ C,e -h


C 2 sin x.
2977. //^-C,e" 3 * {-C 2 e 3V 2978. // -=C t -|- C 2 e x 2979. */
C, cos x
v
2980. //--e' (CicosA'-l-C 2 Sin A-) 2981. y=~ e~** (C t cos 3v-|-C 2 sin 3x) 2982. y
-|

sin x-\
8

H-C'jC

2983. i/-.--e- (C,e r

(C^i-Cx)^-*
*

- CV*

-r

C,e"

.//-4tf
2992.

//=-

sin

^4cos2v

/J

y(A\ -\-B\i-C}

4- xe

av

^-i-3A
2998.

-I

//

--.

C 2 sin

C,c*

7 'v

I-

C2

sin x.

t>

(/;

C a x)

2995. [/^(C.-j

2xJ

nx

+2

-{-

--

f)

3001. y

16*

cos 2x 4-

C 2 sin

2x)

//

= a cosh

+
+

4 cos 2* h
e)

;<

x \-F)x

'

+ 4x

(2x

a x)

-f-

3003.

!,

2996. y

3).

P~ X -[-{-

= ^ (C

ic

<>**.

~ xex

3000. y

= C, cos x-f-

2
-p-

(3 sin 2x-|-cos 2.v). 3002.

i/

== (C 1

3005.

sm2x).
1

sin 2x.

f/-

0,

sin

+ C, sin

i/=l. 2991.

x
e

A;

= C,+C e- x + ~x + ^r.(2cos2x
X

J-

a)

= C,e* + C e~ 2X
.

C, cos

- C,<?* H- C^"*

3004.

C2

-[-

>

ft

i/"TT

/
I

= sin 2x. 2990.

= (C, -f C

J^^fex

If

xe** (Ax*
Bx C); b)
fl sin x),
d) g* (^ cos x
xc* [(Av -j- fiv
C) cos 2x+ (>x*-f

+1

2999. y

ax

2997.

r/

2994.

);

2984.

).

j/-=C, cos

+ CvV

2v

sin

+ C^~* rl

2986. //=.g

2988. f/-=e~*. 2989.

2993.
c)

X sin

-|-C 2 e

+ e lv

/y-0

-|-/Jsin2x;

(C t e

0,

---x
2

*
2

(/=*

<

if

2985.

3006.

//

cos 2x -f

-^-

(sin

(/=

+ sin 2x).

__
Answers

468

$007.

1)

=C

cosco/

+ C 1 sinci)f + -j

-^slnp/;

2)

= Cj cos w/ + C, sin

$/

3010.

y^CfV + CStx**. 3009. ^ =


2X
= **(<:! + C x + x 3011. = C + C e +
-|x.
yA-e
=
^ + 4-(
3cos2x + sin2x
3013.
y
o

-f.cosorf. 3008.

2Jf

).

t/

3012.

CD/

)-

~
= (CjCos3jc + C sln3A;)gx + ~
= C, + C e * ~ (cos* + 3 sin*)
3018.
+ C x + *V w + ^4-^.
1U
o

X*~* + |-e*.

= (C

3017. y

cosx.

+C

ie(4x + l)-^-^ + j.

3021. y
sin

= C 6" * + C
1

2x

(3 sin 2*

-|-

2xcosx). 3024.

i/

3x)+~(2x

(2

x)e

8X
.

= C e~~ x

+ 2cos2x).

2x

^. (sin

3023.

8;f

I)e

j(3x

= C, + C c

IJC

the

(2x

x -f

cos x

-j-

*/

(5, cos x

t/^C, cos 3x-

3028. y

+ x)^^ +

sin

cosines to the

of

product

ex

-x

2jce*-~ x

sin

3022.

3026. t/^

= C e-* + C ^-

y = Cj cos x + C 2

|/

x)e*. 3025.

3027. y

Transform

Hint.

-f r^sinSx.

IJC

+ 2 cos2x) + -j

3029.

8030.

3020.

= C e x H-C e- x + ~ (%
+

*/

3019. y

= C, cos 2x

_^_*.
xslnx

3016.

<

-^-

sum

of cosines.

+ C ex +xe*sin x + e* cos x. 3032. = C, cos x + C


=
tan ~
+ --- 3033
+ cos;cln cot
x
x
x
3035.
3034. y = (C + C x)ex + xe \n\x\.
x
t/=^(C + C x)e-~ + xe^
-=
3036.
3037.
y = Cj cos x C sin ^ + x sin x + cos x In cos x
C, cos x +
=
+ C sinx *cos* + sinxlnjsin*|. 3038. a) y C e* + Cf-* + (e* + e~*)x
V
1/t
Xarctan^; b) = C,^
+e*'.
+Cf-*
3040._EquatIon of motion,
3031. y

t/

\Ti

-f-

|.

(/

2#
-= -5

sin30/

t/

60

-~^

g^sin

900

"^7 cm

sec.

|
.

ff

x-

rec koned

'

3041.

from the position of

4
k (x g
is the distance of
l), where x
-jjt"=*4
the point of rest of the load from the initial point of suspension of the spring,

rest of the load,

is

the length of

then

the spring at rest;

+ xy

therefore, k (*

/)

= 4,

hence, -i^L*

k(x

y),

where *-4, ^ = 981 cm/sec

1
.

3042.

d*x.

dt*

= k(bx)k(b + x)

Answers

and x = ccos

)
J

6~
In (6+ 1^35).
3044.2a)r=
^35). 3044.
~=r=gs;
-gs;=: j/
y -In

3043. 6

e~ wi Hint. The differential equation of motion


+ e-"*); b) r = (e
= o)V. 3045. y = C, + C,e* + C e 12X 3046. y = C, + C <T * + C e*.
ll)t

(e'tf

2
is

469

2,(H

^
or

C 4 e" x 2 3049.
3048. i/ -= C,
C 2 x C,e* 2
C 2 sin x) +e~ x (C 3 cos x -\3050. // c_e* (C, cos x
C 4 x) sin 2x
3051 y
(C,
(C, -j- C 2 x) cos 2x
^ /
i/T

3065. w--_C e" x


1

3066.

//

= Cj

4-

+C

y=-e~

-\-e

sin

3068. y -- (C,

3073.

3075.
3077.

j/

C2

In x)

C, cos (2 In x)

3070. y

3071.

-I-

= C,JC-hC

~
+C

x 2 |-C 3 x 3

r/^Qx-H-

2
.

3074.

-1

sec x

|/-j

cos

3069.

8 /

+ cos x In
i/-j

xH-

I/"Q-

~
\ 4

p^sin ^r- x

cos v

tan x sin x
|

+ x sin x.

\
)

2
/
/3
y
C,x + -%

2.

-|-x

sin (2 In x).
4

3072.

/=- C,-|-C 2 (3jcH-2)-

f/^Q cos

(In x)-f

C 2 sin

/3

(In x).

= (^+l)MC + C ln(jc-H)] f (x-}-l)


= C x + C x +-o * 3076
2
= x(lnx + ln x). 3078. y = C, cos x + C sin x, z = C cos x C sinr
x
y = e~ (C cosx + C
2

i/

!/

f/

3079.

-^*
C 4 sin x)

c,sinjcH-g

c(;sx-l

C 2 cos x-\-C 3
-

3067.

3080. y

(C,

Cj

Qx) e

Answers

470

3081. x

=C

I/-,-

2
e

(^C

3082.

I/"Q~

cos^* + C sin-^

x^CV'
=
= C + C + 2sinx, z= 2C,
2C x)<r* 6jt+14,
y = (C
2C,

tj,

3083. y
3084.

3085.

e~ x
2x(3 + 4e-*),
= 10^~8e ~^ + 6/-l; y=

0=14(1
3086. x

b) In

+ = arc

f/

dz

hence,

y%
..

2
r

Vx +y
2

we

^2

the

find

xdx -\-udy

v=

==C 2

Z
|f

c)

+ (/-|-z

..

..

whence dx + dy+d*

+ +z
2

-=
dx

we have

-- Q and,

In (*
v

J/

9
2

+
,

-ryw

Hint.

6.

dz

cf/y

y_ 2 z _ x
consequently, x + + z = C
>

jc

]/

-f

x __y

9v
2
)
/

we have

of derivative proportions,

//

homo-

In

integral

A;

10.

Hint. Integrating the

Whence lnz^=

of derivative proportions,
properties
r
r
t

&+ 12/+

first

....

0,

f x (5 + 4^-

*"*)

(1

%&* + &** +

Then, using the properties

y dy
/

+i/

+ C,.

= -; xdx r=

+C

--

geneous equation

= arcxtan

tan

8t

]^x

C 2 (2A:+ 1) 3 sin x 2 cos *.


z^Q-fCj,*) e-* + 5x 9;

A;

+ ln C, and,
,

the

Applying

= dx+dt/-\-dz

x dx
1

Similarly,

x dx -\- if dy ~\- z dz
n
z(x
y)
2
C 2 Thus, the integral curves are the circles x + // + z C ,;t 2 -f// 2 -}-z 2 Co
4-z
From the initial conditions, Jt=l (/=!, z
2, we will have C l = 0, C 2 = 6.

y(zx)
.

3089.

/==

z=
3090. y

C
1

*2

2C.A;

~^
+

+ -| (3 In * + In x2

f-

1).

= C^ V

Solution.

mjj-kv x m~-^kvy
\

mg

for the initial conditions:

when

Answers
x

0,

0,

I/Q

+ mg = (kv

kv v

sinTr

t,

Vm

2-2*

3093. [/-

x2

3097.

X2
^
z

+
*%
x

r/

-j

^~

x'

3101. //m

_|

~~t
o

4
t

lions:

3104.

u(0,

//

-.2

JT

-]-~t
o

3105. u

= -4-

"(^.

--qi~~ x

the

(x,

JfH-~

9
-

^ ie

'

method

for

conver ^ es for

serics

undetermined

of
'

"-"

coefficients.

series converges for

oo<x<-}-oo.

undetermined coefficients.

of

SCneS

the

..

.}

3103.

0- -0,

(/.

M (x, 0) -=

cos /m) sin

(1

Conver 6 es

^-

sin

sin

x-

3102.

M-^cos^sin^.

for

'l<-

v
l

l~oT*

~r-

Hint. Use the condi-

-^-

Hint.

Use

the

conditions:

u(x,

~y

cos

~T"

sin

~T~

"

Hint

n-\

du

undetermined coefficients.

of

sin

1)

O-o,

^~

,'+...

tt<"/l 2

= 0.

2ig+ '"'

^""

0----0. "

w(0,

+ r

-}-

method

Hint. Use the


2

Hint. Use the

equations of motion:

...' the series converges

Use the method


x

u~

^=-,1,

X4

77

Hint.

= acos

3092. x

differential

.\ e 2

X3

3099. //--I

3100. r/--'

y=(y +

3094.

+ o4 +
+ pr-r
Z*3

'

3098. f/-^x

<x< -j-oo.

The

^
^^ 1^
.-.I^+-

3096. /t

+ mg)e m

Hint.

vx

mu 2

yl + lx +

3095.

s\na

1.

-\

a-

we obtain

Integrating,

Q
ft

sina.

i>

'"

= -~

UV

cosa,

i;

Xo

471

'

^ se

llie

conf^ tlons:

forO<x<~,

0)

3106. u

=--t A n cos
n=o

^~ r
^

""
sin

n **

^"^

where

the

coefficients

/!=

Answers

472

(2

,*. f

"+ 1)JtJf dx.

Hint. Use the conditions

^-sin
.

400
7 >u===

\^

_2^

,,
-

(1

<,.a)*

-o.

0=0.

to,

nnx

cos rui) sin

Hint. Use the" conditions: u

(0,

ioo a

e
j-rrr-

/)-=0,

u (100,

Chapter

= 0,

(x, 0)

= 0.01

x(100

x).

<1

<1

3108. a) <1"; <0.0023/


mm; <0.26/
gm; <0.0016/ .
c)
b)
3109.
0.0005; <1.45/
0.005; <0.16/
c)
b)
a) <0.05; <0.021/ ;
8
since the number lies between 47,877
3110. a) two decimals; 48-10* or 49- 10
and 48,845; b) two decimals; 15; c) one decimal; 6*10 2 For practice1 -purposes
2
3111. a) 29.5;
there is sense in writing the result in the form (5.90.1)- 10
2
or
18.5
1.643.2.
3112.
10
18470.01;
84.2;
b)
c) the result of
a)
c)
b)
subtraction does not have any correct decimals, since the difference is equal
with a possible
absolute error of one hundredth.
to one hundredth
2
Hint. Use the formula for increase in area of a square.
3113*. 1.80.3 cm
2
19.90.1
3115.
3114.
0.30.1.
30.00.2;
43.70.1;
c)
b)
a)
3116. a) 1.12950.0002; b) 0.1200.006; c) the quotient may vary between
48 and 62. Hence, not a single decimal place in the quotient may be considered certain. 3117. 0.480. The last digit may vary by unity. 3118. a) 0.1729;
2
3
s
3120. a) 1.648; b) 4. 025 0.001;
b) 277- 10 ; c) 2. 3119. (2.050.01)- 10 cm
s
cm 2 Absolute error, 65 cm 2 Relative error,
c) 9.0060.003. 3121. 4.01- 10
0.2 cm; sina
0.440.01, a-2615'
0.16/ 3122. The side is equal to 13.8
3125. The length of the pendulum
35'. 3123. 270.1. 3124. 0.27 ampere
should be measured to within 0.3 cm; take the numbers ;t and q to three
decimals (on the principle of equal effects). 3126. Measure the radii and the
generatrix with relative error 1/300. Take the number n to three decimal places
(on the principle of equal effects). 3127. Measure the quantity / to within
and s to within 0.7/ (on the principle of equal effects).
0.2/
3128.
;

<

<

Answers

473

J129.

3130.

the

4
Hint. Compute the first live values of y and, after obtaining A {/o^24, repeat
number 24 throughout the column of fourth diilerences. After this the

remaining part of the table


from right to left).

is

tilled

in

by the operation

of

addition (moving

Answers

474

b) 0. 229; 0.399; 0.491; 0.664. 3132.

3131. a) 0.211; 0.389; 0.490; 0.660;

=~

1822;

x*
x*. 3134. y
x4
0.1993; 0.2165; 0.2334; 0.2503. 3133. \+x
oe
QC
JC
x2
20 for x^5.2. Hint. When computing
8; J/^ 22 fr x
5.5;
^T
T2
x2
20 take
11. 3135. The interpolating polynomial is y
A; for
10x4- 1;
0=1 when x 0. 3136. 158 kg! (approximately). 3137. a) 0(0.5)-= 1,

+ +

0(2)=11;

= -,

b) 0(0.5)

J/t 2

)^

3138

I-

325

1.01.

3139.

3140.
019. 3143. 0.31 and 4
0.25; 2.11. 3141. 2.09. 3142. 2 45 and
1.86;
3144. 2.506. 3145. 0.02. 3146. 024. 3147. 1 27
3148.
35; 1 53
1.88;
3149. 1.84. 3150. 1.31 and
0.67. 3151. 7.13. 3152. 0.165. 3153. 1.73 and 0.
3154. 1.72.
3155. 138
3156. x
0.56
0.83;
0.83; i/=
056;
1
1997
3160. By the trapezoi3157. *=1.67;
22. 3158. 4 493. 3159.
dal formula, 11.625; by Simpson's formula, 11 417. 3161.
995;
1; 0.005;
3164. 0.79.
3163. 069
1.3-10- s
A^O.005. 3162. 0.3068; A
0.5/
3166. 0.28.
3167. 0.10.
3168. 1 61.
3170. 0.09.
3169. 1.85
3165. 0.84.
3171. 0.67. 3172. 0.75. 3173. 0.79. 3174. 4.93. 3175. 1 29. Hint. Make use
of the parametric equation of the ellipse x
cost, 0-= 0.6222 sin/<:nd trans-

x-

0=1

_JT

foim

is

the

formula

of the arc length to the

the eccentricity of the ellipse. 3176. y l (x)

x7

xn

x 15
-

63
v3

o?9

3177

59535-

Qy2

T+T""*
-2.

+1:
3178.

'

x2

W=~
T~

iW^ - 2
3x

= x,
(x)

f/ 2

=x

'

e 2 cos 2 /-d/,

}/~l

y2

(x)

xs

3x 2

*T T

'

W-|

=x
(x)

|,

2x 2

where

= xT + x^

uo

x3

y3

(x)

To +

~
J X3

yl

22

= 3.36.

form

+ 3x-2,

Zl

(*)=-i

^(x)-x~ + ^.
=

3180.
3181.
z (1)==2. 72
0.80.
3.72;
0(2)
0(1)
3 15.
0-1.80. 31S3. 3.15. 3184. 0.14. 3185. 0(0.5) -3 15; z (0 5)0.18. 3187. 1.16. 3188.0 87. 3189. x (n) -3.58;
0(0.5)^0.55; z (0 5)
3190.
x' (ji) -=0.79.
429+ 1739 cos x 1037 sin x 6321 cos 2x -f- 1263 sin 2x
3191.
96cos x-j-2. 14 sin x
1242cos3x 33sm3x.
1.68 cos 2x
0. 53 sin 2x
1.13 cos 3x
0.04 sin 3x. 3192. 0.960
0.851 cos x
0.915 sin x -|608 sin x [-f-0. 542cos2x 4-0. 620sin2x -[-0.271 cos 3x -|-0. 100 sin 3x. 3193. a)
0.414 cos x 4- 0.1 11 cos 2x |-0.056cos 3x.
4- 0.076 sin 2x4- 0.022 sin 3x; b) 0. 338

3179.
3182.
3186.

0(1)

6491

APPENDIX

I.

II.

Greek Alphabet

Some Constants

476

Appendix

III.

Inverse Quantities, Powers, Roots, Logarithms

Appendix

477

Continued

478

Appendix
IV.

Trigonometric

Functions

Appendix
V.

Exponential, Hyperbolic and Trigonometric Functions

479

480

Appendix

VI.

Some Curves

(for

Reference)

-f

1.

-/
4.

2.

Parabola,

Cubic parabola,

Graph

3.

Rectangular
hyperbola,

-/

of a fractional

5.

The witch

of Agnesi,

function,

6.

Parabola (upper
branch),

7.

Cubic parabola,

481

Appendix

Semicubical

8/7

parabola,
2

8a.

=x

^
*

or

Neile's p arabola,
2
(

y-^-c'

x^t*

or

9.

Sine curve and cosine curve,

10.

--sin

.v

and

//

Tangent curve and cotangent curve,


and /
cot#.

Appendix

482

11.

Graphs

of the functions

//-=secx and

y=arc

f/

sin x-

y^arc cos

Ji

'

A
*

arc cos

\
>

y arcsinx

Graphs of the inverse trigonometric functions


# = arc sin x and y~ arc cos x.

483

Appendix

13.

Graphs

of the

//

= arccot#.

14.

Graphs
f/

of the exponential

= g*

and y

= e~*.

cot

"V \^^ arccot

inverse trigonometric functions

//=-arc tan x and

arc cot

functions

486

Appendix

25.

Bernoulli's lemniscate,

24. Strophoid,
*

u2
y

= ATs^-M

ax
!

27.

Hypocycloid
i

\ y
or

26. Cycloid,

= a(f

(1

(astroid),

a cos 3

fl

sin 8

-f/y

/,

=a v

sin*),

cos

28. Cardioid,

= a(l+coscp).

/)

29.

Evolvent (involute)

of

the circle

sin*)*

{;::

487

Appendix

31.

^30. Spiral of Archimedes,

32.

Logarithmic
r

= *'?.

33.

spiral,

Hyperbolic spiral,
a

Three-leafed rose,
r
a sinSip.

34. Four-leafed rose,


r
asiti2(p.

INDEX

B
Absolute error 367
Absolute value

number

of a real

Bending point 84
11

Absolutely convergent series 296, 297


Acceleration vector 236
Adams' formula 390
Adams' method 389, 390, 392
Agnesi

Witch

of 18,

Bernoulli's equation 333


Bernoulli's
lemniscate
Beta-function 146, 150

155,

Binormal 238

Boundary conditions 363


Branch of a hyperbola 20, 480
Broken-line method
Euler's 326

156,480

Algebraic functions 48

Angle between two surfaces, 219


Angle of contingence 102, 243
Angle of contingence of second kind
243
Antiderivative 140, 141
generalized 143
Approximate numbers 367
addition of 368
division of 368
multiplication of 368
powers of 368
4
roots of 368
subtraction of 368

Approximation
successive 377, 385

Arc length
Arc length
Archimedes

of a curve
of a space

158-161

curve 234

spiral of 20, 65, 66, 105, 487


Area in polar coordinates 155, 256
Area in rectangular coordinates 153,
256
Area of a plane region 256
Area of a surface 166-168, 259

Argument

11

Astroid 20, 63,

105,

Asymptote 93
horizontal 94
inclined 94
right horizontal 93
right inclined 93
left
left

vertical

93

486

Cardioid 20, 105, 486


Catenary 104, 105, 484
Catenoid 168
Cauchy's integral test 295
Cauchy's test 293, 295
Cauchy's theorem 75, 326
Cavalieri's "lemon" 165
Centre of curvature 103
Change of variable 211-217
in a definite integral 146
in a double integral 252-254
in an indefinite integral 113

Characteristic equation 356


Characteristic points 96
Chebyshev's conditions 127

Chord method 376


Circle 20,
of
of

104

convergence 306
curvature 103

osculating 103
Circulation of a vector 289
Cissoid 232
of

Diodes

18,

485

Clairaut's equation 339


Closed interval 11
Coefficients
Fourier 318, 393, 394
Comparison test 143, 293, 294
Composite function 12, 49

486

Index

Concave down 91
Concave up 91

probability 19, 484


sine 481
tangent 481

Concavity

Cusp 230
Cycloid 105, 106, 486

direction of 91

Conchoid 232
Condition
Lipschitz

489

385

Conditions

boundary 363
Chebyshev's 127
Dirichlet 318, 319
initial 323, 363

D'Alembert's test 295


Decreasing function 83
Definite integral 138
Del 288

Conditional extremum 223-225


Conditionally (not absolutely)
convergent series 296
Contingence
angle of 102, 243
Continuity of functions 36
Continuous function 36
proper lies of 38

Dependent variable

Convergence
circle of 306

interval of 305
radius of 305

region of 304

uniform 306
Convergent improper
270
Convergent series 293
Coordinates

integral

of centre of gravity 170


generalized polar 255
Correct decimal places in
sense 367
Correct decimal places in
a narrow sense 367
Cosine curve 481
Cotangent curve 481

143,

broad

Coupling equation 223


Critical point of the second kind 92
points 84
Cubic parabola 17, 105, 234, 480
Curl of a vector field 288
Critical

Curvature
centre of 103
circle of 103
of a

curve 102, 242

radius of 102

second 243

Curve
cosine 481

cotangent 481
discriminant 232, 234
Gaussian 92
integral 322
logarithmic 484

11

Derivative 43
left-hand 44
logarithmic 55
nth 67
right-hand 44
second 66
Derivative of a function
in a given direction 193
Derivative of functions
represented parametrically 57
Derivative of an implicit function 57
Derivative of an inverse function 57
Derivative of the second order 66
Derivatives
of higher orders 66-69
one-sided 43
table of 47
Descartes
folium of 20, 21, 232, 485

Determinant
functional 264

Determining coefficients
first method of 122
second method of 122
Diagonal table 389
Difference of two convergent

series

298

Differential
of an arc 101, 234
first-order 71

higher-order 198
principal properties of 72
second 198
second-order 72
total,

integration

of

202-204

Differential equation 322


homogeneous linear 349
inhomogeneous linear 349
Differential equations
first-order 324
forming 329
higher-order 345

linear 349, 351

490

Index

Differential equations of higher powers


first-order

337

Differentials

method

343
and higher orders 72
Differentiating a composite function
47
Differentiation 43
of implicit functions 205-208
of

of third

tabular 46
Diocles
cissoid of 18, 485

Direction of concavity 91
Direction field 325
Dirichlet
conditions 318, 319
function 40
series 295, 296

theorem 318
Discontinuity 37
of the first kind 37
infinite 38
removable 37
of the second kind 38
Discontinuous function 270
Discriminant 222
Dicriminant curve 232, 234
Divergence of a vector field 288
Divergent improper integral 143, 270
Divergent series 293, 294

Domain
Domain

11

of definition 11

Double
in

integral 246
curvilinear coordinates

in polar coordinates 252


in
rectangular coordinates

253
246

coupling 223
differential 322
Euler's 357
exact differential 335
first-order

differential

324

homogeneous 330, 351, 356


homogeneous linear differential 332,
349

inhomogeneous 349, 351, 356


Lagrange's 339
Laplace's 289, 291
linear 332
of a normal 60, 218
of a tangent 60
of a tangent plane 218
with variables separable
Equivalent functions 33
Error
absolute 367
limiting absolute 367
limiting relative 367
relative 367
Euler integral 146
Euler-Poisson integral 272
Euler's broken-line method
Euler's equation 357
Even function 13
Evolute of a curve 103
Evolvent of a circle 486
Evolvent of a curve 104
Exact differential equation
Exponential functions 49,

Exkemal point
Extremum

327,

328

326

335
55,

483

84

conditional 223-225
of a function 83, 83, 222

Double point 230


Factor

Elimination

method

of

359

Ellipse 18, 20, 104, 485

Energy
kinetic 174

Envelope
equations of 232
of a family of plane curves 232
Epicycloid 283

Equal

effects

principle of 369

Equation
Bernoulli's 333
characteristic 356
Clairaut's 339

integrating 335
Field
direction field 325
nonstationary scalar
potential vector 289
scalar 288
solenoidal vector 289
Field (cont)
stationary scalar or
vector 288
Field theory 288-292
First-order differential
First-order differential

Flow

lines

Flux of
Folium

288

vector 288

71

equations 324

288

a vector field
of

or vector

288

Descartes 20, 21, 232, 485

Index

Force lines 288

491

logarithmic 49
transcendental, integration of 135
trigonometric 48
trigonometric, integrating 128, 129
Fundamental system of solutions 349

Form
Lagrange's 311

Formula
Adams' 390
Green's 276, 281, 282
Lagrange's 145
Lagrange's interpolation 374
Leibniz 67
Maclaurin's77, 220
Newton-Leibniz 140, 141, 275
Newton's interpolation 372
Ostrogradsky-Gauss 286-288
parabolic 382
Simpson's 382-384
Stokes' 285, 286, 289
Taylor's 77, 220
trapezoidal 382

Gamma-function

Formulas
reduction 130, 135
Fourier- coefficients 318,
Fourier series 318, 319
Four -leafed rose 487

146,

150

Gaussian curve 92
General integral 322
General solution 359
General solution (of an equation) 323
General term 294
Generalized antiderivative 143
Generalized
255
polar coordinates
Geometric progression 293, 294
Gradient of a field 288
Gradient of a function 194, 195
of a function 12
Greatest value 85, 225, 227
Green's formula 276, 281,
Guldin's theorems 171

Graph
393,

394

282

Fraction

proper rational 121

Function 11
composite 12, 49
continuous 36

Hamiltonian operator 288

Harmonic

continuous, properties
decreasing 83
Dinchlet 40
discontinuous 270

of

38

even 13
of a

Homogeneous

function 12

implicit 12
increasing 83
Lagrange 223, 224

multiple-valued
periodic 14
single-valued
vector 235

11

11

Functional determinant 264


Functional series 304
Functions
algebraic 48
equivalent 33
exponential 49, 55, 483
hyperbolic 49, 484
hyperbolic, integration of 133
inverse

series 294, 296, 297


Higher-order differential 198
Higher-order differential equations 345
Higher-order partial derivative 197
Holograph of a vector 235
Homogeneous equations 330, 351, 356

12

Functions (cont)
inverse circular 48
inverse hyperbolic 49
inverse trigonometric 482, 483
linearly dependent 349
linearly independent 349

linear

differential

equation 332, 349

Hyperbola

17, 18, 20,

485

rectangular 480
Hyperbolic functions 49, 484
integration of 133

Hyperbolic spiral 20, 105, 487


Hyperbolic substitutions 114, 116, 133
Hypocycloid 283, 486
I

Implicit function 12

Improper integral
convergent 270
divergent 270
Improper multiple integrals 269, 270
Incomplete Fourier series 318, 319
Increasing function 83
Increment of an argument 42
Increment of a function 42
Independent variable 11
Indeterminate forms
evaluating 78 79
t

492

Index

Infinite discontinuities 38
Infinitely large quantities 33
Infinitely small quantities 33
Infinites 33

Interpolation formula
Lagrange's 374
Newton's 372
Interval
of calculations 382
closed 11

Infinitesimals 33
of higher order 33
of order n 33
of the same order 33
Inflection

of
of

Interval (cont)

points of 91

open

Inhomogeneous equation 349, 351, 356


Inhomogeneous linear differential
equation 349

363
322
convergent improper 143
definite 138
divergent improper 143
double 246
Euler 146
Euler-Poisson 272
general 322
improper multiple 269, 270
line 273-278
particular 322
probability 144
singular 337
surface 284-286
triple 262
Integral curve 322
Integral sum 138
Integrating factor 335

Initial conditions 323,

Integral

Integration
basic rules of 107
under the differential sign
direct 107
by parts 116, 117, 149
path of 273, 274, 280

1 1

interval 372
circular functions 48
functions 12

hyperbolic functions 49
interpolation 373
trigonometric functions 482,

483

Jacobian 253, 264

Kinetic energy 174

109

of differential equation
of power series 361, 362
Integration of functions

Integration

by means

differential

Integration of total differentials 202-

204
Integration of transcendental functions 135

Interpolation
of functions 372-374
inverse 373

table
Inverse
Inverse
Inverse
Inverse
Inverse

Involute of a circle 20, 106, 486


Involute of a curve 104
Isoclines 325
Isolated point 230
,
Iterative method 377, 378, 380

region of 246-248
by substitution 1 13

numerical 382, 383


Integration of ordinary
equation
numerical 384-393

convergence 305
monotonicity 83

Lagrange's equation 339


Lagrange's form 311
Lagrange's formula 145
Lagrange's function 223, 224
Lagrange's interpolation formula 374
Lagrange's theorem 75
Laplace equation 289, 291
Laplace transformation 271
Laplacian operator 289

Lamina
coordinates of the centre of gravity
of a, 261
mass and static moments of a 260
moments of inertia of a 261
Least value 85
Left-hand derivative 44
Left horizontal asymptote 94
Left inclined asymptote 94
Leibniz rule 67, 269
Leibniz test 296, 297

linear 13, 372

Lemniscate 20, 105, 232


Bernoulli's 155, 486
Level surfaces 288

quadratic 372

L'Hospital-Bernoulli

rule

78-82

493

Index
of

385,

Limit of a sequence 22
Limiting absolute error 367
Limiting relative error 367
Limits

Minimum
Mixed

of inertia 169
static 168

straight 17, 20
Line integral

Monotonicity

application of 276, 283


of the first type 273, 274, 277, 278
Line integral of the second type 274,
275, 278-281
Linear differential equations 349, 351
Linear equation 332
Linear interpolation 372
of a fa nation 13
Linearly dependent functions 349
Linearly independent functions 349
Lines
flow 288
vector 288
Lipschitz condition 385

21,

105,

point
of a function 151

Mean-value theorems 75,


Mean rate of change 42
Method
1

150

389, 390, 392

chord method 376


of differentials 343
of elimination 359

Method

nth derivative 67

Nnbla 288
Napier's number 28
Natural trihedron 238
Necessary condition for
for

convergence
an extremum

106,

Newton-Leibniz formula 140, 141, 275


Newton's interpolation formula 372
Newton's method 377, 379
Newton's serpentine 18
Niele's parabola 18, 234, 481

Maclaurin's formula 77, 220


Maclaurin's series 31 1, 313
Maximum of a function 84, 222

Adams

trident of 18

derivative 55
functions 49

Mean value

11

Multiplicities
root 121

Newton

curve 484

spiral 20,

intervals of 83
Multiple-valued function

293
Necessary condition
222

force 288

Maximum

point 84

partial derivative 197

Moment

one-sided 22

Line

Logarithmic
Logarithmic
Logarithmic
Logarithmic
487

successive

approximation 381,
389
of tangents 377
of undetermined coefficients 121, 351
of variation of parameters 332, 349,
352
Minimum of a function 84, 222

Pascal's 158
Limit of a function 22
Limit on the left 22
Limit on the right 22

(cont)

Euler's broken-line 326


iterative 377, 378, 380
Milne's 386, 387, 390
Newton's 377, 379
Ostrogradsky 123, 125
Picard's 384, 385
reduction 123

Runge-Kutta 385-387, 390

Node 230
Nonstationary scalar or vector
Normal 217
to a curve 60
equations of 218
principal 238
Normal plane 238

field

288

Number
Napier's 28
real

11

Number

series 293
Numerical integration of functions 382,
383
Numerical integration of ordinary

differential

equations 384-393

One-sided derivatives 43
One-sided limits 22

Open

interval 11

494

Index

Operator

Hamiltonian 288
Laplacian 289
Order of smallness 35
Orthagonal surfaces 219
Orthagonal trajectories 328
Osculating circle 103
Osculating plane 238
Ostrogradsky-Gauss formula 286-288
Ostrogradsky-Gauss theorem 291
Ostrogradsky method 123, 125

Parabola 17, 20, 104,


cubic 17, 105, 234

105,

Niele's 18, 234, 481


safety 234
semicubical 18, 20, 234,

480, 485

481

Parabolic formula 382

critical 84
stationary 222, 225
Polar subnormal 61
Polar subtangent 61
Potential (of a field) 289
Potential vector field 289

Power

series

305

Principal normal 238


Principle
of equal effects 369
Runge 383, 386
of superposition of solutions
Probability curve 19, 484
Probability integral 144

353

Product of two convergent series 298


Progression
geometric 293, 294
Proper rational fraction 121
Proportionate parts
rule of 376

Parameters
variation of 332, 349, 352
Parametric representation of
a function 207

Quadratic interpolation 372


Quadratic trinomial 118, 119,
Quantity

Partial derivative

hirheg-order 197
"mixed" 197
second 197

infinitely large 33
infinitely small 33

Partial sum 293


Particular integral 322
Particular solution 339
Pascal's lima^on 158
Path of integration 273, 274, 280

Period of a function 14
Periodic function 14
Picard's method 384, 385

Plane
normal 238
osculating 238
rectifying 238
tangent 217
Point
bending 84
the second
(of
discontinuity 37
double 230
extremal 84

critical
of

of inflection 91

isolated 230

maximum
minimum

84
84
singular 230
stationary 196
of

tangency 217

Points
characteristic 96

123

kind)

92

Radius of convergence 305


Radius of curvature 102, 243
Radius of second curvature 243
Radius of torsion 243
Rate of change
of a function 43
mean 42
Ratio (of a geometric progression) 294
Real numbers 11
Rectangular hyperbola 480
Rectifying plane 238
Reduction formulas 130, 135, 150
Reduction method 123
Region of convergence 304
Region of integration 246-248
Relative error 367

Remainder 31
Remainder of a series 293, 304
Remainder term 311
Removable discontinuity 37
1

Right-hand derivative 44
Right horizontal asymptote 93
Right inclined asymptote 93
Rolle's theorem 75
Root multiplicities 121

495

Index

four-leafed 487
three-leafed 20, 487
Rotation (of a vector field) 288

Solenoidal vector field 289


Solution (of an equation) 322
general 323, 359
particular 339

Rule

Spiral

Rose

Leibniz 67, 269


1'Hospital-Bernoulli 78-82
of proportionate parts 376

method 385-387,
principle 383, 386

Runge-Kutta

Runge

of

390

Safety parabola 234


Scalar field 288

Subnormal

Scheme
twelve-ordinate 393-395

Second curvature 243


Second derivative 66
Second deferential 198
Second-ordeP differential 72
Second partial derivative 197
Segment of the normal 61
Segment of the polar normal 61
Segment of the polar tangent 61
Segment of a straight line 20
Segment of the tangent 61
Semicircle 20
Semicubical parabola

Archimedes 20, 65, 66,

18, 20, 1M4, -181

Series

convergent 296, 297


with complex terms 297
conditionally (not absolutely)
convergent 296
convergent 293
absolutely

Scries (cont)
Dirichlet 295, 296
divergent 293, 294
Fourier 318, 319
functional 304

harmonic 294, 296, 297


incomplete Fourier 318, 319

487

61

polar 61
Substitutions

hyperbolic 114, 116, 133


trigonometric 114, 115, 133

Subtangent 61
polar 61

Successive

method

approximation 377,
384, 385, 389

385

of

Sufficient conditions (for an

extremum)

222

Sum
integral 138
partial 293
of a series 293, 304
of two convergent series 298
Superposition of solutions
principle of 353
Surface integral of the first type 284
Surface integral of the second type 284
Surface integrals 284-286
Surfaces

level

288

orthogonal 219

Table

Maclaurin's 311, 313


number series 293
operations on 297
power 305
Taylor's 311, 313
Serpentine

diagonal table 389


of standard integrals 107
Table interval 372
Tabular differentiation 46
Tacnode 230

Newton's 18
Simpson's formula 382-384

point "of 217


Tangent 238
Tangent curve 481
Tangent plane 217
equation of 218
Tangents
method of 377
Taylor's formula 77, 220

Sine curve 481


Single-valued function 11
Singular integral 337
Singular point 230
Slope (of a tangent) 43
Smallest value 225, 227

105,

hyperbolic 20, 105, 487


logarithmic 20, 21, 105, 106, 487
Static moment 168
Stationary point 196, 222, 225
Stationary scalar or vector field 288
Stokes' formula 285, 286, 289
Straight line 17, 20
Strophoid 157, 232, 234, 486

Tangency

Index

496
Taylor's series 311, 313

Term
general 294
remainder 311
Test
d' Alembert's 295
Cauchy's 293, 295
Cauchy's integral 295
comparison 143, 293, 294
Leibniz 296, 297

Weierstrass' 306

computing volumes by means


evaluating a 265

of

268

in rectangular coordinates 262


Trochoid 157
Twelve-ordinate scheme 393-395

U
Undetermined coefficients
method of 121, 351
Uniform convergence 306

Theorem
Cauchy's

75, 326

Dirichlet's 318

Theorem

(cont)

Lagrange's 75
Ostrogradsky-Gauss 291
Rolle's 75

Theorems
Guldin's 171

mean-value
Theory

75, 150

288-292
Three-leafed rose 20, 487
Torsion 243
field

Tractrix 161
Trajectories

orthogonal 328
Transcendental functions
integration of 135
Transformation
Laplace 271
Trapezoidal formula 382
Trident of Newton 18
Trigonometric functions 48
integrating 128, 129
Trigonometric substitutions 114, 115,
133
Trihedron
natural 238
Trinomial
quadratic 118, 119, 123
Triple integral 262
applications of 265, 268
change of variables in 263

Value
greatest 85, 225, 227
least

85

mean

(of a function) 151, 252


smallest 225, 227

Variable

dependent 11
independent 11
Variables separable
an equation with 327, 328
Variation of parameters 332, 349, 352
Vector
acceleration 236
of binomial 238

normal 238
tangent line 238
velocity 236
Vector field 288
Vector function 235
Vector lines 288
Velocity vector 236
Vertex of a curve 104
Vertical asymptote 93
Vertices of a curve 104
of principal

of

Volume
Volume

of a cylindroid 258
of solids 161-166

W
1

Weierstrass test 306


Witch of Agnesi 18, 156, 480

Work

of a force

174, 276,

277

You might also like